You are on page 1of 266

Krishna's

TEXT BOOK

PHYSICAL
CHEMISTRY-II
B.Sc. Second Year Third Paper

As per U.P. UNIFIED Syllabus

Alok Bariyar
Ph.D. (CSIR fellow), M.Sc.(IIT Delhi),
G.A.T.E., N.E.T.
Ex. Scientist (Nuclear)
Bhabha Atomic Research Centre,
Mumbai

R.P. Singh Ashish Dwivedi


M.Sc., Ph.D. M.Sc., Ph.D.
Head, Dep’t of Chemistry Dep’t of Chemistry

KNI of Physical Social Science, Ganpat Sahai P.G. College,


Sultanpur (U.P.) Sultanpur (U.P.)

KRISHNA Prakashan Media (P) Ltd.


KRISHNA HOUSE, 11, Shivaji Road, Meerut-250 001 (U.P.), India
Jai Shri Radhey Shyam

Dedicated
to

Lord

Krishna
Author & Publishers
P REFACE
e are happy to present this book entitled “Physical Chemistry-II”. It has been written

W according to the latest U.P. Unified Syllabus to fulfil the requirement of B.Sc. IInd year
students of all Colleges & Universities in Uttar Pradesh.

The book is written with the following special features:

1. It is written in a simple language so that all the students may understand it easily.

2. It has an extensive and intensive coverage of all topics.

3. In each Chapter, Solved Examples are given based on different Topics.

4. The complete Syllabus has been divided into Seven Chapters under Four Units.

5. Sufficient Subjective Questions and Objective type questions with Hints & Solutions given at the end of
each Chapter will enable Students to understand the concept .

First of all we want to express our sincere gratitude to Purnima Sinha, Dr. S.B.P Sinha, Prof.
J.C. Ahluwalia, Prof. N.K. Jha for their invaluable guidance, immense interest and constant
encouragement for the successful completion of the work. We are also thankful to Bandana Bariyar,
Ashish Bariyar, Abhishek Bariyar, Archi Bariyar & Aradhyaa Bariyar for their kind help at many
occasions.

We are extremely grateful to our respected and beloved Parents whose incessant inspiration
guided us to accomplish this work. We also express gratitude to our respective Families for their moral
support.

We are immensely thankful to Mr. S.K. Rastogi (Managing Director), Mr. Sugam Rastogi
(Executive Director), Mrs. Kanupriya Rastogi (Director) and entire team of Krishna Prakashan Media
(P) Ltd., for taking keen interest in this project and outstanding Management in getting the book
published.

The originality of the ideas is not claimed and criticism and suggestions are invited from the
Students, Teaching community and other Readers.

June 2015 –Authors

(iv)
Syllabus
Physical Chemistry-II
B.Sc. II nd Year Paper III
U.P. UNIFIED (w.e.f. 2012-13) M.M. 50

UNIT-I : (Thermodynamics & Chemical Equilibrium)


I. Thermodynamics -I
Definitions of thermodynamics terms: System, surroundings etc. Types of systems, intensive and extensive
properties, state and path functions and their differentials, thermodynamic processes, concept of heat and work.
First Law of Thermodynamics: Statement, definition of internal energy and enthalpy, heat capacity, heat
capacities at constant volume and pressure and their relationship, Joule's law- Joule-Thomson coefficient and
inversion temperature. Calculation of w, q, dU & dH for the expansion of ideal gases under isotheral and adiabatic
conditions for reversible process.
Thermochemistry: Standard state, standard enthalpy of formation- Hess's Law of heat summation and its
applications, heat of reaction at constant pressure and at constant volume, enthalpy of neutralization, bond
dissociation energy and its calculation from thermo-chemical data, temperature dependence of enthalpy,
Kirchhoff's equation.

UNIT-II
II. Chemical Equilibrium
Equilibrium constant and free energy, Thermodynamic derivation of law of mass action, Le-Chatelier's principle.
Reaction isotherm and reaction isochore- Clapeyron-clausius equation and its applications.
III. Thermodynamics-II
Second Law of Thermodynamics: Need for the law, different statements of the law, Carnot's cycle and its efficiency,
Carnot's theorem. Thermodynamic scale of temperature.
Concept of entropy: Entropy as a state function, entropy as a function of V &T, entropy as a function of P & T,
entropy change in physical change, clausius inequality, entropy as a criteria of spontaneity and equilibrium,
equilibrium change in ideal gases and mixing of gases.
Gibbs and Helmholtz functions: Gibbs function (G) and Helmholtz function (A) as thermodynamic quantities, A
& G as criteria for thermodynamic equilibrium and spontaneity, their advantage over entropy change, variation of
G and A with P,V and T.
Third Law of Thermodynamics: Nernst heat theorem, statement and concept of residual entropy. Nernst
distribution law- thermodynamic derivation, applications.

UNIT-III : (Electrochemistry-I & Solutions)


IV. Electrochemistry -I
Electrical transport: Conduction in metals and in electrolyte solutions, specific conductance molar and equivalent
conductance, measurement of equivalent conductance, variation of molar equivalent and specific conductance
with dilution.
(v)
Migration of ions and Kohlrausch's law, Arrhenius theory of electrolyte dissociation and its limitations, weak and
strong electrolytes, Ostwald's dilution law its uses and limitations, Debye-Huckel-Onsager’s equation for strong
electrolytes (elementary treatment only), Transport number, definition and determination by Hittorf ’ s method
and moving boundary method. Applications of conductivity measurements: determination of degree of
dissociation, determination of Ka of acids, determination of solubility product of a sparingly soluble salt,
conductometric titrations.
V. Solutions
Liquid- Liquid mixtures- Ideal liquid mixtures, Raoult's and Henry's law, non-ideal system-azeotropes-HCl-H2O
and ethanol-water systems. Partially miscible liquids- phenol-water, trimethylamine-water, nicotine-water
systems, immiscible liquids, steam distillation.

UNIT-IV: (Electrochemistry-II & Phase Equilibrium)


VI. Electrochemistry-II
Types of reversible electrodes- gas-metal ion, metal-ion, metal-insoluble salt-anion and redox electrodes, Electrode
reactions, Nernst equation, derivation of cell E.M.F. and single electrode potential, standard hydrogen electrode-
reference electrodes and their applications, standard electrode potential, sign conventions, electrochemical series
and its significance. Electrolytic and Galvanic cells-reversible and irreversible cells, conventional representation of
electrochemical cells. EMF of a cell and its measurements, computation of cell EMF, calculation of
thermodynamic quantities of cell reactions (∆ G, ∆ H and K). Concentration cell with and without transport, liquid
junction potential, application of concentration cells, valency of ions, solubility product and activity coefficient,
potentiometric titrations. Definition of pH and pKa, determination of pH using hydrogen, quinhydrone and glass
electrodes, by potentiometric methods; buffers- mechanism of buffer action, Henderson-Hazel equation,
application of buffer solution, hydrolysis of salts.
VII. Phase Equilibrium
Statement and meaning of the terms-phase, component and degree of freedom, derivation of Gibb's phase rule,
phase equilibria of one component system-water, ‘CO2’ and ‘S’ systems. Phase equilibria of two component
system-solid liquid equilibria simple eutectic-Bi-Cd, Pb-Ag systems, desilverisation of lead. Solid solutions-
compound formation with congruent melting point (Mg-Zn) and incongruent melting point, (FeCl3-H2O) and
(CuSO4-H2O) system.

(vi)
Unit - I

Chapter 1 Thermodynamics-I P–03-40

1.1 Introduction 03
1.2 Some Basic Terms in Thermodynamics 03
1.3 First Law of Thermodynamics 07
1.4 Heat Capacity, Specific Heat Capacity and Molar Heat Capacity 08
1.5 Work Done by Ideal Gas in Different Processes 11
1.6 Joule−Thomson Effect 16
1.7 Thermochemistry 19
1.8 Different Types of Heats of Reaction 21
1.9 Hess's Law 22
1.10 Kirchhoff 's Law 26

Unit - II

Chapter 2 Chemical Equilibrium P–41-69

2.1 Introduction and Basic Terms 41


2.2 Equilibrium Constant for Different Types of Equilibria 48
2.3 Le-Chatelier's Principle 56
2.4 Free Energy Change and Spontaneity of a Reaction 61

Chapter 3 Thermodynamics - II P–70-102

3.1 Introduction 70
3.2 Carnot Cycle 70
3.3 Second Law of Thermodynamics 73
3.4 Entropy (S) 75
3.5 Gibbs Free Energy, G
3.6 Nernst Heat Theorem and Third Law of Thermodynamics 89
3.7 Nernst's Distribution Law 90
3.8 The Distribution Law 90

(vii)
Unit - III

Chapter 4 Electrochemistry - I P–103-136

4.1 Introduction and Basic Terms 103


4.2 Specific Conductance, Equivalent Conductance and Molar Conductance 105
4.3 Transport Number 113
4.4 Kohlrausch's Law 118
4.5 Arrhenius Theory of Electolytic Dissociation 122
4.6 Conductometry 126

Chapter 5 Solution P–137-160

5.1 Introduction 137


5.2 Vapour Pressure 137
5.3 Ideal Solution 139
5.4 Solubility of Partially Miscible Liquids 143
5.5 Solutions of Gases in Liquids 149

Unit - IV

Chapter 6 Electrochemistry - II P–161-230

6.1 Introduction 161


6.2 Galvanic Cell 161
6.3 Electrochemical Series and its Significance 169
6.4 Concentration Cells 182
6.5 Potentiometric Titration 190
6.6 Ionic Equilibrium 196
6.7 Different Types of Commercial Cells 214

Chapter 7 Phase Equilibrium P–231-260

7.1 Different Basic Terms 231


7.2 One Component System 237

❍❍❍

(viii)
Book-3
P hysical C hemistry–II
Unit-I
Chapter 1: Thermodynamics-I

Unit-II
Chapter 2: Chemical Equilibrium
Chapter 3: Thermodynamics-II

Unit-III
Chapter 4: Electrochemistry-I
Chapter 5: Solutions

Unit-IV
Chapter 6: Electrochemistry-II
Chapter 7: Phase Equilibrium
UnitP-3
-I

C HAPTER 1
Thermodynamics-I

1.1 Introduction
Thermodynamics is a branch of natural science which deals with the bulk behaviour of the body. It does not
describe the microscopic behaviours of the molecules. Historically, thermodynamics developed out of a
desire to increase the efficiency and power output of early steam engines. And, now a days it is applies to a
wide variety of topics in science,engineering and various other fields of life. Thermodynamics deals with heat
and temperature and their relation to energy and work. It defines macroscopic variables, such as internal
energy, entropy and pressure which partly describe a part of matter or radiation. It states that the behaviour
of those variables is subject to general constraints. These general constraints are expressed in terms of the four
laws of thermodynamics – zeroth law, first law, second law and third law.

1.2 Some Basic Terms in Thermodynamics


1.2.1 Thermodynamic Function or Thermodynamic Property of a System
Those properties of a system, which depend on state of a system only and not on manner by which that state
is achieved or changed to some other state, are known as thermodynamic functions. Internal energy – E or U,
enthalpy - H, entropy S and free energy - G are the four thermodynamic functions. G is also known as Gibbs
free energy. There is one more type of free energy – A, known as Helmholtz free energy. Mathematically, the
change in such functions like dE or dU, dH, dS , dG and dU are perfect differentials.

1.2.2 System and Surroundings


A primary goal of the study of thermochemistry is to determine the quantity Surroundings
of heat exchanged between a system and its surroundings. The system is the
part of the universe being studied, while the surroundings are the rest of the
universe that interacts with the system. A system and its surroundings can be
as large as the rain forests in South America or as small as the contents of a
System
beaker in a chemistry laboratory. The type of system one is dealing with can
have very important implications in chemistry because the type of system
dictates certain conditions and laws of thermodynamics associated with that
Energy
system. Fig. 1: System and surroundings
Thermodynamic systems are of three types,
P-4

1. Open System
2. Closed System
3. Isolated System
1. Open System: An open system is a system that freely exchanges energy and matter with its
surroundings. For instance, when one is boiling soup in an open saucepan on a stove, energy and
matter are being transferred to the surroundings through steam. The saucepan is an open system
because it allows for the transfer of matter for example, adding spices in the saucepan and for the
transfer of energy for example, heating the saucepan and allowing steam to leave the saucepan.
One can examine how matter and energy are exchanged in an open system. Matter can be exchanged
rather easily - by adding matter like spices or removing matter. Energy exchange is a little bit more
complicated than matter exchange. There are a couple of ways energy can be exchanged - through
heat and through work. Energy induced through heat can be demonstrated by bringing the system
close to an object that dissipates heat like Bunsen burner, stove etc. By doing so, one is able to change
the temperature of the system and therefore, induce energy through heat. Another way to increase the
energy is through work. An example of inducing work is by taking a stirrer and then mixing the coffee in
the cup with the stirrer. By mixing coffee, work is done as the coffee is being moved against a force.

Matter
Matter

Energy gy
Ener
Open Energy Energy
Energy
System

Fig.
Fig:
Fig: 2
2:
2 Open
Opensystem
Open system
system
2. Closed System: Putting a lid on the saucepan makes the saucepan a closed system. A closed
system is a system that exchanges only energy with its surroundings, not matter. By putting a lid on the
saucepan, matter can no longer transfer because the lid prevents matter from entering the saucepan
and leaving the saucepan. Still, the saucepan allows energy transfer. Imagine putting the saucepan on
a stove and heating it. The saucepan allows energy transfer as the
saucepan heats up and heats the contents inside it. For example, when a lid is put on a beaker, and
when the contents in the beaker are boiled, the sides of the beaker will start getting foggy and misty.
This fog and mist is the steam which covers the sides of the container because it cannot escape the
beaker due to the lid. The fact that the beaker is able to produce this steam means that the beaker
allows for energy transfer. Thus, even though a closed system cannot allow matter transfer, it can still
allow energy transfer. The methods of energy transfer in a closed system are the same as those
described for an open system above.
P-5

Matter does
Ene not go in or
rgy rgy
Ene out
Closed
System

Energy Energy

Fig. 3: Closed system

3. Isolated System: Now a type of system is examined in which a thermos flask substitutes for the
saucepan. A thermos is used to keep things either cold or hot. Thus, a thermos does not allow for
energy transfer. Additionally, the thermos, like any other closed container, does not allow matter
transfer because it has a lid that does not allow anything to enter or leave the container. As a result, the
thermos is an isolated system. An isolated system does not exchange energy or matter with its
surroundings. For example, if soup is poured into an insulated container and closed, there is no
exchange of heat or matter. The fact that, in reality, a thermos is not perfect in keeping things
warm/cold illustrates the difficulty in creating a truly isolated system. In fact, there are a only few
systems that exist in this world that are completely isolated systems.
Matter does
not go in or
out

Isolated
System Energy does Energy does
not go in or not go in or
out out

Fig. 4: Isolated system

1.2.3 Intensive and Extensive Properties


An intensive property is a physical quantity whose value does not depend on the amount of the substance for
which it is measured. For example, the temperature of a system in thermal equilibrium is the same as the
temperature of any part of it. If the system is divided the temperature of each subsystem is identical. The same
applies to the density of a homogeneous system; if the system is divided in half, the mass and the volume
change in the identical ratio and the density remains unchanged. Additionally, the boiling point of a
substance is another example of an intensive property. For example, the boiling point for water is 100°C at a
pressure of one atmosphere, a fact which remains true regardless of quantity.
P-6

Extensive properties are the counterparts of intensive properties, which are intrinsic to a particular system. An
extensive property is defined as a physical quantity which is proportional to the size of the system it describes,
or to the quantity of matter in the system. The value of such an additive property is the sum of the properties
of separate non interacting subsystems that compose the entire system. Taking on the example of melting of
ice, the amount of heat required to melt ice is an extensive property. The amount of heat required to melt one
ice cube would be much less than the amount of heat required to melt an iceberg.

Dividing one type of extensive property by a different type of extensive property will in general give an
intensive value. For example, mass (extensive property) divided by volume (extensive property)
gives density which is an intensive property.

1.2.4 State and Path Functions


In thermodynamics,a state function or state variable is a property of a system that depends only on the
current state of the system, not on the way in which the system acquired that state. So, state functions are
independent of path. A state function describes the equilibrium state of a system. For example, internal
energy, enthalpy and entropy are state quantities because they describe quantitatively an equilibrium state of
a thermodynamic system, irrespective of how the system arrived in that state. Some of the examples of the
state functions are mass,Pressure (P), Temperature (T),Volume (V), Internal energy (E), Enthalpy (H),
Entropy (S),Gibbs free energy (G), Helmholtz energy (F).

In contrast,mechanical work and heat are process quantities because their values depend on the path
between two equilibrium states. So, work and heat are examples of non state or path function. As such, the
opposite of a state function is a path function.

1.2.5 Various Processes


1. Isothermal process: Processes taking place at constant temperature are called isothermal process,
such that ∆T = 0.
2. Isobaric process: Those processes carried out at constant pressure are called isobaric processes,
such that ∆P = 0.
3. Isochoric process: Those processes carried out at constant volume are called isochoric processes
such that ∆V = 0.
4. Adiabatic process: In an adiabatic process, no heat exchange takes place between the system and
surroundings, such that q = 0.
5. Cyclic processes: Those processes for which the initial and final states are the same are referred as
cyclic process.
6. Reversible and irreversible process: In thermodynamics, a process is said to be reversible when
the energy change in each step of the process can be reversed in direction by changing the variables
such as pressure, volume or temperature acting on them. Here the process is carried out infinitesimally
slowly such that at every stage the system is in pressure and temperature equilibrium with its
surroundings. In such a process, the driving and opposing forces differ by a very small margin and the
process can be reversed completely by increasing the opposing force by an infinitesimal amount. Any
process which does not take place in the above mentioned manner is said to be an irreversible process.
In an irreversible process, the driving and opposing force differ by a larger amount. The fact that the
P-7

process is irreversible does not mean that the system cannot be restored to the original state. The water
that has come down to the ocean can be taken back to the top of the hill. But such reversals would
involve work by the surroundings or external agencies greater than that is obtained during the forward
change.

1.2.6 Enthalpy or Heat Content of a System (H)


The pressure and volume of a system are thermodynamic variables and their product PV is expressed in units
of energy. The sum of E and PV (E + PV) is thus a form of energy. The sum, E + PV is universally denoted by
H and is called enthalpy or heat content of the system at constant pressure. dH or ∆H is a measure of the heat
energy. It is also defined as dq or ∆q supplied to the system from the surroundings at constant pressure.

1.2.7 Internal Energy of a System, E or U


Every substance is made up of molecules, which are themselves made up of still smaller species like electrons
and nuclei. Each of these particles has kinetic energy as well as potential energy. The sum of the kinetic and
potential energies of all the particles making up a substance is referred to as the internal energy. Total energy,
E T of a quantity of substance is the sum of its kinetic and potential energies as a whole (E K + E P ) plus its
internal energy.
ET = EK + EP + U
A gas molecule possesses energy due to various motions, such as translational, vibrational and rotational
motions. Furthermore, it is already known that the gas molecules do not have potential energy at all as
suggested by kinetic theory of gases. All the different modes that contribute to the total internal energy of a
gas molecule are called degrees of freedom. A monoatomic gas molecule has three degrees of freedom, a
diatomic molecule has five degrees of freedom while a each triatomic molecule and subsequent polyatomic
molecules are considered to have six degrees of freedom.

1.2.8 Zeroth Law of Thermodynamics


It is also known as law of thermal equilibrium. Despite the fact that the law was put forward much after the
establishment of first and second law, it is placed before the first and second law. According to this law, if
system A and B are separately in thermal equilibrium with ‘C’ then A and B will belong to the same
equilibrium.
For example, when a thermometre is place in the system, it comes to thermal equilibrium with the system.
Statement of zeroth law: Two systems in thermal equilibrium with a third system are also in thermal
equilibrium with each other. Thermal equilibrium means having same temperature.

1.3 First Law of Thermodynamics


First law of thermodynamics is nothing but law of conservation of energy stated in different manner.
According to this law, “different forms of energy are interchangeable but whenever a quantity of one kind of
energy disappears, an equivalent amount of another kind appears.” From this statement, it follows that
various changes and transformations might occur but the total energy of universe must remain constant. If on
supplying dq amount of heat energy to a system from its surroundings, its internal energy changes by dE or
dU and it performs dw amount of work, then according to first law of thermodynamics.
dq = dE + dw
P-8

That is, heat taken up by a system from the surroundings would be equal to the sum of internal energy
increase of the system and the work done by or on the system. This is mathematical statement of the first law
of thermodynamics. q and w are not state functions, rather they depend on the path followed by the system
in going from one state to another. Internal energy, E or U is a state function and does not depend on the path
followed in reaching that state. Before moving into the application part of first law of thermodynamics, it is
important to know the concept of work done.

1.3.1 Pressure-Volume Work ( P-V Work)


If a gas sample expands at constant temperature keeping its pressure unchanged the gas sample has to do
some work at the cost of its internal energy. The magnitude of work done by it is equal to P∆V, where ∆V =
change in volume and P is the pressure of the sample.

Similarly, if an external agent compresses the volume of the gas sample isothermally at constant pressure by
say ∆V, then the external agent will have to do work over sample equal to P∆V and this work done will be
stored in the gas sample as internal energy , so, the internal energy of system will increase. Now, the first law
of thermodynamics is applied to various cases.
1. Adiabatic change: In an adiabatic change, since no heat exchange takes place between the system
and surroundings, so, dq = 0. Hence from equation
0 = dE + dw
or dw = − dE
Hence, in the case of an adiabatic change the system will do work at the cost of its internal energy.
2. Cyclic change: Since internal energy of a system is a state function, hence in a cyclic process, dE = 0.
So, dq = 0 + dw
Or, dq = dw
It means that in a cyclic process, the whole of the heat energy supplied to the system is used up in
performing mechanical work.
3. Isochoric change: The process, which takes place under constant volume conditions is termed as an
isochoric process. If the system under consideration is an ideal gas for which dw is of mechanical
nature only, involving expansion or contraction, then dw can be replaced by PdV, that is,
dq = dE + PdV
If the change is isochoric, dV = 0,
∴ dq = dE
It means that at constant volume, the whole of the heat energy absorbed by the system would be used
up in increasing its internal energy.
Work done in isothermal process will be discussed later in this chapter.

1.4 Heat Capacity, Specific Heat Capacity and Molar Heat Capacity
Heat capacity is the measurable physical quantity of heat energy required to change the temperature of a
substance through 1Kelvin or 1° C of a given amount. The SI unit of heat capacity is joule per Kelvin . Heat
capacity is an extensive property of matter, meaning it is proportional to the size of the system. When
P-9

expressing the same phenomenon as an intensive property, the heat capacity is divided by the amount of
substance-say mass or sometimes even by volume. It makes the quantity independent of the size of the
sample. The specific heat capacity, often simply called specific heat, is the heat capacity per unit mass of a
material.Similarly, the molar heat capacity is defined as heat capacity for one mole of substance. It is
calculated separately at constant volume and constant pressure for gases only. For solids and liquids both the
values are the same. Heat capacities at constant volume and constant pressure for gases are frequently
termed as C V and C P respectively. It is given by the formula,
∆H = n C P ∆T where ∆H is enthalpy change of the system and C V is expressed in terms of ∆U and given by
the formula ∆U = n C V ∆T where ∆U is internal energy change of the system.

1.4.1 Thermodynamic Relations and Definition of Heat Capacity


The internal energy of a closed system changes either by adding heat to the system or by the system
performing work. Written mathematically it can be stated as
dU = δQ + δW .
For work as a result of an increase of the system volume it can be written as,
dU = δQ − PdV.
If the heat is added at constant volume, then the second term of this relation vanishes and one readily
obtains,
 ∂U   ∂Q 
  =   = CV .
 ∂T  V  ∂T  V

This defines the heat capacity at constant volume, C V , which is also related to changes in internal energy.
Another useful quantity is the heat capacity at constant pressure, C P . This quantity refers to the change
in enthalpy of the system and is given by
H = U + PV.
A small change in the enthalpy can be expressed as
dH = δQ + VdP.
and therefore, at constant pressure, it can be said that,
 ∂H   ∂Q 
  =   = CP .
 ∂T  P  ∂T  P

These two equations:


 ∂U   ∂Q 
  =   = CV .
 ∂T  V  ∂T  V

 ∂H   ∂Q 
  =   = CP .
 ∂T  P  ∂T  P

are property relations and are therefore independent of the type of process. In other words, they are valid for
any substance going through any process. Both the internal energy and enthalpy of a substance can change
with the transfer of energy in many forms like heat. It is easier to measure the heat capacity at constant
pressure, allowing the material to expand or contract freely and solve for the heat capacity at constant volume
using mathematical relationships derived from the basic thermodynamic laws. Starting from the fundamental
thermodynamic relation one can show
P-10

 ∂P   ∂V 
CP − CV = T    
 ∂T  V,N  ∂T  P,N

where the partial derivatives are taken at constant volume and constant number of particles, and constant
pressure and constant number of particles, respectively. This can also be rewritten as
α2
C P − C V = VT
βT
Here, α is the coefficient of thermal expansion, β T is the isothermal compressibility.
For an ideal gas, evaluating the partial derivatives above according to the equation of state where R is the
gas constant for an ideal gas
PV = RT
 ∂P   ∂V 
CP − CV = T    
 ∂T  V  ∂T  P
2
 ∂P   ∂V 
C P − C V = −T    
 ∂V  T  ∂T  P
RT  ∂P  − RT − P
P= ⇒  = =
V  ∂V  T V2 V
2
RT  ∂V  R2
V= ⇒  =
P  ∂T  P P 2

Substituting these values,


2 2
 ∂P   ∂V   −P   R 
− T     = −T    2 = R
 ∂V  T  ∂T  P  V  P 

This equation reduces simply to


CP − CV = R

It is also known as Mayer's relation.

The heat capacity ratio (γ) is the ratio of the heat capacity at constant pressure to heat capacity at constant
volume. It is sometimes also known as the expansion factor.
CP
γ=
CV

1.4.2 Calorimetry
An object undergoing a temperature change without a chemical reaction or change of state, absorbs or
discharges an amount of heat equal to its heat capacity times the temperature change, such that,

heat exchange = (heat capacity)×(temperature change)

On the basis of this formula, enthalpy change accompanying a chemical reaction can be very earily
calculated.
P-11

Example 1: A sample of solid naphthalene C10 H 8 , weighing 0.600 g is burnt to CO 2(g) and
H 2O(l) in a constant volume calorimeter at T = 298K. In this experiment, the observed
temperature rise of the calorimeter and its content is 2.27° C. In a separate experiment, the
total heat capacity of the calorimeter was found to be 2556 cal/deg. What is ∆E for the
combustion of one mole of naphthalene? What is ∆H for this reaction? Also calculate the
enthalpy of formation of naphthalene.
∆H 0f (H 2O, l) = − 68.32 kcal/mol
∆H 0f (CO 2, g) = − 94.05 kcal/mol

Solution : C10H8 (s) + 12O 2 (g ) → 10CO 2 (g ) + 4H2O (g )


Heat released = 2.27 × 2556 = 5.802 kcal
5.802 × 128
∆E =− = − 1237.76 kcal/mol
0.6
Also, ∆H = ∆E + ∆nRT
For the reaction,
C10H8 (s) + 12O 2 (g ) → 10CO 2 (g ) + 4H2O (l )
∆n = 10 −12 = −2
∴ ∆H = −1237.76 − 2 × 2 × 10 −3 × 298 = − 1238.952 kcal / mol
Also − 1238 . 952 = 10∆H f CO 2 (g ) + 4 ∆H f H2O(l ) − ∆H f C10H8 (s)
−1238.952 = 10 × (− 94.05) + 4 (− 68.32) − ∆H f C10H8
∴ ∆H f C10H8 = 25.172 kcal / mol

1.5 Work Done by Ideal Gas in Different Processes


As already studied, the basic concept for calculating the work done is known. There are different cases for
calculating the work done by an ideal gas. Some of these cases are discussed below :

1.5.1 Work Done in an Isothermal Reversible Process


For n mole of an ideal gas, which expands isothermally and reversibly from volume V 1 and pressure P 1 to
final volume V 2 and pressure P 2 , the total work done by the system is given by the expression.
V2
Wrev = − 2.303 nRT log …(1)
V1
P1
Wrev =− 2.303 nRT log …(2)
P2
It can also be given by,
V2
Wrev = − 2.303 P 1V 1 log as it is known that, P 1V 1 = nRT.
V1
P-12

1.5.2 Derivation
The reversible compression or expansion of the gas proceeds in an infinite number of steps, where the
external pressure applied is always essentially equal to the internal pressure of the gas. The work done in the
expansion or compression is given by
W = − P ext ∆V,
As the process is reversible, P ext = P int .
Following PV = nRT, P ext = nRT / V,
Putting these values and integrating,
V2
w=−
∫ PdV …(1)
V1

Substitute for pressure in (1), it can be said


V2
nRT
w=−
∫ V
dV
V1
V2
dV
w = −nRT
∫ V
V1

V  dx 
w = −nRT [ ln V]V2
1 Q ∫ x
= ln x 

w = −nRT [ln V2 − ln V1]
V2
w = −nRT ln
V1
V2
w = − 2.303 nRT log (ln x = 2.303 log x )
V1
Since P1V1 = P2 V2
V2 P1
=
V1 P2
P1
w = − 2. 303 nRT log
P2
Other thermodynamic parameters are evaluated as discussed below. (1) ∆H = nC p ∆T, as the process is
isothermal, ∆T = 0, hence ∆H= 0.(2) Similarly, ∆U = nCv∆T = 0. (3) From the first law of thermodynamics,
V
∆U = q +w, so q = − w = 2.303 nRT log 2 .
V1

1.5.3 Irreversible Isothermal Expansion (When external pressure is constant)


If the same change is carried out irreversibly, work done by the system is given by the expression
wirrev = − P ext (V 2 − V 1),
P-13

∆H = nC P ∆T, as the process is isothermal, ∆T = 0 , hence ∆ H= 0. Similarly, ∆U = nC V ∆T = 0. From the first


law of thermodynamics, ∆U = q +w, so q = −w = P ext (V 2 − V 1).
It is known that,
w rev > wirrev
That is, work done by the system in a reversible isothermal process is greater than the work done in the same
change which is carried out in an irreversible manner. For this reason, w rev is the maximum possible work,
w max obtained out of the change of state.
There is one more case of constant external pressure when external pressure is zero (P ext =0)
In this case w = − Pext (V2 − V1) = 0
∆H = 0, ∆V = 0
Even q = 0 this process is also known as free expansion.

1.5.4 Work Done in an Adiabatic Expansion


Adiabatic processes are those processes in which heat is neither allowed to enter nor leave the system. So,
there is no exchange of energy between the system and the surroundings. For such processes,
CP
PV γ = constant where, γ =
CV

TV γ −1 = constant
T γ P 1− γ = constant
Relation between T − V and T − P and P − V for a reversible adiabatic process:
For an adiabatic process, q = 0, so, ∆U = w
As P is the external pressure against which expansion is taking place and ∆V is the change in volume so that
the work done is − P∆V . So, ∆U = − P∆V
Now, ∆U = C V (∆T ) (From Kirchhoff’s law)
RT  RT 
⇒ C V dT = − PdV ⇒ C V dT = − dV Q P = 
V  V 
dT dV
⇒ CV = −R
T V
Integrating,
T2 dT V2 dV
CV
∫T1 T
=−R
∫V1 V
T2 V T R V
C V ln = − R ln 2 ⇒ ln 2 = ln 1
T1 V1 T1 C V V2
γ −1
T2 V 
⇒ ln = ln  1 
T1  V2 
 CP − CV R R
Q C P − C V = R ⇒ = ⇒γ −1= 
 C V C V C V
P-14
γ −1
T2  V1 
So, = 
T1  V2 
−1
⇒ TV γ = constant. (Here γ = C P / C V ).
Case 1. As P1V1 = RT 1 and P2 V2 = RT2 .
T2 P2 V2
So, = ; Using this relation,
T1 P1V1
γ −1 γ −1 γ
P2 V2  V1  P2 V1  V1  V  γ γ
=  ⇒ =   =  1  ⇒ P1V1 = P2 V2
P1V1  V2  P1 V2  V2   V2 
V1 RT1 / P1 V T P
Case 2. As, = ⇒ 1= 1× 2
V2 RT2 / P2 V2 T2 P1
γ −1 γ γ−1
T2  T1 P2  T  P 
Putting this values, = ×  ⇒  2 =  2
T1  T2 P1   T1   P1 
γ 1– γ
 T2   P1 
⇒   = 
 T1   P2 
γ 1− γ γ 1− γ
⇒ T2 P2 = T1 P1
1− γ
⇒ TγP = constant.
The work done in an adiabatic reversible expansion is governed by
γ γ
P1V1 = P2 V2
where γ = CP / CV ,
C P is the heat capacity of a gas at constant pressure and C V is the heat capacity of the gas at constant volume.
Now, the work done is calculated as below. For adiabatic process,
q = 0, So, dU = dw.
P
As a matter of fact ∆U and ∆ H can be calculated very easily.
Isothermal process
dU = nC V ∆T = dw, so w = nC V (T f − Ti ),
So work done in an adiabatic process is,
dw = dU or w = ∆U
or w = nC V ∆T = nC V (T f − Tv )
Similarly, dH = nC V ∆T = nC V (T f − Ti ).
Adiabatic process
The work done in adiabatic reversible expansion is lesser than
isothermal expansion process as shown in the graph below:
Work done V
Work done can also be calculated in adiabatic irreversible
Fig 5: Work done isothermal
process, but there are two cases − (i) the intermediate and adiabatic process
expansion, (ii) free expansion.
(i) The intermediate expansion: If the gas is expanding from V1 to V2 against a constant external
pressure Pext , then the work done is, w = − Pext (V2 − V1)
Moreover ∆U = w (For adiabatic process)
And ∆U = C V (T2 ′ − T1); here T2 ′ is final temperature and T1 is initial temperature.
P-15

So, − Pext (V2 − V1) = C V (T2 ′ − T1)


 RT ′ RT1 
⇒ C V (T2 ′ − T1) = − Pext  2 −
 P2 P1 
T ′ T 
⇒ C V (T2 ′ − T1) = − RPext  2 − 1 
 P2 P1 
Solving, T2 ′ is calculated.
(ii) Free expansion: In case of free expansion, Pext = 0, so,
w = 0 ⇒ ∆U = 0 ⇒ ∆T = 0 ⇒ ∆H = 0.
Example 2: One mole of an ideal gas with CP , m = 30.0 J / k, Cv , m = 20.7 J / k (both considered
constant)is expanded from 500 K and 4.00 atm to atmospheric pressure. Calculate the work
done if.
(a) The expansion is reversible and isothermal.
(b) The expansion is reversible and adiabatic.
Vf
Solution: (a) w = −nRT ln
Vi
Vf Pi 4
= = =4
Vi Pf 1

w = − (1 mol)(8. 3145 Jk −1mol −1) (500 K) ln 4


w = −576
. kJ
Tf   Pf 
(b) C p ,m ln   = R ln  
 Ti   Pi 
 Tf  R  1 atm.  8 . 3145 Jk −1mol −1  1 atm. 
ln   = ln   = ln  
 500 K  C P,M  4 atm. 30 . 0 Jk −1mol −1  4 atm.
 Tf 
ln   = − 0 . 38421
 500K 
ln T f = ln 500 − 0 . 38421 = 5 . 8304
T f = 340 . 49 K = 340 K
dU = dw = nC V,mdT
w = nCv,m(T f − Ti ) = (1 mol)(20 .7 Jk −1mol −1)(340 K − 500 K)
w = − 3 . 31 kJ
Example 3: Calculate the final temperature, the work done and the change in internal energy
when ammonia is subjected to a reversible adiabatic expansion from 1.00 L to 6.00 L. The
initial pressure is 1.00 atm and the initial temperature is 25° C.
Solution: Assume heat capacity of C V, M = C P,M − R = 35.06 Jk −1mol −1 −8.3145 Jk −1mol −1 = 26.7455
Jk −1mol −1 as first approximation. After computation of the first approximation of the final temperature, we
can compute the heat capacity at the mean temperature using
P-16

C P, M = 29.75 + 25.10 × 10– 3T – (1.55 × 105)/T2


Tf   Vf  (1 atm. )(1 L)
C V,M ln   = − R ln   n =
 Ti   Vi  (8 . 3145 Jk −1mol −1) (298 K)
 Tf  8 . 3145 Jk −1mol −1  6 . 00
ln   =− ln   n = 0 . 04089 mol
 298K  26 .7455 Jk −1mol −1  1 . 00 
ln Tf = ln 298 − 0 . 31088 ln 6 = 5 .1400
T f = 170 .7 K
| 298 − 170 .7|
T = + 170 .7 = 234 . 35 K
2
1. 55 × 105
C V,M = 29 .75 + (25 .10 × 10−3 )(234 . 4 K) − −R
(234 . 4 K)2
= 24 . 498 Jk −1mol −1
 Tf  8 . 3145 Jk −1mol −1  6 . 00
  =− ln  
 298K  24 . 498 Jk −1mol −1  1 . 00 
T f = 162 . 2 K
| 298 − 162. 2|
T = + 162 . 2 = 230 .1 K
2
1.55 × 105
. + (25 .10 × 10−3 )(230 .1 K) −
C V,M = 2975 −R
(230 .1 K)2
= 24 . 282 Jk −1mol −1
 Tf  8 . 3145 Jk −1mol −1  6 . 00
ln   =− ln  
 298K  24 . 282 Jk −1mol −1  1 . 00 
T f = 161 . 3 K
| 298 − 161 . 3 |
T = + 161 . 3 = 229 . 6 K
2
Next iteration also gives 161 .3K = 161 K
∆T = 161 K − 298 K = −137 K
w = ∆U = n C V.m ∆T = (0 . 04089 mol) (24 . 282 Jk −1mol −1)(− 137 K)
w = ∆U = −136 J

1.6 Joule−Thomson Effect


In thermodynamics, the Joule–Thomson effect or Joule–Thomson expansion describes the lowering of
temperature of a gas when it is forced through a porous plug from a high to low pressure zone under adiabatic
conditions. Adiabatic condition means the system is insulated so that no heat is exchanged between system
and surroundings. However, hydrogen and helium behave as exceptions and get heated up in the
Joule–Thomson process. It is because the inversion temperature for these two gases is lower than room
temperature. The inversion temperature for hydrogen is −80°C and for helium it is −240°C.
P-17

The inversion temperature is defined as the temperature below which a gas becomes cooler on expansion.
On the other hand nitrogen and oxygen,the two most abundant gases in air, have inversion temperatures of
348 °C and 491 °C respectively. So, these gases can be cooled at room temperature due to the
Joule–Thomson effect.

Nature of Joule Thomson Effect


Thermometers Adiabatic
It is presumed that a certain amount of gas is passed
wall
through the porous plug. So,
Change in volume on the left hand side = − V1
Work involved on the left hand side = P1V1
Change in volume on the right hand side = V2 P1, V1, T1 P2, V2, T2

Work involved on the right hand side = − P2 V2


Net work involved in the system = P1V1 − P2 V2 .
As the process is adiabatic, dq = 0, Throttling device Weightless piston

Fig 6: Nature of Joule Thomson


∆U = w
or, U 2 − U 1 = − P2 V2 + P1V1 ⇒ U 2 + P2 V2 = U 1 + P1V1
⇒ H2 = H1
So, the enthalpy of the gas which has moved across remain unchanged and it can be said that Joule
Thomson expansion is an isoenthalpic process.

1.6.1 Explanation of Joule−Thomson Effect


The cooling of the gas in Joule –Thomson effect can be explained in the following manner. When the gas
expands from a high pressure into a low pressure zone, certain amount of work is done by the gas. This is the
work of expansion. The work is done in overcoming the Vander Waals forces of attraction operational
between real gas molecules. This causes a decrease in the internal energy of gas molecules which is
manifested in the form of decrease of temperature.

For ideal gases, the temperature drop of the system is zero in the case of Joule–Thomson effect. It means no
cooling is observed for ideal gases. It is due to the fact that there is no intermolecular force of attraction
between ideal gas molecules, hence no work is to be done in case of ideal gases. So, no drop in internal
energy takes place, subsequently no decrease in temperature is recorded.

1.6.2 Joule−Thomson Coefficient µ JT


The rate of change of temperature T with respect to pressure P in a Joule–Thomson process is
the Joule–Thomson coefficient µ JT . The value of µ JT is typically expressed in °C/ bar(SI units is K/Pa) and
depends on the type of gas and on the temperature and pressure of the gas before expansion. This coefficient
can be expressed in terms of the volume of the gas V , its heat capacity at constant pressure C P , and
its coefficient of thermal expansion as
 ∂T  V
µ JT =   = (αT − 1); α is co-efficient of thermal expansion.
 ∂P  H C P
Derivation of Joule Thomson Co-efficient
As H = f (T , P), so
P-18

 ∂H   ∂H 
dH =   dT +   dP
 ∂T  P  ∂P  T
As enthalpy (H ) is constant in Joule Thomson effect and dividing by ∂P,
 ∂H   ∂T   ∂H 
0=     +  
 ∂T  P  ∂P  P  ∂P  T
  ∂H  
T   ∂P  

  T  = − 1  ∂H  ...(1)
⇒   =−  
 ∂P  H   ∂H   C P  ∂P  T
 
  ∂T  P 

Since, H is a state function,


 ∂H   ∂V 
  = −T   + V ...(2)
 ∂P  T  ∂T  P

 ∂T  1   ∂V  
⇒   =− −T   + V
 ∂P  H C P   ∂T  P 
 ∂V 
T   −V
 ∂T  P  T  ∂V 
V 
⇒ µ JT =
CP
=  V  ∂T  − 1
CP
 P 
V 1  ∂V 
µ JT = (αT − 1) Here α = ×  
CP V  ∂T  P
µ JT for an ideal gas:
 ∂V  nR
For an ideal gas   = . So,
 ∂T  P P
 nR 
T   −V
 P
µ JT = =0
CP
As µ JT = 0 for an ideal gas, it means for any ideal gas, there is no impact on temperature as the gas is
transferred from a high pressure to low pressure zone.
All real gases have an inversion point at which the value of µ JT changes sign. The temperature of this point is
known as inversion temperature. Inversion temperature depends upon the pressure of the gas before
expansion. The inversion temperature in thermodynamics is the critical temperature below which a real or
non-ideal gas (as such all the gases in reality are non-ideal gases) that is expanding will experience a
temperature decrease under adiabatic conditions. Above the inversion temperature, the gas will experience a
temperature increase. This temperature change is exploited in the liquefaction of gases.
In a gas during expansion the pressure decreases, so the sign of ∂P is negative by definition. With that in
mind, the following table explains when the Joule–Thomson effect cools or warms a real gas. It is important to
keep in mind. that ∂P is always negative because the gas is expanding.

If the Sgas temperature is then µ JT is since ∂P is thus ∂Tmust be so the gas


below the inversion temperature positive always negative negative cools
above the inversion temperature negative always negative positive warms
P-19

For an ideal gas, µ JT is always equal to zero. It means that ideal gases neither warm nor cool upon being
expanded under adiabatic conditions.

1.6.3 Inversion Temperature


The temperature of the gas at which µ JT = 0 is known as inversion temperature.
V Ti  ∂V  
As µ JT = 0 ⇒    − 1 = 0

C P  V ∂T P 
1 1
⇒ Ti = = ; here αi is co-efficient of thermal expansion at Ti .
 1   ∂V  αi
   
 V   ∂T  P

1.7 Thermochemistry
Thermochemistry is the branch of chemistry which deals with the thermal or heat changes caused by
chemical reactions.

1.7.1 Different Terms in Thermochemistry


1. Sign Conventions: If heat is absorbed by the system (q>0), then the reaction is said to be
endothermic and ∆E or ∆H value is a positive sign. If heat is evolved (q<0) the reaction is said to be
exothermic, and ∆E or ∆H value is assigned negative sign.
2. Exothermic and Endothermic Reactions :
(i) Endothermic reactions: Reactions which are accompanied by the absorption of heat are
termed as endothermic reactions. Heat absorbed during the reaction increases the energy
content of the products and hence the enthalpy of the products becomes greater than that of
the reactants from which the products are formed. As it is known, ∆H = ΣH (Products) − ΣH
(reactants)
And, Σ H (products) > Σ H (reactants) ⇒ ∆H > 0.
(ii) Exothermic reactions: Reactions which are accompanied by the evolution of heat are termed
as exothermic reactions. Heat evolved during the reactions is transferred to the surroundings,
Therefore, the enthalpy of the products is less than enthalpy of the reactants from which the
products are generated.
Here ∆ H < 0, Σ H (products) < Σ H (reactants)
3. Heat of Reaction : Heat of reaction is defined as the amount of heat absorbed or evolved at a given
temperature when the reactants have combined to form the products as represented by a balanced
chemical equation. If the heat of reaction is denoted by q, then the numerical value of q depends on the
manner in which the reaction is performed. ∆U is referred as heat of reaction at constant volume and it
is measured in a bomb calorimeter. Similarly, ∆H is referred as heat of reaction at constant pressure
and it is measured in an open mouthed calorimeter. So if measurements are done at.
(i) Constant volume: w = 0 and qv = ∆E or ∆U;
(ii) Constant pressure: w = − p∆V and qp = ∆E + P∆V;

4. Reactions Performed at Constant Volume and at Constant Pressure : A chemical change can
take place either at constant volume or at constant pressure. The heat change in the former case is
known as heat of reaction at constant volume and is commonly denoted by ∆H V or ∆E or ∆U. The
P-20

heat change in the latter case is known as heat of reaction at constant pressure and is commonly
denoted by ∆H P or ∆H. However, most of the chemical reactions and physical transformations take
place at constant pressure of one atmosphere. Hence, the quantity ∆H as a measure of heat of reaction
is of greater practical importance than ∆E or ∆U.
5. Relation between ∆H and ∆E or ∆U : The quantities ∆H and ∆E or ∆U are related to each other by
the well known thermodynamic equation.
∆H = ∆E + P∆V …(1)
or, by replacing ∆H and ∆E by ∆H P and ∆H V respectively, it can be said that,
∆H P = ∆H V + P∆V …(2)
Assuming gas laws to be valid, the general gas equation can be used, which states,
PV = nRT
Since P and T remain constant for a reaction carried out at constant pressure and temperature.
So, P∆V = ∆nRT
Here ∆n = n 2 −n 1 = (sum of moles of gaseous products)−(sum of moles of gaseous reactants.)
Therefore, equation (2) can now be expressed as
∆H P = ∆H V + ∆nRT
In case of reactions involving solids and liquids, difference between the two values is negligible. But in
case of gaseous reactions the difference between ∆H P and ∆H V is by no means negligible.
6. Factors Influencing the Heat of Reaction : There are multiple factors which influence the heat
exchange during a chemical reaction. Some of these determinants are discussed here at length.
(i) Physical state of substances: Heat of reaction changes with the change of physical state of
the substances involved in the reaction.
1
H2 (g ) + O 2 (g ) → H2O(g ), ∆ H = − 57 kcal
2
1
H2 (g ) + O 2 (g ) → H2O(l ), ∆ H = − 68 . 3 kcal
2
Apparently, there is a difference in the value of heat of reaction, if water is obtained in gaseous
or liquid state. The higher value in second case is due to the fact that some heat is evolved
when steam is converted into water.
(ii) Allotropic forms of the elements: Heat energy is involved when one allotropic form of an
element is converted into another. So, the value of ∆H depends upon the allotropic form used
in the reaction.
C(diamond) + O 2 (g) → CO 2 (g) ∆ H = −97.6 kcal
C(graphite) + O 2 (g) → CO 2 (g) ∆ H = −94.3 kcal
The difference between the two values equals to the heat absorbed when 12 g of diamond is
converted into 12 g of graphite.
(iii) Temperature: Heat of reaction depends upon the temperature at which the reaction is carried
out. This is due to the variation in the heat capacity of the system with temperature. Kirchhoff
has related heat of reaction (∆H or ∆E) with temperature as
∆ H 2 − ∆ H 1 = ∆C P (T2 − T1)
The same relation can be expressed in terms of internal energy.
∆E 2 − ∆E 1 = ∆C V (T2 − T1)
P-21

∆Cp is the difference in the heat capacities (at constant pressure) of products and reactants.
Similarly, ∆CV is the difference in the heat capacities (at constant volume) of products and
reactants.
(iv) The condition of constant volume or constant pressure: The heat of reaction depends on
the fact that whether the reaction is carried out at constant volume (∆E) or at constant
pressure (∆H). The two are, however related as follows:
∆H = ∆E + ∆nRT
∆n = moles of gaseous product − moles of gaseous reactant

1.8 Different Types of Heats of Reaction


There is a variety of reaction taking place in chemistry. Each of these types are associated with its own heat of
reaction. Some of the important ones are discussed below;

1.8.1 Heat of formation


The change in enthalpy that takes place when one mole of the compound is formed from its elements.
1
H2 (g ) + O 2 (g ) → H2O(l ), ∆ H = − 68 . 38 kcal / mol
2
C (s) + O 2 (g ) → CO 2 (g ), ∆ H = − 94 .13 kcal / mol
Standard heat of formation: It is defined as the change in enthalpy that takes place when 1 mole of a
compound is formed from its elements. All the elements are taken in their standard states is at 1 atm pressure
and 25°C.
Standard states: In the computation of heat of reactions, it is a convention to assume that the heat of
formation of elements in their standard states is zero. The standard state is taken as 1 atm pressure and a
constant temperature. Standard states for various forms of matter are summarized below:
Table 1: Standard states for various forms of matter

State Standard State


Gas Ideal gas at 1 atm and the given temperature.
Liquid Pure liquid at 1 atm and the given temperature.
Solid Stable crystalline form at 1 atm and given temperature.

1.8.2 Heat of Combustion


The change in enthalpy when one mole of the substance is completely burnt in excess of air is known as heat
or enthalpy of combustion. Some of the examples are
1
H2 (g ) + O 2 (g ) → H2O(l ), ∆ H = − 68 . 38 kcal / mol
2
C (s) + O 2 (g ) → CO 2 (g ), ∆ H = − 94 .13 kcal / mol

1.8.3 Heat of Neutralisation


It is defined as the quantity of heat exchanged when one gram equivalent of an acid is completely neutralised
by one gram equivalent of a base in dilute solution. Examples are
P-22

HNO 3 (aq) + NaOH(aq) → NaNO 3 (aq) + H 2O (l ); ∆ H = −13 . 69 kcal / mol or − 57 . 2 kJ


HCl(aq) + NaOH(aq) → NaCl(aq) + H 2O (l ); ∆ H = −13 . 69 kcal
The heat of neutralisation of a strong acid and a strong base is a fixed value, −13.69 kcal. On the basis of
electrolytic dissociation theory, it has been clearly explained that this heat of neutralisation is merely the heat
of formation of water from H + of an acid and OH − of a base.
H + (aq) + OH − (aq) → H 2O (l ); ∆ H = −13 . 69 kcal

The heat of neutralization in case of a weak acid or a weak base is less than 13.69 kcal because some energy is
used up in dissociating the weak electrolyte – weak acid or weak base. The difference in the values gives the
dissociation energy of the weak acid or a weak base. The heat of neutralisation of HF is more than 13.69 Kcal

due to extremely high enthalpy of hydration of F O ion.

1.8.4 Heat of ionisation


As stated earlier the heat of neutralisation of weak acid or weak base must be less than 57 . 2 KJ or 13.69
Kcal. This energy is used in getting the weak acid or base dissociated. It is known as heat of ionisation; such
that,
∆ H i = (∆H H )weak acid / base − (∆H H )strong acid / base

1.8.5 Heat of Solution


It is defined as the quantity of heat evolved or absorbed when one mole of the solute is dissolved completely
in a large excess of water, so that further dilution of solution does not produce any heat change. Examples are
KCl (s) + H 2O → KCl (aq); ∆ H = + 44.0 kcal
HCl (g ) + H 2O → HCl (aq); ∆ H = − 39 . 3 kcal

1.8.6 Heat of Dilution


It is defined as the quantity of heat evolved or absorbed when solution containing one mole of a solute is
diluted from one concentration to another. Examples are
KCl (s) + 20H 2O → KCl (20H 2O ); ∆ H 1 = + 3 . 8 kcal
KCl (s) + 200 H 2O → KCl (200 H 2O ); ∆ H 2 = + 4.44 kcal
∴ Heat of dilution = ∆H 2 − ∆H 1
= 4.44 − 3.80 = 0.64 kcal

1.9 Hess's Law


Hess’s Law of Constant Heat Summation, also known as Hess’s law, is a relationship in physical chemistry
named after Germain Hess. The law deals with the fact that the total enthalpy change during the course of a
reaction is the same whether the reaction is completed in one step or in several steps. Hess’s law is now
understood as an expression of the principle of conservation of energy. Hess’s law can be used to determine
the overall energy required for a chemical reaction, when it can be divided into synthetic steps that are
individually easier to characterize. The law is based upon the fact that enthalpy change is a state function. So,
its value remains path independent, it only depends on the initial and final states. On this basis Hess
enunciated a law which states that, the enthalpy change in a chemical or physical process is same whether the
process is carried out in a single step or in more than one steps. Diagrammatically,
P-23

As suggested, A can be directly converted into B or it can be converted into B via C. However, the enthalpy
change in both the cases would be the same so,

Path I
A B
∆H + 1 O2 + 1 O2
2 2
PathII ∆H1 CO ∆H2
∆H1 C CO2
∆H2
+O2
C ∆H
Fig. 8: Hess's law in formation of CO2
Fig:
Fig.
7 Hess
7: Hess's
law CO
law,
2,∆H=∆H2+∆H
∆H=∆H2+∆H1 1

∆H = ∆H 1 + ∆H 2
It can also be discussed with the help of following examples, like formation of CO 2
Another example of formation of NH3 is considered.
route 1

(a) ∆Hf
N2(g)+3H2(g) 2NH3(g)

∆H1 ∆H2
route 2
2N(g)+6H(g)
From Hess's law ∆Hf=∆H 1+∆H2
Fig. 9: Hess's law for NH3

1.9.1 Application of Hess's Law


There are a number of practical applications of Hess’s Law like calculating heats of very slow reactions,
determination of lattice energies of ionic substances by constructing Born-Haber cycles if the electron affinity
to form the anion is known, finding heat changes in phase transitions and allotropic transitions or calculating
electron gain enthalpies using a Born-Haber cycle with a theoretical lattice energy data. Some of these are
discussed below in detail:
1. Determination of enthalpy of formation: With the help of Hess’s law, one can calculate the
enthalpy changes for the reactions which are experimentally not possible. For example, there are large
number of compounds such as C 6 H 6 , C 2 H 6 etc. whose direct synthesis from their constituent
elements is not possible. The enthalpy of formation of such compounds can be computed from Hess’s
Law.
2. Calculation of standard enthalpies of reactions: On the basis of standard enthalpies of
formation of reactants and products, the standard enthalpy of a reaction can be calculated by using
Hess’s law.

Elements in their ∆H1=Σ∆H°f(R) Reactants in their


standard states standard states

∆Η° = ?
∆H2=Σ∆H°f(P) Products in their
standard states

From Hess’s law,


P-24

∆H 2 = ∆H °+ ∆H 1
∆H ° = ∆H 2 − ∆H 1
Hence, ∆ H° = [Sum of standard enthalpies of formation of products] − [Sum of standard enthalpies of
formation of reactants].

For the reaction: aA + bB → cC + dD.


∆ H ° = [c ∆ H °f (C) + d ∆ H °f (D)] − [a ∆ H °f (A) + b ∆ H °f (B)]
Similarly, ∆H° = [Sum of standard enthalpies of combustion of reactants] − [Sum of standard enthalpies of
combustion of products].
Example 4: The standard enthalpies of formation at 298 K for CCl 4 (g), H 2O(g), CO 2(g) and
HCl(g) are −106.7, −241.8, −393.7 and −92.5 kJ mol −1, respectively. Calculate ∆H 298
°
K for the
reaction
CCl 4 ( g) + 2H 2O ( g) → CO 2( g) + 4HCl ( g)
Solution: The enthalpy change of the given reaction will be given as
∆ H = ∆ H f (CO 2 , g ) + 4 ∆ H f (HCl, g ) − ∆ H f (CCl 4 , g ) − 2∆ H f (H 2O , g )
= (− 393.7− 4 × 92.5 + 106.7 + 2 × 241.8) kJ mol −1

= −173.4 kJ mol −1
3. Born−Haber cycle: This cycle devised by Born and Haber in 1919 relates the lattice energy of a
crystal to other thermochemical data. The elements in ∆H°f
Na(s) +1/2Cl2(g) NaCl(s)
their standard state are first converted to gaseous atoms,
then to ions and finally packed into the crystal lattice. The ∆Hsub ∆Hdiss
Born Haber − cycle is shown below by taking an example
of NaCl. Na(g) Cl(g) U
According to Hess's law, "The overall energy change in a process
I.E. E.A.
depends only on the energy of the initial and final states and not
on the route taken" Hence -
Na+(g) + Cl (g)
∆ H ° f = ∆ H sub + I . E. + ∆ H diss + E. A. + U
Fig. 10: Born Haber cycle for NaCl
Here ∆H sub is enthalpy of sublimation of sodium I.E. is
ionisation energy of Na. ∆H diss . is enthalpy of dissociation of chlorine gas and E.A. is electron affinity or
electron gain enthalpy of Cl. U is the lattice enthalpy of NaCl. Hence from above, one can calculate any
unknown value such as lattice energy, ionization energy, electron affinity or electron gain enthalpy etc. by
putting other various known values.

Example 5: Use the Born-Haber cycle and the following data to calculate the electron affinity of
chlorine.
∆H f (RbCl) = − 102.9 kcal/mol
Ionization energy of Rb = 95 kcal/mol
∆H sub (Rb) = +20.5 kcal/mol
B.E. (Cl 2) = 54 kcal/mol
Lattic energy of RbCl = − 166 kcal/mol
P-25

Solution: Rb(s)+1/2Cl2(g) –102.9 RbCl(s)


20.5 27 –166
∆H=+E.A.
Cl(g) Cl–(g)
95
Rb(g) Rb+(g)

E.A. = −102.9 − (20.5 + 27 + 95 − 166) = −79 kcal/mol

Example 6: Calculate the enthalpy change for the following reaction:


XeF4 ( g) → Xe + ( g) + F − ( g) + F2( g) + F( g)
The average Xe − F bond energy is 34 kcal/mol, first I.E. of Xe is 279 kcal/mol, electron affinity
of F is 85 kcal/mol and bond dissociation energy of F is 38 kcal/mol.
Solution:
(a) XeF4 (g ) → Xe (g ) + 4F(g ) ∆ H 1 = 136 kcal
(b) + −
Xe (g ) → Xe (g ) + e ∆ H 2 = 279 kcal / mol
(c) F(g ) + e − → F − (g ) ∆ H 3 = −85 kcal / mol
(d) F2 (g ) → 2F(g ) ∆ H 4 = 38 kcal / mol
The required reaction is
XeF4 (g ) → Xe + (g ) + F − (g ) + F2 (g ) + F(g ), ∆H5 = ?
The enthalpy change ∆ H 5 can be easily obtained as
eq.(i) + eq (ii) + eq. (iii) − eq. (iv)
∆ H 5 = ∆ H 1 + ∆ H 2 ∆ H 3 − ∆H 4 = 292 kcal / mol.

1.9.2 Bond Dissociation Energy and Bond Energy or Enthalpy


The energy required to break bond of between two atoms in a gaseous molecule is known as bond
dissociation energy. On the other side, the average value of different bond dissociation enthalpies is known
as bond enthalpy. For a diatomic molecule, the bond dissociation energy is same as bond energy.
However, in a polyatomic molecule, the bond dissociation energy is not the same for successive bonds
desprite the fact that bonds are of the same type. Therefore, in such case the bond energy is not equal to the
bond dissociation energy. An example is considered where H 2 O is undergoing stepwise dissociation:
H − OH(g ) → H (g ) + OH (g ); ∆ H 1 = 498 kJmol −1

O − H (g ) → H(g ) + O (g ); ∆ H 2 = 430 kJ mol −1


________________________________________________________
H − OH (g ) → 2H (g ) + O (g ); ∆ H = ∆ H 1 + ∆ H 2 = 928 kJmol −1
So, bond energy in the above case is
498 + 430
∆HO− H = = 464 kJmol −1
2
498 and 430KJ/mol represent successive bond dissociation energies. Thus for polyatomic molecule having
various types of same bonds, one needs to define bond energy. And it is defined as the average amount of
energy required to break bonds of one type in a gaseous molecule.
P-26

1.9.3 Bond Enthalpy and Heat of Reaction


One can use the bond energy data to estimate heats of reaction. To illustrate this, let us find ∆H for the
reaction.
CH 4 (g ) + Cl 2 (g ) → CH 3Cl(g ) + HCl (g )
One can imagine that the reaction takes place in steps involving the breaking and forming of bonds. Starting
with the reactants, we suppose that one C−H bond and the Cl−Cl bond break. The enthalpy change for this is
BE(C−H) + BE (Cl − Cl).
H H

H—C—H+Cl—Cl H—C—H+Cl—Cl

H H
Now the fragments are reassembled to give the products
H H

H—C—H+Cl—Cl H—C—Cl+H—Cl

H H
In this case, C−Cl and H− Cl bonds are formed, and the enthalpy change equals the negative of the bond
energies −BE(C − Cl) − BE(H − Cl). In general, the enthalpy of reaction is equal to the sum of the bond
energies for bonds broken minus the sum of the bond energies for bonds formed. So, ∆H = BE(C− H) + BE
(Cl − Cl) − BE(C − Cl)− BE(H− Cl). Hence, Enthalpy of reaction, ∆H = [Sum of bond energies of reactants] −
[Sum of bond energies of products].

1.10 Kirchhoff 's Law


It is known that, at constant pressure, the heat capacity is equal to change in enthalpy divided by the change
in temperature.
 ∂H 
Cp =  
 ∂T  p

Therefore, if the heat capacities do not vary with temperature then the change in enthalpy is a function of the
difference in temperature and heat capacities. On the basis of this relation Kirchhoff’s stated that
T2
∴ ∆H (T2 ) = ∆H (T1) +
∫T
1
∆C p dT

Kirchhoff’s Law describes the variation of enthalpy of a reaction with temperature changes. In general,
enthalpy of any substance increases with temperature, which means both the products and the reactants’
enthalpies increase. The overall enthalpy of the reaction will change if the increase in the enthalpy of products
and reactants is different.
∆H Tf = ∆H Ti + ∆C p (T f − Ti )
Here ∆C p is the difference in heat capacity, Cp f − Cpi . ∆H Tf , ∆H Ti are the enthalpy changes at the
respective temperatures. The amount by which that the enthalpy changes is proportional to the product of
temperature change and change in heat capacities of products and reactants. A weighted sum is used to
calculate the change in heat capacity to incorporate the ratio of the molecules involved since all molecules
P-27

have different heat capacities at different states. The Kirchh off Law can only be applied to small temperature
changes, (<100 K) because over a larger temperature change, even the heat capacity is not constant.

1.10.1 Variation of Enthalpy of Reaction with Temperature


The enthalpy of a chemical reaction changes with temperature for the reaction
2A + bB → cC + dD
The enthalpy change is given by, ∆H = ΣH P − ΣH R .
So, ∆ H = [c(H C ) + d(H D ) ] − [a (H A ) + b (H B )] ...(1)
Differentiating with respect to temperature,
 ∂∆H    ∂H c   ∂H D     ∂H A   ∂H B  
  = c  +d  – a  + b 
 ∂T  P   ∂T  P  ∂T  P    ∂T  P  ∂T  P 

= [c (C P )C + d (C P )D ] − [a (C P ) A + b (C P )B ] = ∆C P ...(2)
Eq. (2) is kirchhoff equation for a chemical reaction.
 ∂ ∆H 
As,   = ∆ C P ⇒ d ∆H = ∆ C P dT ...(3)
 ∂T  P
 ∂ ∆H 
Similarly,   = ∆ C V ⇒ d ∆U = ∆ C V dT ...(4)
 ∂T  V
Case 1. Temperature range is small.
For small temperature range, heat capacity is independent if temperature, so,
T2 T2
∫T 1
d∆H =
∫T
1
∆ C p dT ⇒ ∆ H 2 − ∆ H 1 = ∆ C p (T2 − T1)

Similarly, ∆ U 2 − ∆ U 1 = ∆ C V (T2 − T1).


Case 2. Temperature range is large.
For large temperature ranges, C p is a temperature dependent parametre. If C p is expressed as,
C p = α + βT + γ T 2 (α, β, γ are constant)
So, ∆ C p = [(c α C + d α D ) − (a α A + b α B )] + ..... ...(5)
= [(c β C + d β D ) − (a β A + b β B )] T + .....
T2 T2
(∆ α + ∆ β T + ∆ γ T 2 ) dT
So,
∫T1
d (∆ H ) =
∫T
1

1 1 3
⇒ ∆ H 2 − ∆ H 1 = ∆ α (T2 − T1) + (∆ β) (T22 − T12 ) + (∆ γ ) (T23 − T1 ) ...(6)
2 3
Eq. (6) is integrated Kirchhoff equation for a reaction.

Example 7: Magnetite Fe 3O 4 is reduced to iron by hydrogen or carbon monoxide as


(a) Fe 3O 4 ( s) + 4H 2( g) → 3Fe ( s) + 4H 2O ( g)
(b) Fe 3O 4 ( s) + 4CO ( g) → 3Fe ( s) + 4CO2 ( g)
Calculate the enthalpy change involved during the reduction to get 1 g of iron. Data given are
Substance: Fe 3O 4 ( s) CO( g) CO 2( g) H 2O ( g)
∆H f ( 298 K ) 1117.1 110.5 393.5 241.8 [kJ/mol]
P-28

Solution: (a) Fe 3O 4 (s) + 4H 2 (g ) → 3Fe(s) + 4H 2O (g )


∆ H 1 = 4 ∆H f (H 2O , g ) − ∆ H f (Fe 3 , s)
= 4 × 241 . 8 − 1117 .1 = − 149 . 9
149 . 9
Enthalpy change during the reduction to get 1 g of Iron = = 0 . 89226 kJ / g
3 × 56
= 892.26 J/g
(b) Fe 3O 4 (s) + 4CO (g ) → 3Fe(s) + 4CO 2 (g )
∆ H 2 = 4 ∆H f (CO 2 , g ) − 4 ∆ H f (CO , g ) − ∆ H f (Fe 3O 4 , s)
= 4 × 393 . 5 − 4 × 110 . 5 − 1117 .1 = 14 . 9 kJ
Enthalpy change during the reduction to get 1 g of iron
14 . 9
= = 0 . 08869 kJ / g = 88 . 69 J / g
3 × 56

Example 8: In order to get maximum calorific output, a burner should have an optimum fuel to
oxygen ratio which corresponds to 3 times as much oxygen as is required theoretically for
complete combustion of the fuel A burner which has been adjusted for methane as fuel (with x
litre/hour of CH 4 and 6x litre/hour of O 2) is to be readjusted for butane, C 4H10 . In order to get
the same calorific output what should be the rate of supply of butane and oxygen? Assume that
losses due to incomplete combustion etc. are the same for both fuels and that the gases behave
ideally. Heats of combustion:
CH 4 = 809 kJ / mol and C 4H10 = 2878 kJ / mol
Solution: Let the volume occupied by 1 mole gas be V lit.
CH 4 (g ) + 2O 2 (g ) → CO 2 (g ) + 2H 2O (l )
x
Mol. of CH4 burnt =
V
x
Heat generated from methane = × 809
V
13
C 4H 10 (g ) + O 2 (g ) → 4CO 2 (g ) + 5H 2O (l )
2
x
Heat generated from butane = × 809
V
V x
∴ rate of supply of butane = × × 809 = (0. 28x ) lit hr −1
2878 V
 13 × 3 −1
∴ Corresponding supply of O 2 = . 28 ×   = 5.48x lit
 2 

Example 9: When maltose, C12H 22O11( s) burns in a calorimetric bomb at 298 K yielding water
and carbon dioxide, the heat of combustion is-1350 kcal/mol. Calculate the heat of combustion
of maltose at constant pressure if the water vapour resulting from the combustion is (a)
condensed and (b) not condensed.
Solution: The combustion reaction of maltose can be shown for (a) and (b) as follows.
(a) C 12H 22O 11(s) + 12 O 2 (g ) → 12 CO 2 (g ) + 11 H2O ( › )
(b) C 12H 22O 11(s) + 12 O 2 (g ) → 12 CO 2 (g ) + 11 H 2O ( g )
P-29

Case (a): When water is condensed.


It is given that,
∆ H V = −1350 kcal / mol
∆n = 0; R = 1 . 987 × 10−3 kcal / mol −1 K −1
T = 298 K
On inserting these values in the expression
∆ H P = ∆ H V + ∆ nRT
∆ H P = −1350 + 0 × RT = − 1350 kcal / mol
Case (b): When water is not condensed.
It is given that
∆ H V = − 1350 kcal / mol
∆n = (12 + 11 − 12) = 11
R = 1 . 987 × 10−3 kcal / mol
T = 298 K
On inserting these values in the expression
∆ H P = ∆ H V + ∆ nRT
∆ H P = [−1350 + (11 × 1 . 987 × 10−3 × 298)] kcal
∆ H P = −1343 kcal / mol
Example 10: The ∆H °f for CO 2(g), CO(g) and H 2O(g) are –393.5, –110.5 and –241.8 kJ mol −1
respectively. The standard enthalpy change (in kJ) for the reaction CO 2(g) + H2 (g)→ CO(g) +
H 2O(g) is:
(a) 524.1 (b) 41.2 (c) − 262.5 (d) − 41.2
Solution: ∆H °reaction = ∆ H °f (CO ) + ∆ H °f (H 2O , g ) − ∆ H °f (CO 2 , g )
= −110. 5 − 241. 8 + 393 . 5 = 41 . 2
∴ Ans: (b)
Example 11: The amount of heat required to raise the temperature of 1 mole diatomic gas by 1°C
at constant pressure is 60 cal. The amount of heat which goes as internal energy of the gas is
nearly.
(a) 60Cal (b) 30 Cal (c) 42.6 Cal (d) 49.8 Cal
Solution: For a diatomic gas C V = 7 / 2R and C V = 5 / 2 R
5
Only . of energy supplied increases the internal energy of the gas. The rest is used to do work against
= 071
7
external pressure
0.71 × 60 = 42.6 Cal
∴ Ans: (c)
Example 12: When a polyatomic gas undergoes an adiabatic process, its temperature and volume
are related by the equation TV n= constant the value of n will be:
(a) 1.33 (b) 0.33 (c) 2.33 (d) 1
Solution: For adiabatic process,
P-30

TV γ −1 = constant
For polyatomic gas
4
γ = = 1.33
3
n = 1.33 –1 = 0.33
∴ Ans: (b)
Example 13: For the reaction F2( g) + 2HCl( g) l 2HF( g) + Cl 2( g)
∆H° at 298 K is − 84.4 kcal, ∆H° f (HF) = − 64.2 kcal/mole.
∆ H ° f for the HCl(g) per gram is:
(a) − 0.603 kcal (b) − 0.603 cal (c) 0.0603 kcal (d) 6.03 kcal
Solution: F2 (g ) + 2HCl(g ) l 2HF(g ) + Cl 2 (g )
∆ H = − 84 . 4 = − 128 . 4 − 2∆ H f (HCl)
∆ H f (HCl) = −22 kcal / mol
∆ H f (HCl) = − 0 . 603 kcal / mole.
∴ Ans: (b)
Example 14: Which is intensive property?
(a) Boiling point (b) Molarity (c) Freezing point (d) All
Solution: An extensive property of a system is that property which depends upon the amount of the
substance or substances present in the system e.g. mass, volume, energy, entropy, Gibbs potential, enthalpy
whereas intensive properties are those properties of a system, which are characteristic of the substance or
substances present, and are independent of its mass. Temperature, pressure, density, heat capacity, boiling
point etc. are intensive property.
∴ Ans: (d)
Example 15: Heat of solution of BaCl 2 . 2H 2O = 200 kJ mol −1
Heat of hydration of BaCl2 = − 150 kJ mol −1
Hence heat of solution of BaCl2 is
(a) 350 kJ (b) 50 kJ (c) −350 kJ (d) None.
Solution: We have
BaCl2 . 2H 2O + aq → Ba 2 + (aq) + 2Cl − (aq) + 2H 2O ; ∆ H = 200 kJ mol −1
BaCl 2 + 2H 2O → BaCl 2 . 2H 2O ; ∆ H = −150 kJ mol −1
On adding both equation, we get
BaCl 2 + aq → Ba 2 + (aq) + 2Cl −(aq); ∆ H = 50 kJ mol −1
This is the reaction for solution of BaCl2 . Therefore, heat of this reactions is in fact the heat of solution of
BaCl2 .
∴ Ans: (b)
Example 16: Heat of neutralization of oxalic acid is −53.35 kJ mol −1 using NaOH. Hence ∆H of
H 2C 2O 4 l C2O24 − + 2H + is:
(a) 5.88 kJ (b) −5.88 kJ (c) −13.7 kcal (d) 7.9 kJ
Solution: By the definition of heat of neutralization, we have
P-31

1/ 2 H 2C 2O 4 + NaOH → 1/ 2 Na 2C 2O 4 + H 2O ; ∆ H = −53 . 35 kJ
or 1/ 2 H 2C 2O 4 + OH − → 1/ 2 C 2O 24 − + H 2O ; ∆ H = −53 . 35 kJ ...(1)
H + + OH − → H 2O ; ∆ H = − 57 . 3 kJ ...(2)
Subtracting equation (2) from (1), we get
1 1
H2C 2O 4 → C 2O 24 − + H + ; ∆ H = −3.95
2 2
∴ H2C 2O 4 → C 2O 24 − + 2H + ; ∆ H = 2 × 3.95 = 7.9 kJ
∴ Ans: (d)
Example 17: At a particular temperature H + ( aq) + OH − ( aq) → H 2O ( l); ∆ H = −57. 1 kJ The
approximate heat evolved when 400 ml of 0.2M H2SO4 is mixed with 600 ml of 0.1 M KOH
solution will be
(a) 3.426 kJ (b) 13.7 kJ (c) 5.2 kJ (d) 55 kJ
0. 2 × 2 × 400
Solution: Number of equivalent of H 2SO 4 taken = = 0.16
1000
600 × 01 .
Number of equivalent of KOH added = = 0.06
1000
Number of equivalents of acid and bases which neutralized each other = 0.06
∴ Heat evolved = 0.06 × 57.1 kJ = 3.426 kJ
∴ Ans: (a)
Example 18: Standard molar enthalpy of formation of CO 2 is equal to
(a) Zero
(b) The standard molar enthalpy of combustion of gaseous carbon
(c) The sum of standard molar enthalpies of formation of CO and O 2
(d) The standard molar enthalpy of combustion of carbon (graphite).
Solution: C (s) + O 2 → CO 2 (g ) ; ∆H ° = ∆H °f
Standard molar enthalpy of formation of a compound is the enthalpy change associated with formation of
one mole of a compound from constituent elements on their pure state under standard condition.
∴ Ans: (d)
Example 19: 4g calcium is dissolved in hydrochloric acid at 27°C in an open vessel at the
atmosphere pressure 0.821 atm. The work done is
(a) 0 (b) −24.94 J (c) −2.463 J (d) 0.0821 J
Solution: We know when a reaction is carried out in a closed vessel of fixed volume, change in volume will be
zero. Hence the system has not done any work because
W = P∆V = P × 0 = 0
But here the system increases its volume.
Ca (s) + 2HCl(aq) → CaCl 2 (aq) + H 2 (g )
From stoichiometry we know the number of moles of H 2 produced
4 1
= mole = mole
40 10
P-32

nRT
∴ Volume of hydrogen gas produced at 27°C =
P
1 × 0 . 0821 × 300
= = 3 litre
10 × 0 . 821
∴ Change in volume during the reaction ∆V = 3 – 0 = 3 litre
∴ Work done by the system
= − P × ∆V
= − 0.821× 3 = − 2.463 J
∴ Ans: (c)

1. When 2 moles of C 2H 6 (g ) are completely burnt, 3129 kJ of heat is liberated. Calculate the heat of
formation of C 2H 6 (g ). ∆ H f for CO 2 (g ) and H 2O (l ) are –395 and –286 kJ mol −1 respectively.
2. The standard enthalpy of combustion at 25° C of H 2 , C 6H 10 and cyclohexane (C 6H 12 ) are –241,
–3800, –3920 kJ mol −1 respectively. Calculate heat of hydrogenation of cyclohexene (C 6H 10 ).
3. The heat liberated on complete combustion of 7.8 g benzene is 327 kJ. This heat has been measured at
constant volume and at 27°C. Calculate heat of combustion of benzene at constant pressure at 27°C.
(R = 8.3 J mol −1 K −1).
4. Calculate ∆H° for 2Al + Fe 2O 3 → 2Fe + Al2O 3 given that standard enthalpy of formation of Fe 2O 3
and Al2O 3 are –196.5 and –399.1 kcal.
5. The specific heat at constant volume for a gas 0.075 cal/g and at constant pressure is 0.125
cal/g.Calculate
(i) The molecular weight of gas (ii) Atomicity of gas
(iii) Number of atoms of gas in its one mole.
6. The bond dissociation energy of gaseous H 2 , Cl 2 and HCl are 104, 58 and 103 kcal mol −1
respectively. Calculate the enthalpy of formation for HCl gas.
7. Estimate the average S–F bond energy in SF 6 . The standard heat of formation values of SF 6 (g), S(g)
and F2 (g) are –1100, 275 and 80 kJ mol −1 respectively.
8. At 300 K, the standard enthalpies of formation of C 6H5COOH(s), CO 2 (g ) and H 2O (l ). are –408, –393
and –286 kJ mol −1 respectively. Calculate the heat of combustion of benzoic acid at (i) constant
pressure and (ii) constant volume.
9. A sample of argon gas at 1 atm pressure and 27°C expands reversibly and adiabatically from 1.25
dm 3 to 2.50 dm 3 . Calculate the enthalpy change in this process. Cv,m for argon is 12.48 Jk −1 mol −1.
10. Calculate the bond enthalpies of C – H, C – C and C = C bonds from the following data:
∆ H 0f (CH 4 , g ) = −74 . 9 kJ mol −1
∆ H 0f (C 2H 6 , g ) = − 84 .7 kJ mol −1
0
∆H f (C 2H 4 , g ) = 52 . 3 kJ mol −1
0
∆H sub (C , graphite) = 718 . 4 kJ mol −1
∆ H H− H = 435 . 9 kJ mole
P-33

Long Answer Type Questions PV γ = constant [Avadh 2010]


Based on Hess's Law Based on Bond Energy
1. (a) What is main difference between an open, 7. (a) Define ‘bond energy’. How is bond energy of
closed and isolated systems? a bond calculated for a polyatomic molecule?
(b) State and explain Hess’s law of constant (b) Calculate ∆H° for the reaction,
heat summation. How can it be used to
C 2H 4 + 3O 2 → 2CO 2 + 2H 2O
evaluate the heat of formation in those cases
Given that the average bond energies of the
where it cannot be directly determined?
different bonds are
Calculate the enthalpy of formation of
methane, given that the exthalpies of
Bond : C −H O=O C = O O −H C=C
combustion of methane, graphite and
hydrogen are 890.2 kJ, 393.4 kJ and 285.7 Bond 414 499 728 460 619 (kJ
kJ mol −1 respectively. [Avadh 2011] energies: mol −1 )
2. (a) Discuss Hess's Law of constant heat [Avadh 2009]
summation. Explain with suitable example. 8. (a) Explain bond energy. How is it estimated?
(b) The formation of benzene can be written as: (b) Calculate the bond energy of HCl. Given that
3C2 H 2 (g) → C6 H 6 (g) H-H bond energy is 433 kJ mol −1, Cl-Cl
Given that heats of combustion of C2 H 2 (g) bond energy is 242 kJ mol −1 and ∆H f for
and C6 H 6 (g) at 298 K are −1300 kJ mol −1, HCl is −91 kJ mol −1. [Purv. 2010]
calculate the heat of the above reaction. Based on Adiabatic Process
[D.D.U. 2008] 9. Show that for reversible adiabatic expansion of an
3. Calculate ∆H 0 for the reaction ideal gas. TV γ −1=constant. [Lko. 2008]
3
CH3OH(l ) + O2 (g) → CO2 (g) + 2H2O( g ) 10. (a) Prove that PV γ = constant for an adiabatic
2
process. Draw P − V diagram for the process.
from the following data:
(b) Derive Kirchhoff's equation.
∆H of CH3OH (liquid) = −238.57 kJ mol −1 [D.D.U. 2010, Purv. 2008, 09]
∆H of H2O (gas) = −241.84 kJ mol −1 Based on Heat Capacity
11. (a) Calculate the expressions for w.q. ∆E and
∆H of CO2 (gas) = −393.55 kJ mol −1.
∆H for reversible and irreversible adiabatic
[D.D.U. 2011] expansion of an ideal gas. Derive
4. Enthalpy of neutralisation of HCl and HCN with TV γ −1=constant.
NaOH are −55.9 kJ and 12.1kJ respectively. (b) Derive Kirchhoff's equation for variation of
Calculate enthalpy of ionisation of HCN. enthalpy of reaction with temperature.
[Purv. 2008] [Lko. 2009]
5. (a) State explain Hess's Law of constant heat of 12. (a) Define heat capacity of gases at constant
summation and discuss its applications pressure (C p ) and at constant volume (Cv )
(b) Drive: ∆H = ∆E + ∆nRT why Cv is greater than Cv ?
[Meerut 2008, 09, 10] (b) Explain bond energy with a suitable example.
Based on Basic Terms [D.D.U. 2009]
6. (a) (i) Distinguish between reversible and 13. (a) Define C p and C v, Derive the relationship
irreversible processes. between them.
( ii) Distinguish between state function and (b) State Hess's law of constant heat summation
path function. and discuss its important applications.
(b) Derive that for reversible adiabatic [Purv. 2009, 11]
expansion of an ideal gas : 14. (a) Derive thermodynamically Kirchoff's equation.
P-34

(b) The heat of reaction H 2 + Cl 2 → 2HCl at 27°C Based on Basic Terms


is −22.1 kcal, Calculate the heat of eaction at 4. Define isolated system, closed and open system.
77°C. The molar heart capacities at constant [Avadh 2008]
pressure and at 27°C for hydrogen, chlorine and 5. Define: isothermal and adiabatic process.
HCl and 6.82, 7.70 and 6.80 cal mol −1 [Avadh 2008]
respectively. 6. Why is ∆E = 0 for an isotheremal expansion of an
[Purv. 2010]
ideal gas. [D.D.U. 2008]
15. Explain thermodynamical reversible process and
7. Why is more convenient to measure ∆H their ∆E?
show that internal energy is a function of state.
[D.D.U. 2008]
Prove also C P − C V = R . [Purv. 2011]
8. Define zeroth law of thermodynamics. What is its
16. What is Kirchhoff's law? Derive equation for it. relevance? [Lko. 2009]
Based on Joule-Thomson Effect 9. Differentiate between system and surroundings.
17. Derive the expression for Joule-Thomson [D.D.U. 2009]
coefficient and inversion temperature in terms of 10. The property whose magnitude is independent of
Van der Waals constant. [Lko. 2010] amount of the substance is called intensive/
18. (a) State and explain the first law of extensive/colligative. Give correct answer with
thermodynamics. suitable example. [D.D.U. 2010]
11. (a) Distinguish between closed and isolated
(b) What is inversion temperature?
[Purv. 2009] system.
(b) Give mathematical statement of the first
19. What is Joule-Thomson effect? Derive an
law of thermodynamics of a closed
expression for Joule-Thomson coefficient. Show
system and isolated system.
that for an ideal gas the effect is zero.
(c) Which of the following thermodynamic
[Meerut 2009, 10, 11]
properties are intensive?
Based on First Law
E, S, T, P, V, H
20. Calculate ∆E and ∆H when 10 litres of an ideal gas (d) What is the change in internal energy
at 10°C is heated to 100°C in a closed vessel when an ideal gas expands isothermally?
Given C = 2 R . [Purv. 2010]
 v
3  [Lko. 2011] 12. Write short note on thermodynamic criteria of
21. State & explain 1st law of thermodynamics. Derive equilibrium. [Alld. 2010]
an expression of work done when an ideal gas Based on Heat Capacity
expand isothermally and reversibly. Calculate q, w, 13. Define CP and CV . How are they related to one
∆E for the isothermal reversible expansion of one another? Derive the relationship. [Avadh 2010]
mole of an ideal gas from an initial pressure of 1 14. Derive thermodynamically Kirchhoff equation.
atm, to a pressure of 0.1 atm at a temperature of [Avadh 2009]
273K. [Meerut 2008] 15. Write short note Kirchhoff's equation.
Based on Heat of Reaction [D.D.U. 2009; 11; Meerut 2008]
22. Heat of combustion of ethanol is −330 kcal Heat of 16. Why heat capacity at constant pressure is more
formation of CO 2 (g) and H 2 O(l) is −94.3 kcal than heat capacity at constant Volume of a gas?
[Purv. 2008]
−68.5kcal respectively. Calculate the heat of
17. Write short note on heat capacity. [Purv. 2010]
formation of ethanol. [Purv. 2010]
18. What do you understand by the term C p and C v ?
Explain why C p is always greater than C v .
Short Answer Type Questions [Agra 2009]
Based on Hess's Law
19. Calculate the work performed by the system during
1. State Hess’s law of constant heat summation and reversible isothermal expansion of two moles of an
its important applications. [Avadh 2010] ideal gas from 2lit. to 10lit. at 20°C. [Agra 2009]
2. What are the laws of thermochemistry? Explain with 20. Derive thermodynamically Kirchoff's equation.
an example of each. [D.D.U. 2009] [Agra 2009]
3. What is Hess's law? Given its application. 21. Define the relationship between Cp and Cv .
[Agra 2008] [Kanpur 2012]
P-35

Based on Joule-Thomson Effect 39. Heat of combustion of carbon monooxide at


22. Show that for an ideal gas Joule-Thomson constant volume and at 17°C is 67710 calorie.
coefficient is zero. [Avadh 2011] Calculate the heat of combustion at constant
23. Explain Joule-Thomason's effect and explain why pressure. [Avadh 2010]
for ideal gas Joule Thomson's coefficient is zero. 40. Why the heat of neutralization of a strong acid by a
[Purv. 2008] strong base is always constant. [Lko. 2011]
24. (a) Define intensive property and extensice 41. A process is carried out adiabatically. Will ∆H of
property with example. the system be zero? Justify. [Lko. 2011]
(b) Explain 'heat of neutralization'. 42. What do you understand by enthalpy of
(c) Explain inversion temperature with neutralization? [Kanpur 2008]
example. [Purv. 2011]
25. Explain Joule-Thomson effect and Joule-Thomson Very Short Answer Type Questions
coefficient of a real gas. [Agra 2008] Based on Hess's Law
26. What is Joule-Thomson coefficient? Prove that 1. Define Born Haber cycle.
Joule-Thomson coefficient for an ideal gas is zero.
2. What is Hess's law?
[Kanpur 2009, 10]
Based on First Law 3. Differentiate between bond energy and bond
dissociation energy.
27. State and explain the first law of thermodyna- mics.
[Avadh 2011] Based on Basic Terms
28. Define first law of thermodynamics. What is the 4. Define different types of system.
mathematical statement of the law? 5. What are extensive and intensive properties?
[Lko. 2008, Kanpur 2012] 6. What are state and non state functions?
29. Prove that reversible work done for an ideal gas Based on Heat Capacity
maximum. [Lko. 2008]
7. Define molar heat capacity.
30. Maximum/Minimum work is obtained when gas
8. Explain Kirchhoff's Law. [Agra 2009]
expands against vacuum. Which one is correct?
Justify your answer. [D.D.U. 2010] Based on Joule Thomson Effect
31. Calculate the work done by the system when an 9. What is Joule Thomson effect?
ideal gas is allowed to expand isothermally from Based on First Law
volume V1 to V2 . [D.D.U. 2011] 10. Write the expression of work done by an ideal gas
32. Derive an expression for maximum work in the under isothermal reversible condition.
reversible isothermal expansion of an ideal gas. 11. What is first law of thermodynamics?
[Kanpur 2009] [Meerut 2009]
33. What is First Law of Thermodynamics? Explain. Based on Heat of Reaction
[Kanpur 2008] 12. Define heat of neutralisation.
34. Write short note on the following:
(a) Kirchhoff's equation
(b) First law of Thermodynamics. Objective Type Questions
[Kanpur 2010, 11]
35. What are the limitations of first law of Multiple Choice Questions
thermodynamics ?
1. For the combustion of n-octane:
Based on Heat of Reaction
C 8H 18 + O 2 → CO 2 + H 2O at 25°C (ignore
36. A process is carried out adiabatically. Will ∆H of the
resonance in CO2 )
system be zero? Explain. [Lko. 2011]
(a) ∆H = ∆E − 5 . 5 × 8 . 31 × 0 . 298 in kJ / mol
37. What do you understand by enthalpy of
neutralization? Explain. [Avadh 2011] (b) ∆H = ∆E − 4 . 5 × 8 . 31 × 0 . 298 in kJ / mol
38. Write an equation for the work down by an ideal (c) ∆H = ∆E − 4 . 5 × 8 . 31 × 298 in kJ / mol
gas under isothermal conditions. [Garhwal 2006] (d) ∆H = ∆E − 4 . 5 × 8 . 31 × 0 . 298 in kJ / mol
P-36

2. H 2 (g) + ½O 2 (g) → H 2 O(l) ; ∆H = − 68.39 kcal 9. The value of ∆E for ideal gas in isothermal process
K(s) + H 2 O(l) + aq → KOH(aq) + ½ H 2 (g) ; ∆H = is:
− 48 kcal (a) zero (b) finite (c) infinity (d) one.
KOH(s) + aq → KOH(aq) ; ∆H = −14 kcal [Agra 2008]
10. CP > CV by:
The heat of formation of KOH(s) is (in kcal):
(a) − 68.39 + 48 − 14 (a) T (b) P (c) S (d) R.
[Agra 2008]
(b) − 68.39 − 48 + 14
11. A gas absorbs 250 J of heat and expands from 1
(c) 68.39 − 48 + 14
litre to 10 litres against the pressure 0.5 atm at
(d) 68.39 + 48 + 14 . constant temperature. The values of q, w and U are
3. The bond energy of H 2 is 104.3 kcal/mol. It means respectively:
that: (a) + 250, − 455 J, −205 J
(a) 104.3 kcal heat is needed to break one bond to (b) 250 J, 455 J, 205 J
form two atoms
(c) − 250 J, 205 J, 455 J
(b) 104.3 kcal is required to break 6.02 × 1023
(d) − 205 J, −250 J, − 455 J .
molecules into atoms of hydrogen
12. Which one of the following is the correct equation?
(c) 104.3 kcal is required to break 3.015 × 1023
(a) ∆H = ∆Ε + w
hydrogen molecules to 6.02 × 1023 hydrogen
(b) ∆ H = ∆ E − w
atoms
(c) ∆H = ∆Ε + (∆Ε − w)
(d) None of these.
(d) ∆ H = − ∆ H − w. [Garhwal 2011]
4. The value of bond energies (kJ mole −1) of N−N, N
13. The mathematical form of the 1 st law of
= N, N ≡ N are respectively:
thermodynamics is :
(a)159.0, 418.4, 945.6
(a) dq = dE + dw
(b) 945.6, 418.4, 159.0
(b) dq = dE – dw
(c) 418.4, 159.0, 945.6
1
(d) 945.6, 159.0, 418.4 . (c) dE = d + dw
dq
5. The dissociation energy of methane is 360 kcal mol (d) None of these.
−1
[Garhwal 2010]
and that of ethane is 620 kcal mol −1. The C−C
14. One male of a gas absorbs 200 J of heat at constant
bond energy is:
volume Its temperature rises from 298k to 30 k.
(a) 210 (b) 130 (c) 180 (d) 80. The change in internal energy is
6. Given: 308 298
(a) 200 J (b) –200J (c) 200 × (d) 200 ×
NH 3 (g) + 3Cl 2 (g) → NCl 3 (g) + 3HCl(g) + X 1 298 308
N 2 (g) + 3H 2 (g) → 2NH 3 (g) + X 2 15. In a cyclic process :
H 2 (g) + Cl 2 (g) → 2HCl(g) – X 3 (a)q = w (b) q = T (c) q=E (d) q = S.
The heat of formation of NCl 3 (g) is [Garhwal 2009]
X 3X 3 X 3X 3 16. The word standard in standard molar enthalpy
(a) X 1 + 2 + (b) X 1 + 2 –
2 2 2 2 change implies:
X 2 3X 3 3X 3 (a) Temperature 298 K
(c) X 1 – + ` (d) X 1 + X 2 +
2 2 2 (b) Pressure 1 atm
7. An example of extensive property is: (c) Temperature 298 K and pressure 1 atm
(a) Temperature (b) Internal energy (d) All temperatures and all pressures
(c) Viscosity (d) Molar heat capacity. 17. The factor which does not influence the heat of
8. Identify the intensive quantities from the following: reaction is:
(a) Enthalpy (b) Temperature (a) The physical state of reactants and products.
(c) Volume (d) Internal energy . (b) The temperature of the reaction.
P-37

(c) The method by which the final products are (a) − 46 kJ


obtained. (b) 46 kJ
(d) Whether the reaction is carried out at constant (c) 184 kJ
pressure or constant temperature. (d) − 184 kJ .
18. Heat of neutralisation of NaOH and HCl is −57.46 25. The difference between the heats of reaction at
kJ/equivalent. The heat of ionisation of water will be : constant pressure and constant volume for the
(a) −57.46 kJ/mol (b) 57.46 kJ/mol reaction
(c) −114.92 kJ/mol (d) 114.92 kJ/mol . 2C 6 H 6 (l) + 15O 2 (g) →12CO 2 (g) + 6H 2 O(l) at
19. At 25°C and 1 atm, which one(s) of the following 25°C in kJ is:
has a non-zero ∆H °f ? (a) − 7.43
(a) Fe (b) O (c) C(s) (d) Ne . (b) 3.72
20. The lattice energy of solid NaCl is 180 kcal per mol. (c) − 3.72
The dissolution of the solid in water in the form of (d) 7.43 .
ions is endothermic to the extent of 1 kcal per mol. 26. The following is not an endothermic reaction(s) :
If the solvation energies of Na + and Cl − ions are in (a) Combustion of methane
the ratio 6 : 5, what is the enthalpy of hydration of
(b) Decomposition of water
sodium ion?
(c) Dehydrogenation of ethane to ethylene
(a) −85.6 kcal/mol (b) − 97.5 kcal/mol
(d) Conversion of graphite to diamond.
(c) 82.6 kcal/mol (d) +100 kcal/mol.
21. The heats of neutralization of four acids A, B, C and
D when neutralized against a common base are
Fill in the Blank
13.7, 9.4, 11.2 and 12.4 kcal respectively. The 1. In an open system only ……… exchange takes
weakest among these acids is: place between system and surrounding.
(a) A (b) B (c) C (d) D. 2. Bribing point is an ……… property.
22. For the allotropic change represented by the 3. The µ JT (Joule Thomson co-efficient) is given by
equation µ JT =……….
C(diamond) → (graphite) 4. According to first law of thermodynamics,
∆U = q +……….
The enthalpy change is ∆H = − 1.89 kJ
5. Workdone by an ideal gas under isothermal
If 6 g of diamond and 6 g of graphite are separately
reversible is given by w =……… expansion.
burnt to yield carbon dioxide, the heat, liberated in
the first case is:
(a)Less than in the second case by 1.89 kJ/mole True and False
(b)More than in the second case by 1.89 kJ 1. Density is an intensive property.
(c) Less than in the second case by 11.34 kJ 2. In terms of molar heat capacities, CP − CV = R
∂H 
(d)More than in the second case by 0.945 kJ. 3. Kirchhoff law is CP =  
 ∂T  P
23. The enthalpy change for which of the following
processes represents the enthalpy of formation of 4. Joule Thomson effect is shown by ideal gases.
AgCl? 5. Work done by a system is a state function.
+ −
(a)Ag (aq)+Cl (aq) → AgCl(s) 6. Workdone in an isothermal free expansion is zero.
(b) Ag(s) + ½Cl 2 (g) → AgCl(s) 7. The feasibility of a process can be predicted by the
(c) AgCl(s) → Ag(s)+½ Cl 2 (g) first law of thermodynamics. [Garhwal 2009]
(d) Ag(s)+ AuCl(s) → Au(s) + AgCl(s). 8. The heat of neutralisation of HF is more
24. The enthalpy of formation of ammonia is − 46 kJ exothermic than − 13 . 69 kcal.
mol −1. The enthalpy for the reaction, 2N 2 (g) + 6H
2 (g) → 4NH 3 (g)
P-38

Numerical Questions
1. –83.5 kJ 2. –121 kJ 3. ∆H = –3273.735 kJ
4. ∆H° = –202.6 kcal 5. (i) M = 40, 6. –22.0 kcal
(iii) Monoatomic,
7. 309 kJ 8. –3201 kJ, –3199.75 kJ 9. –115.87 J
10. 416.3 , 331.4 kJmol −1, 591.1 kJ mol −1

Objective Type Questions

Multiple Choice Questions


1. (d) 2. (b) 3. (b) 4. (a) 5. (d)
6. (a) 7. (b) 8. (b) 9. (a) 10. (d)
11. (a) 12. (b) 13. (b) 14. (a) 15. (a)
16. (b) 17. (c) 18. (b) 19. (b) 20. (b)
21. (b) 22. (d) 23. (b) 24. (d) 25. (a)

26. (a)

Fill in the Blank


1. energy 2. intensive 3.  ∂T  4. w 5. V 
 
 ∂P  H − nRT ln  f 
 Vi 

True and False


1. T 2. T 3. T 4. F
5. F 6. T 7. F 8. T

Hints & Solutions

Numerical Questions
1. C2H6 + 3 . 5 O2 → 2CO2 + 3 H2O
So, ∆ H r = 2 × (∆ H f )CO + 3 × (∆H f )H O − (∆ H f )C H
2 2 2 6
2. C6H10 + H2 → C6H12 . So, ∆ H r = (∆ H c )C H + (∆ H c )H − (∆H c )C H
6 10 2 6 12
3. As, ∆H = ∆U + ∆ H RT
For the reaction C6H6 + 7 . 5 O2 → 6CO2 + 3H2O. So, ∆H = − 1 . 5
4. ∆ H r = (∆ H f ° )Al O − (∆ H f ° )Fe O
2 3 2 3
P-39

5. CP
(i) Using (CP − CV ) × M = R (ii) γ = suggests atomicity
CV
0 . 125 5
As V= =
0 . 75 3
6. For reaction, H2 + Cl2 → 2HCl
∆H r = (BE)R − (BE)P = (BE)H + (BE)Cl − 2 (BE)HCl
2 2
7. S( g ) + 6F(g) → SF6
∆ H r = (∆ H f )SF − [(∆ H f )S + 6 (∆ H f )F ]
6
And, ∆ H r = 6 (BE)S − F
8. Use ∆ H = ∆U + (∆ H) RT
For, C6H5COOH + 7.5 O2 → 7CO2 + 3H2O
(s) (g ) (g ) (l)
∆H = − 0 . 5
9. Using TV γ − 1 = constant; T1V1γ − 1 = T2 V2 γ − 1
T2 is calculated
And the ∆ H = nCP (T2 − T1)
10. Use ∆H = (BE)R − (BE)P

Long Answer Type Questions


1. (b) We have to find, C + 2H 2 → CH4 ∆ Hg = ?
Here, ∆ H f = (∆ H c )C + 2(∆ H c )H − (∆ H c )CH
2 4
∆ H c is heat of combustion
So, ∆ H f = 393. 4 + 2 × 285 . 7 − 890 . 2 = 74 . 6 kJ/mol
2. (b) ∆ H r = (∆ H c )R − (∆ H c )P = 3 × 260 − (−1300)
= 2080 kJ / mol
3. ∆ H r = (∆ H f ) p − (∆ H f ) r
= (− 393 . 55) + 2 × (− 241 . 84) − (− 238 . 57)
= − 638 . 66 kJ/mol.
4. ∆ Hi = − 12 . 1 − (− 55 . 9) = 43 . 8 kJ
 
7. (b) ∆ H r = (BE)C = C + 4 (BE)C— H  − 2 × 2 (BE)C = O + 2 × 2 (BE)O—H 
   
(Q ∆H r = BE R − BE P )
So, ∆ H r = {619 + 4 × 414} − {4 × 728 + 4 × 460}
= (2275) − (4752) = − 2477 kJ / mol
8. (b) ∆ H γ = (BE)R − (BE)P
As the reaction is, H 2 + Cl2 → 2HCl
So, ∆ H f = (BE)H + (BE)Cl − 2 × (BE)HCl
2 2
− 91 × 2 = [433 + 242] − 2 x
Solving x = 428 . 5 kJ / mol
14. (b) ∆ H 2 = ∆ H 1 + (∆ CP )γ (∆ T)
⇒ ∆ H77 ° C = ∆H 27 ° C + [2 × 6 . 8 − (6 . 82 + 7 . 7)] × 50
= − 22 . 1 × 103 + [− 46] = − 22 . 146 kcal.
w T − T1 100 
16. As, = 2 ⇒ w = 1897 . 86 × 103 
q2 T2  373 

= 508 . 81 kJ
P-40

20. Using ∆ E = HCV (∆T) Pressuring n = 1


3
∆ E = × R [90] = 270 cal
2
5
∆ H = × R [90] = 450 cal. (Q CP = R + CV )
2
21. Work done in isothermal reversible case is
P
w = − nRT × 2 . 303 log 10 i
Pf
1
= − 1 × 2 × 273 × 2 . 303 × log = − 1 . 257 kcal
0.1
For isothermal process, ∆ E = HCV (∆ T) = 0
So, ∆E = q + w (∆E and ∆U are same).
⇒ q= −w
22. C2H5OH+ 3O2 → 2CO2 + 3H 2O
∆ H γ = 2(∆ H f )CO + 3(∆H f )H O − (∆H f )C H OH
2 2 2 5
= − 330 = 2 × (− 94. 3) + 3 (− 68.5) − (∆H f )C H OH
2 5
(∆H f )C H OH = − 64 . 1 kcal / mol.
2 5

Multiple Choice Questions


13. In a cyclic process, ∆E = 0.

Fill in the Blank


2. Bf does not depend on mass content of a system.

True and False


1. m
d= , both
V
mass and volume equally depend on the mass content.
4. Ideal gases have no inter molecular force of attraction.
6. As in case of free expansion, P ext =0.

8. FO has very high value of enthalpy of hydration.

mmm
UnitP-41
-II

C HAPTER 2
Chemical Equilibrium

2.1 Introduction and Basic Terms


The concept of chemical equilibrium was proposed after Berthelot observed that certain chemical reactions
are reversible when these are allowed to take place in a closed contain er. An example is decomposition of
PCl 5 as, PCl5 (g) a PCl3 (g)+ Cl2 (g).
According to him, any reversible reaction is found to attain equilibrium only when the rates of forward and
backward reaction equal each other.
He further stated that if composition of the system does not change with time, the system is said to be in
chemical equilibrium. It is the state in which there is no net reaction. By the way it is important to remind that
the concept of chemical equilibrium is strictly applicable to reversible reactions only.

2.1.1 Reversible Reactions


Those reactions in which the reactants transform to give products and products are simultaneously
transformed into reactants are called reversible reactions. So, such reactions proceed in both forward and
reverse directions and interestingly never completed. The state of chemical equilibrium is achieved if the
forward and reverse rates of reaction are equal. The common examples of reversible reactions are
N 2 (g) + O 2 (g)L 2NO(g)
PCl 5 (g)L PCl 3 (g) + Cl 2 (g)
2NO 2 (g)L N 2 O 4 (g)
H 2 (g) + I 2 (g)L 2HI(g)
CH 3 COOH(aq) + C 2 H 5 OH(aq)L CH 3 COOC 2 H 5 (aq) + H 2 O(aq)
State of Equilibrium: In reversible reactions, a stage is Forward Rate
reached when the rate of transformation of reactants into
products equals the rate of transformation of products into
Rate

reactants. At this stage, the composition of reactants and


products does not change with time. However it never means Reverse Rate
that the reaction has stopped taking place. As a fact, both time
backward and forward reactions are still taking place but the
Fig. 1: Chemical equilibrium in terms of rate
P-42

two rates are exactly the same. This stage


of reaction is known as state of Reactants
equilibrium. This is being graphically At equilibrium

Amount
represented as below. The first graph No change in
states that the two rates of reaction- concentration
forward as well as backward have become
equal. (Fig. 1)
Products
Similarly in the second graph, the time
concentrations of the product and reactant
have acquired a constant equilibrium value. Fig. 2: Chemical equilibrium in terms of concentrations
(Fig. 2) of reactants and products

2.1.2 Law of Mass Action and Equilibrium Constant


The law of mass action was proposed by Guldberg and Waage. The law states that “at constant temperature,
the rate of reaction is directly proportional to active masses of the reactants raised to the power or exponents
of proper stoichiometric coefficients”.
Active Mass: Active mass of the reactant is directly proportional to its molarity. Active mass of any species is
represented by writing the concentration of that species in square brackets.
Active mass of reactant = γ × Molarity
where γ = Activity coefficient
∴ a = γ × Molarity
For very dilute solutions, the value of γ is unity.
∴ a = Molarity
Equilibrium Constants, K c and K P: Let us consider a reaction of the type
A(g) + B(g) L C(g) + D(g).
According to law of mass action, rate of forward reaction α [A] [B]
So, rate of forward reaction = k f [A] [B]
here K f is the rate constant of forward reaction.
Similarly rate of backward reaction α [C] [D]
So, rate of backward or reverse reaction = k r [C] [D]
here k r is the rate constant of backward reaction.
At equilibrium the forward rate of reaction is equal to the backward rate of reaction. So,
k f [A] [B] = k r [C] [D]
k f [C ][D]
∴ =
k r [A ][B]
As k f and k r are the constants at a given temperature, thus their ratio (K f / K r ) would also be constant.
[ ][D]
Therefore, the ratio of C is also constant called K eq .
[A][B]
[C ][D]
∴ = K eq
[A][B]
P-43

For instance, if it is assumed that the K eq=10 for this equilibrium, then no matter what one takes initially,
[C][D]
once the equilibrium is established the ratio of will always be equal to 10. This certainly looks
[ A][B]
surprising.
[ ][D]
The ratio of C would be called K c , when the active masses are expressed in the terms of molar
[A][B]
[ ][D]
concentration of reactants and products. Similarly, the ratio C is called K p when the active masses are
[A][B]
expressed in the terms of partial pressures, so that,
PC × PD
∴ Kp =
PA × PB
There is one more type of equilibrium constant K x . It is used when the active masses are expressed as mole
(X c )(X D )
fractions. So, K x =
(X A )(X B)
Concentration of Pure Solids and Pure Liquids: The concentration of all pure solids and pure liquids is
a constant and it is conventionally taken as (1) This is because if initially one takes w gm of A, then the moles
of A are w/M. The volume of A is w/d where d is the density of A. Therefore, the initial concentration of A is
w/M d
= . Thus, it is evident that at equilibrium the concentration of A remains as d/M which is constant (
w /d M
d and M are constants). In fact even if A were a pure liquid, its concentration would have remained constant.
It is important to note that for aqueous solutions, concentrations are not considered as unity.
Relation Between K P And K c : Let us consider a reversible reaction of the type
x 1A(g) + x 2 B(g) L y 1C(g) + y 2 D(g)
The expression of equilibrium constant K c is given by
[C ]y 1 [D]y 2
∴ Kc =
[A] x1 [B]x 2
Here, all the active masses are expressed as concentrations.
As it is known, the concentration of a gas is given by P/RT. Thus,
Pc P P PB
[C] = , [D] = D , [A] = A , [B] =
RT RT RT RT
y1 y2
 Pc  P 
  ×  D
 RT   RT 
∴ Kc =
x1 x2
 PA  P 
  ×  B
 RT   RT 

y 1 + y 2 ) −( x1 + x2 ) (Pc )y 1 (p D )y 2
∴ K c (RT)( =
(PA )x1(PB)x2
RHS of the equation is a constant called K p . By definition,
( )y 1 (PD )y 2
K p = Pc
(PA )x1 (PB )x2
P-44

So, K p = K c (RT)∆n
Here, ∆n = number of moles of gaseous products - number of moles of gaseous reactants
R = Gas constant = 0.0821 litre atm per mole per Kelvin
And, T = Absolute temperature
(i) If ∆n = 0, then K p = K c ,
For N 2 (g) + O 2 (g) L 2NO(g), ∆n = 0
(ii) If ∆n > 0, then K P > K c ,
For PCl 5 (g) L PCl 3 (g) + Cl 2 (g), ∆n = 2 – 1 =1
(iii) If ∆n < 0, then K p< K c ,
For N 2 (g) + 3H 2 (g) L 2NH 3 (g), ∆n = 2 – 4=–2
Relation between K p and K x : It is known that P A = (X A )Pt . Here P A is partial pressure of A, X A is mole
fraction of A and Pt is total pressure.
y y
(PC )y 1 (PD )y 2 (x C p t ) 1 (x D p t ) 2
So, Kp = =
(PA )x1 (PB)x2 (x A p t )x1 (x Bp t )x2
⇒ K p = K x (Pt )∆n

Units of K p and K c : When the concentration of species are expressed in moles per litre, then the unit of K c
∆n
 moles 
would be  . Similarly, when the pressure is expressed in atmospheres, the unit of K P would be
 litre 
(atm)∆n .
There is one more way in which partial pressures and concentrations are expressed as ratios. In this way of
expressing concentrations and partial pressures, K p and K c become unit less.

Example 1: For the reversible reaction, N 2(g) + 3H 2(g) L 2NH 3 (g) at 500°C, the value of K P
−5
is 1.44 × 10 when partial pressure is measured in atmospheres. The corresponding value of K
−1
c , with concentration in mole litre , is

(a) 1.44 × 10 −5 /(0.082 × 500) −2 (b) 1.44 × 10 −5/(8.314 × 773) −2


(c) 1.44 × 10 −5/(0.082 × 773) 2 (d) 1.44 × 10 −5/(0.082 × 773) −2
−∆ng
Solution: K c =K p(RT)

Since ∆ng = −2
∴ K c = 1.44 × 10 −5 × (0.082 × 773) 2
Ans: (d).

2.1.3 Factors Affecting Equilibrium Constant


There are certain prominent factors which affect the value of equilibrium constant. Some of these are
discussed in details.
P-45

1. Nature of Reactants and Products: The value of equilibrium constant depends on the nature of
reactants and/or products. By changing the reactant(s) or product(s) of a reaction, its equilibrium
constant will change. For example
N 2 (g) + O 2 (g) L 2NO(g) ...(1)
N 2 (g) + 2O 2 (g) L 2NO 2 (g) ...(2)
Although the reactants are same but products in the two reactions being different, the value of
equilibrium constant for the two reactions will be different.
[ NO]2
KP = for reaction (1), Similarly ...(1)
[ N 2 ][O2 ]
[ NO2 ]2
for reaction (2) KP = ...(2)
[ N 2 ][O2 ]2
2. Temperature: The variation of equilibrium constant with temperature is given by the relation
K2 ∆H  1 1
log =  − . …(1)
K1 2.303 R  T1 T2 
This relation is also termed as Van't Hoff equation
This can be simply obtained by Arrhenius equation. Starting with Arrhenius equation of rate constant
and applying for forward reaction,
k f = A fe – Eaf / RT …(2)
here, k f = rate constant for forward reaction, A f = Arrhenius constant of forward reaction,and, Ea f =
Energy of activation of forward reaction. Similarly for backward reaction,
–E / RT
k r = A re a r …(3)
where, k r = rate constant for backward reaction, A r = Arrhenius constant of backward reaction and,
Ea r = Energy of activation of backward reaction.
Dividing (2) by (3),
(E a r − E a )
f
kf A f RT
= e
kr A r
k
It is Known that f = K eq (equilibrium constant )
kr
(E a r − E a )
f
k A RT
∴ K eq = f = f e
kr A r
For Sake of convenience, K eq will be represented as K.
At temperature T 1,
(E a r − E a )
f
Af RT1
K T1 = e ….(4)
Ar
At temperature T 2 ,
(E a r − E a )
f
A RT2
K T2 = fe …(5)
Ar
P-46

Dividing (5) by (4) we get


 Ea r −Ea f   1 1 
  − 
K T2  R   T2 T1 
=e
K T1

Taking log of both the sides,


K T2 Ea − Ea f  1
r 1
log =  − 
K T1 2.303 R  T2 T1 

The enthalpy of a reaction is defined in terms of activation energies of forward and reverse reactions as
Ea f − Ea r = ∆H
K T2 − ∆H  1 1
∴ log =  − 
K T1 2.303 R  T2 T1 
K T2 ∆H  1 1
∴ log =  −  …(6)
K T1 2.303 R  T1 T2 

(i) If ∆H is +ve (endothermic) an increase in temperature (T2 > T1) will make K2 > K1. So, the
reaction goes more towards the forward direction and vice-versa.
(ii) If ∆H is –ve (exothermic) an increase in temperature(T2 > T1), will take K 2 < K 1. It implies that
the reaction goes in the reverse or backward direction.
3. Stoichiometry of Reaction: The value of K p and K c depends upon the stoichiometry of reaction as

N 2 (g) + 3H 2 (g) L 2NH 3 (g)


[NH3 ]2
K c= …(1)
[N2 ][H2 ]3

N 2 (g) + H 2 (g) L
1 3
NH 3 (g)
2 2
[NH3 ]
K 'c = …(2)
[N2 ]1/ 2 [H2 ]3 / 2

From (1) and (2), K 'c = Kc

In general, if for a reaction A a B equilibrium constant is K, then, for,


nA a nB,
the equilibrium constant becomes (K) n .
4. Mode of writing a Chemical Equation: The value of K P and K c also depend on the method of
representing a chemical equation. So, if N2 and H 2 combine to give ammonia, then,
N 2 (g) + 3H 2 (g) L 2NH 3 (g)
[NH3 ]2
Kc =
[N2 ][H2 ]3

If the equilibrium reaction is reversed, then the new equilibrium constant change as well
P-47

2NH 3 (g) L N 2 (g) + 3H 2 (g)


[N2 ][H2 ]3 1
K 'c = =
[NH3 ]2 Kc

5. Equilibrium Constant for the sum of Reactions: It is possible to determine the equilibrium
constant for a reaction and then use them to obtain the equilibrium constant of other reactions. So,if a
given chemical equation can be obtained by taking the sum of other equations, the equilibrium
constant for the given equation equals the product of the equilibrium constants of the constituent
equations.

So, for A a B ; equilibrium constant is K. If this reaction is written as a sum of different reactions like,

Aa C K1 Ca D K2

Da B K 3 ; Here K 1, K 2 and K 3 are individual equilibrium constants. Then K = K 1 × K 2 × K 3 .

Example 2: Determine K c for the reaction ½N 2(g) + ½O 2(g) + ½Br 2(g)l NOBr(g) from the
following information (at 298 K)
2NO (g) l N 2(g) + O 2(g) ; K1 = 2.4 × 10 30

NO(g) + ½Br 2(g) l NOBr(g) ; K 2 = 1.4

Solution:
(i) ½N 2 (g) + ½O 2 (g) l NO(g)
1 1 −30
K 'c = = × 10 = 6.45 × 10−16
K1 2.4

(ii) NO(g) + ½Br 2 (g) l NOBr(g) ; K 2 = 1.4


Adding (i) & (ii) gives the net reaction :
½N 2 (g) + ½O 2 (g) + 1/2Br 2 (g)l NOBr(g)
[NO] [NOBr] [NOBr]
K= × =
1 1 1 1 1 1
[N2 ] 2 [O 2 ]2 [NO][Br2 ]2 [N 2 ] 2 [O 2
2 ] [Br2 ]
2

= K c × K2 = 6.45 × 10−16 × 1.4


'

= 9.03 × 10 −16

2.1.4 Reaction Quotient and Prediction of Direction of Reaction


Suppose a gaseous mixture consists of 0.02 M CO, 0.02 M H 2 , 0.001 M CH 4 and 0.001 M H 2 O. If the
mixture is passed over a catalyst at 1200 K, the following reaction occurs,
CO(g)+3H 2 (g) l CH 4 (g)+H 2 O(g);
with an equilibrium constant value K c =3.0 × 10−2 . The question to be answered here is will the reaction start
from the right or from the left to attain equilibrium with these concentration values of the reactants and
products.
P-48

To answer this problem, a new term is introduced called reaction quotient, Q. Reaction quotient is an
expression that has the same form as the equilibrium constant expression, but concentration values are not
those at equilibrium, it can be at any instant of time. To predict the direction, we substitute the concentrations
of substances into the reaction quotient and compare its value to K c .
0.001 × 0.001
Q= = 6.25
0.02 × (0.02)3
As the value of Q is more than K c , this value will tend to approach the value of K c. This means Q has to
decrease which is possible only when products are converted to reactants so that the reaction starts
proceeding in the reverse direction to attain equilibrium. So, the reaction will proceed in backward or reverse
direction.
For the general reaction :
aA + bB L cC + dD
[C]ci [D]di
Qc=
[A]ai [B]bi
where the subscript i indicates concentrations of species at a particular instant, i.
Then, if Q c > K c , the reaction will go to the backward direction.
If Q c < K c , the reaction will go to the forward direction.
If Q c = K c , the reaction mixture is at equilibrium.

2.2 Equilibrium Constant for Different Types of Equilibria


Primarily, depending upon the phases of reactants and products, chemical equilibrium is of two types-
1. Homogeneous equilibrium
2. Hetrogeneous equilibrium
These two are discussed below at length.
2.2.1 Homogeneous Equilibria
Homogeneous equilibria are those in which phases of all the reactants and products are the same. so, all the
reactants and products may be either gases or in solution phase.
1. Homogeneous equilibria in gaseous phase: A homogeneous equilibrium in gaseous phase is an
equilibrium that involves reactants and products in gaseous phase. For example
PCl 5 (g) L PCl 3 (g) + Cl 2 (g)
[ ] × [Cl2 ] PPC l3 × PC l2
K c = PCl3 , Kp =
[PCl5 ] PpC l
5

Here, [ ] represents molar concentration. It is important to note that the molar concentrations of PCl 3
and Cl 2 at equilibrium need not be equal. Similarly the equilibrium pressure of PCl 3 and Cl 2 need not
be the same because equilibrium amount - molar concentration or pressure depends upon initial
situation of the reaction mixture.
2. Homogeneous equilibria in solution phase: A homogeneous equilibrium in solution phase is an
equilibrium that involves all reactants and products in the same phase - solution phase. For example,
P-49

CH 3 COOH(aq) + C 2 H 5 OH(aq) L CH 3 COOC 2 H 5 (aq) + H 2 O(aq)


[CH3COOC 2H5 ] × [H2O]
Kc =
[CH3COOH] × [C 2H5OH]

For equilibrium in solution phase only K c exists. K p does not exist for such cases because one cannot
find the pressure of a liquid or a solution. In the above equilibrium, all components are in solution
phase.
Example 3: Initially 1 mole each of acetic acid, ethanol, ester and water are present in a vessel.
If equilibrium constant is 4 then find the amount in grams of all the constituents of equilibrium
mixture.
CH 3 COOH(aq) + C 2H 5OH(aq) L CH 3 COOC 2H 5(aq) + H 2O(aq)

Solution: The first thing to be kept in mind is that for an equilibrium having a constituent in solution phase,
K P would not exist. Thus, the equilibrium constant given is K c .
CH 3 COOH(aq) + C 2 H 5 OH(aq) L CH 3 COOC 2 H 5 (aq) + H 2 O(aq)
Initially 1 1 1 1
At equilibrium (1-x) (1-x) (1+x) (1+x)
(1 + x )(1 + x )
Equilibrium constant, Kc =
(1 − x )(1 − x )
2
1 + x  1+ x 1
4=  ; 2= ; x=
1 − x  1− x 3
 1
Amount of acetic acid at equilibrium =1 −  × 60 = 40 g
 3
 1
Amount of ethanol at equilibrium =1 −  × 46 = 30.67 g
 3
 1
Amount of ester at equilibrium =1 +  × 88 = 117.33 g
 3
 1
and amount of water at equilibrium =1 +  × 18 = 24 g
 3
Example 4: The equilibrium constant for the reaction 2SO 2 + O 2 L 2SO 3 at 1000K is 3.5.
What would the partial pressure of oxygen gas have to be to give equal moles of SO 2 and SO 3 ?
(a) 0.29 atm (b) 3.5 atm (c) 0.53 atm (d) 1.87 atm
(PSO3 )2 1
Solution: KP = =
2 PO2
(PSO2 ) PO2

Here, partial pressure of SO 3 and SO 2 are same because moles of SO 2 and SO 3 are equal at equilibrium.
1
PO2 = = 0. 285
Kp

Ans: (a).
P-50

Example 5: PCl 5 is 50% dissociated into PCl 3 and Cl 2 at 1 atmosphere. It will be 40%
dissociated at:
(a) 1.75 atm (b) 1.84 atm (c) 2.00 atm (d) 1.25 atm
Solution: PCl 5 L PCl 3 + Cl 2
Let x = degree of dissociation of PCl 5 at pressure p
y = degree of dissociation of PCl 5 at pressure p'
px 2 p' y2
Kp = =
2
(1 – x ) (1 − y2 )
given, x = 0.50, p = 1 atm
y = 0.40, p' = ?
1 × 0. 25 p' × 0.16
=
075
. 0.84
p' = 1.75 atm
Ans: (a).

2.2.2 Heterogeneous Equilibria


Heterogeneous equilibria are those which involve more than one phases among the reactants and products.
Some of the examples are
1. CaCO 3 (s)l CaO(s) + CO 2 (g)
2. NH 4 HS(s)l NH 3 (g) + H 2 S(g)
3. 2Ag 2 O(s))l 4Ag(s) + O 2 (g)
4. NH2COONH4 (S) a 2NH3 (g)+ CO 2 (g)
Decomposition of solid CaCO 3 into solid CaO and gaseous CO 2 is a heterogeneous equilibrium. It is being
discussed below. Let a moles of CaCO 3 are taken in a vessel of volume V litre at temperature TK.
CaCO 3 (s) l CaO(s) + CO 2 (g)
Moles initially a 0 0
Moles at equilibrium (a– x) x x
[CaO] × [CO 2 ]
Kc =
[CaCO 3 ]
Since, CaO and CaCO 3 are pure solids, their concentrations are constants. So, the given equilibrium
expression can be rearranged as
[CaCO 3 ]
Kc × = [CO 2 ]
[CaO]
It can be seen that left hand side of the equation is a constant represented by K c '
x
∴ K c = [CO 2 ] = …(1)
v
PCO
2
Assuming CO 2 gas to behave ideally, the molar concentration of CO 2 , [CO 2 ] can be written as .
RT
PCO
2
∴ K' c =
RT
Since, K' c (RT) = K p .
P-51

xRT
∴ K p = p CO2 = …(2)
V
From equation (1) and (2), it is evident that whenever the equilibrium would be attained at T K, in a vessel of
volume V litre, the moles of CO 2 present at equilibrium must be x.
Any amount of CaCO 3 more than x would be sufficient to establish the equilibrium. If rather than starting
with a mole of CaCO 3 , we start with x moles of CaCO 3 in the vessel of volume V at T K, then the whole of
CaCO 3 would have decomposed to give CO 2 but the equilibrium cannot be maintained as there would be
no CaCO 3 . Moreover, if the moles of CaCO 3 taken in the same vessel at same temperature are less than x,
the equilibrium would never be attained. The given equilibrium can be made to move in the forward
direction by either removing some moles of CO 2 or increasing the volume of the container or by increasing
the temperature of the reaction as the reaction is endothermic. Addition of solid CaCO 3 or CaO to the
equilibrium mixture would not affect the equilibrium situation.
The variation of K p with temperature is given by the relation.
(K p)T2 ∆H  1 1
log = −
(K p)T1 2.303R  T1 T2 

Example 6: For the reaction


NH 2COONH 4 (s) L 2NH 3 (g) + CO 2(g)
The equilibrium constant K P = 2.9 × 10 −5
atm 3 . The total pressure of gases at equilibrium when
1.0 mole of reactant was heated will be:
(a) 0.0194 atm (b) 0.0388 atm (c) 0.0580 atm (d) 0.0667 atm.
Solution: NH 2 COONH 4 (s) L 2NH 3 (g) + CO 2 (g)
Initially 1 0 0
At equi. (1– x) 2x x
Total moles of gaseous substances at equilibrium = 3x
Equilibrium pressure = p
2
PNH3 = P, PCO2 = P
3 3
Kp = (PNH3 )2 p CO
2
−54 3
2.9 × 10 = P
27
p = 0.0580 atm
Ans: (c)
Example 7: Dissociation pressure of CO 2 at 500°C is 0.773 mm of Hg. At this temperature, ∆H
of the reaction is 43.2 kcal mol −1. Calculate the dissociation pressure at 600°C, assuming that
∆H remains constant in this temperature range. The equilibrium reaction is
CaCO 3 (s) L CaO(s) + CO 2(g)
(K p)T2 ∆H  1 1
Solution: log = −
(K p)T 1 2.303R  T1 T2 

For the given equilibrium, K p = p co2


(p co2 )T 2 ∆H 1 1
∴ log = T − T 
(p co2 )T1 2.303R  1 2
P-52

(p co2 )873 43200  100 


log =
0773
. 2. 303 × 1.98 773 × 873
∴ p co2 at 873 k=19.6 mm of Hg
Some other examples of Heterogeneous Equilibria
(a) For the decomposition of solid NH 4 HS into gaseous NH 3 and H 2 S.
NH 4 HS(s) L NH 3 (g) + H 2 S(g)
K c = [NH 3 ] × [H 2 S] and,
Kp =p NH3 × p H2S
(b) For the decomposition of silver oxide, Ag 2 O(s).
2Ag 2 O(s) L 4Ag(s) + O 2 (g)
K c = [O 2 ]
K p = p o2
Example 8: The mass ratio of steam and hydrogen is found to be 1 : 2 at equilibrium. Find the
value of equilibrium constant for reaction
3Fe(s) + 4H 2O(g) l Fe 3 O 4 (s) + 4H 2(g).
Solution: 3Fe(s) + 4H 2 O(g) L Fe 3 O 4 (s) + 4H 2 (g)
[H2 ]4
Kc = Given WSteam : WH2 = 1 : 2
[H2O]4
Let the mass of steam be x g, then the mass of hydrogen would be 2x g.
nH O x×2 1
2
Therefore molar ratio is = =
nH2 18 × 2x 18
4
 nH2 
 
 V 
∴ Kc = where V is the volume of the vessel
4
 nH2 O 
 
 V 
= (18) 4 = 1.05 × 10 5
Example 9: In a mixture of N 2 and H 2 in a ratio of 1:3 at 30 atm and 300°C, the percentage of
ammonia under the equilibrium is 17.8. Calculate the equilibrium constant (K P ) of the mixture,
for the reaction
N 2(g) + 3H 2(g) l 2NH 3 (g).
Solution: Let the initial number of moles of N 2 and H 2 be 1 and 3 respectively. (This assumption is valid as
K P will not depend on the exact number of moles of N 2 and H 2 taken initially. One can start even with x and
3x).
N 2 (g) + 3H 2 (g) l 2NH 3 (g)
Initially 1 3 0
At equi. 1− x 3 − 3x 2x
Since % by volume of a gas is same as % by mole.
P-53

2x
∴ = 0.178
4 − 2x
x = 0.302
3 − 3x
∴ Mole fraction of H 2 at equilibrium = = 0.6165
4 − 2x
Mole fraction of N 2 at equilibrium = 1 − 0.6165 − 0178
. = 0.2055
(X NH3 × PT )2 (0178
. × 30)2
∴ Kp = = = 7.31 × 10 −4 atm −2
(X N2 × PT)(X H2 × PT )3 (0 . 2055 × 30)(0.6165 × 30)3
Alternatively,
N 2 (g) + 3H 2 (g) l 2NH 3 (g)
Initially x 3x 0
At equi. x−a 3x − 3a 2a
2a
∴ = 0.178
4x − 2a
2a / x
∴ = 0178
.
4x − 2a
a
∴ =0.302
x
Similarly, We can calculate the mole fraction of N2 (g) and H2 (g) at equilibrium.
Example 10: At temperature, T, a compound AB 2(g) dissociates according to the reaction
2AB 2(g) l 2AB(g) + B 2(g)
with a degree of dissociation x, which is small compared with unity.
Deduce the expression for x in terms of the equilibrium constant, K p and the total
pressure, P.
Solution: Let the initial pressure of AB 2 (g) be P 1
2AB 2 (g) l 2AB(g) + B 2 (g)
P 1(1−x) P 1x P1x
2
 x
Total pressure = P 11 +  ≈ P 1 (Q x <<1)
 2
∴ P1 = P
(p x)2 × px / 2 P x3
So, KP = ≈
[P(1- −x)]2 2

2K p
∴ x =3
p
Example 11: One mole of N 2O(g) is kept in a closed container along with gold catalyst at 450 K
under one atmosphere. It is heated to 900 K when it dissociates to N 2(g) and O 2(g) giving an
equilibrium pressure of 2.4 atm. The degree of dissociation of N 2O(g) is:
(a) 20% (b) 40% (c) 50% (d) 60%
P-54

Solution: N 2 O(g) L N 2 (g) + ½O 2 (g)


α
P 0 (1− α) P 0α P0
2
When temperature of N 2 O is doubled, its pressure also doubles.
∴ Pressure of N 2 O at 900K, P 0 = 2 atm
Total pressure at equilibrium = 2.4atm
α
P 0 (1− α) + P 0 α + P 0 = 2.4
2
α
P0 1 +  = 2.4
 2
α = 0.4
∴ Degree of dissociation of N 2 O(g) = 40%
Ans: (b)
Example 12: For the reaction: 2HI(g) l H 2(g) + I 2(g); the degree of dissociation (α) of HI(g)
is related to equilibrium constant Kp by the expression:
1 + 2 Kp 1 + 2K p 2K p 2 Kp
(a) (b) (c) (d)
2 2 1 + 2Kp 1+ 2 K p

Solution : 2HI(g) l H 2 (g) + I 2 (g)


α α
1 −α
2 2
2
α 
 P
2 
KP =
(1 − α)2 P2
α
= 2 Kp
1−α
2 Kp
α=
1+ 2 Kp
Ans: (d)
Example 13: At a certain temperature the following equilibrium is established.
CO(g) + NO 2(g) l CO 2(g) + NO(g)
One mole of each of the four gases is mixed in one litre container and the reaction is allowed to
reach equilibrium state. When excess of baryta water is added to the equilibrium mixture, the
weight of white precipitate obtained is 236.4 g. The equilibrium constant K c of the reaction is:
(a) 1.2 (b) 2.25 (c) 2.1 (d) 3.6
Solution: CO(g) + NO2 (g) l CO2 (g) + NO(g)
initially 1 1 1 1
at equi. 1−x 1− x 1+x 1+x
CO 2 + Ba(OH) 2 −→ BaCO 3 + H 2 O
236.4
Moles of BaCO 3 = = 1.2
197
P-55

∴ Moles of CO 2 at equilibrium = 1.2


or, 1 + x = 1.2 ; x = 0.2
2 2
1 + x  1.2 
∴ K c=   =   = 2.25
1 − x   0.8
Ans: (b)
2.2.3 Simultaneous Equilibrium
If more than one equilibria has been established in a vessel and any reactant(s) or product(s) is (are) common
to both the equilibria then the equilibrium concentration of the substance(s) in both the equilibria will be
same. For example,
A + B l C
At equi. a−x a−x−y x
B + Dl E
At equi. a−x−y b−y y
As reactant B is common to both the equilibrium, therefore the equilibrium concentration of B should be
same for both the equilibria. Such cases of equilibria are known as simultaneous equilibria
Example 14: In a vessel two equilibria are simultaneously established as follows
N 2(g) + 3H 2(g) l 2NH 3 (g)
N 2(g) + 2H 2(g) l N 2H 4 (g)
Initially the vessel contains N 2 and H 2 in molar ratio 9 : 13. The equilibrium pressure is 7P 0 in
which due to ammonia the pressure is P 0 and due to hydrogen pressure is 2P 0 . Find thes values
of equilibrium constants for both the reactions.
Solution: Let the initial pressure of N 2 and H 2 is 9P and 13P respectively.
N 2 (g) + 3H 2 (g) l 2NH 3 (g)
9P − x − y 13P−3x−2y 2x
N 2 (g) + 2H 2 (g) l N 2 H 4 (g)
9P − x − y 13P−3x−2y y
Pressure due to NH 3 = 2x = P 0
P0
x= …(1)
2
Pressure due to H 2 = 13P − 3x − 2y = 2P 0
3
13P − P 0 − 2y = 2P 0
2
7
13P − 2y = P 0 …(2)
2
Total pressure at equilibrium
9P − x − y + 13P − 3x − 2y + 2x + y = 7P 0
P
9P − 0 + 2P 0 + P 0 = 7P 0
2
[Putting the value from equation (1) and (2)]
5
9P = 7P 0 − P 0
2
P-56

9
9P = P0
2
P
P= 0
2
[Putting the value in equation (2)]
13 P0 7P
− 2y = 0
2 2
3
y = P0
2
(2x )2
K P1 =
(9P – x – y )(13P – 3x – 2y )3
2
 P0 
2 × 
 2 P02
= =
 9 P0 P0 3 P0  3  5 P0  3
 − −  (2P0 )  P
 2 2 2   2  0
1
K P1 =
20 P02
(y)
K p2 =
(9P – x – y )(13P – 3 x – 2 y )2
(y)
K P2 =
(9P – x − y)(13P − 3x − 2y)2
 3P0 
 
 2 
=
 5P0  2
  (2P )
 2  0
3
K P2 =
20P02

2.3 Le-Chatelier's Principle


This principle is based on the fundamentals of a chemical reactions in equilibrium. Whenever a chemical
equilibrium is disturbed, it shifts in a direction to nullify the effect of change produced so that the same value
of equilibrium constant is maintained. It states that “a chemical reaction at equilibrium is a stable
equilibrium and always reverts back to original equilibrium state when disturbed from this
state by any external factor such as, concentration, pressure, temperature etc”.
Le Chatelier's Principle can be explained with the help of following illustration. An equilibrium of
decomposition of ammonia is considered. The reaction takes place as,
2NH3 (g) l N2 (g) + 3H2 (g)
Let the moles of N 2 , H 2 and NH 3 at equilibrium be a, b and c moles respectively. Since the reaction is at
equilibrium,
P-57

PN2 × (PH2 )3
= KP
(PNH3 )2
3
 a  b 
 × PT   × PT 
a + b + c  a + b + c 
= KP
2
 c 
 × PT 
a + b + c 
Here P T is the total pressure of the system.
ab 3 (PT )2
× = KP
c2 (a + b + c)2
(a + b + c)RT
Since, PT = (assuming all gases to be ideal)
V
PT RT
=
(a + b + c) V
2
ab 3  RT 
∴ ×  = KP …(1)
c2  V

Now, let us examine the effect of change in certain parameters such as number of moles, pressure,
temperature, addition of inert gas etc.
1. Effect of Concentration: If we increase a or b, the left hand side expression becomes Q p ( as it is
disturbed from equilibrium) and we can see that Q P > K P . The reaction therefore moves backward to
make Q P = K P . If we increase c, Q P < K P and the reaction has to move forward to revert back to
equilibrium.
Thus generalization about effect of change of concentration of reactants or products is:
(i) When more reactant is added to an equilibrium mixture, the net reaction occurs in the forward
direction (as Q < K) to give a new equilibrium, and more products are produced.
(ii) When more product is added to an equilibrium mixture, the net reaction moves in the reverse
direction (as Q > K) to give a new equilibrium, and more reactants are produced.
2. Effect of Pressure: If the volume of the container is increased which amounts to decreasing the
pressure, Q P < K P as suggested by equation (1). The reaction moves forward to attain equilibrium.
Thus, it can be generalized that If the pressure is decreased by increasing the volume of a reaction
mixture, the reaction shifts in the direction of more moles of gas similarly if the pressure is increased by
decreasing the volume of a reaction mixture, the reaction shifts in the direction of fewer moles of gas.
3. Effect of Temperature: An increase in temperature is the result of the flow of heat into the system.
Conversely a flow of heat out of the system reduces the temperature. A reaction itself can be a source or
sink for heat. The heat flow for a reaction is characterized by the enthalpy of reaction or the heat of
reaction, ∆H reaction . If ∆H reaction > 0, the reaction is said to be endothermic, this means that the
reaction draws heat from its surroundings. If ∆H reaction < 0, the reaction is said to be exothermic, this
means that the reaction releases heat into its surroundings as the reaction occurs.
Suppose an exothermic reaction is at equilibrium, and then the temperature of the system is increased.
The increase in temperature corresponds to introducing heat into the system. This influx of heat pushes
the system away from equilibrium and nature restores equilibrium by removing some of this additional
P-58

heat. Because the reaction is exothermic, the reaction produces heat when it proceeds in the forward
direction. Le- Châtelier’s Principle states that the system will react to remove the added heat, thus the
reaction must proceed in the reverse direction. Conversely, if the temperature of the system were
decreased (heat removed from the system), the system would react in a direction that opposed the
removal of the heat. The forward reaction would thus occur to release heat in an attempt to offset the
heat that was removed from the system.
Bear in mind that the system is being held at a specific temperature, so the net reaction which either
produce or release heat does not actually change the temperature of the system. That extra heat is
always removed or replaced as necessary to maintain the temperature. Thus the effect of change of
temperature can be generalized as:
(i) If ∆H is +ve (endothermic) an increase in temperature (T 2 > T 1) will make K 2 > K 1. So, it
means that the reaction goes more towards the forward direction and vice-versa.
(ii) If ∆H is –ve (exothermic) an increase in temperature (T 2 > T 1) implies K 2 < K 1. So,
automatically the reaction goes in the reverse direction.
4. Effect of Addition of Inert Gas: Inert gas is introduced to the system first at constant volume and as
a secand case at constant pressure.
(i) Addition of inert gas at constant volume: Since V remains fixed, a, b and c also remains
fixed and RT is any way a constant, thus one can see that Q P = K P . Therefore, the reaction does
not move at all. Thus, generalization is when the addition of inert gas is carried out at constant
volume, the equilibrium remains unaffected for reactions whether it does not even depend
whether these reactions have ∆n = 0 or ∆n ≠ 0.
(ii) Addition of inert gas at constant pressure: If one adds the inert gas at constant pressure it
means there is an increase in the volume of vessel therefore Q P < K P . As a result, the reaction
move forward to attain equilibrium. Thus, it can be generalized that:
(a) The addition of an inert gas at constant pressure to an equilibrium, whose ∆n ≠ 0, the
equilibrium shifts in the direction of greater number of moles.
(b) When ∆n = 0, there will be no effect of addition of inert gas at constant pressure on chemical
equilibrium reaction as V would not exist in the expression of K p.

2.3.1 Application of Le-Chatelier Principle to Physical Equilibria


Effect of Pressure on Boiling Point: When pressure is raised then condensation of vapour into liquid take
place, thus vapour pressure will decrease. Now more heat is required to equate vapour pressure with
atmospheric pressure, hence boiling point will increase.
Effect of Pressure on Melting Points
(a) When volume of solid decreases on melting or in other words in liquid state volume is lesser, then by
increasing pressure on such solids the melting point will go down, because at high pressure melting is
facilitated It is observed in the cases of ice, diamond, carborundum (SiC) etc.
Solid L Liquid
(b) When volume increases on melting then by increasing pressure, the equilibria will shift in the reverse
direction on increasing the pressure. In such cases, process of melting will be inhibited and more heat is
required thus melting point will rise. Examples are Iron, Copper, NH 4 Cl, NaCl etc.
Solid L Liquid
P-59

Example 15: What is the effect on the following equilibrium if each of the indicated stresses is
applied?
½ N2 + O2 L NO 2 + heat
(a) Increase in N 2 concentration (b) Decrease in temperature
(c) Increase in volume of vessel (d) Decrease in O 2 concentration
(e) Addition of a catalyst
Solution: The equilibrium will shift to the
(a) right (b) right (c) left (d) left
(e) no shift will occur As catalyst lowers the activation energy of forward and reverse reacion by same
amount, thus increasing the forward and reverse rate equally and hence, no forward or backward shift is
observed. The only observable behaviours is that the equilibrium is attained at a much faster rate.
Example 16: ½N 2 + ½O 2 L NO, ∆H = 179.50 kJ. State the effect upon the reaction
equilibrium of:
(a) Increased temperature (b) Decreased pressure
(c) Higher concentration of O 2 (d) Lower concentration of N 2
(e) Higher concentration of NO (f) Presence of a catalyst
Solution:
(a) The reaction is endothermic, so adding heat to raise the temperature favors the forward reaction.
(b) Since there are same number of moles of gases on the two sides of the equation, neither is favored.
(c) Forward reaction is favored by higher concentration of reactant.
(d) Reverse reaction is favored by lower concentration of reactant.
(e) Reverse reaction is favored by higher concentration of product.
(f) No change, as catalyst does not affect the position of the equilibrium.
Example 17: For the gaseous equilibrium at high temperature,
PCl 5(g) l PCl 3 (g) + Cl 2(g), ∆H = 87.9 kJ
explain the effect upon the material distribution of (a) increased temperature (b) increased
pressure (c) higher concentration of Cl 2 (d) higher concentration of PCl 5 (e) presence of a
catalyst.
Solution:
(a) When the temperature of a system in equilibrium is raised (by addition of heat), the equilibrium is
displaced in the direction which absorbs heat. Hence increasing the temperature will cause more PCl 5
to dissociate.
(b) When the pressure of a system in equilibrium is increased, the equilibrium is displaced in the direction
of the smaller volume. Hence a pressure increase will promote the reaction to form more PCl 5 .
(c) Increasing the concentration of any component will displace the equilibrium in the direction which
tends to lower the concentration of the component added. Increasing the concentration of Cl 2 will
result in the consumption of more PCl 3 and the formation of more PCl 5 and this action will tend to
offset the increased concentration of Cl 2 .
(d) Increasing the concentration of PCl 5 will result in the formation of more PCl 3 and Cl 2 .
(e) A catalyst accelerates both forward and backward reactions equally. It speeds up the approach to
equilibrium but does not favor reaction in either direction.
Example 18: Predict the effect of decreasing the volume of the system on each of the
following equilibria.
P-60

(a) Heat + MgCO 3 (s)l MgO(s) + CO 2(g)


(b) 2C(s) + O 2(g)l 2CO(g) + heat
Solution: Decreasing the volume, increases the partial pressure of the gases. Thus the equilibria will shift in
the direction where the number of moles of gases decreases.
(a) Equilibrium shifts to the left (b) Equilibrium shifts to the left.

2.3.2 Relation Between Degree of Dissociation and Vapour Density


Degree of dissociation may be defined as the fraction of a mole of the reactant undergoing dissociation. It is
usually denoted by ' α'
number of moles dissociated
α=
number of moles present initially
For example, let us consider the decomposition of SO 3 is considered. Let the initial number moles of SO 3 be
‘a’ and the moles of SO 3 dissociated at equilibrium be ‘x’.
2SO 3 (g) L 2SO 2 (g) + 1/2O 2 (g)
Initial moles a 0 0
At equilibrium a–x x x/2
Now if x moles dissociate from a moles of SO 3 . then degree of dissociation of SO 3
2x '
would be x/a. If the moles dissociated would have been 2x, then α =
a
2SO 3 (g) L 2SO 2 (g) + 1/2O 2 (g)
Initially a 0 0
At equilibrium a–2x 2x x
Let us now derive the relation between vapour density and the degree of dissociation. As it is known that,V.D.
= M. mass/2, and molecular mass is inversely proportional to number of moles. So,
1
∴ V. D. ∝
n
Total moles at equilibrium initial vapour density
∴ =
Initial number of moles Vapour density at equilibrium
Let us consider a general equilibrium reaction,
A(g) l nB(g)
Initial moles c 0
Equilibrium number of moles c(1– α) cnα
Total moles at equilibrium = c – c α + c n α = c[1 + α(n−1)]
Let the initial vapour density and the vapour density at equilibrium be D and d respectively.
c[1 + α(n − 1)] D
∴ =
c d
D
∴ 1 + α(n–1) =
d
(D − d)
∴ α=
(n − 1)d
Knowing the values of D and d, α can be calculated.
For example,
SO 3 (g) l SO 2 (g) + ½O 2 (g)
P-61

Initially c 0 0

At equilibrium c – cα cα
2
 α
Total moles at equilibrium = c 1 + 
 2
 α
c 1 + 
 2 D
∴ = =
c d
2(D − d )
∴ α=
d
This method is valid only for those equilibrium reactions whose K P exists and ∆n ≠ 0.
Example 19: The vapour density of PCl 5 at 250°C is found to be 57.9. Calculate percentage
dissociation at this temperature.
Solution: Degree of dissociation may be related with vapour density as
D−d
α=
(n − 1)d
For, PCl 5 (g) L PCl 3 (g) + Cl 2 (g)
208. 5
where, D = Initially vapour density = = 104.25
2
d = Vapour density at equilibrium = 57.9
n=2
104. 25 − 57.9
∴ α= = 0.8
57.9(2 − 1)
% dissociation = 0.80 × 100 = 80 %

2.4 Free Energy Change and Spontaneity of a Reaction


Every reaction proceeds with a decrease in free energy. The free energy change in a process is expressed by
∆G.
If ∆G = –ve,reaction goes in the forward direction
∆G = +ve, reaction goes in the backward direction
∆G = 0, reaction is at equilibrium
Free energy, G denotes the self intrinsic electrostatic potential energy of a system. It tells us about the stability
of a molecule with respect to another molecule. Lesser the free energy of a molecule more stable it is. As
stated earlier, ∆G is the free energy change at any given concentration of reactants and products. If all the
reactants and products are taken at a concentration of one mol per litre, the free energy change of the
reaction is becomes ∆G° or standard free energy change.
° °

∆G° = ∑
∆G f of products – ∆G f of reactants

and ∆G =∑ ∆ G f of products –∑ ∆G f of reactants

2.4.1 Relation Between ∆G° and K


∆G° = –RT ln K
P-62

T is always taken in Kelvin, and if R is in joules, ∆G° will be in joules, and if R is calories then ∆G° will be in
calories. K may either be K c or K P or any other equilibrium constant.

2.4.2 Clapeyron-Clausius Equation


An equation of fundamental importance which finds extensive application in phase equilibrium was derived
by Clapeyron, and independently by Clausius. It is derived from the second law of thermodynamics and is,
generally, known as the Clapeyron-Clausius equation. The two phases in equilibrium are any of the following
types:
(i) Solid and Liquid, S L L, at the melting point of the solid.
(ii) Liquid and Vapour, L L V, at the boiling point of the liquid.
(iii) Solid and Vapour, S L V, at the sublimation temperature of the solid.
(iv) One crystalline form and another Crystalline form as, for example, rhombic and monoclinic sulphur,
SR L SM , at the transition temperature of the two allotropic forms.
In general, any two phases of a substance are considered which are in equilibrium with each other at a given
temperature and pressure. It is possible to transfer any definite amount of the substance from one phase to
the other in a very slow manner such that thermodynamic reversibility is maintained. For example, by
supplying heat infinitesimally slowly to the system, it is possible to change any desired amount of the
substance from the liquid to the vapour phase at the same temperature and pressure. Similarly, by
withdrawing heat very slowly from the system, it is possible to change any desired amount of the substance
from the vapour to the liquid phase without changing the temperature and pressure. Since the system is in a
state of equilibrium, the free energy change of either process will be zero. One may conclude, therefore, that
equal amounts of a given substance must have exactly the same free energy in the two phases at equilibrium
with each other.
In general, the change of a pure substance from phase A to another phase B in equilibrium is considered. If G
A is the free energy per mole of the substance in the initial phase A and G B is the free energy change per mole
in the final phase B, then, since G A = G B , hence, there will be no free energy change,
∆G = G B – G A = 0
If the temperature of such a system is increased, say, from T to T + dT, the pressure will also have to change,
say, from P to P + dP, in order to maintain the equilibrium. The relationship between dT and dP can be
derived from thermodynamics. It is assumed that the free energy of the substance in phase A at the new
temperature and pressure be G A + dG A and that in phase B be G B + dG B. Since the two phases are still in
equilibrium, hence,
G A + dG A = G B + dG B
Now it is known that,
dG = VdP – SdT
Equation for phase A may be written as
dGA = VA dP − S A dT
and for phase B, as dGB = VB dP − S B dT
Since G A = G B, hence, from eq.
dGA = dGB
∴ VA dP − S A dT = VB dP − S B dP
dP S B − S A
or =
dT VB − VA
P-63

It is noted that since V A and V B are the molar volumes of the pure substance in the two phases A and B,
respectively, V B – V A represents the change in volume when one mole of the substance passes from the
initial phase A to the final phase B. It is represented by ∆V. Similarly, S B – S A , being the change in entropy for
the same process, may be put as ∆S. Hence,
dP ∆S
=
dT ∆V
If q is the heat exchanged reversibly per mole of the substance during the phase transformation at
temperature T, then the change of entropy (∆S) in this process is given by ∆S = q/T.
dP q
Hence, =
dT T∆V
dP q
Thus, =
dT T(VB − VA )
This equation, evidently, gives change in pressure dP which must accompany the change in temperature dT
or vice versa, in the case of a system containing two phases of a pure substance in equilibrium with each
other. Some of the simple cases are taken to explain the equation.
(a) An equilibrium like below at the temperature T is considered,
H2O (liquid) v H2O (vapour)
If, q = Molar heat of vaporization, ∆H V , V B = volume of one mole of water in the vapour state, say, Vg ,V A
= volume of one mole of water in the liquid state, say, Ve , then the equation takes the form
dP ∆H v
=
dT T(Vg − Ve)
(b) Another system consists of water at its freezing point. Now, the two phases in equilibrium
are,
H 2 O(S) v H 2 O (liquid)
Clapeyron-Clausius equation is then written as,
dP ∆H f
=
dT T(V1 − Vs)
Here, ∆H f is the molar heat of fusion of ice.
(c) Integrated form of Clapeyron-Clausius equation for liquid v Vapour equilibrium: The
Clapeyron-Clausius equation as applied to liquid v vapour equilibrium, can be easily integrated.
The molar volume of a substance in the vapour state is considerably greater than that in the liquid state.
In the case of water, for example, the value of Vg at 100°C is 18 × 1670 = 30060 ml while that of V 1 is
only a little more than 18 ml. Thus Vg – V 1 can be Considered as Vg without introducing any serious
error. The Clapeyron-Clausius equation, therefore, is written as
dP ∆H v
=
dT TV g

Assuming that the gas law is applicable, so, PV = RT (per mole), Vg = RT/P
dP ∆H v P ∆H V
Hence, = × =P
dT T RT RT2
1 dP ∆H V d(In P) ∆H V
or × = or =
2
P dT RT dT RT2
Assuming that ∆H V remains constant over a small range of temperature, it can be said,
P-64

∆H V dT ∆H V  1 
∫ d (In P)= R ∫ T2 orP = −   +C
R  T
here C is constant of integration. It is mathematical equation of a straight line when ln P is plotted against 1/T
∆H V
with slope =− and intercept C. This enables evaluation of ∆H V .
R
Upon integration,
P2 ∆H V T2 dT
∫P 1
d (In P) =
R ∫T
1 T2
T
P2 ∆H V  1  2 ∆H V 1 1  ∆H V  T2 − T1 
∴ In =− = T − T  = R  T T 
P1 R  T  T R  1 2  1 2 
1

This integrated form of relation is known as Clapeyron-Clausius equation.


2.4.3 Application of Clapeyron-Clausius Equation
Clapeyron clausius equation can be applied to a number of cases. Some of these cases are discussed below,
1. For Liquid l Vapour equilibria: The Clapeyron–Clausius equation can be used for calculating
the molar heat of vaporization, ∆H V , of a liquid, provided the vapour pressures at two temperatures
are known. Further, if ∆H V is known, vapour pressure at a desired temperature can be calculated from
the knowledge of a single value of vapour pressure at a given temperature. It is also used for calculating
the effect of pressure on the boiling point of liquid. Some more application are Possible like
(i) Calculation of molar heat of vaporization, ∆H V : The molar heat of vaporization of a
liquid can be calculated if its vapour pressures at two different temperatures are known.
(ii) Effect of pressure on boiling point: If the boiling point of a liquid at one pressure is known,
that at another pressure can be calculated.
Example 20: Vapour pressures of H2O at 95°C and 100°C are 634 and 760 mm, respectively.
Calculate the molar heat of vaporization, ∆H V , of water between 95°C and 100°C.
Solution: Using Clausius Clapeyron equation,
760 ∆H V  1 1 
log = –
634 2.303 × 8.314  368 373
Solving, ∆H V = 41.363 KJ/mol.
Example 21: The vapour pressure of H2O at 100°C is 760 mm. What will be the vapour
pressure at 95°C ? The heat of vaporization of water in this temperature range is 41.27 kJ per
mole.
Solution: Using the equation.
P2 ∆H V  1 1
log = –
P1 2.303 R  T1 T2 

P2 41.27 × 10 3
⇒ log =
760 2.303 × 8.314
Solving, P2 = 634.3mm
Example 22: Diethyl ether boils at 33.5°C at one atmosphere pressure. At what temperature
will it boil at a pressure of 750 mm, given that the heat of vaporization of ether is 369.86 joules
per gram.
P-65

P2 750 369.86 × 74  1 1
Solution: log = = –
P1 760 8.314 × 2.303 306.5 T2 

Solving, T2 = 305.9 K
2. For Solid l Vapour Equilibria: Clapeyron−Clausius equation for solid l vapour equilibrium
is expressed as,
dP ∆Hs
=
dT T(Vg − Vs )

Here, ∆Hs stands for the molar heat of sublimation of the substance. Since the molar volume of a substance
in the gaseous state is very much greater than that in the solid state, Vg – Vs is taken as Vg. Eq. can thus be
easily integrated, as before, to given the following expression :
P2 ∆Hs  T2 − T1 
In =
P1 R  T1T2 

3. For Solid l Liquid Equilibria: The Clapeyron-Clausius equation for solid l liquid
equilibrium cannot be integrated easily since V S cannot be ignored in comparison with Ve. Also the
laws of liquid state are not so simple as those for gaseous state. However, this equation as such can be
used for calculating the effect of pressure on the melting point of a solid. Apart from it, the equation is
also used for calculating heats of fusion from vapour pressure data obtained at various temperatures.
P-66

Long Answer Type Questions Calculate enthalpy of vaporization per mole of


water. [Lko. 2010]
Based on Basic Terms
1. What is thermodynamics criteria of an equilibrium? 11. Derive Clausius-Clapeyron equation for various
Derive the law of mass action thermodynamically. phase equilibria. [Agra 2008]
[Garhwal 2010] Short Answer Type Questions
2. What is law of mass action? Derive law of mass Based on Basic Terms
action thermodynamically. [Purv. 2008] 1. Define equilibrium for a reaction and derive Law of
3. If K1 and K 2 are equilibrium constants of a chemical equilibrium. [Garhwal 2011]
reaction at temperatures T1 and T2 respectively. 2. Write the equations of Van't Hoff reaction isotherm
Derive thermodynamically the relation: and Van't Hoff reaction isochore. [Garhwal 2007]
K ∆H  1 1  3. Give thermodynamic derivation of law of mass
log 2 =  − 
K1 2.303R  T1 T2  action. [Avadh 2010]
[Lko. 2009]
4. Discuss the effect of temperature on K c and K p in
Based on Le-Chatelier's Principle
the form of vant Hoff equation. [Avadh 2009]
4. Define Le–Chatelier’s principle. Discuss its
application to the formation of ammonia by Haber
process. [Garhwal 2005] Reactant
5. State Le–Chatelier’s principle : Discuss its Energy
application to the following reactions:
Products
(i) N2 + 3H2 → 2NH3 ; ∆H = −22.08 Kcal
(ii) 2SO2 + O2 → 2SO3 ; ∆H = −23.15 Kcal Progress of reaction
[Avadh 2008] 5. Consider the following diagram:
6. Discuss Le-Chatelier's principle application for Does it describe a reaction in which K is very large
equilibrium reaction. Show its effect with change in of K is very small? Explain. [D.D.U. 2009]
concentration, P and T on the equilibrium of a 6. Distinguish between equilibrium and steady state.
reaction on: [D.D.U. 2010]
N2 + 3H2 l 2NH3 , ∆H =-22.8 k.cal. 7. Derive the law of mass action thermodynamically.
[Purv. 2011] [D.D.U, 2011]
7. State and explain Le-Chatelier's principle. 8. Give relationship between K p and K c for chemical
[Lko. 2011] equilibrium in a gaseous reaction. [Purv. 2011]

Based on Free Energy Change and Spontaneity of a 9. What happens to the concentration of the different
reactants and products when equilibrium is
Reaction
attained? Give reason. [Lko. 2011]
8. Define law of chemical equilibrium. Derive the
10. Establish relation between K c , K p and K x . Under
relationship .
what condition all of them are equal? [Lko. 2009]
d(InK P ) ∆H °
= 11. Give thermodynamic derivation of law of mass
dT RT2 [Avadh 2009] action. [Kanpur 2008]
9. Describe the Clausius-Clapeyron equation and show 12. What are reversible and irreversible processes?
its integrated form can be written as: Describe. [Kanpur 2009]
P ∆H  T2 − T1  13. Derive Law of Mass Action thermodynamically.
log 2 =  
P1 2.303R  T1T2  [Purv. 2009] [Kanpur 2010, 12]
Based on Equilibrium constant for Different types
10. The normal boiling point of water is 100°C. Its of Equilibria
vapour pressure at 90°C is 0.705 atmosphere. 14. Discuss the law of chemical equilibrium.
[Garhwal 2010]
P-67

15. How the decomposition of CaCO 3 would be 26. The Value of K p for the reaction: N 2 O4 l
different if it is carried out in an open vessel? 2NO2 at 25°C is 0.14. What is the value of ∆G° ?
[Garhwal 2008] [Lko. 2009]
16. Give an expression for equilibrium constant (K P ) 27. For gaseous reaction derive the following relationship:
for the reaction. ∆G ° = −RT ln Kp [Kanpur 2012]

NO2 (g) + SO2 (g) a NO(g) + SO 3 (g)


[Purv. 2010] Objective Type Questions
17. Explain reversible heterogeneous reaction with
example. [Kanpur 2012] Multiple Choice Questions
18. The concentration equilibrium constant (K c ) for 1. The variation of equilibrium constant with
the reaction N2 + 3H2 g 2NH3 - at 400° C is temperature is given by :
0.5. Find the value of K p. [D.D.U. 2010] (a) Van’t Hoff isotherm
Based on Le-Chatelier's Principle (b) Van’t Hoff isochore
19. Discuss the effect of temperature and pressure (c) Law of Mass Action
according to Le–Chatelier’s principle on the (d) Le–chatelier’s principle [Garhwal 2009]
following reactions :
2. Which is correct relationship?
(i) N 2 (g) + 3H 2 (g) x 2NH 3 (g),
∆H = −99.38 kJ (a) K p = K c (RT) (b) K p= K c (RT)−∆n

(ii) N 2 (g) + O2 (g) x 2NO3 (g), (c) K p / K c =(RT) ∆n (d) K p=RT(K c ) ∆n


∆H = +180.75 kJ [Garhwal 2012] [Garhwal 2009]
20. Explain the effect of temperature on equilibrium 3. A Chemical reaction is at equilibrium when the rate
state through Le-Chatelier’s principle. of forward and backward reactions are:
[Garhwal 2010]
(a) Unequal (b) Constant
21. Write notes on the following :Le-Chatelier’s
(c) Equal (d) Increase
principle. [Garhwal 2009]
[Garhwal 2010]
Can the equilibrium CaCO3 (s)l
22. State Le Chatelier’s principle. Discuss its
4. CaO(s) +
application to the following :
CO2 (g) be attained in an open vessels:
(i) 2HI v H 2 +I 2 , ∆H=x K cal
(a) Yes (b) No
(ii)N 2 +3H 2 v 2NH 3 , ∆H=−22.8 K cal
[Avadh 2010] (c) Can't say (d) None of these
[Agra 2008]
23. Which of the followoing equilibrium are not
5. For which forward reaction, high pressure is
affected by pressure change and why?
(a) PCl5 (g) j
helpful?
PCl3 (g)+ Cl2 (g)
(a) H 2 +I 2 l (b) N 2 +3H 2 l
(b) N2 (g) + O2 (g) j 2NO(g)
2HI 2NH 3
(c) N 2 +O 2 l 2NO (d) PCI 5 l
(c) H2 (g) + I2 (g) j 2HI(g)
PCI 3 +CI 2
[Garhwal 2007, 12]
(d) 2O3 (g) j 3O2 (g) [Lko. 2010] 6. Le–Chatelier's principle is applicable only to :
Based on Free Energy Change and Spontaneity of a (a) Homogeneous reactions
Reaction
(b) Heterogeneous reactions
24. Derive Clapeyron–Clausius equation. Discuss its
applications for the equilibrium liquid x vapour.
(c) Systems in equilibrium
(d) Systems not in equilibrium [Garhwal 2011]
[Garhwal 2010]
25. What are the thermodynamic conditions for a 7. For the manufacture of ammonia gas by the reaction:
reaction to take place, or not to take place, and for N 2 (g) + 3H 2 (g) → 2 NH 3 (g) + 21.9 kcal, we
equilibrium? [Purv. 2009] require:
P-68

(a) Low temperature, low pressure and catalyst 11. The temperature at which the solid and liquid form
(b) Low temperature, high pressure and catalyst of a pure substance are in equilibrium at the
(c) High temperature, high pressure and catalyst atmospheric pressure is called the……… .

(d) High temperature, low pressure and catalyst. 12. The amount of the solid in grams that dissolves in
[Garhwal 2010] 100 g of the solvent to form a saturated solution at

In reaction 2SO2 (g) + O2 (g) l


a particular temperature is called .................... .
8. 2SO3 (g) +
32k. cal. formation of SO 3 (g) is favoured by: 13. For equilibrium state ∆G is ……………. .
[Garhwal 2011]
(a) Low temperature and low pressure
(b) High temperature and high pressure
(c) Low temperature and high pressure
True and False
1. The value of K c = K p (RT) ∆n .
(d) High temperature and low pressure. [Garhwal 2011]
[Garhwal 2008] 2. A chemical reaction is at equilibrium when
9. The effect of concentration, temperature and reactants are completely transformed into
pressure on a system in equilibrium can be products. [Garhwal 2010]
Kp
predicted with the help of: 3. For a reaction N 2 (g)+O2 (g) a 2 NO(g) = 1.
(a) Ostwald's duilution law (b) Charle's law Kc

(c) Arrhenius theory (d) Le Chatelier's Principle 4. Increasing temperature cause a rise in the value of
[Agra 2009] equilibrium constant.
5. K ∆H  1 1
Fill in the Blank(s) log 2 =  –  is a mathematical
K1 2.303 R  T2 T1 
1. If concentration quotient of reaction is less than its
equilibrium constant, then the reaction will representation of Van't Hoff uation.
proceed in the ....................direction. 6. There is no effect of addition if inert gas to the
2. On adding a catalyst to a reaction, the equilibrium equilibrium PCl5 (g) a PCl3 (g)+Cl2 (g) at
constant .................... . constant pressure.
3. Equilibrium constant of a reaction does not change 7. Clapeyron–Clausius equation is used for determining
the molecular conductivity of an electrolyte.
with .................... but changes with .................... .
[Garhwal 2005,12]
4. According to law of mass action, the rate at which a
8. ∆G ° = −RT ln Q
substance reacts is proportional to its ....................
5. In terms of rate constant for forward and backward
reactions, equilibrium constant of a reaction is
equal to……… .
6. The equilibrium constant for the reaction 2A +2B
l 2C+2D is 100. The equilibrium constant for

the reaction A+B a C+D at the same


temperature is ………………… . [Garhwal 2012]
7. The rate of reaction of a substance is
………………. to its active mass. [Garhwal2007]
8. Molar concentration of pure solid or a pure liquid is
taken as……...… .
9. If pressure is applied on the ice ⇔ water
equilibrium, more of .................... will be formed.
10. Exothermic reactions are favoured by………
temperature.
P-69

Objective Type Questions


Multiple Choice Questions
1. (a) 2. (c) 3. (c) 4. (b) 5. (b)
6. (c) 7. (b) 8. (c) 9. (c)

Fill in the Blank(s)


1. forward 2. remains unchanged 3. initial concentration of reactants, change
in temperature
4. active mass or molar 5. K f /K b 6. 10
concentration
7. directly proportional 8. constant 9. water
10. low 11. melting point 12. saturated solution
13. 0

True and False


1. True 2. False 3. True 4. False
5. False 6. False 7. False 8. False

Hints & Solutions


Long Answer Type Questions
VP2 ∆H Vap  1 1 
10. Using, log =  –  (Since VP at boiling point is 1 atm)
VP1 2.303 R  T1 T2 
1 ∆H Vap  1 1 
⇒ log = – .
0.705 2.303 × 2  363 373 
⇒ ∆H Vap = 9.459K cal / mol.

Short Answer Type Questions


5. K is very low.
18. K p =K c(RT)∆n

⇒ K p = 0.5(0.0821 × 673)–2
= 0.00016
23. (b) and (c) are not affected by pressure because for these reactions ∆H=0.
26. ∆G° = –2.303(RT) log 10 K p.
. .
= –2.303 × 8.314 × 298 × log 10 014
= + 4.872 KJ/mol.

True and False


8. ∆G ° = −RT ln K
❍❍❍
P-70 Unit-II

C HAPTER 3
Themodynamics-II

3.1 Introduction
The study of thermodynamics offers the opportunity to predict the stability of chemical substances and the
extent and direction of chemical reactions. However, it is always important to note that thermodynamics
does not deal with the rate or speed of a chemical reaction. Thermodynamics predicts the spontancity of a
chemical reaction, with the help of a number of parametres like ∆S total or ∆Gsys etc. In this chapter, the
domain of discussion will include these parameters and some more.

3.2 Carnot Cycle


The Carnot cycle is a theoretical thermodynamic cycle proposed by Carnot in 1823 and later expanded by
various scientists. It can be shown that it is the most efficient cycle for converting a given amount of thermal
energy into work, or conversely, creating a temperature difference by doing a given amount of work.
Example of creating a temperature difference is refrigeration. Every single thermodynamic system exists in a
particular state. When a system is taken through a series
of different states and finally returned to its initial state, a
thermodynamic cycle is said to have occurred. In the
process of going through this cycle, the system may
perform work on its surroundings, thereby acting as
a heat engine. A system undergoing a Carnot cycle is
called a Carnot heat engine, although such a perfect
engine is only a theoretical limit and cannot be built in
practice. The Carnot cycle when acting as a heat engine
consists of the following steps as shown in the following
pressure -volume graph:
It is consisted of the following steps: Fig. 1: Carnot cycle illustrated on a PV diagram

1. Reversible isothermal expansion of the gas at


the higher temperature, T2: During this step 1 to 2 on Fig.1, the gas is allowed to expand and it does
work on the surroundings. The temperature of the gas does not change during the process, and thus
the expansion is isothermal. The gas expansion is propelled by absorption of heat energy q 2 . The state
of the gas is changing from P 1V 1 to P 2 V 2 shown by points A and B respectively. As the process is an
isothermal expansion, ∆U = 0. Now, from the first law of thermodynamics, ∆U = q+w ; q = – w. So,
P-71

the total heat absorbed is equal to the work done by the system. As q 2 is the heat absorbed by the
system at temperature at temperature T2 and w 1be the work done by the system, then
q 2 = – w 1 = RT2 ln(V2 / V1)
2. Reversible adiabatic expansion of the gas : In this step. which is shown as 2 → 3, the system is
assumed to be thermally insulated due to adiabatic nature, thus the system neither gains nor loses heat.
The gas continues to expand, doing work on the surroundings, and losing an equivalent amount of
internal energy. The gas expansion causes it to cool from T 2 to the lower temperature, T 1. The state of
the gas is changing from P 2 V 2 to P 3 V 3 shown by points B and C respectively.
Since work is being done adiabatically, q = 0. From the first law of thermodynamics, one can say that,
∆U = –w; Negative sign shows that the work is done by the ges. Using Kirchhoff’s law,
∆U = C V ∆T = C V (T1 − T2 ) = −w
If the work done in this stage is w 2 ; then
−w 2 = C V (T1 − T2 )
3. Reversible isothermal compression of the gas: In the graph, it is shown by transition 3 to 4. As
the gas is subjected to a reversible isothermal compression at the lower temperature T1, the work is
done on the gas. The volume of the gas decreases from V 3 to V 4 . The state of the gas is changing from
P 3 V 3 to P 4 V 4 . This time heat is produced by the system and given out to the surroundings. As the
compression is isothermal, ∆U = 0. If q 1 is the heat given out to the surroundings at temperature T 1
and w 3 is the work done on the system, then,
− q1 = w 3 = RT1 ln(V4 / V3 )
4. Reversible adiabatic compression of the gas : In the graph, it is shown by transition 4 to 1.
During this step, the surroundings do work on the gas, increasing its internal energy. The state of the
gas is changing from P 4 V 4 to P 1V 1 shown by points D and A respectively. As it is a compression
process, work is done on the system, so positive. Using Kirchhoff’s law,
∆U = C V ∆T = C V (T 2 –T 1) = –w
If the work done in this stage is w 4 then,
w 4 = C V (T 2 –T 1)
The net heat absorbed is given by q 2 + (− q1 ) = RT2 ln(V2 / V1) + RT1 ln(V4 / V3 )
= RT2 ln(V2 / V1) − RT1 ln(V3 / V4 )
As, it is known that,
CV ln(T2 / T1) = R ln(V3 / V2 ) for stage 2
CV ln(T2 / T1) = R ln(V4 / V1) for stage 4
or V3 / V2 = V4 / V1
V2 / V1 = V3 / V4
Hence, the net heat absorbed is given by q 2 − q1 = R(T2 − T1) ln(V2 / V1).
Similarly, the net work done by the gas is,
w = –w1 − w 2 + w 3 + w 4
= RT2 ln(V2 / V1) + CV (T1 − T2 ) + RT1 ln(V4 / V3 ) + CV (T2 − T1)
RT2 ln(V2 / V1) − RT1 ln(V3 / V4 )
P-72

Since, V2 / V1 = V3 / V4
Hence, w = R(T2 − T1) ln(V2 / V1).
Since, it is a cyclic process, q = w. The relationship between the net work done by the system, w and q 2
can be obtained as below,
w = R(T2 − T1) ln(V2 / V1).
q 2 = RT2 ln(V2 / V1),
hence, w = q 2 (T2 − T1 / T2 )
w / q 2 = (T2 − T1) / T 2

As T2 − T1 is less than T2 it means even w will be lesser than q 2 . As the work done is less than the heat
absorbed,it means only a part of heat absorbed is used in the work done. The rest part is given out to
the surroundings in the form of q1 .

3.2.1 Efficiency of Carnot Cycle


The efficiency η is defined as the fraction of heat absorbed which is converted into work. So,
η = w / q 2 = (T2 − T1) / T2
As suggested by the mathematical expression, η is always less than 1. No heat engine has yet been
constructed which has an efficiency equal to 1. It follows from the equation that greater is the difference of the
two temperatures more will be the efficiency of the Carnot cycle. This explains why superheated steam is used
in a steam engine. Moreover, the net heat absorbed by the system is q, and it is given by, q = q 2 − q1 .
Combining this, with the efficiency expression,
(q 2 – q 1 )/ q 2 = (T 2 – T 1)/ T 2
So, η = (q 2 – q 1 )/ q 2 = w/q 2 = (T 2 – T 1) / T 2

3.2.2 Carnot Theorem


The theorem was developed in 1824 by Sadi Carnot, also called Carnot’s rule is a principle that specifies
limits on the maximum efficiency any heat engine can obtain, which thus solely depends on the difference
between the hot and cold temperature reservoirs. Carnot’s theorem primarily deals with all heat engines.
According to him, every Carnot engine between a pair of heat reservoirs is equally efficient, regardless of the
working substance employed.
Carnot theorem states that no heat engine working in a cycle between two constant
temperature reservoirs can be more efficient than a reversible engine working between the
same reservoirs. In other words, it means that all the engines which operate between a given
constant temperature source and a given constant temperature sink, does not have a higher efficiency than
a reversible engine.
Proof: Suppose there are two engines E A and E B operating between the given source at temperatureT1 and
the given sink at temperature T2 . Let E A be any irreversible heat engine and E B be any reversible heat
engine. It has to be proven that efficiency of heat engine E B is more than that of heat engine E A . Suppose
both the heat engines receive same quantity of heat Q from the source at temperatureT1. Let W A and W B be
the work output from the engines and their corresponding heat rejections be (Q – W A ) and (Q – W B)
respectively.
P-73

Assume that the efficiency of the irreversible engine be more than the reversible engine , so, ηA > ηB.
Hence, WA / Q > WB / Q
or WA > WB
Now both the engines are coupled and E B is reversed which will act as a heat pump. It receives (Q – W B) from
sink and W A from irreversible engine E A and pumps heat Q to the source at temperature T1. The net result is
that heat W A – W B is taken from sink and equal amount of work is produced. This violates second law
of thermodynamics. Hence the assumption which has been made that irreversible engine having higher
efficiency than the reversible engine is wrong. So, it is concluded that reversible engine working between
same temperature limits is more efficient than irreversible engine thereby proving Carnot’s theorem.

3.3 Second Law of Thermodynamics


It has been observed that in a particular process the energy difference between the final and initial states is
equal to the difference of heat absorbed by the system and the work done by the system as suggested by the
first law of thermodynamics. It gives no information as to whether any particular change will take place
spontaneously or not. The second law of thermodynamics enables us to predict the direction of a
spontaneous chemical change under a given set of conditions. There are multiple statements available so far
as second law is concerned. Some of these are discussed here.
1. Carnot statement: Carnot interpreted thermodynamics in the light of the first law. It is physically
equivalent to the second law of thermodynamics, and remains valid today. It deals with the efficiency
of a reversible Carnot cycle. According to Carnot efficiency of an engine depends only on the
temperatures of the two heat reservoirs, and is independent of the working substance. A Carnot engine
operated in this way is the most efficient possible heat engine using those two temperatures.
Carnot stated that it is impossible to construct a machine functioning in cycles which can
convert heat completely into the equivalent amount of work without producing changes
elsewhere.
2. Clausius statement: The German scientist Rudolf Clausius laid the foundation for the second law of
thermodynamics. His statement is based upon the relation between heat transfer and work. His
formulation of the second law, which was published later, is known as the Clausius statement. He
stated that, heat can never pass from a colder to a warmer body without some other change,
connected therewith, occurring at the same time.
In simpler language, heat cannot spontaneously flow from cold regions to hot regions without external
work being performed on the system, which is evident from ordinary experience of refrigeration, for
example. In a refrigerator, heat flows from cold to hot, but only when forced by an external agent, the
refrigeration system.
3. Kelvin statement: Lord Kelvin expressed the second law as, it is impossible, by means of
inanimate material agency, to derive mechanical effect from any portion of matter by
cooling it below the temperature of the coldest of the surrounding objects.
4. Entropy statement: It was being observed that entropy of a spontaneous process always increases.
For example, a gas expands when transferred from a high pressure to low pressure zone or, entropy of
a system in two different time zones as shown below. Here time-2 is a later time period. On this basis,
the second law was stated as. Entropy of an isolated system is ever increasing.
P-74

Time1 Time 2

Entropy increases in isolated system


Fig. 2: Entropy in two time zones

So, the different aspects of second law of thermodynamics can be summarised as,
1. Heat flows spontaneously from a hot body to a cool one.
2. One cannot convert heat completely into useful work.
3. Every isolated system becomes disordered in time.

3.3.1 Clausius Inequality


The Clausius theorem is a mathematical explanation of the second law of thermodynamics. Also referred to
as the “Inequality of Clausius”, the theorem was developed by Rudolf Clausius who intended to explain the
relationship between the heat flow in a system and the entropy of the system and its surroundings. Clausius
developed this in his efforts to explain entropy and define it quantitatively. In more direct terms, the theorem
gives us a way to determine if a cyclic process is reversible or irreversible. The Clausius theorem provides a
quantitative formula for understanding the second law.
Clausius was one of the pioneers to work on the idea of entropy and is even responsible for giving it that
name. Clausius sought to show a proportional relationship between entropy and the energy flow by heating (
δQ) into a system. In a system, this heat energy can be transformed into work, and work can be transformed
into heat through a cyclic process. Clausius wrote that the algebraic sum of all the transformations
occurring in a cyclic process can only be less than zero, or, as an extreme case, equal to zero. In
other words, the equation
δq
∫ T
=0

where δq is energy flow into the system due to heating and T is absolute temperature of the body when that
energy is absorbed is found to be true for any process that is cyclic and reversible. Clausius then took this a
step further and determined that the following equation must be found true for any cyclic process that is
possible, reversible or not. This equation is the “Clausius Inequality”.
δq
∫ T
≤0

Now that it is known, there must be a relation developed between the Clausius inequality and entropy. The
amount of entropy S added to the system during the cycle is defined as
δq
∆S =
∫ T
It has been determined, as stated in the second law of thermodynamics, that the entropy is a state function. It
depends only upon the state that the system is in, and not what path the system took to get there. This is in
contrast to the amount of energy added as heat (δq) and as work (δw), which may vary depending on the
path. In a cyclic process, therefore, the entropy of the system at the beginning of the cycle must equal the
entropy at the end of the cycle. In the irreversible case, entropy will be created in the system, and more
P-75

entropy must be extracted than was added (∆S < 0) in order to return the system to its original state. In the
reversible case, no entropy is created and the amount of entropy added is equal to the amount extracted
(∆S = 0).
If the amount of energy added by heating can be measured during the process, and the temperature can be
measured during the process, the Clausius inequality can be used to determine whether the process is
reversible or irreversible by carrying out the integration in the Clausius inequality.
δq
Proof: A system absorbs heat δq at temperature T. Since the value of does not depend on the details of
T
how the heat is transferred, it is assumed that it is coming from a Carnot engine, which in turn absorbs heat
δq 0 from a heat reservoir with constant temperature T 0 . According to the nature of Carnot cycle,
δq δq 0
=
T T0
δq
⇒ δq 0 = T0
T
Therefore in one cycle, the total heat absorbed from the reservoir is
δq
q 0 = T0
∫ T
Since after a cycle, the system and the Carnot engine as a whole return to its initial status, the difference of the
internal energy is zero. Thus according to first law of thermodynamics,

q 0 = ∆U + w + w 0 = w + w 0 = w total (Q ∆U = 0 for a cyclic process)


According to the Kelvin-Planck statement of drained from one reservoir and converted entirely into work
without making any other changes. So,
Wtotal ≤ 0
Combining all the above, Clausius inequality is obtained.
δq
∫T
≤0

If the system is reversible, then reversing its path and doing the experiment again,the following relation is
obtained:
δq
∫T
≤0

δq
Thus
∫T
=0

3.4 Entropy (S)


Entropy (S) is a property of thermodynamic system as suggested by Rudolf Clausius. Clausius has named it
from the Greek word transformation. Later Ludwig Boltzmann described entropy as a measure of the
thermodynamic probability, and he defined entropy as S = klnΩ, here Ω is the number of possible microscopic
configurations of the individual atoms and molecules of the system also referred as microstates. The factor k is
known as Boltzmann's constant. From a macroscopic perspective, in classical thermodynamics, the entropy is
a state function of a thermodynamic system, that is, a property depending only on the current state of the system,
independent of how that state came to be achieved. Entropy is a key ingredient of the second law of
thermodynamics, which has important consequences. Mathematically, it is defined as
dq rev ∆ q rev
dS = or ∆S =
T T
P-76

Entropy of a system is a measure of its disorder or randomness. If a liquid is converted into its vapour state,
disorderness in the system increases and therefore its entropy increases as well. Cooling of water,
crystallization of a solid from solution and alike are attended by more ordered arrangement of molecules or
ions, so these changes cause decrease in entropy of the system.
Standard Entropy S°: The standard entropy or standard absolute entropy S°, is the entropy of substance in
its standard state. The standard state refers to 298 K and 1 atm pressure.
Entropy as a Criterion to Spontaneity: Entropy-S, is a measure of the energy distribution within a
system. Entropy changes, ∆S, are associated with changes in the way in which a energy of a system is
distributed among available energy levels. As a matter of fact, the direction of spontaneous change is the
direction in which total entropy increases. There is a pertinent question that what is total entropy change. The
answer is total entropy change is the sum of the entropy change of Spontaneous change
the system and its surroundings or in other words the entropy
change of the universe. So,
∆Suniv = ∆Ssys + ∆Ssurroundings
According to the second law of thermodynamics, the entropy of the Increasing entropy
universe, must always increase for a spontaneous process, that is, Fig. 3: Increasing disorderness in
∆Suniv > 0. spontaneous process

3.4.1 Calculation of Entropy Change of an Ideal Gas with Change in P, V and T


Since entropy of a system varies with the state of the system, its value for a pure gaseous substances will
depend on any two of the three variables T, P and V. Since T is taken generally as one of the variables, the
second variables to be considered are either V or P. Thus, the two cases to be considered are either when T
and V are variable or T and P together are variable. An ideal gas is considered for the present discussion.
From the first law of thermodynamics, it can be said that,
dU=dq + dw
If the work involved due to expansion of the gas, for an infinitesimal increase in volume dV against a pressure
P is considered, it can be said that, –dw = PdV, similarly, dq = TdS. Putting these values,
dU=TdS–PdV
⇒ TdS=dU+PdV
For an ideal gas dU= C V dT , thus,
TdS=C V dT +PdV
⇒ dS=(C V /T)dT+(P/T)dV ...(1)
For a finite change of state of a system, the entropy change ∆S is obtained by integrating the above equation
between the limits of the initial state 1 and the final stage 2. Using the equation of state for an ideal
gas PV=RT, the following relation is obtained,
C  R 
dS= V  dT +   dV
 T   V
So, upon integrating
∆S = C V ln(T2 / T1) + R ln(V2 / V1)
This gives the change in entropy in terms of temperature and volume. In general, for an ideal gas, the total
entropy change for n moles of the gas is given by
 T V 
∆S = n C V ln(T2 / T1) + n R ln(V2 / V1) ⇒ ∆S = 2.303 nC V log 2 + nR log 2
 T1 V1 
P-77

When isothermal reversible expansion or compression of an ideal gas takes place, entropy change is given by
the expression below; as temperature is constant,
V P
∆S = 2.303nR log 2 = 2.303nR log 1
V1 P2
where n = Number of moles of the gas, V1 = Initial volume; V 2 = Final volume of gas, P 1 = Initial
pressure; P 2 = Final pressure of gas.
When T and P are the two variables: If P 1 is the pressure of the ideal gas in the initial state and P 2 in the
final state, then it is evident from the above equations that the entropy change for the change in state of an
ideal gas depends on the initial and final pressures as well as on the initial and final temperatures. So,
T T P 
∆S = C V ln 2 + R ln 2 × 1 
T1  T1 P2 
PV
As, for ideal gas, = constant (when number of moles is constant)
T
T V T P
⇒ V× So, 2 = 2 × 1
P V1 T1 P2
T  P T P
Now, ∆S = (C V + R) ln  2  – R ln 2 = C P ln 2 – R ln 2
 T1  P1 T1 P1
For n moles,
 T P 
∆S = n C p ln (T2 / T1) − nR ln(P2 / P1) ⇒ ∆S = 2.303 nC P log 2 – nR log 2
 T1 P1 

3.4.2 Entropy Changes of an Ideal Gas in Different Processes


The entropy change of an ideal gas depends upon the conditions applied. There are a number of processes
for which entropy change can be calculated. Three special types of processes are discussed here.. These are:
1. Isothermal Processes
2. Isobaric Processes and
3. Isochoric Processes.
1. Isothermal Processes: In the isothermal process, there is no change in temperature, So T 2 =T 1 Hence
the entropy change for the process as stated above, is,
∆S = n R ln (V2 / V1) = n R ln(P2 / P1)
If V 2 > V 1, ∆S will be positive. This means that isothermal expansion of an ideal gas is accompanied by
increase in entropy. Conversely, isothermal compression of an ideal gas is accompanied by decrease in
entropy.
2. Isobaric Processes: It is a process which takes place at constant pressure. In this case, therefore, P 2
= P 1. Hence change in entropy is given by
∆S = n C p ln(T2 / T1)
It follows from the equation that the increase in temperature of an ideal gas at constant pressure is
accompanied by increase in entropy of the ideal gas.
3. Isochoric Processes: Isochoric processes are those in which volume of the gas remains constant. So,
V2 = V1, and change in entropy is given by,
∆S = n C V ln(T2 / T1)
Evidently, increase in temperature of an ideal gas at constant volume is accompanied by increase in
entropy. Here, the constant-volume molar heat capacity C V is constant and there is no phase change
taking place as well.
P-78

3.4.3 Entropy Change for Phase Transition


Reversible phase transitions occur at constant temperature and pressure. The reversible heat is the enthalpy
change for the transition, and the entropy change is the enthalpy change divided by the thermodynamic
temperature. There are two prominent types of phase changes known to us– one involves conversion of solid
of solid to liquid known as fusion. Another is liquid to gas–known as vaporization. For fusion or melting of a
solid to a liquid at the melting point T m, the entropy of fusion is
∆H fus
∆S fus =
Tm
Similarly, for vaporization of a liquid to a gas at the boiling point T b , the entropy of vaporization is
∆H vap
∆Svap =
Tb
Trouton's Rule: Trouton's rule deals with entropy of vaporization. Trouton’s rule states that the standard
entropy change for the equilibrium vaporization of non–polar liquids at their normal boiling points is
approximately constant.
°
∆H vap
°
∆Svapn = ≈ 87Jmol -1K −1
Tb

3.4.4 Entropy of Mixing


In thermodynamics the entropy of mixing is the increase in the total entropy when several initially separate
systems of different composition, each in a thermodynamic state of internal equilibrium, are mixed without
chemical reaction. In general, the mixing may be constrained to occur under various prescribed conditions.
In the customarily prescribed conditions, the different materials are initially at a common temperature and
pressure, and the new system may change its volume, while being maintained at that constant temperature,
pressure, and chemical component masses.
The volume available for each material to explore is increased, from that of its initially separate compartment,
to the total common final volume. The final volume need not be the sum of the initially separate volumes, so
that work can be done on or by the new closed system during the process of mixing, as well as heat being
transferred to or from the surroundings. It is because of the maintenance of constant pressure and
temperature. The internal energy of the new closed system is equal to the sum of the internal energies of the
initially separate systems.
In a process of mixing of ideal gases or ideal solution, the final common volume is the sum of the initial
separate compartment volumes. There is no heat transfer and no work is done. The entropy of mixing is
entirely accounted for by the diffusive expansion of each material into a final volume not initially accessible to
it. On the mixing of non-ideal materials, the total final common volume may be different from the sum of the
separate initial volumes, and there may occur transfer of work or heat, to or from the surroundings. Also,
there may be a departure of the entropy of mixing from that of the corresponding ideal case. That departure is
the main reason for interest in entropy of mixing. These energy and entropy variables and their temperature
dependences provide valuable information about the properties of the materials. In ideal materials,
intermolecular forces are the same between every pair of molecular kinds, so that a molecule experiences no
difference between itself and its molecular neighbours. This is the reference case for examining
corresponding mixings of non-ideal materials. For example, two ideal gases, at the same temperature and
pressure, are initially separated by a dividing partition. Upon removal of the dividing partition, they expand
into a final common volume which is the sum of the two initial volumes, and the entropy of mixing ∆Smix is
given by
∆Smix = −nR(x 1 ln x 1 + x 2 ln x 2 )
P-79

Here R is the gas constant, n the total number of moles and x i the mole fraction of component i, which
initially occupies volume Vi Such that, Vi = x i V.

Example 1: One mole of an ideal mono – atomic gas expands reversibly from a volume of 10 dm 3
3
and temperature 298 K to a volume of 20dm and temperature 250K. Assuming that
CV = 3 / 2R, calculate the entropy change for the process.
Solution: As it is known that,
T  V 
∆S = C V ln  2  + R ln 2 
 T1   V1 
250 20
= 1.5×8.314 ln + 8.314 ln
298 10
= –2.19 +5.76 = 3.57 JK –1 mol –1

3.4.5 Entropy of a Mixture of Ideal Gases


For one mole of an ideal gas,change in entropy is calculated as
dS = C V dT / T + RdV / V
Integrating, assuming that C V remains constant for an ideal gas, it can be said that
S = C V ln T + R ln V + S0
here S0 is the constant of integration.
Remembering that C P -C V = R and V = RT/P, the expression becomes,
S= C P ln T – R ln P + R ln R+S o
= C P ln T − R ln P + S0 '
here S0 ’ is another constant,such that S 0 ' = R ln R+ S 0 .
Now, entropy of a mixture of gases would evidently be given by the sum of the individual entropies of the
constituents are at pressures or even concentrations prevalent in the mixture. If n 1, n 2 , n 3 etc. are the
numbers of moles of the various gases present in the mixture. The partial pressures of these components are p
1, p 2 , p 3 etc. then the entropy of the mixture is given by,
′ ′
S=n1(C P lnT – R lnP1 + S0 ) + n2 (C P lnT – R lnP2 + S0 ) + …

= ∑ n(C P lnT – R lnP + S0 ′ );


as partial pressure is defined a P= xP; S 0 ,
S= ∑ n(C P lnT – R lnP – R ln x + S0 ′ )
3.4.6 Overview of Entropy
In thermodynamics, entropy is a measure of the number of specific ways in which a thermodynamic system
may be arranged, often taken to be a measure of disorder. The entropy of an isolated system never
decreases, because isolated systems spontaneously evolve towards thermodynamic equilibrium, which is the
state of maximum entropy.
The change in entropy (∆S) was originally defined for a thermodynamically reversible process as
dq rev
∆S =
T ∫
P-80

The above definition is sometimes called the macroscopic definition of entropy because it can be used
without regard to any microscopic picture of the contents of a system. In thermodynamics, entropy has been
found to be more generally useful and it has several other formulations. Entropy was discovered when it was
noticed to be a quantity that behaves as a function of state, as a consequence of the second law of
thermodynamics. Entropy is an extensive property, but the entropy of a substance can be expressed as
an intensive property. Entropy becomes intensive properly when it is expressed either as specific entropy
(entropy per unit mass) or molar entropy (entropy per mole). The absolute entropy S rather than ∆S was
defined later, using the third law of thermodynamics.
Entropy as a Measure of the Disorder of the System: It has already been shown that all spontaneous
processes, such as flow of heat from a hot end to a cold end of a conductor, flow of electricity from a point at a
higher potential to a point at lower potential, expansion of gas in vacuum, diffusion of solute from a
concentrated to a dilute solution are accompanied by increase in the disorder of the system. Spontaneous
processes are accompanied by increase in entropy as well as increase in the disorder of the system. It is
known that melting of a solid or evaporation of a liquid is accompanied by increase of entropy. At the same
time it is known that a solid has a definite crystal lattice, so that the atoms or the ions or molecules in a solid
are arranged in a definite order. The order is much less in a liquid and the least in a gas. Thus, increase of the
entropy implies increase in disorder. Thus, entropy is regarded as a measure of the disorder of a system.
Entropy as a Measure of Probability: As seen above, all spontaneous processes lead to increase in
entropy and also to increase in disorder. A little consideration will show that when a process is spontaneous, it
is proceeding from a less probable to a more probable state. So, there is a close relationship between entropy
S and the thermodynamic probability W of the state of the system both of which increase at the same time.
This relationship was expressed by Boltzmann as
S =k lnW + c
here, k is Boltzmann’s constant (=R/Avogadro’s number). Boltzmann defined thermodynamic probability of
a system as the ratio of probability of the actual state to the probability of the state in which there is complete
order for the same energy and volume. According to Planck, the constant, c , in the above equation is zero.
Hence,
S = k ln W
Example 2: A mole of steam is condensed at 100°C, the water is cooled to 0°C and frozen to
ice. What is the difference in entropies of the steam and ice? The heats of vaporization and
fusion are 540 cal gm−1 and 80 cal gm−1 respectively. Use the average heat capacity of liquid
water as 1 cal gm−1degree −1.
Solution: Entropy change during condensation of steam
18 × 540
∆S1 = cal / ° C = −26.06cal / ° C
373
Entropy change during cooling of water from 100°C to 0°C
273
∆S2 = 18 × 1 ln cal / ° C = −5.62cal / ° C
373
Entropy change during freezing of water at 0°C
18 × 80
∆S3 = − cal / ° C = −5. 27 cal / ° C
273
So total entropy change = –26.06 – 5.62 – 5.27 = –36.95 cal/°C
Hence difference in entropy between steam and ice = 36.95 cal/°C
P-81

3.5 Gibbs Free Energy, G


The thermodynamic functions, A (or F) and G were introduced to indicate the direction of any physical or
chemical change. In thermodynamics, the Gibbs free energy or Gibbs energy is a thermodynamic
potential that measures the usefulness or process-initiating work obtainable from a thermodynamic system at
a constant temperature and pressure.The Gibbs free energy (SI units J/mol) is the maximum amount of
non-expansion work that can be extracted from a closed system. This maximum work can be attained only in
a completely reversible process. When a system changes from a well-defined initial state to a well-defined
final state, the Gibbs free energy ∆G equals the work exchanged by the system with its surroundings, minus
the work of the pressure forces, during a reversible transformation of the system from the same initial state to
the same final state.
G is an important thermodynamic parameter. As such it is also considered as the chemical potential that is
minimized when a system reaches equilibrium at constant pressure and temperature. Its derivative with
respect to the reaction coordinate of the system vanishes at the equilibrium point. As such, it is also treated as
a convenient criterion of spontaneity for processes with constant pressure and temperature.
The Gibbs free energy is defined as
G(P,T) = U + PV – TS
which is the same as
G(P,T) = H – TS ⇒ ∆G = ∆H– ∆(TS) ⇒ ∆G = ∆H – T∆S – S∆T
As temperature is to be kept constant, so
∆G = ∆H – T∆S
here, U is the internal energy (SI unit is joule), P is pressure (SI unit is pascal), V is volume (SI unit is m 3 ), T is
the temperature (SI unit is kelvin), S is the entropy (SI unit is joule per kelvin) and H is the enthalpy (SI unit is
joule).
The expression for the infinitesimal reversible change in the Gibbs free energy as a function of its ‘natural
variables’ P and T, for an open system, subjected to the operation of external forces can be stated as below
with the help of the first law of thermodynamics.
As stated by first law,
TdS = dU + PdV
⇒ d(TS) – SdT = dU + d(PV) – VdP
⇒ d(U – TS + PV) = VdP – SdT
So, dG= VdP – SdT (Q U+PV=H and G= H–TS)
This is one form of Gibbs fundamental equation. Each quantity in the equations above can be divided by the
amount of substance, measured in moles, to form molar Gibbs free energy. The Gibbs free energy is one of
the most important thermodynamic functions for the characterization of a system. It is a factor in determining
outcomes such as the voltage of an electrochemical cell, and the equilibrium constant for a reversible
reaction.
Standard Free Energy Change (∆G°): Standard free energy change is the change in free energy when all
the reactants and products are at unit concentration (for K C ) or at the unit pressure (for K P ). Here, K C and K P
are equilibrium constant of a reversible reaction.
The standard free energy change, ∆G° can be calculated as
1. By substituting standard enthalpies and entropies of reaction and a Kelvin temperature into the Gibbs
equation. or
2. By combining standard free energies of formation through the expression
∆G° = [Σv p∆G°f (products)] − [Σv r ∆G°f (reactants)]
P-82

Free Energy Change and Equilibrium: The condition of equilibrium is one for which ∆G = 0. The
standard free energy change is a particularly useful property to use in
describing equilibrium because of its relationship to the equilibrium

Free energy, G
constant, K eq . As it is known that, Q>Keq
∆G=∆G°+RT ln Q (here Q is reaction quotient)
Q<Keq
At equilibrium, ∆G=0 and Q=K eq ;
So, 0 =∆G°+RT ln K eq
⇒ ∆G ° = −RT ln K eq
The value of ∆G° is in itself often sufficient to determine whether a reaction
Equilibrium
is likely to
Q=Keq
1. Go to completion ( ∆G° very large and negative).
2. Occur hardly at all (∆G° very large and positive). or Reactants Products
3. Reach an equilibrium condition requiring an equilibrium calculation Fig. 4: Change of free energy
( ∆G° not very large and either positive or negative).
For ∆G° neither very large not very small
Sometimes a non spontaneous reaction can be made to occur by combining it with a spontaneous reaction.
Together, these two coupled reactions produce a spontaneous overall reaction.
The Dependence of ∆G° and K eq on Temperature: Values of ∆G°f , ∆H °f and S°are usually tabulated for
25 °C. To obtain values of K eq at other temperatures, it is generally assume that ∆H ° and ∆S° are
independent of temperature. With this assumption, one can derive the Van’t Hoff equation to relate the
equilibrium constant and temperature.
K ∆H °  1 1
ln 2 =  − 
K1 R  T1 T2 

3.5.1 ∆G as a Criterion of Spontaneity of a Process


Before discussing ∆G as a criterion of spontaneity, one must understand a spontaneous change or
a spontaneous process. A spontaneous change or a spontaneous process is one that occurs by itself without
outside intervention. A non–spontaneous process is one that cannot occur on its own. For a chemical or
physical process, if change in Gibbs free energy, ∆G has negative value that process will spontaneously takes
place under conditions of the experiment. In a state of equilibrium in reversible reactions and in reversible
physical changes, no net changes takes pace and no useful work is being done, hence ∆G = 0. If, on the other
hand, ∆G has a positive value, the change will take place in reverse direction causing products of a reversible
change to convert into reactants. On the other hand, if ∆G is negative, then the process becomes
spontaneous. A tabular information is provided below, which deals with criteria for spontaneity in various
conditions. The spontaneity is predicted on the basis of signs of ∆H and ∆S.
Table 1: Criteria for spontaneity in various conditions
∆H ∆S ∆G Reaction characteristics

– + Always negative Reaction is spontaneous at all temperatures.

+ – Always positive Reaction is non spontaneous at all temperatures.

– – Negative at low temperature but Reaction is spontaneous at low temperature but becomes non
positive at high temperature spontaneous at high temperature.

+ + Positive at low temperature but Reaction is non spontaneous at low temperature but becomes
negative at high temperature spontaneous at high temperature.
P-83
3.5.2 The Variation of Free Energy Change with Temperature and Pressure
The variation of free energy change with change in temperature and pressure is discussed below. Consider
the following equation
G = H – TS …(1)
As H = U + PV
Substituting the value of H in equation (1), we will get
G = U + PV – TS …(2)
Now differentiating the equation (2), we will get
dG = dU + PdV + VdP – TdS – SdT …(3)
The first law equation for an infinitesimal change may be given as
dq = dU – dw
If the work done is only due to expansion, then –dw = PdV
Therefore, dq=dU+PdV
For a reversible process,
dS = dq / T
TdS = dq = dU + PdV … (4)
Combining equations (3) and (4), one will get:
dG = VdP – SdT …(5)
This equation gives the change of free energy when a system changes reversibly with a change in pressure as
well as in temperature.
At constant pressure when dP = 0, then equation (5) will be as
dG = – SdT ... (6)
or (∂G / ∂T) P = –S
It is known ∆G = ∆H – T∆S and,
 ∂∆ G 
– ∆S =  ; so,
 ∂Τ  P
 ∂∆ G 
∆G = ∆H – T 
 ∂Τ  P
At constant temperature , dT = 0, then equation (5) will be as
dG = VdP ... (7)
or (∂G / ∂P) T = V ; and from Maxwell relation,
 ∂S   ∂V 
  = –  
∂P T ∂T  P
 ∂S   ∂V 
We have to prove that,   = – 
 ∂T  P  ∂T  P
Proof: As it is known that, S=f(T,P) so the differential form is,
 ∂S   ∂S 
dS =   dT +   dP …(1)
 ∂T  P  ∂P  T
1 P
Moreover dS = dU + dV
T T
As H = U + PV ⇒ dU = dH – PdV – VdP.
  ∂H   ∂H  
Taking H = f (T,P) ⇒ dU =  dT +  dP – PdV – VdP
 ∂T  P  ∂P  T 
P-84

1   ∂H   ∂H   PdV VdP PdV


So, dS = dT +  dP – – +
T  ∂T  P  ∂P  T  T T T
1   ∂H   1   ∂H  
⇒ dS = dT + dP – VdP …(2)
T  ∂T  P  T  ∂P  T 
Comparing (1) and (2),
 ∂S  1  ∂H 
  =   …(3)
∂T P T ∂T  P
 ∂S  1   ∂H  
and   = –V …(4)
∂P T T  ∂P  T 
Differentiating eq. (3) wrt P at constant temperatures,
∂   ∂S   1  ∂  ∂H   1 ∂ 2H
  =   = …(5)
∂T  ∂T  P  T T  ∂P  ∂P  P  T ∂P∂T
Differentiating eq. (4) wrt T at constant pressures,
∂   ∂S   ∂  1   ∂H  
    =     – V 
∂T  ∂P T  P ∂T  T  ∂P T  P
1  ∂ 2H   ∂V   1   ∂H  
=  ∂T∂P  –  ∂T   – 2 –V …(6)
T T  ∂P  T 
  P

As 'S' is a state function.


∂ 2S ∂ 2S
= ; so,
∂T∂P ∂P∂T
1 ∂ 2H 1 ∂ 2H 1  ∂V  1   ∂H  
= –   – –V
T ∂P∂T T ∂T∂P T  ∂T  P T2  ∂P  T 

 ∂V  1   ∂H  
⇒   = – – V ; Using eq. (4)
∂T P T  ∂T  T 
 ∂V   ∂S 
⇒   = – 
∂T  P  ∂P  T

3.5.3 Gibbs Free Energy of Mixing


The Gibbs free energy change ∆Gmix = ∆H mix − T∆Smix determines whether mixing at constant (absolute)
temperature T and pressure P is a spontaneous process. This quantity combines two physical
effects—the enthalpy of mixing, which is a measure of the energy change, and the entropy of mixing
considered earlier. For an ideal gas mixture or an ideal solution, there is no enthalpy of mixing (∆H mix ), so
that the Gibbs free energy of mixing is given by the entropy term only
∆Gmix = −T∆Smix
For an ideal solution, the Gibbs free energy of mixing is always negative, meaning that mixing of ideal
solutions is always spontaneous. The lowest value is when the mole fraction is 0.5 for a mixture of two
components, or 1/n for a mixture of n components.

3.5.4 Fugacity and Activity


Making use of the free energy function, G, Lewis introduced the concept of fugacity for representing the
actual behavior of real gases which is different from the behaviour of ideal gases. Variation of free energy with
pressure at constant temperature is given below using equation (7).
P-85

 δG  = V
 δP  T
This equation is applicable to all gases whether ideal or non-ideal. If one mole of a gas is under consideration,
then V refers to molar volume. For an ideal gas, the above equation may be written as
dP
(dG)T = RT
P
dP
and for n moles (dG)T = nRT = nRT (ln P)
P
Integrating the above relation,
G = G * +nRT ln P
where G*is the integration constant. It is the free energy of n moles of the ideal gas a temperature T when the
pressure P is unity. This equation gives the free energy of an ideal gas at temperature T and pressure P.
Integrating between pressures P 1 and P 2 , at constant temperature T, yields
P2 dP P
= nRT ln 2
∆G =
∫P 1
nRT
P P1
The corresponding equation for 1 mole of the gas would be
P
∆G = RT ln 2
P1
This equation is definitely not valid for real gases since V is not exactly equal to RT/P. In order to make these
simple equations applicable to real gases, Lewis introduced a new function f, called fugacity function. It
replaces P in the above equation. So that the form of equation changes to
(dG)T = nRTd(ln f )
So that,
G = G* + nRT ln f
here G* is the free energy of n moles of a real gas when its fugacity happens to be 1. Thus, fugacity is a sort of
hypothetical pressure which is used for real gases. So, the equation, evidently, gives the free energy of a real
gas at temperature T and pressure P at which its fugacity can be taken as f. On integration between fugacities f
1 and f 2 , at constant temperature T, the above equation yields
f 
∆G = nRT ln  2 
 f1 
The corresponding equation for 1 mole of the gas would be
f 
∆G = RT ln  2 
 f1 
Example 3: Calculate the free energy change accompanying the compression of 1 mole of a gas
at 37°C from 25 to 200 atm. The fugacities of the gas at 57°C may be taken as 23 and 91 atm,
respectively, at pressure of 25 and 200 atm.
P 200
Solution: ∆G = nRT ln 2 = 1×8.314×330 ln
P1 25
= 5702.85
It we use fugacities instead of pressures, for more accurate values are obtained.
P-86

P2 91
∆G = nRT ln = 1×8.314×330 ln
P1 23
= 3730 J.

3.5.5 Fugacity at Low Pressures


The ratio f/P, where P is the actual pressure, approaches unity when P approaches zero since in that case a
real gas approximates to ideal behaviour. The fugacity function, therefore, is defined as
lim f
P→ 0 =1
P
Evidently, at low pressures, fugacity is equal to pressure. The two terms differ only at high pressures.

3.5.6 Determination of Fugacity of a Gas


The fugacity equation for one mole of a real gas is
G=G*+RT ln f
Differentiation of equation with respect to pressure at constant temperature and constant number of moles of
the various constituents in a closed system, gives
 δG   δ(ln f )
  = RT  
δP T δP 
 δG 
Since,   = V
δP T
 δ(ln f ) V
So,   =
δP  T RT
Thus, at a definite temperature, the above equation is converted into
RTd(ln f)=VdP
As one mole of the gas is under consideration, V is the molar volume of the gas. Knowing that for an ideal gas,
V = RT/P, the quantity α, defined as departure from ideal behavior at a given temperature, is given by
RT
α= −V
P
Multiplying by dP throughout,
 dP 
αdP = RT   − VdP
 P
Combining equations, it can be said that
dP
RTd(ln f)=RT − αdP
P
dP
Such that, d(ln f) =d(ln P) – α
RT
Integrating between zero and pressure P,
f 1 P
ln = −
P RT ∫0α(dP)
Here, α can be determined experimentally, at different pressures.These values of are then plotted against
corresponding pressures, as shown in fig.5. The area under the curve between pressure P = 0 and any given
P
∫0
pressure P, yields the value of the integral , α(dP), as illustrated by the shaded portion in fig. 5.
P-87

Positive
RT–V

Positive field
P

Negative field
α=

Negative
Pressure P

Fig. 5: Graph between α and p (for determination of fugacity of gas)

Since α, the departure from ideal behaviour, can be both positive as well as negative, the area under the
curve can be both positive as well as negative. Thus, the fugacity of the gas can be both less than or more than
P
∫0
the pressure P. As is evident from figure, the area and hence the value of α(dP) is positive at low pressures

and negative at very high pressures. Hence, fugacity f of the gas would be less than the pressure P at low
pressures and more than the pressure P at very high pressures.

3.5.7 Helmholtz Free Energy


The Helmholtz free energy was developed by Helmholtz, a German physicist, and is usually denoted by the
letter A or sometimes the letter F. A is derived from the German word arbeit meaning work. The IUPAC
recommends the letter A for Helmholtz energy. In thermodynamics, the Helmholtz free energy is
a thermodynamic that measures the “useful” work obtainable from a closed thermodynamic system at a
constant temperature. The negative of the difference in the Helmholtz energy is equal to the maximum
amount of work that the system can perform in a thermodynamic process in which temperature is held
constant. If the volume is not held constant, part of this work will be performed on the environment. The
Helmholtz energy is commonly used for systems held at constant volume, so that in this case no work is
performed on the environment. For a system at constant temperature and volume, the Helmholtz energy is
minimized at equilibrium.
While Gibbs free energy is most commonly used as a measure of thermodynamic potential, it is an
inconvenient tool for some applications that do not occur at constant pressure. For example, in the research
work of explosives, Helmholtz free energy is often used since explosive reactions by their nature induce
pressure changes. It is also frequently used to define fundamental equations of state of pure substances.
The Helmholtz energy is mathematically expressed as
A ≡ U − TS
Here, A is the Helmholtz free energy (SI unit is joules), U is the internal energy of the system (SI unit is
joules),T is the absolute temperature (Kelvin),S is the entropy (SI unit is joules per Kelvin). The Helmholtz
energy is the Legendre transform of the internal energy, U, in which temperature replaces entropy as the
independent variable.
The first law of themodynamics, with a constant number of particles, is given by
dU = δq − δw,
where U is the internal energy, δq is the energy added by heating and δW is the work done by the system.
From the second law of thermodynamics, for a reversible process it can be said that δq = TdS. Also, in case of
a reversible change, the work done can be expressed as δW = PdV, so
dU = TdS − PdV
Applying the product rule for differentiation to d(TS) = TdS + SdT, it can be said that
P-88

dU = d(TS) − SdT − PdV


d(U − TS) = −SdT − PdV
The definition of A = U – TS enables to rewrite this as
dA = − SdT − PdV
This relation is also valid for a process that is not reversible because A is a thermodynamic function of state.
From the relation, dA = –SdT – PdV; it follows,
 ∂A   ∂A 
  = – P;   = –S
∂V T ∂T  V
 ∂S   ∂P 
and   =  
∂V T ∂T V
(It is one of the Maxwell derivations which is taken from T and V dependence of entropy.)
 ∂S   ∂P 
We have to prove that  = .
 ∂V  T  ∂T  V
Proof: As entropy is a function of temperatures and volume,
 ∂S   ∂S 
S = f(T, V); so, dS =   dT +  dV …(1)
 ∂T  V  ∂V  T
From first law, dU = dq rev – PdV
dU P dq rev dU P
⇒ + dV = ⇒ dS = + dV …(2)
T T T T T
 ∂U   ∂U 
As U = f (T, V) ⇒ dU =  dT +  dV …(3)
 ∂T  V  ∂V  T
Rearranging eq. (2)
1   ∂U   1   ∂U  
dS =   dT + P+ dV …(4)
T  ∂T  V  T   ∂V  T 
Comparing eq. (4) and eq. (1)
 ∂S  1  ∂U  nC v
  =   = …(5)
∂T V T ∂T V T
 ∂S  1  ∂U  
and   = P+ …(6)
∂V  T T   ∂V  T 
Differentiating (5),
∂   ∂S   1 ∂2U
= …(7)
∂V  ∂T  V  T ∂V∂T
T
Differentiating (6),
∂   ∂S   ∂ 1   ∂U   
 ∂V  T 
=
T P +   
∂T V
∂T   ∂V  T  

∂   ∂S   1   ∂P  ∂ 2U  1   ∂U  
⇒ = + – P +  …(8)
∂T  ∂V  T  T
 
  ∂ T  ∂ T ∂ V  T2

 
∂V  T 
V  
From (7) and (8),
1 ∂ 2U 1  ∂P  1 ∂ 2U 1   ∂U  
=   + – P +   
T ∂ V ∂ T T  ∂ T  V T ∂ T ∂ V T2  ∂V  T 
P-89

 ∂P  1  ∂U  
⇒   = P +    …(9)
∂T V T  ∂V  T 
From (6) and (9),
 ∂P   ∂S 
  =   .
∂T V ∂V  T
∆A as a Criterion of Spontaneity of a Process: It is presumed that our system is also kept at constant
volume to prevent work from being done. If temperature and volume are kept constant, then dA = 0. This is a
necessary, but not sufficient condition for equilibrium. Similarly, for the spontaneity of a process, the change
in Helmholtz free energy must be negative, that is A f ≤ Ai .

3.6 Nernst Heat Theorem and Third Law of Thermodynamics


The Nernst heat theorem says that as absolute zero is approached, the entropy change ∆S for a chemical or
physical transformation approaches 0. This can be expressed mathematically as follow
lim ∆S = 0
T→ 0
The above equation is a modern statement of the theorem. Another way of looking at the theorem is to start
with the definition of the Gibbs free energy (G),
G = H – TS,
For a change from reactants to products at constant temperature
and pressure the equation becomes ∆G = ∆H –T∆S. In the limit of
∆G
T = 0 the equation reduces to just
∆G = ∆H, as illustrated in the fig. 6 shown here, which is
supported by experimental data.
Energy

However, it is known from thermodynamics that the slope of the


∆G curve is -∆S. Since the slope shown here reaches the horizontal
limit of 0 as T → 0 then the implication is that ∆S → 0, which is the ∆H
Nernst heat theorem. The significance of the Nernst heat theorem
is that it was later used by Max Planck to give the third law of
thermodynamics,
O
T
Fig. 6: Energies at low temperatures
3.6.1 Third Law of Thermodynamics
The third law says if an object reaches the absolute zero
of temperature (0 K = –273.15C = –459.67°F), its atoms will stop moving. So, the third law of
thermodynamics states that regarding the properties of systems in equilibrium, the entropy of a perfect
crystal, at absolute zero kelvin, is exactly equal to zero. Experimentally, it is not possible to obtain –273.15°C.
It is found that most of the gases either liquefy or solidify before reaching such a temperature, gaseous
molecules no longer remaining. At zero kelvin the system must be in a state with the minimum
possible energy, and this statement of the third law holds true if the perfect crystal has only one minimum
energy state. Quite interestingly, if the system does not have a well-defined order like glassy solids, then in
practice there will remain some finite entropy as the system is brought to 0 kelvin. It is because the system
becomes locked into a configuration with non-minimal energy. The constant value is called the residual
entropy of the system.
For the transformation,
solid(0 Kelvin, P) → solid (T.P)
the entropy change can be calculated using.
P-90

T2 TC
P dT
∆S = nC P ln
T1
⇒ ST – S0 =
0 T ∫
T dT
⇒ ST – S0 +
0 ∫
CP
T
T dT
As suggested by Planck, S0 = 0 ⇒ ST =
0 ∫CP
T
Entropy of solid: It is important to know that heat capacity of solid must be known from 0 Kelvin to the
desired temperature. Normally, the heat capacities of solids are known till 10-15 K. Below this temperature,
the heat capacity is determined from Debye T-cubed law,
C P = αT3 (Here α is constant)
Tmin dT 0 dT
So, ST =
0 ∫ CP
T
+

Tmin
CP
T
Entropy of Liquid: The following steps are involved for the evaluation,
1. Heating the solid from 0 Kelvin to melting point (T m )
2. Conversion of substance from solid to liquid at T m .
3. Heating the liquid from T m to desired temperature (T) So,
 Tm dT   ∆H fusion   T dT 
(ST )Liquid = 
0
∫C P (S) +
T   Tm   Tm  +

C P (l)
T

Entropy of a gas: In a similar fashion,


 Tm dT   ∆H fusion   Tb dT   ∆H vap   T dT 
(ST )gas =  ∫
 0
C P (S) +
T   Tm   Tm  +
∫ C P (l) + +
T   Tb   Tb ∫C P (g)
T

3.6.2 Residual Entropy


Residual entropy is the difference in entropy between a non-equilibrium state and crystal state of a substance
at absolute zero. This term is used to describe the entropy at zero kelvin of a glass or plastic crystal referred to
the crystal state, whose entropy is zero according to third law of thermodynamics. It occurs if a material can
exist in many different states when cooled. A common example is the case of carbon monoxide, which has a
very small dipole moment. As the carbon monoxide crystal is cooled to absolute zero, a few of the carbon
monoxide molecules have enough time to align themselves into a perfect crystal with all of the carbon
monoxide molecules oriented in the same direction. Another example is an amorphous solid like glass. These
have residual entropy, because the atom–by–atom microscopic structure can be arranged in a huge number
of different ways across a macroscopic system.

3.7 Nernst's Distribution Law


Distribution law or the Nernst’s distribution law gives a generalisation which governs the distribution of a
solute between two non miscible solvents. This law was first given by Nernst who studied the distribution of
several solutes between different appropriate pair of solvents. It has been experimentally observed that, at a
constant temperature, if a solute distributes itself between two immiscible phases, then the ratio of its
concentrations in the two phases is constant.

3.8 The Distribution Law


On the basis of these observations, Nernst stated that if a solute X distributes itself between two immiscible
solvents A and B at constant temperature and X is in the same molecular condition in both the solvents, then
(Concentration of X in A) / (Concentration of X in B) = K d ,
P-91

here K d is called the distribution coefficient or the partition coefficient. K d is temperature dependent
constant. This law is one of the laws applying to ideal dilute solutions. It was discovered by W. Nernst in 1890.
The Nernst distribution law permits us to determine the most favorable conditions for the extraction of
substances from solutions. For example, Iodine a violet cyrstalline solid when added to a mixture of two
immiscible liquids say water and carbon tetrachloride, distributes in both of them. The value of K in this case
is 85 at 298 K.
Applications: The priniciple of distribution law is quite helpful in metallurgical operations. A popular
example is desilverisation of lead also known as Parke’s process. Other applications are
1. Solvent extraction
2. Partition chromatography etc.
Thermodynamic derivation of Nernst law: It is pressured that solve A is present in two immiscible
solvents 1 and 2. The chemical potential of A is µ 1 in solvent 1 and µ 2 in solvent 2. At equilibrium,
µ 1 = µ 2 ⇒ µ °1 + RT ln a1 = µ °2 + RT ln a 2 (Qµ = µ°+RT ln a)
a
⇒ µ 2°– µ °1 = RT ln 1
a2
a1
⇒ =constant (Here a1 and a 2 are activities)
a2
x1
⇒ =constant (Q a×x ; x is mole fraction)
x2
C1
⇒ = constant (Q a×C ; C is concentration)
C2
C1
∴ = K d; K d is the constant
C2
This is Nernst distribution law. Now, some of the cases are considered where the solute might associate or
dissociate.
Case I. Association of solute
PHASE I
a n×a
PHASE II
X (X)n Total conc. C2
C1

Here association of solute is taking place in phase II. 'X' is the molecular formula of solute in phase I where
its concentration is C1 and it because (X) n having concentration C 2 in phase II for the equilibrium,
(X)n a nX ; the equilibrium constant is,
[X]n
K = ⇒ [X]n = K[(X)n ]
[(X)n ]
⇒ [X] = n K[(X) ] = constant × n [(X) ]
n n

nC C1
⇒ C1 = constant 2 ⇒ = Kd
nC
2
Case II. Dissociation of solute.

a a
PHASE I PHASE II
X X A +B
C1 C2

αC 2
P-92

C 2 (1 – α) (αC 2 )(αC 2 )
Here solute X has concentration C1 in phase I. Due to dissociation in phase II, its concentration has reduce to
C 2 (1 – α) from C 2 . Here α is degree of dissociation of X in phase II.
X a A + B
Initial concentration in phase II C2 — —
Conc. dissociated αC 2 — —
Conc. at equilibrium C 2 (1 – α) αC 2 αC 2
So, according to Nernst law,
C1
= Kd
C 2 (1 – α)
Nernst law is highly useful. Some of the major applications are study of associations or dissociation of solute,
study of complex ion, solvent extraction, desilverisation of lead by Parke's process etc.
Solvent extraction: Solvent extraction refers to removal of dissolved organic substance from aqueous
solutions with the help of organic solvents like benzene, CHCl3 etc. A general formula is derived below for
the determination of the amount of a solute left un-extracted after a given number of extractions.
'v' mL of an extracting is used to extract from a solution containing W gram of solute in V mL solvent. Let
w 1 be the mass of solute leftover unextracted after first step; so,
w1 / V  K dV 
Kd = ⇒ w1 =
(W – w1) / v K dV + v 

Similarly, the mass of solute leftover after second step is,


2
 K dV   K dV 
w 2 = w1 =W
K dV + v  K dV + v 

So, amount leftover after 'n' operations or steps is,


n
 K dV 
wn = W
K dV + v 

To keep w n as small or possible , 'n' must be large. Moreover, n × v = 0 total volume of extracting liquid
which is constant. So, reiterating, larger efficiency is observed if 'n' is large rather than v.
Parke's process: It is used for desilverisation of lead. The argentiferrous Pb is fused to 800°C. Molten zinc is
added subsequently. Molten Zn and Pb behave as two immiscible liquids and Ag behaves as solute. So, Ag
passes readily into Zn layer and separated. By repeating the process 3-5 times, almost a good percentage of
Ag passes from heavier Pb layer to lighter Zn layer.
Conditions for validity of law: There are basically two limitations. They are
1. The solute which is to be distributed should not react with any of the solvents.
2. The solute should not undergo any change in its molecular state in the solvents. So, it should neither
dissociate nor associate. If the dissolved compound in one of the solvents can associate like below,
nA Û A n
Then, the ratio C 1 / C 2 is not stable at constant temperature.
P-93

3. The solutions are dilute. When the concentration is small, deviation from law is less. In an extreme
S
case, both the solvents way be saturated with respect to solute. Then partition co-efficient K d = 1 .
S2
Here S 1 & S 2 are solubilities in the two solvent layers.
4. The two liquid solvents must be mutually insoluble or sparingly soluble like water and benzene.
Presence of solute most not change their mutual solubilities.
Example 4: Calculate the entropy increase in the evaporation of 1 mole water at 373 K. Heat of
Vaporization of water at 373 K is 540 cal/g.
∆H V 540 × 4.1814 × 18
Solution: ∆S V = =
Tb 373

=109.03 JK –1mol –1
Example 5: Calculate the entropy change when one mole of steam at 100°c is converted into ice
at 0°c. Average specific heat of water = 4.184 JK –1Kg. –1 Heat of vaporization at boiling point=
2.4906×10 3 J Kg. –1; heat of fusion at 0°c = 333.55×10 3 J/Kg.
J Kg
Solution: C P = 4.184 × 10 3 × 18 × 10 –3 = 75.31JK –1mol –1
K × Kg mol
J Kg
∆H V = 2490.6 × 10 3 × 18 × 10 –3 = 44831.56 J/mol
Kg mol
J Kg
∆H f = 333.55 × 10 3 × 18 × 10 –3 = 6004 J/mol
Kg mol
The process is,
H2O(g,100 ° C) q H2O(l,100 ° C) → H2O(l,0 ° C) q H2O(s,0 ° C)
 – ∆H V  273   – ∆H f 
∆Stotal = +C P ln +
 Tb   373  2  Tf ,
1 

=–165.73 JK –1mol –1
Example 6: Calculate the entropy change when 1 mole of O 2 (g) and 2 moles of H 2 (g) are
mixed. It is assumed that gases and mixture behave ideally and in chemical reaction takes
place.
Solution: nH2 =2, n O2 = 1
2 2
xH2 = =
1+ 2 3
1
x O2 = .
3
So, ∆S mixing = –RΣni ln x i
1 2
=–R1 ln + 2 ln 
 3 3
J
=15.88
K
P-94

Example 7: Free energy change accompanying a process is –85.77 KJ at 25°C and –83.68 KJ at
35°C. Calculate enthalpy change at 30°C.
Solution: We have to use,
 ∂∆G 
∆G =∆H+T 
 ∂T  P

 ∂∆G  –83.68 + 85.77 KJ


Now,   = = 0.209
∂T  P 308 – 298 Kelvin
∆G at 30°c is presumed to be average of (∆G) 25° C and (∆G) 35° C
85.77 + 83.68 
(∆G) 30°C =–  = –84.725 KJ
 2 
So, –84.725 = ∆H+303×(0.209)
∆H =–148.05 KJ.

1. Show that the reaction CO(g) + ½O 2 (g) → CO 2 (g) at 300 K, is spontaneous and exothermic, when
the standard entropy change is –0.094 kJ mol −1 K −1. The standard Gibbs free energies of formation for
CO 2 and CO are –394.4 and –137.2 kJ mol −1, respectively.
2. For the reaction at 25°C, X 2 O 4 (l) → 2XO 2 (g)
∆E = 2.1 Kcal and ∆S = 20 cal/K.
Calculate ∆G for the reaction and predict whether the given reaction is spontaneous.
3. Two moles of a perfect gas undergo the following processes:
(i) A reversible isobaric expansion from (1.0 atm, 20.0 L) to (1.0 atm, 40.0 L);
(ii) A reversible isochoric change of state from (1.0 atm, 40.0 L) to (0.5 atm, 40.0 L);
(iii) A reversible isothermal compression from (0.5 atm, 40.0 L) to (1.0 atm, 20.0L).
(a) Sketch with labels each of the processes on the same P-V diagram.
(b) Calculate the total work (w) and the total heat change (q) involved in the above proceses.
(c) What will be the values of ∆U, ∆H and ∆S for the overall process?
P-95

Long Answer Type Questions 9. Define entropy and discuss its physical
Based on Carnot Cycle significance. How its is related to spontanity of any
reaction? [D.D.U. 2010]
1. Describe the working of Carnot cycle. Show that
the thermodynamic efficiency. η of the cycle is Based on Gibbs Free Energy, G
given by: 10. Derive Gibbs-Helmholtz’s equation in terms of free
T – T1 energy at constant pressure. [Avadh 2009]
η= 2
T2 11. (i) Describe free energy functions.
2. (i) Prove that efficiency of Carnot engine is (ii) Two moles of an ideal gas are allowed to
given by, expand reversibly at constant temperature
T − T1 300 K from a pressure of 1 atm. to a
η= 2
T2 pressure 0.1 atm. Calculate the change in
free energy.
where, T2 = temperature of source, T1 =
(R = 8.314 JK −1 mol −1)
temperature of sink
[Garhwal 2010]
(ii) Why is heat of neutralization of a strong acid
12. Show that following thermodynamic relations:
by a base always same whereas it is less if
 ∂V  = −  ∂S 
either the acid or the base or both are weak?    
 ∂T  P  ∂P  T
[Avadh 2008]
 ∂E  = T  ∂P  − P
Based on Entropy(s)    
 ∂V  T  ∂T  V [Avadh 2011]
3. Derive an expression for entropy change for an ideal
13. (i) Write down the condition of spontaneity and
gas associated with temperature and pressure.
equilibrium in terms of entropy change.
[Avadh 2010]
(ii) Calculate (∆S) fusion of 1 mol of ice at 0°C
4. What is entropy? Calculate the entropy change in and 1 atm. Given that ∆H fusion of ice is 6
heating of 1 mole of helium gas at 300 K to 400 K KJ. mol –1
at constant pressure. (Given C P = 5.0 cal deg −1
(iii) A sample of n moles of an ideal gas expands
mol −1) [Avadh 2011] isothermally and reversibly at a temperature
5. (i) Derive the concept of entropy from the second T so that the final volume is 10 times of its
law of thermodynamics and show that change initial value. Calculate the entropy change.
in entropy is a state function. [Alld. 2009]
14. (i) Derive an expression for the entropy change
(ii) Two moles of an ideal gas expands isothermally
associated with me change in temperature
and reversibly from 15 lit to 30 lit at 300 K.
and pressure of an ideal gas.
Calculate the change in entropy.
[Alld. 2010] (ii) Derive the following thermodynamic relations:
∂S ∂P ∂T ∂V 
6. Derive a relationship for entropy as a function of V (a)   =  (b)   = 
 ∂T  T  ∂T  V  ∂P  S  ∂S  P
and T. [Purv. 2011]
[Alld. 2009]
7. Explain the entropy for an ideal gas with change in 15. What is the concept of free energy? Deduce the
P, V and T. [Agra 2008] Gibbs–Helmholtz equation. [Alld. 2011]
8. (i) Show that for an irreversible process: 16. ClF 3 (g) can be prepared by the following reaction:
∆Ssys + ∆Ssurr > 0 Cl 2 (g)+3F 2 (g) →2Cl F 3 (g)
(ii) Prove that entropy of mixing of two gases in Given that ∆ f H° and ∆ f G° values of Cl F 3 (g) are
positive. [Lko. 2010] –163 KJ mol –1 and –123KJ mol –1 respectively:
P-96

(i) Calculate equilibrium constt. (ii)How is the concept of residual entropy


(ii) Calculate standard entropy change ∆S°. originated? How is it calculated? [Avadh 2011]
(iii) If Cl F 3 (g) is collected as liquid how would 24. State Nernst heat theorem. Describe the method
the sign and mangnitude of ∆S change? for determination of absolute entropy.
Explain. [Avadh 2008]
(iv) Standard absolute entropies of Cl 2 (g) and Cl
Based on Nernst's Distribution Law
F 3 (g) are 223 JK –1and 282 JK –1. Calculate
25. State and explain Nernst distribution law. Derive
absolute entropy of F 2 (g). [D.D.U. 2008]
the law thermodynamically. How can it be applied
17. (i) Mention the physical significance of entropy. to determine the solubility of solute in a particular
How does thermodynamic entropy differ from solvent? [Avadh 2010]
statistical entropy?
26. State distribution law. What are its limitations?
(ii) Considering entropy (S) as a function of V
Discuss application of this law.
and T show that:
[Garhwal 2005, 10]
∂S   ∂U  = P, and
T   – P =  27. Explain Nernst distribution law. Partition
 ∂V  T  ∂V  T
coefficient of succinic acid between water and
 ∂E  = T  ∂S 
    ether is 5.5 . How much succinic acid would be
 ∂T  V  ∂T  V [D.D.U. 2008] extracted by 50 ml ether from its 5% solution
18. (i) On the basis of first and second laws of thermo- (wt/V)? [Alld. 2009]
dynamics, prove that dG=VdP–SdT and find
28. State the Nernst distribution law? Derive this law
value of (dG) T.P .
thermodynamically. Mention two important
(ii)Predict conditions for the feasibility of a
applications of the law. [Alld. 2011]
thermodynamic process in terms of ∆G,∆H and
∆S values. [D.D.U. 2009] 29. What are the limitation of Nernst distribution law?
19. Show that: On the basis or this law discuss the theory of
∂(∆G) extraction process. [Alld. 2012]
∆G=∆H+T 
 ∂T  P 30. Derive a suitable expression to prove that multi
20. Prove any two of the following relations: step extraction is more economical than single step
∂(∆G) extraction. [Lko. 2011]
(i) ∆G=∆H+T 
 ∂T  P
(ii) C P –C V =R Short Answer Type Questions
(iii) PV r = constant Based on Carnot Cycle
[Agra 2008]
1. State and explain Carnot theorem and its
21. (i) State and explain Nernst heat theorem and third
significance. [Garhwal 2011]
law of thermodynamics.
(ii)Derive Gibbs-Helmholtz equation. [Lko. 2010] 2. What is Carnot cycle? How is it useful in calculating
the efficiency of heat engine? [Garhwal 2010]
22. Prove the following:
(i) –∆A = W max 3. Derive an expression for the efficiency of a Carnot
(ii) − ∆G = Wmax − P∆V engine working between two temperatures T 2 and
T 1. [Garhwal 2007]
(iii)  ∂A  =  ∂G 
   
 ∂T  V  ∂T  P [Avadh 2011] 4. What do you understand by Carnot Cycle? With
Based on Nernst Heat Theorem and third law of the help of this cycle derive a relation for the
thermodynamics efficiency of heat engine, calculate the efficiency of
heat engine working between 110°C and 25°C.
23. (i) State third law of thermodynamics. How does
[Garhwal 2006]
this law help in the determination of absolute
entropies of chemical compounds at any 5. State Carnot theorem.Whether it is possible to
desired temperature? convert heat completely into work? [Lko. 2009]
P-97

6. Give Carnot theorem. [Avadh 2011] 18. Calculate change in entropy for vaporization of 1
7. A Carnot engine operates between two near mole of ethanol at 351 K. Molar heat of
reservoir at 27°C and 27°C, and absorbs 1000 cal vaporization of ethanol is 9522 cal mol −1.
[Garhwal 2008]
from the reservoir at higher temperature. How
much work is done per cycle and what is the 19. Define entropy. Discuss the entropy of mixing of
efficiency of the engine? ideal gases. [Garhwal 2008]
[Alld. 2010]
20. 5 moles of an ideal gas are expanded isothermally
8. Write note on efficiency of Carnot cycle.
[Meerut 2011]
and reversibly to twice its volume. Calculate ∆S.
[Garhwal 2007]
9. State and explain Carnot cycle and Carnot
21. Derive an equation representing the change of
theorem. [Agra 2008]
entropy for an isothermal expansion of an ideal
10. Efficiency of Carnot's cycle is maximum. Justify. gas. [Garhwal 2005]
[Kanpur 2010,12]
22. Justify the statement that ‘entropy is a measure of
11. It is possible to construct a reversible heat engine of disorder of the system’. [Avadh 2010]
unit efficiency? Give reasons for your answer.
23. Show that entropy change in an isothermal
[Lko. 2008]
transformation is given by:
Based on Second Law of Thermodynamics ∆H
∆S =
12. What is Clausius inequality? Show for spontaneous T [D.D.U. 2011]
cooling dS total > 0 . [Agra 2008] 24. Discuss the entropy change in irreversible process.
Based on Entropy(s) [Kanpur 2008]

13. Calculate the entropy change in the melting of 0.1 25. Derive an equation for entropy change in
gm of ice at 0°C. Latent heat of fusion of ice at 0°C isothermal expansion of an ideal gas.
is 80 cal per gram. [Garhwal 2012]
or
Drive an equation for free energy change of an
14. State which of the following will have more entropy
ideal gas in isothermal expansion. [Agra 2009]
and why?
26. Four moles of an ideal gas expands isothermally
(i) Ice or water
from 1 litre to 10 litres at 300K. Calculate change in
(ii) 1 mole of O 2 at 25°C and at 45°C free energy and entropy. (R=8.314 JK –1mol –1)
(iii) 1 mole of CaCO 3 (solid) or mixture of CaO [Lko. 2009]
(solid) and CO 2 (gas) having same moles. 27. Entropy of a substance is maximum in gaseous
(iv) 1 mole gas placed in a container of 1 litre or state Justify. [Lko. 2010]
2 litre capacity. [Garhwal 2012] 28. Calculate the change in entropy accompanying the
15. Explain the change of entropy in irreversible heating of one mol of helium gas, assumed ideal,
process. [Garhwal 2009] from a temperature of 298 K to a temperature of
1000K at constant pressure. Assuming that
16. 1 mole of water at 1 atom, pressure, vapourises at
C V = 3 / 2R . [Alld. 2011]
100°C. This vaporization absorbs 9710 cal/mole of
heat. Calculate the change in entropy in the Based on Gibbs Free Energy, G
process. [Garhwal 2009] 29. Derive : dG = VdP – SdT [Garhwal 2012]
17. Explain with the help of equation ∆G = ∆H – T∆S 30. Derive Gibbs– Helmholtz equation.
whether a chemical reaction would be possible or [Garhwal 2011]
not in following cases : 31. The ∆G for a reaction at 300 K and 310 K are
Case I : When ∆H = positive and ∆S = positive –127.7 KJ mol −1 and –107.9 KJ mol −1 respecti-
Case II : When ∆H = negative and ∆S = negative vely. Calculate ∆H for the above reaction at 310 K.
[Garhwal 2010]
Case III : When ∆H = positive and ∆S = negative
32. Complete the following equations :
Case IV : When ∆H = negative and ∆S = positive
[Garhwal 2009]
P-98

q −q q 49. Write notes on the following:


(i) 2 1 = ……… (ii) ……… = rev
q T (i) Residual entropy
2
(ii) Third law of thermodynamics [Purv. 2010]
(iii) A = ………. – TS (iv) dG = ………
[Garhwal 2009] 50. Benzene acid is distributed between benzene and
33. Explain the terms free energy and work function. water it forms dinner in benzene. What is the
Differentiate between the two. [Garhwal 2007] formula for partition co-efficient in the case?
[Lko. 2008]
34. Derive Gibbs-Helmholtz equation in terms of free
energy. [Garhwal 2006]
Based on Nernst's Distribution Law
51. In the distribution of benzoic acid between water
35. Write short notes on the following :
and benzene, it distributed itself between the two
(i) Helmholtz function. [Avadh 2010]
layers as follows:
36. Show that :
Aqueous layer (in grams) : 1.50 1.95 2.97
∂(∆G) 
∆G = ∆H + T  Benzene layer (in grams) : 24.20 41.20 97
 ∂T  P [Avadh 2010] Assuming that benzoic acid exists as a simple
37. Using the first and second laws of thermodynamics, molecule in water, decide the molecular state of
prove that: dG = − SdT + VdP [Avadh 2009] benzoic acid in benzene. [Garhwal 2012]
38. Write short notes on fugacity. [Garhwal 2006] 52. Derive thermodynamically the Nernst distributive
39. Considering entropy as a function of T and P show Law. [Garhwal 2011]
that: 53. Define Gibbs free energy and derive the equation
 ∂S  =C P , and  ∂S  = 1   ∂H  – V  for Nernst distribution law. [Garhwal 2011]
     
 ∂T  T  ∂P  T   ∂P  T 
54. Explain Nernst law of distribution. How partition
[Alld. 2009] coefficient of iodine between CCl 4 and water is
40. Show that: determined? [Garhwal 2008]
∂S ∂G  55.
(i)   = –S (ii)   =V
State distribution law. How the distribution law is
 ∂T  P  ∂P  T [Alld. 2009] modified when solute undergoes association on
41. Show that: one of the solvents? [Garhwal 2007]
 ∂T  =  ∂V  and dG=–SdT+VdP 56. Explain Nernst distribution law and is applications.
   
 ∂P  S  ∂S  P 57. Benzoic acid is distributed between benzene and
[Alld. 2009]
water. If forms a dimer in benzene. What is the
42. What is Helmholtz function? [Kanpur 2008] formula for partition law in this case? [Lko. 2008]
43. What does the term (G) T.P.<0 indicate? 58. Give condition of applicability of Nernst
[Purv. 2010]
distribution law. [Lko. 2009]
44. Give the concept of free energy with example. How
is related with enthalpy? [Purv. 2010]
Very Short Answer Type Questions
Based on Carnot Cycle
45. Show that ∆G=∆A for an isothermal expansion of
1. Can the efficiency of a heat engine be greater than
an ideal gas. [Lko. 2010]
unity? If not, why? [Lko. 2011]
Based on Nearnst Heat Theorem and Third law of
Based on Second Law of Thermodynamics
Thermodynamics
2. State second law of thermodynamics.
46. Write the concept of residual entropy?
[Meerut 2010, 11]
[Avadh 2008]
Based on Entropy (s)
47. Describe concept of residual entropy. [Purv.2008]
3. Find entropy change when liquid is vaporized.
48. (i) State Nernst heat theorem. Describe the method [Meerut 2009]
for determination of absolute-entrophy.
4. Define entropy. What are its unit? [Kanpur 2012]
(ii) Prove the following:
5. What is the mathematical statement of entropy?
(a) dG=VdP-S.dT [D.D.U. 2009]
(b) ∆H = ∆E+P∆V [Purv. 2009] 6. Define entropy. What are its uses? [Kanpur 2009]
P-99

7. Define entropy. What are its units? [Kanpur 2010] 4. Which one has the highest entropy?
8. Define entropy. [Purv. 2009] (a) Ice (b) Water
Based on Gibbs Free Energy, G (c) Water vapours (d) Piece of iron
9. Derive Gibbs-Helmholtz equation. What are its [Garhwal 2008]
main applications? [Kanpur 2011] 5. At constant temperature, for reversible process in
10. Write in brief Helmholtz function. [Kanpur 2012] equilibrium is the correct relation is :
11. Explain in brief about Gibbs function. q q
(a) ∆S > (b) ∆S <
[Kanpur 2012] T T
12. Write equation for relation between free energy q
(c) ∆S = (d) None of these
change and enthalpy change of a reaction. T [Garhwal 2007]
[Garhwal 2007]
6. The standard entropies of N 2 (g), H 2 (g) and
13. Write the equation for the mutual relation of
NH 3 (g) are 191.5, 130.5 and 192.6 J K −1 mol −1
equilibrium constant and free energy.
[Garhwal 2006] respectively. The value of ∆S° of formation of
ammonia is
14. Write the equation for free energy change in terms
of volume for one mole of an ideal gas. (a) −98.9 J K −1 mol −1 (b) Zero
−1 −1
[Garhwal 2005] (c) +129.4 J K mol (d) −29.4J K −1 mol −1
15. Write in brief about Gibbs function. 7. Identify the correct statement regarding entropy.
[Avadh 2010]
(a) At absolute zero of temperature, the entropy
16. Show how Gibbs free energy can be used a
of perfectly crystalline substance is positive.
criterian for the spontaneity of a chemical change?
(b) At absolute zero of temperature entropy of
[Avadh 2009]
perfectly crystalline substance is taken to be
17. For a given process A→B; ∆ r H° =–102KJ and
zero.
∆ r S°=–330KJ –1. Is this process spontaneous?
(c) At 0°C the entropy of a perfectly crystalline
Explain. [D.D.U. 2008] substance is taken to be zero.
18. What is the criteria of spontancity of a chemical
(d) At absolute zero of temperature, the entropy
reaction in terms of its free energy change?
of all crystalline substance is taken to be
[D.D.U. 2011]
negative.
Based on Nearnst Heat Theorem and Third law of
Thermodynamics 8. In a reversible process ∆Ssys + ∆Ssurr is:
19. State third law of thermodynamics. (a) 0 (b) ≥ 0 (c) ≤ 0 (d) <1
[Meerut 2012] [Agra 2008]
Objective Type Questions 9. Which one of the following is the correct equation ?
(a) ∆G = ∆H + T∆S (b) ∆G = ∆H – T∆S
Multiple Choice Questions (c) ∆G = T∆S – ∆H (d) ∆G = ∆H – S∆T
1. The maximum efficiency n of an engine working
[Garhwal 2010]
between temp. T 1 and T 2 is :
T − T1 10. For spontaneous reaction ∆G is :
(a) η = 2 (b) η = T1 × T2
T2 (a) > 0 (b) < 0 (c) ≥ 0 (d) ≤ 0
T − T2
(c) η = 1 (d) None of these [Garhwal 2010]
T1 [Garhwal 2011] 11. When enthalpy and entropy change for a chemical
2. The entropy change of a spontaneous process is : reaction are −2.5 × 103 cals and 7.4 cal deg −1
(a) Positive (b) Negative respectively. Predict that reaction at 298 K is:
(c) Zero (d) None of these
(a) Spontaneous (b) Reversible
[Garhwal 2009] (c) Irreversible (d) Non-spontaneous
3. In the reversible isothermal expansion of a gas, the
total entropy change of the system and its 12. The efficiency of the heat engine is always less
surroundings is : than:
(a) More than 1 (b) Less than 1 (a) 0 (b) unity (c) T (d) P
(c) Zero (d) Only 1 [Agra 2008]
P-100

2. The second law of thermodynamics does not


Fill in the Blank(s) restrict the direction of flow of heat.
1. The efficiency of heat engine is always ……… than [Garhwal 2008]
unity. 3. Fall of water from clouds in spontaneous.
[Garhwal 2010] [Garhwal 2011]
2. Formula to calculate change in entropy at constant 4. All natural processes are spontaneous process.
temperature for an ideal gas is ……… . [Garhwal 2010]
[Garhwal 2010] 5. In spontaneous process, entropy increases.
3. In S.I. system, the unit of entropy is ……… . [Garhwal 2010]
[Garhwal 2009] 6. The entropy of nature always tends towards
4. When liquid is converted into vapour, then the minimum. [Garhwal 2006]
value of entropy ……… . [Garhwal 2006] 7. At equilibrium the change in work function is zero.
[Garhwal 2010]
5. A = ……… – TS [Garhwal 2012] C1
8. Distribution coefficient, K = is applicable when
6. The partition coefficient is the ratio of ……… of a C2
solution in two immisible solvents at constant
complex compound is formed. [Garhwal 2010]
……… when the system is in equilibrium.
[Garhwal 2012] 9. If solute forms dimer in phase II, then the
C
distribution law is 1 = K.
True and False C2 [Garhwal 2012]
1. The efficiency of the Carnot cycle does not depend
on the nature of the working substances.
[Garhwal 2009]

Objective Type Questions


Multiple Choice Questions
1. (a) 2. (a) 3. (a) 4. (c)
5. (c) 6. (a) 7. (b) 8. (a)
9. (b) 10. (b) 11. (a) 12. (b)

Fill in the Blank(s)


1. less 2. V 3. J/K
∆S=nR ln 2
V1

4. increases 5. U 6. concentrations, temperature

True and False


1. True 2. False 3. True 4. True 5. True
6. False 7. True 8. False 9. True
P-101

Hints & Solutions

Numerical Questions
1. Use ∆G = ∆H – T∆S to find ∆H
When ∆G=(∆G f ) CO –(∆G f ) CO .
2
2. 2.67 kcal; spontaneous
Step 1. ∆H= ∆E+(∆H)RT. (Here ∆H=+2).
Step 2. ∆G= ∆H–T∆S
3. (i)

Fig. 7
(ii) w = 620.14 J; q = 620.14 J
(iii) ∆U = 0 ; ∆H = 0 and ∆S = 0

Long Answer Type Questions


T 400
3. ∆S =C P ln 2 = 5 × 2.303 × log
T1 300
=1.438 cal deg –1mol –1.
30
4. (ii) ∆S = 2 × 8.314 × 2.303 log = 11.53 JK –1mol –1
15
10. P 0.1
(ii) ∆G=nRT ln 2 = 2 × 8.314 × 300 × 2.303 × log
P1 1
=–11.488 KJ.

∆H fusion 6000
12. (ii) ∆Sfusion = = = 21.98 JK –1mol –1
Tm 273
V
(iii) ∆S = nR ln f = nR ln 10 =2.303 (nR)
Vi
15. (i) ∆G ° = –2.303 RT log10K
123 × 103
= log 10 K. ⇒ K=356×10 21
2.303 × 8.314 × 298
(ii) –123 × 103 + 163 × 103
∆G ° = ∆H ° –T∆S° ⇒ = ∆S°.
298
∆S° = 134.23 JK –1mol –1.
(iii) As the process becomes more exothermic, ∆S° still remains positive with a larger magnitude.
(iv) (∆S° )reaction = 2(S° ClF ) – [(S° Cl ) + 3(S° F )]
3 2 2
P-102

134.23 = 2×282 – [223+3x]


S° F = x = 68.92 JK –1mol –1
2
w1 /100
26. Kd = ; here w 1 is the amount of succinic acid leftover in water, so that 5-w, is the amount extracted
(5 – w1) / 50
into ether.
Putting value of K d = 5.5 , we find w 1.

Short Answer Type Questions


383 – 298
4. η= = 0.22
383
10. (i) water
(ii) at 45°C
(iii) Mixture of CaO and CO 2 .
(iv) 2 litre container
9710
12. ∆Svap = = 26.03 cal mol –1K –1
373
9522
14. ∆Svap = = 27.13 cal mol –1K –1
351
V
16. ∆S = nR log c f = 5 × 8.314 × 2.303 × log10 2
Vi
= 28.82 JK –1mol –1
T 5 1000
24. ∆S =C P ln 2 = × R × 2.303 log
T1 2 298
⇒ ∆S =25.17 JK –1mol –1 (QC P – C V =R )
∂∆ G 
27. As it is known, ∆G = ∆Η + Τ  
 ∂T  P
 ∂∆G  = (∆G)310K – (∆G)300K
 
 ∂T  P 10
–107.9 + 127.7
= = 1.98 KJ/K.
10
∆H = ∆G310 – 1.98 × 310 = –107.9 × 103 – 613.8
= –108.51 KJ mol –1
28. (i) η
(ii) ∆S
(iii) U
(iv) VdP–SdT
C1
46. In case of association, = Kd
nC
2
1.5 1.95
So, = Solving n = 2.
n 24.2 n 41.2

Very Short Answer Type Questions


17. To check spontaneity, ∆G must be negative.
∆G = ∆H – T∆S ⇒ ∆G = –102 × 103 + 298 × 330
= –21.66KJ
As ∆G is negative, the process is spontaneous.

mmm
UnitP-103
-III

C HAPTER 4
Electrochemistry-I

4.1 Introduction and basic terms


Electrochemistry deals with the study of electrochemical processes and their applications which involve the
conversion of electrical energy into chemical energy and the conversion of chemical energy into electrical
energy. It is possible to carry out a non spontaneous chemical reaction by applying sufficient electrical voltage
in an electrochemical cell. In this case, the electrical energy is converted to chemical energy, for example,
preparation of sodium metal by Down's process. Similarly, it is also possible to produce electrical energy by
carrying out a spontaneous chemical reaction in a suitable device called galvanic cell or voltaic cell, for
example, batteries and fuel cells. Some of the fundamental terms used in electrochemistry are discussed here.

4.1.1 Different Terms


1. Electricity: The term electricity refers to a form of energy or variety of phenomenon resulting from the
accumulation of static charges or flow of charged particles.
2. Electric charge: The electric charge is the fundamental and conserved property of some subatomic
particles which determines their electromagnetic interactions. It exhibits two states - positive and
negative. In SI units, the electric charge is measured in Coulombs (C). The smallest value of “free”
charge known in nature is the charge of an electron or a proton, which has a magnitude of 1.602 × 10
−19
Coulomb. The charge on one electron is: −1.602 × 10 −19 C, whereas the charge on a proton is
equal to +1.602 × 10 −19 C.
3. Electric field: It is an influence produced by an electric charge on other charges in its vicinity. A
charged particle experiences force when placed in an electric field. The electric field exerts a force on
charged particles, accelerating them in the direction of the force, in either the same or the opposite
direction of the electric field. Hence some amount of work has to be done against this force to bring the
charged particle to a certain point in an electric field.
4. Electric potential: The electric potential at any point in an electric field is defined as the energy
required to bring a unit charge from an infinite distance to that point. It is usually measured in volts (V).
Volt can be formally defined as the ratio of energy in joules to the charge in coulombs. The absolute
electric potential cannot be measured. However, the difference in electric potential between two points
can be measured and has more practical usage.
5. Electric potential difference or voltage: It is the energy required to move a unit charge between
two specified points in an electric field. However, it is usually referred to as electric potential or some
times as voltage.
P-104

Energy required in joules


Electric potential difference (Voltage) =
charge in coulombs

One Volt (V) is equal to 1 Joule per 1 Coulomb, or,


1 Joule (J)
1 Volt(V) =
1 Coulomb (C)

6. Electromotive force (emf): Formally, it is same as electric potential difference. It is the potential
energy divided by electric charge and is measured in volts. It can be defined as the external work done
per unit of charge to produce an electric potential difference thereby causing the current to flow in the
circuit.
7. Current: Formally, it is the flow of charged particles like electrons, protons, ions. The strength of
current (I) is defined as the charge transferred in unit time. It is measured in amperes (A).
Charge(Q)
Current (I) =
Time (t)

One ampere (A) is defined as 1 coulomb of electricity transferred in one second.


1 Coulomb (C)
1 Ampere (A) =
1 Second (sce)

4.1.2 Ohm's Law


Ohm’s law states that the current through a conductor between two points is directly proportional to
the potential difference across the two points. Introducing the constant of proportionality, the resistance, one
arrives at the usual mathematical equation that describes this relationship:
V
I= ,
R
here I is the current through the conductor in units of amperes, V is the potential difference
measured across the conductor in units of volts, and R is the resistance of the conductor in units of ohms.
More specifically, Ohm’s law states that the R in this relation is constant, independent of the current.
The law was named after the German physicist Georg Ohm who described measurements of applied voltage
and current through simple electrical circuits containing various lengths of wire.
Resistance (R): The tendency of a material to stop the flow of current is known as resistance. It is measured
in ohms (Ω). According to Ohm’s law, the resistance offered by a substance is directly proportional to its
length (l), but inversely proportional to its cross sectional area (A).
1
R∝
A
In case of electrolytic solutions, ‘l’ represents the distance between two electrodes and the 'A' is the cross
sectional area of the electrodes. The above equation can also be written as:
l
R=ρ …(1)
A
Here ρ is the proportionality constant and is known as specific resistance or resistivity.
If A = 1 cm 2 and l=1cm, then R = ρ.
P-105

4.1.3 Types of Substances on the Basis of Electrical Conductivity


Based on electrical conductivity, the materials are divided into two types as follows:
1. Insulators: The substances which resist the flow of electric current through them are called insulators.
They do not have free electrons or freely moving charged particles. For example, organic polymers
(like plastics), glass, diamond, quartz etc.
2. Conductors: The substances which allow the flow of electricity through them with little resistance are
known as conductors. The conductors are divided into two types:
(i) Metallic or electronic conductors: The conductors which conduct the electricity through
the electrons. For example, all metals, graphite etc. In case of metallic conductors;
(a) No chemical reaction occurs during the conduction of electricity.
(b) Conductivity decreases with increase in temperature due to vibrational disturbances.
(ii) Electrolytes: The substances which furnish oppositely charged ions for the conduction of
electricity. Some of the examples are NaCl, KCl, CH 3 COOH, HCl etc. In case of electrolytes;
(a) There is flow of ions towards the oppositely charged electrodes.
(b) During conduction of electricity through electrolytes, oxidation occurs at anode whereas
reduction occurs at cathode. So that it can be said that a chemical reaction occurs.
(c) The conductivity increases with increase in temperature as the extent of ionization
increases.
The electrolytes undergo dissociation to furnish ions either in molten state or in aqueous solutions.
Depending on the extent of ionization in water, the electrolytes are further divided into two types:
1. Strong electrolytes: These undergo complete ionization in water, some of the examples are NaCl,
KCl, K 2 SO 4 , HCl, H 2 SO 4 ,NaOH, NaNO 3 etc.
2. Weak electrolytes: These undergo partial or feeble ionization in aqueous solutions. Some examples
of weak electrolytes are HF, CH 3 COOH, NH 4 OH, HCOOH etc.
Non-electrolytes: The substances which do not furnish ions for electrical conduction are called
non-electrolytes. Examples are urea, glucose, sucrose etc.

4.2 Specific Conductance, Equivalent Conductance and Molar Conductance


In a metallic conductor, the length (l), cross-sectional area (a) and the resistance (R), are inter related as
below,as stated previously,
R ∝l
1
and R∝ ; on combining these two, it can be said, that,
a
l l
R∝ ;R =ρ
a a
Here ρ is the specific resistance. It is defined as the resistance of unit length of a conductor of unit
cross−section.
P-106

4.2.1 Specific Conductance or Conductivity (κ)


The reciprocal of specific resistance is termed the specific conductance or conductivity. It is the conductance
of one centimeter cube of a conductor. It is denoted by the symbol k and read as kappa. Thus,
κ = 1/ρ …(1)
l
Specific conductance is also called conductivity. It is known that, R = ρ ×
a
1 1 l
So, = ×
ρ R a
or, κ = (l/a) × C
Here, l/a = cell constant. l represents the the length of the liquid column between two electrode or simply the
distance between two electrode and a is the cross sectional area of the electrodes or the area of one electrode
presuming both have the same area. The cell constant is not found by measuring distances but is determined
by measuring the conductance of an electrolyte whose specific conductivity is already known.
So that, specific conductance = conductance × cell constant
As stated earlier, in the case of electrolytic solutions, the specific conductance is defined as the conductance
of a solution of definite dilution enclosed in a cell having two electrodes of unit area separated by one
centimeter apart as shown in figure below. In the diagram a unit cube is shown and the corresponding
conductance data is specific conductance.
-C
The unit of specific conductance is ohm −1 cm −1 or S cm −1 (S ath
od
represent Siemen). The major drawback of the term specific + e
An

conductance is that it does not involve the concentration


od

term. As such, the conductance of a solution depends upon


e

Anode
the number of ions present and hence on the concentration. -
To compare the conductivity of solutions, having different Cathode + Solution
concentrations. It is necessary to take the concentration of the
solutions into consideration. It is done by using terms like 1c
m
equivalent conductance −Λ e and molar conductance −Λ m . 1cm
Fig.1: Conductance through unit cube
4.2.2 Molar Conductance ( Λ m )
Molar conductance is the conductiong ability of all the ions produced by 1 mole of an electrolyte present per
litre of a solution. It measures the efficiency with which a given electrolyte conducts electricity in solution. The
normally used symbol is Λ m .
Molar conductance Λ m is defined as the conductance of a solution having concentration of solution as 1 M or
1mole litre −1. The solution is being placed between two electrodes kept 1 cm apart. Its unit is Siemens meter
squared per mol or S m 2 mol −1 or Scm2 mol −1. Mathematically,
κ
Λm =
concentration
−1
So, Λ Μ = 1000 κ / molarity S mol
cm2

1000κ ohm −1 cm −1 cm 3
Derivation of unit of Λ m : Λm = =
M mol
P-107

= ohm −1cm 2mol −1


= Ω −1cm 2mol −1
= S cm 2mol −1

4.2.3 Equivalent Conductance


The equivalent conductance is a parallel term to molar conductance and it is defined as the conductance of a
solution containing 1 equivalent or 1 gm-equivalent of the dissolved electrolyte present per litre of solution.
Moreover, the entire solution is placed between two electrodes which are 1 cm apart. As direct determination
of this quantity would need electrodes of enormous sizes, the equivalent conductance (Λ e ) is always
evaluated through measurement of specific conductance (κ) and putting the value in the equation,
Λ e = 1000 κ /normality
here, κ is the specific conductance.
Derivation of unit of Λ e : As suggested by its mathematical definition, the unit is siemens meter squared
per equivalent or S m 2 equiv −1.Dimensionally,Λ e is given by

ohm −1. cm −1
Λe =
equivalents. cm −3

= cm 2 . ohm −1. equiv −1 = cm 2 . mho. equiv −1


= m 2 . Siemens. equiv −1
The relation between equivalent conductance and molar conductance is given by,
κ κ × 1000
Λe = × 1000 =
N n−f ×M
κ × 1000 1
⇒ Λe = ×
M n − factor
ΛΜ
⇒ Λe =
n − factor
molar conductance
Equivalent conductance =
n − factor
Molecular mass
where, n − factor =
Equivalent mass

4.2.4 Measurement of Conductance


It is now known to us that when the solution of an electrolyte is taken between two parallel electrodes, each
having cross-sectional area ‘a’ and ‘l’ cm apart, then the specific conductance, κ, is:
l 1
κ= ×
a R
Thus, knowing the values of R, l and a, the specific conductance can be measured. The resistance of the
solution between two parallel electrodes is determined by using Wheatstone bridge method. The diagram of
the apparatus is shown in fig. 2.
P-108

Resistance box Conductivity


R cell

X
A T B
Telephone head
Thermostate
Secondary Test solution
S Primary

Induction coil
Fig. 2: Determination of conductivity

AB is a uniform wire and X is a sliding contact which moves over it. C is the conductivity cell containing the
solution of the electrolyte and S represents the source of alternating current. R is the resistance box and T is a
headphone to detect the flow of current. A suitable resistance is taken out from the resistance box and the
sliding contact X is moving on the wire to search a point of minimum sound in the headphone. At this point,
the bridge is balanced. It can be said that,
(Resistance of solution)/(Resistance from resistance box)
=(Resistance XB)/(Resistance XA)
=(Length XB)/(Length XA)
Thus, resistance of solution can be used although there are complications like
1. Change in the concentration of the solution due to electrolysis which will change the resistance.
2. Polarisation at the electrode sets.
Thus, an alternating current (A.C.) is used to overcome the above complications. The solution whose
conductance is to be measured is taken in a special type of cell known as conductivity as conductivity cell.
Various types of conductance cells are shown in the fig.3 below:
Generally the electrodes consist of platinum discs coated with
finely divided platinum black and welded to platinum wires
fused in two glass tubes. The glass tubes contain mercury and
are finely fixed in the cover of cells. Contact with the platinum
is made by dipping the copper wires of the circuit in the
mercury contained in the tubes. As the conductivity changes
with temperature, the cell is usually placed in a constant (a) (b) (c)
temperature bath during the experiment. Cells with long Fig.3: Various types of conductance cells
paths are used for concentrated solution and cells with short
paths and large electrodes are used for dilute solutions.

4.2.5 Cell Constant


Since the electrodes are not exactly 1 unit apart and may not possess a surface area of 1 square unit, the
measured resistance does not give the specific conductance of the solution. Actual measurements of l and a
P-109

are inconvenient. So, an indirect method is employed to determine the value of cell constant. It is known
that,

(Specific conductance) / (Conductance) = l/a = cell constant

The resistance of cell or its conductance is measured when filled with a standard solution (say N/10 KCl
solution) at a given temperature. The standard values of specific conductance of KCl solutions of various
concentrations at different temperature are known. Thus, the cell constant is calculated by using the above
equation. The same cell constant applies to a measurement with any other solution. The determination of
specific conductance of an electrolytic solution, consists of two steps:

1. Determination of cell constant by using a standard KCl solution of known concentration in the
conductivity cell.

2. Determination of resistance of the given solution using the same cell

The reciprocal of this resistance gives the value of conductance. Multiplication of conductance and cell
constant gives the value of specific conductance of the solution. In order to determine equivalent
conductance or molar conductance, the concentration of the experimental solution should be known. In
conductance measurements, the solutions are always prepared in conductivity water which has no
conductance due to dissolved impurities. It is prepared by distilling a number of times the distilled water to
which a little KMnO 4 and KOH have been added in a hard glass distillation assembly. Such water has very
low conductance of the order of 4.3 × 10 −8 ohm −1. For ordinary purposes, double distilled water may be
used.

4.2.6. Effect of dilution on conductance


Effect of dilution of a solution on its conductivity is discussed here. The specific conductivity (κ) decreases on
dilution as number of ions decreases with respect to volume. The equivalent and molar conductivities both
increase with increase of dilution. The equivalent and molar conductances tend to acquire maximum value
with increasing dilution, such that maximum value is obtained at ∞ dilution or zero concentration.

Explanation: In case of weak electrolytes, Λ m increases with dilution because of enhanced dissociation of
weak electrolyte into ions with dilution. Oswald dilution law states that as the dilution increases, the degree of
dissociation also increases hence the number of ions also increases. Increase of number of ions ensures that
Λ Μ of weak electrolyte increases with increasing dilution. However, this line of explanation does not hold
good for strong electrolyte. It is because for strong electrolyte. The degree of dissociations (∝) remain always
1, so dilution has no impact on ∝.

In case of strong electrolytes, molar conductance Λ m increases on dilution as inter-ionic attraction decreases
along with dilution. For an electrolyte, Λ Μ is expressed by Debye-Huckel Onsager equation as follows:
Λ m = Λ°m − b c (In place of Λο one can also use Λ ∞ )

Here, ΛoM = molar conductance at infinite dilution or limiting molar conductance. Λ M = Molar conductance
at a given dilution. b = It is a constant which depends upon nature of solvent and temperature, c =
concentration. In the following graph, equivalent conductances of some strong and weak electrolytes are
plotted against concentration.
P-110

Observation from the graph


1. Electrolytes like KCl have high value of conductance even at low concentration and there is no rapid
increase in their equivalent conductance on dilution. Such electrolytes are termed as strong
electrolytes. In the case of strong electrolytes, there is a tendency for equivalent conductance to
approach a limiting value when the concentration approaches zero. When the whole of the electrolyte
has ionized, further addition of the water does not bring any change in the value of equivalent
conductance. This stage is called infinite dilution. The equivalent conductance has a limiting value at
infinite dilution and is represented by Λ ∞ .
2. Electrolytes like acetic acid have a low value of Λ Μ or Λ e at high concentration and there is a rapid
increase in the value of equivalent conductance with dilution. Such electrolytes are termed weak
electrolytes. There is no indication that a limiting value of equivalent conductance can be attained even
when the concentration approaches zero. Thus, graphically, Λ ∞ of weak electrolytes cannot be
obtained. It is thus concluded that equivalent conductance of electrolytes whether strong or weak
increases with dilution and reaches to a maximum or limiting value which is termed as Λ ∞ (equivalent
conductance at infinite dilution). Λ ∞ in the case of strong electrolytes can be obtained by extrapolation
of the graph of equivalent conductance to zero concentration but in the case of weak electrolytes it
cannot be obtained accurately. An indirect method for obtaining Λ ∞ or Λ° for weak electrolyte has
been given by Kohlrausch. It is known as Kohlrausch law.

4.2.7 The Debye-Huckel Theory of Strong Electrolytes


The Debye–Huckel equation provides a starting point for modern treatments of non–ideality of electrolyte
solutions. A description of Debye–Huckel theory includes a very detailed discussion of the assumptions and
their limitations as well as the mathematical development and applications. Debye–Huckel treatment deals
with the distribution of ions around a given ion and the combined effects of these neighboring ions have on
the central ion. Debye and Huckel derived an equation based on the quantitative treatment of inter ionic
attraction. This equation was later on modified by Onsager and the eventual expression is known as
Debye–Huckel-Onsager (DHO) equation for strong electrolyte.
According to this theory, strong electrolytes which exist as ions even in the solid state must be completely
ionized in solution at all concentrations. If the solvent has a high dielectric constant like water, the electrostatic
P-111

forces will be small. Further, if the solution is very dilute, the distance between the ions will be large.
Therefore, the electrostatic forces which vary inversely as the square of the distance between the ions will be
weakened all the more. Under these circumstances, the forces of attraction between the oppositely charged
ions, or in other words inter ionic forces, will become very small. Consequently, the ions will lie far apart from
one another. If, on the other hand, the solvent has low dielectric constant like ethanol or if the solution is of a
high concentration even when a high dielectric constant solvent like water is used, the inter ionic forces will be
appreciably high. Under these circumstances, some of these ions will not separate out completely from one
another and will remain in pairs, such as A B, known as ionic doublets. This state of solution is expressed by
the statement that the electrolyte is completely ionized but not completely dissociated. As the solution is
diluted, its conductance increases. Arrhenius attributed this increase to increase in the degree of dissociation
and used it as a method of calculating degree of ionization of an electrolyte by the equation.
Λm
α=
0
Λm
However for strong electrolytes, the degree of dissociation of strong electrolytes is unity even at moderate
concentrations. Therefore, the above relationship is not correct in the case of strong electrolytes. Debye and
Huckel suggested that increase in conductance with dilution in the case of strong electrolytes is due to
increase in the mobilities of ions due to weaker interionic attraction. So, increase in conductance with dilution
in the case of strong electrolytes has nothing to do with the increase in the degree of ionization. To understand
the inter ionic effect, a very dilute solution of sodium chloride is considered. The ions will be so far apart in this
case that the forces of attraction between Na + and Cl − ions will be almost negligible. The ion would behave
independently of each other. Now the concentration of NaCl is increased. The ions now come closer to one
another, and the inter ionic forces of attraction are no longer negligible. In other words, the ionic mobility falls
due to force of attraction between oppositely charged ions. Their contribution towards electrical conductance
decreases correspondingly. As the concentration is increased further, the oppositely charged ions come
closer all the more. The inter ionic forces, therefore, become still more appreciable and the ionic mobility
decreases further.
Debye and Huckel derived an equation which enabled them to calculate the magnitude of the inter ionic
effects. The fundamental idea underlying the calculations is that each ion is surrounded by ions of opposite
charge giving rise to an ionic atmosphere. The formation of the ionic atmosphere is explained as follows. Two
opposing factors control the state of affairs in solutions of electrolytes:
1. The coulombic interaction which tends to arrange the ions in an ordered and organized structure.
2. Numerous thermal collisions between the ions and solvent molecules which tends to prevent the
existence of an organized structure in solution.
At higher temperature, the structure is highly dis−organized because of increased thermal collisions. As a
result of above two opposing factors a situation arises when the negative ions end up as the nearest neighbors
of a given central positive ion and vice versa. Thus a cation is
surrounded by more anions than cations.
This gives rise to ionic atmosphere where the central ion is
surrounded by a group of opposite charge. For example, in fig. 5,
the central ion is positively charged and is surrounded by an
atmosphere of negatively charged ions.
Fig.5: Symmetrical ionic atmosphere
P-112

When an electric field is applied, the ions are set in motion. The + –
central ion moves in one direction and the atmosphere in the
opposite direction. Thus, a central positive ion tends to move –
towards the cathode on the right while its negative ionic atmosphere – –
tends to move towards the anode on the left, as represented in fig.6. +
The symmetry of the atmosphere about the ion is thus destroyed – –
and the atmosphere becomes distorted. In other words, whereas the –
force of attraction exerted by the atmosphere on the central ion,
before the passage of electricity, is uniform in all directions and
Fig. 6: Asymmetric ionic atmosphere
therefore cancels out, it becomes greater behind the ion than in

front, on the passage of electricity, as in fig.6. Consequently the ion experiences a retarding force, a force
which tends to drag it backward. The movement of the ion is therefore slowed down. The drag on the central
ion is known as the asymmetry effect because it arises from a lack of symmetry in the atmosphere of a
moving ion. Similarly, a case of negative central ion being surrounded by the atmosphere of positively
charged ions is considered.
Another factor that slows down the motion of the ions at higher - Electric field strength
+ - -
concentrations arises from the tendency of the ionic atmosphere - +
associated with the molecules of water of hydration to move in a - -
direction opposite to that in which the central ion is moving. Thus, a + - +
-
positive ion, for example, which migrates towards the cathode, has to + - - - -
- - + ++ -
Electrostatic +
make its way through the medium (water) which, itself, is moving with + +
force
the negative ionic atmosphere towards the positive electrode. Similarly + +
a negative ion has to move towards the anode through the medium - + ++Frictional viscous
- ++ -drag of solvent
(water) associated with positive ionic atmosphere which is moving -- - - +
Electrophoretic
towards the negative electrode. These counter currents slow down the - retardation force
ions in the same way as counter currents in a stream slow-down a + + + - +
swimmer. This effect is known as electrophoretic effect. As shown
in fig. 7. Fig. 7: Movement of ion in
The third factor which affects the mobility of ions is the viscous an ionic atmosphere
effect. It arises from the viscous drag of the solvent on the movement
of ions. This ion tends to move in the direction of the applied electric field. This electrical force is opposed by
the frictional viscous drag exerted by the solvent. For an ion having a given charge and size, the greater is the
viscosity of the solvent, the greater is the viscous drag and hence the smaller is the ionic mobility and the
conductance. It is shown in fig.7.
In 1926 Debye, Huckel and Onsager worked out mathematically the magnitudes of asymmetry and
electrophoretic effects in terms of such factors as valency of the ion, ionic concentration and dielectric
constant and viscosity of the medium. For uni-univalent electrolytes such as NaCl, which produce two
univalent ions and for which the molar mass is the same as the equivalent mass, the following equation was
derived:
0  82 . 4 8 . 2 × 10−5 0 
Λm = Λm −  + Λ m c
 ( (DT ) η ( DT ) 
Here D and η are the dielectric constant and coefficient of viscosity of the medium, respectively, at the
absolute temperature T and c is the concentration of the solution in moles per litre. This equation commonly
is known as the Onsager equation.
P-113

0 0
The Onsager equation accounts for the difference between Λ m and Λ m . As is evident, Λ m is less than Λ m .
This is partly due to electrophoretic effect and partly due to asymmetry effect. The first term in the bracket
gives a measure of the asymmetry effect. The sum of these two effects multiplied with the square root of the
0
concentration gives the decrease of molar conductance Λ m from its limiting value Λ m .
For a given solvent and at given temperature, the above equation may be expressed as
Λ m = Λ 0m − ( A + B Λ0m) c
where A and B are the Debye–Huckel constants. The values of A and B for water at 25°C comes out to be
60.2 and 0.229 respectively.
0 0
Λ m = Λ m − (60.2 + 0.229 Λ m ) c
If the equation is correct, then by plotting molar conductances ( Λ m ) against the square roots of the
0
concentrations ( c ), a straight line of slope equal to (60.2 + 0.229 Λ m ), should be obtained. This has been
checked in the case of a number of uni-univalent electrolytes and found to be true up to concentrations of the
order of 0.02M as shown in the following graph (fig. 7). At higher concentration, minute deviations are
noticed which increase with further increase in concentration. These are attributed to certain approximations
in the derivation of the equation which are not valid when concentration is high. If a solution is at infinite
dilution, so that c is almost zero, then the second term on the right hand side of the equation becomes
0
negligible and Λ m approaches as Λ m .

4.3 Transport Number


Transport number or transference number or ion transport number is a quantity which is frequently used in
electrochemical calculations. Ion transport number indicates the different contribution of ions to the electric
current in electrolytes due to different electrical mobility. It has been introduced by Wilhelm Hittorf in 1853.
Broadly, the transport number is the fraction of the total current carried by an ion. It is denoted by
symbols like t + and t − or tc and ta.
Definition
It is defined as the ratio of the current carried by a given ionic species through a cross section of
an electrolytic solution to the total current passing through the cross section.
current carried by an anion
ta =
total carrent passed through the solution
current carried by a cation
tc =
total carrent passed through the solution
Evidently, ta + tc = 1 .
The transport number is also equal to the ratio of the velocity or mobility of a given ion to the sum of the
velocities or mobilities, of the cation and anion. It is a characteristic dependent on :
1. The mobilities of all the ions in the electrolytic solution.
2. On the concentrations of the ions and
3. On the temperature of the solution.
The transport number is usually determined by Hittorf method—that is, by the change in the concentrations
of the ions near the electrodes. During electrolysis, ions transport not only electric charge but also the solvent
P-114

in their solvation sheaths. It is due to solvation. For this reason, transport numbers determined by the Hittorf
method are called apparent transport numbers. Transport number can be determined either by Hittorf 's
method or moving boundary method or emf method or from ionic mobility. A rise in temperature tends to
bring the transport number of cation and anion closer to 0.5.

4.3.1 Transport Number and Ionic Mobility


Ionic mobility or ionic conductance is the conductivity of a solution containing 1 g ion, at infinite dilution,
when two sufficiently large electrodes are placed 1 cm apart.
Definition wise ionic usability refers to the speed of an ion under unit potential gradient (potential difference
per unit length)
So, ionic mobilities (λ a or λ c ) ∝ speeds of ions (ua or u c )
Unit of ionic mobility is m 2 s −1 V −1 or cm 2 S−1 V −1. Ionic mobility and transport number are related as
λ a or λ c = i a or te × ta
Absolute ionic mobility is given by,
Ionic mobility
Absolute ionic mobility =
96,500

4.3.2 Determination of Transport Number by Hittorf Method


This method is based upon observed concentration changes around the electrodes. As mentioned above,
Mobility of cation (u + ) Fall of Concentration round anode
=
Mobility of anion (u − ) Fall of concentration round cathode

u+
And transport number of cation, t+ =
u+ + u −

It follows, therefore, that


Fall of concentration around anode
t+ =
Fall of concentration around + Fall of concentration around cathode
Fall of concentration around anode
=
Total fall of concentration
If the concentrations are measured in terms of gram equivalents, then
Number of gram equivalents lost at the anodic compartment
t+ =
Total number of gram equivalent lost at both the compartments

However, the total number of gram equivalents lost from both the compartments is equal to the number of
gram equivalents deposited on each electrode. This is shown by referring to Hittorf’s theoretical device once
again. It is observed that in each case the total number of ions lost from the anodic and the cathodic
compartments put together is invariably equal to the number of ions discharged or deposited on each
electrode.
Number of gram equivalents lost from the anodic compartment
Therefore, t+ =
Number of gram equivalents deposited on each electrode
P-115

Since, according to Faraday’s second law, the number of gram equivalents, deposited on each electrode
must be equal to the number of gram equivalents of copper deposited in a copper coulometer by the same
quantity of electricity, it is more convenient to include a copper coulometer in series with the experimental
solution and to find out the mass of the copper deposited on the cathode. So,
Number of gram equivalents lost from the anodic compartment
t+ =
Number of gram equivalents of copper deposited in the coulometer

Moreover, (t+ ) + (t− ) = 1


∴ (t− ) = 1 − (t+ )
1. The Apparatus: For a precise determination of transport numbers, intermixing of solutions is avoided
in anodic and cathodic compartments. A ‘middle compartment’ is provided for this purpose. The
concentration of solution in this compartment must remain unchanged at the end of the experiment.
One form of the apparatus is shown in Fig.8.

The set up consists of two vertical glass tubes connected via U-tube as shown in fig 5. The end tubes contain
anode and cathode and constitute anodic and cathodic compartments while the U-tube constitutes the
middle compartment. Each tube is having a stop-cock at the bottom position. The electrodes are made of
suitable metal. They are sealed in narrow glass tubes which pass through rubber stoppers as shown. Some
mercury is placed in the tubes to make adequate electrical contacts. In the present example, the transport
number of silver ions in silver nitrate solution is determined. The electrodes are made up of either platinum or
pure silver metal and the electrolyte is a dilute solution of silver nitrate. The concentration of silver nitrate is
maintained around N/15 to N/20.
P-116

The three-limbed electrolytic cell, after filling in the solution, is connected in series with a C u or a A g
coulometer and a battery through a variable resistance. A milliammetre is also included in the circuit to read
the current strength. A current of 10-20 milliamperes is usually passed for about 2 to 3 hours. The purpose is
to get an appreciable but not too large change in the concentrations around the electrodes. If the change is
large, diffusion sets in. After electrolysis, solution is withdrawn from the anode as well as the central chamber
by opening stop cocks at bottom. The change of concentration in each solution is determined. There must not
be any change in the concentration of the solution withdrawn from the central chamber. It is important to
make that since change in concentration of a solution results in change of density and hence change of
volume, the change of concentration must be determined with reference to a definite mass of the solvent.
Therefore, the estimations are made with the known mass and not the known volume of the solution before
and after electrolysis.

Suppose while finding out the transport number of silver ions in silver nitrate, a gram of the anodic solution
after electrolysis, on titration with a standard solution of potassium chloride, is found to contain b gram of
silver nitrate. Therefore, the mass of water in the solution is (a−b) gram. Evidently, (a−b) gram of water
contains b gram of silver nitrate or b/170 gram equivalents of silver nitrate or of silver ions. Let this value be y.

Before electrolysis, suppose c gram of silver nitrate solution is found to contain d gram of silver nitrate.
Evidently, (c−d) gram of water is associated with d gram of silver nitrate. Therefore, (a−b) gram of water,
before electrolysis, would have been associated with d/(c−d) x (a−b) gram of silver nitrate or d/(c−d) x
(a−b)/170 gram equivalents of silver nitrate of silver ions. Let this value be x.

If the electrolysis of silver nitrate in this case was carried out by using platinum electrodes which are not
attacked by any of the ions, y will be less than x.

Thus fall of concentration round anode = x−y

Suppose, in the copper coulometer connected in series, the mass of copper deposited in the same time was w
gram or w/31.8 gram equivalent. Let this value be z.

Therefore, Transport number of Ag + ion = (x−y)/z

In the above example it has been presumed that the electrodes are not attacked by the ions discharged. While
this is generally true for cation, it is not so always with anion. For example, during finding out the transport
number of silver ions in silver nitrate using silver electrodes, nitrate ions attack the silver anode bringing silvers
ions into the anodic compartment. Consequently, the concentration of the anodic compartment will tend to
increase due to this factor. In such cases, y will be greater than x. Therefore, increase in concentration round
the anode = (y − x) gram equivalent. Further, it is known that

Gram equivalents of copper deposited in copper coulometer

= Gram equivalent of silver deposited on the cathode of the electrolytic cell


= Gram equivalent of silver dissolved from the anode of the cell by using the action of nitrate ions.

Since gram equivalent of copper deposited in the coulometer is z, it follows that increase in the concentration
of silver ions in the anodic compartment should have been z gram equivalent. But actual increase is (y−x)
gram equivalent. Therefore, z−( y−x) gives the fall of concentration due to migration of Ag + ions from the
anodic to the cathodic compartment. Hence, transport number of Ag + ion = z − (y − x )/ z .
P-117

Example 1: A N /10 solution of AgNO3 is electrolysed between Pt electrodes. After passing a


small current for 2 hours, a fall of concentration of 0.0005124 gram equivalent occurred in the
anodic solution. The mass of copper deposited in copper coulometer placed in series was found
to be 0.03879 gram. Calculate the transport numbers of silver and nitrate ions in silver nitrate.
( Equivalent mass of copper=31.8)
Solution: Loss of concentration of Ag + around anode = 0.0005124 g eq.

Similarly g- equivalent of Cu deposited in coulometer


0 . 03879
= = 0 . 001219
31 . 8

g eq. of Ag + lost 0 . 0005124


So, t + = = = 0 . 42
Ag g eq. of Cu deposited 0 . 001219

So, t = 1 − 0 . 42
NO3−
= 0 . 58
Example 2: A solution of AgNO3 was electrolyzed between silver electrodes. Before
electrolysis, 10 gram of the solution contained 0.01788 gram of silver nitrate. After the
experiment, 20.09 gram of the anodic solution contained 0.06227 gram of silver nitrate. At the
same time, 0.009479 gram of the copper was deposited in the copper coulometer placed in
series. Calculate the transport number of silver and nitrate ions.(Ag=108, Cu=63.6)
Solution: (i) After electrolysis,
20.09 g of solution at anode contains 0.06227 g of AgNO 3
So that mass of water = 20 . 09 − 0 . 06227 = 20 . 02773 g
0 . 06227
No. of equivalents of Ag + = = 0.0003664 g eq.
170
(ii) Before electrolysis
0.01788 g of AgNO 3 is present in 10 grams of solution.
Mass of water = 10 − 0 . 01788 = 9 . 98212 g
So, 20.02773 g of water must contain
0 . 01788 × 20 . 02773
AgNO 3 = = 0 . 03588 g
9 . 98212
0 . 03588
= = 0 . 00021 g eq.
170
So, increase in conc. of Ag + = 0 . 0003604 − 0 . 00021 = 0 . 001553 g eq.
0 . 009479
Cu deposited in coulometre = = 0 . 0002982 g eq.
31 . 8
P-118

If no Ag + had migrated from anode, the increase in construction of Ag + would have been the same as the
no. of eq. of Cu deposited. But actual increase is lesser and it is equal to 0.0001553 g eq. So, fall of conc. of
Ag + ion = 0.0002982 − 0.0001553 = 0.0001429 g eq.
0 . 0001429
So, t+ = = 0 . 48; t− = 1 − 0 . 48 = 0 . 52
0 . 0002982

4.3.3 Determination of Transport number by moving boundary method


The moving boundary method is based upon the direct observation of migration of ions in an electric field.
The principle is explained with the reference to determination of transport number of H + ions in hydrochloric
acid. The conductivity cell, in this method, consists of a vertical tube of uniform bore filled with cadmium
chloride and hydrochloric acid, as in Fig. 9. Hydrochloric acid is the principal or main electrolyte whereas
cadmium chloride serves as the indicator electrolyte. It is always the indicator electrolyte which enables
formation of a boundary. The concentrations of the solutions are so adjusted that hydrochloric acid solution
is lighter than the cadmium chloride solution and therefore floats over the cadmium chloride solution. A
sharp boundary appears between the two solutions. The selection of the indicator electrolyte has to be made
carefully. The cation of the indicator electrode must not move faster than the cation whose transport number
is to be determined. Moreover, it must have the same anion as the principal electrolyte. Keeping these
parameter in mind, cadmium chloride solution is chosen because cadmium chloride fulfills both these
requirements. The mobility of cadmium ion is less than that of hydrogen ion and it has a common anion with
hydrochloric acid. The anode inserted at the bottom is a stick of cadmium metal while the cathode at the top
is a platinum metal.
When a small current is passed through the conductivity cell, chloride
ion moves towards the anode. Similarly, hydrogen ions followed by
cadmium ions move towards the cathode. The boundaries –
separating the two solutions also move upwards. It is assumed that Platinum Cathode
the boundary moves through a distance l cm − presently, from aa’ to
HCl Solution
bb’. Then the volume of the liquid that has moved up is lA cm 3
where A is the cross-sectional area of the tube in cm 3 . Let the b b'
concentration of the acid be c gram equivalents per litre. Then the
Moving Boundary
number of gram equivalents of H + ions carried towards the cathode a a'
= Acl/1000. As each gram equivalent carries one faraday of
electricity, the electricity carried by H + ions = lAc/1000 faradays.
CdCl2Solution
It is presumed that the total quantity of electricity that flows in the + Candimum Anode
same time, as measured in a coulometer also included in the circuit
= Q faradays. Then evidently,the transport number of H + ions is +
– Coulometer
given by
Transport number of H + ions = lAc/(1000Q). Fig.9:The
Fig.9: TheMoving
MovingBoundary
Boundarymethod
method

4.4 Kohlrausch's Law


While investigating the equivalent conductances at infinite dilution (Λ 0 ), Kohlrausch found that when salts of
potassium and sodium with a common anion are taken, the difference in their Λ 0 values was found to be
same irrespective of the nature of the anion.
P-119

That is Λ0KCl − Λ0NaCl = Λ0KNO − Λ0NaNO = Λ01/ 2( K − Λ01/ 2 Na etc. Similar results were also
3 3 2SO4 ) 2SO4
obtained with other pairs of salts having either a common anion or a common cation. A few of the results are
shown below.
Table 1: Values Λ0 or Λ∞ for pairs of electrolytes (25° C)
Electrolyte Λ0 or Λ∞ Difference Electrolyte Λ0 or Λ∞ Difference
Λ0(1) − Λ0(2) Λ0(1) − Λ0(2)
(1) KCl 149.9 (1)LiCl 115.0
(2) NaCl 128.8 21.1 (2)LiNO 3 110.1 4.9

(1) KNO 3 145.0 (1)KCl 149.9


(2) NaNO 3 123.9 21.1 (2)KNO 3 145.0 4.9

(1) KNO 3 145.0 (1)HCl 426.16


(2) LiNO 3 110.1 34.9 (2)HNO 3 421.3 4.9

(1) KClO 4 140.0 (1)NaCl 128.8

(2) LiClO 4 105.08 34.92 (2)NaNO 3 123.9 4.9

(1) KCl 149.9


(2) LiCl 115.0 34.9

Such constancy in the difference of Λ 0 values of two salts having a common ion is possible only if each ion
makes a definite contribution towards the conductivity of the solution irrespective of the value of the other
ion. Thus,
Λ0 ( NaCl ) = Λ0( Na+ ) + Λ0( Cl − ) and Λ0 (KCl ) = λ 0( K + ) + λ 0( Cl − )

Hence, Λ0 (KCl ) − Λ0 ( ΝaCl ) = λ 0( K + ) − λ ο( Na+ )

4.4.1 The Limiting Law


Conclusively, whatever salts of sodium and potassium be taken, provided the salts are taken at infinite
dilution with common anion, the difference in Λ0 value will be the same. Kohlrausch, therefore, proposed
that at infinite dilution, each ion makes a definite contribution towards the equivalent conductance of the
electrolyte. This contribution is independent of the other ion with which it is present in the solution. The value
of the molar or equivalent conductance at infinite dilution (Λ0 ) of an electrolyte is the sum of contributions of
the two ions. Mathematically,
Λ0m = v + λ 0+ + u − λ 0− …(1)

where λ 0+ and λ 0− are the conductivities of the cation and anion respectively at infinite dilutions. These are
also called as ion conductances. v + and v − are the number of cations and anions respectively. That is, at
infinite dilution, the equivalent conductance is the sum of ion conductances, which are characteristic of each
ion species. This is also known as the law of independent migration of ions. According to Kohlrausch law,
some of the limiting molar conductances of different electrolytes are,
Λ∞m for NaCl = λ ∞ + λ∞
Na+ Cl −
P-120

Λ∞m for BaCl 2 = λ ∞ + 2λ ∞


Ba2+ Cl −

Λ∞m for Al 2 (SO4 )3 = 2λ ∞ 3 + + 3λ ∞


Al SO2−
4

4.4.2 Application of Kohlrausch's Law


Kohlrausch’s law is an important law of electrochemistry. It has a list of applications, some of these
applications are discussed here.
1. Calculation of limiting conductivities of weak electrolytes: The Kohlrausch law can be used to
calculate the limiting conductivities of weak electrolytes.In the case of weak electrolytes Λ0 , the
conductance at infinite dilution cannot be directly obtained. In fact, the law of independent migration of
ions is used for finding out Λ 0 for weak electrolytes. To illustrate, Λ0 for acetic acid is evaluated by
determining the equivalent conductance of three strong electrolytes separately. These three electrolytes
are HCl, NaCl and sodium acetate. It is easily observed that,
Λ0 HCl + Λ0 ΝaAc − Λ0 ΝaCl = (λ 0 + λ0 ) + (λ 0 + λ0 ) − (λ 0 + λ0 )
H+ Cl − Na+ Ac − Na + Cl −

= λ0 + + λ0 − = Λ0 Ac OH
H Ac
Since the quantities on the left−hand side are experimentally determined, λ 0 is known. This method is
employed for other weak electrolytes also.
Example 3: The molar conductivity at infinite dilution for HCl, KCl and CH 2ClCOOK are 4.26 ×
10 −2, 1.5 × 10 −2 and 1.13 × 10 −2 S m 2 mol −1 respectively. Calculate the molar conductivity at
infinite dilution for monochloroacetic acid ( CH 2ClCOOH).
Solution: For CH 2 ClCOOH
Λ0 CH 2CICOOH = Λ0 CH 2CIOOK + Λ0 HCl − Λ0 KCl
= 1.13 × 10 −2 + 4.26 × 10 −2 −1.5 × 10 −2 = 3.89 × 10 −2 S m 2 mol −1
2. Determination of degree of ionization (α) of weak electrolyte: The degree of ionization of a
weak electrolyte at a particular concentration is equal to the ratio of actual number of ions formed due
to partial ionization to the expected number of ions formed upon complete dissociation. As the degree
of dissociation, α is defined as,
0 Λm
α = Λm / Λm ⇒ α =
λ 0+ + λ 0−
0
Here, Λ m = equivalent conductivity at given concentration, Λ m = limiting equivalent conductivity. It
can be further simplified as
ΛΜ
α=
(λ 0+ + λ 0− )
λ +0 = limiting equivalent conductivity of cation, λ 0− = limiting equivalent conductivity of anion.
3. Determination of acid dissociation constant: Ostwald dilution law is a relationship between
the dissociation constant K a and the degree of dissociation α of a weak electrolyte. The law takes the
form
[ A + ][B − ] (α 2 ) c
Ka = =
[ AB] 1−α
P-121

where the square brackets denote concentrations of the reacting species, and c is the initial concentration
of electrolyte. As the degree of dissociation, α is defined as,
0
α = Λm / Λm
On solving the following relation is obtained
(Λ m / Λ 0m)2 c
Ka =
 Λ 
1 − m 
 Λ 0m 
 2
Λ 1  
⇒ 1− M =  Λ M  c ; dividing by Λ throughout,
 Λ 0   M
Λ 0 Ka
M  M  

1 1 1 (Λ M ) c 
− =   So,
Λ M Λ 0 Ka (Λ 0 )2 
M  M 
0 0
1 / Λ m = 1 / Λ m + c Λ m / K a(Λ m )2 87
which is known as the Ostwald’s dilution law. A plot of 1 / Λ m against c Λ m is drawn to get a straight
0
line. The values of Λ m and K a can be obtained from the intercept and slope respectively.
4. Determination of transport number: As such the transport number of ion is related to their molar
conductance as
λ 0±
t±0 =
λ 0+ + λ −0
More importantly, molar conductances of ions are related to their mobilities as follows,
λ 0± = F u 0±
The combination of these two equations gives,
u 0±
t±0 =
u 0+ + u −0
So, if the molar conductances of ions or their mobilities at infinite dilutions are known, the transport
number at infinite dilution can be calculated.

4.4.3 Ionic Strength


As such, ionic strength is a mathematical parameter to give an idea of the average electrostatic interactions
among ions in an electrolyte. It is one of the main characteristics of a solution with dissolved ions. Ionic
compounds, when dissolved in water, dissociate into ions. The ionic strength of a solution is a measure of the
concentration of ions in that solution. The total electrolyte concentration in solution will affect important
properties such as the dissociation or the solubility of different salts.
The ionic strength, I, of a solution is a mathematically determined as,
n
1
I= ∑ ci zi2
2
i=1
P-122

where ci is the molar concentration of individual ion i. It is expressed in molarity or mol/L. zi is


the charge number of that ion, and the sum is taken over all ions in the solution. For a 1:1 electrolyte such
as sodium chloride, the ionic strength is equal to the concentration, but for MgSO4 the ionic strength is four
times higher. Generally, multivalent ions contribute strongly to the ionic strength. For example. the ionic
strength of a mixture of 0.050 M Na 2 SO 4 and 0.020 M KCl solution is calculated as below:
I = 1/2 × [(Concentration of Na 2 SO 4 in M × Number of sodium ions × (Charge of sodium ion) 2 +
(Concentration of Na 2 SO 4 in M × Number of SO 2− 2− 2
4 ions × (Charge of SO 4 ions) + (Concentration of KCl
× Number of potassium ions × (Charge of potassium ion) 2 + Concentration of KCl in M × Number of
chloride ions × (Charge of chloride ion) 2 .
So, ionic strength of the mixture is,
I = 1/2 × [(0.050 M × 2 × (+1) 2 ) + (0.050 M × 1 × (−2) 2 ) + (0.020 M × 1 × (+1) 2 ) + 0.020 M × 1 ×
(−1) 2 )] = 0.17 M. The ionic strength plays a central role in the Debye–Huckel theory that describes the strong
deviations from ideality typically encountered in ionic solutions.

4.4.4 Activity and Activity Coefficient


In the chemistry of electrolyte solutions, an ideal solution is a solution whose colligative properties are
proportional to the concentration of the solute. Real solutions show departures from this kind of ideality. In
order to accommodate these effects in the thermodynamics of solutions, the concept of activity was
introduced. According to thermodynamic treatment solution the colligative properties are now proportional
to the activities of the ions. Activity, a, is proportional to concentration, c. The proportionality constant is
known as an activity coefficient γ.
a=γc
In an ideal electrolytic solution the activity coefficients of all the ions are equal to one. Non−ideality arises
because ions of opposite charge attract each other due to electrostatic forces, while ions of the same charge
repel each other. Consequently, ions are not randomly distributed throughout the solution, as they would be
in an ideal solution.
Activity coefficients of single ions cannot be measured experimentally because an electrolyte solution must
contain both positively charged ions and negatively charged ions. Instead, a mean activity coefficient, γ ± is
defined. For example, with the electrolyte NaCl,
1
γ ± = [γ Na + × γ ] 2
Cl −

In general, the mean activity coefficient of a fully dissociated electrolyte of formula AnBm is given by
γ ± = (γ nΑ × γ m
Β)
1/( n+ m)

Activity coefficients are themselves functions of concentration as the amount of inter-ionic interaction
increases as the concentration of the electrolyte increases. Debye and Hückel developed a theory with which
single ion activity coefficients could be calculated. By calculating the mean activity coefficients from them, the
theory could be tested against experimental data. It was found to give excellent agreement for “dilute”
solutions.

4.5 Arrhenius Theory of Electolytic Dissociation


In order to explain the properties of electrolytic solutions, Arrhenius put forth, in 1884, a comprehensive
theory which is known as theory of electrolytic dissociation or ionic theory. The main points of the theory are:
P-123

1. An electrolyte, when dissolved in water, breaks up into two types of charged particles, one carrying a
positive charge and the other a negative charge. These charged particles are called ions. Positively
charged ions are termed cations and negatively charged as anions.
AB → A + + B −
For example, NaCl → Na + + Cl −
K 2 SO 4 → 2K + + SO42−
In its modern form, the theory assumes that solid electrolytes are composed of ions which are held
together by electrostatic forces of attraction. When such an electrolyte is dissolved in a solvent, these
forces are weakened and the electrolyte undergoes dissociation into ions. The ions are said to be
solvated as shown below.
A +B − → A + + B −
or A + B − + aq → A + (aq)+B − (aq)
2. The process of splitting of the molecules into ions of an electrolyte is called ionization. The fraction of
the total number of molecules present in solution as ions is known as degree of ionization or degree of
dissociation. It is denoted by α such that,
α= (Number of molecules dissociated into ions)/(Total number of molecules).
It has been observed that all electrolytes do not ionize to the same extent. Some are almost completely
ionized while others are weakly ionized. It is because the degree of ionization depends on a number of
factors.
3. Ions present in solution constantly re-unite to form neutral molecules and, thus, there is a state of
dynamic equilibrium between the ionised and non-ionised molecules, as shown here,
AB L A+ + B−
Applying the law of mass action to above equilibrium,
[A + ][B − ] /[AB] =K
K is known as ionization constant. The electrolytes having high value of K are termed as strong
electrolytes and those having low value of K as weak electrolytes.
4. When an electric current is passed through the electrolytic solution, the positive ions (cations) move
towards cathode and the negative ions (anions) move towards anode and get discharged, so that
electrolysis occurs. The ions are discharged always in equivalent amounts, no matter what their relative
speeds are.
5. The electrolytic solutions is always neutral in nature as the total charge on one set of ions is always
equal to the total charge on the other set of ions. However, it is not necessary that the number of two
sets of ions must be equal always.
AB → A + + B − ( Number of both ions are equal)
+ −
NaCl → Na + Cl (Number of both ions are equal)
2+ −
AB 2 → A + 2B (Anions are double that of cations)
BaCl 2 → Ba 2+ + 2Cl − (Anions are double that of cations)
A 2 B → 2A + + B 2− (cations are double that of anions)
+ 2−
Na 2 SO 4 → 2Na + SO 4 (cations are double that of anions)
P-124

6. The properties of electrolytes in solution are the properties of ions present in solution. For example,
acidic solution always contains H + ions while basic solution contains OH − ions and characteristic
properties of solutions are those of H + ions and OH − ions respectively.
7. The ions act like molecules towards depressing the freezing point, elevating the boiling point, lowering
the vapour pressure and establishing the osmotic pressure.
8. The conductivity of the electrolytic solution depends on the nature and number of ions as the current is
carried through solution by the movement of ions.
Limitations of Arrhenius Theory
1. Ostwald dilution law which is based on Arrhenius theory is not applicable to strong electrolytes.
2. Strong electrolytes conduct electricity in fused state, which means in the absence of water. This is in
contradiction of Arrhenius theory according to which the presence of solvent is a must for ionization.
3. Arrhenius theory assumes independent existence of ions but fails to account for the factors which
influence the mobility of the ions.

4.5.1 Ostwald's Dilution Law


According to Arrhenius theory of electrolyte dissociation, the molecules of an electrolyte in solution are
constantly splitting up into ions and the ions are constantly reuniting to form unionized molecules. Therefore,
a dynamic equilibrium exists between ions and unionized molecules of the electrolyte in solution. It was
pointed out by Ostwald that like chemical equilibrium, law of mass action is applicable to such cases of ionic
equilibria as well. Let us consider a binary electrolyte AB which dissociates into A + and B − ions and the
equilibrium state is represented by the equation:
AB l A+ + B−
Initially, t = 0 c 0 0
At equilibrium c (1−α) cα cα
So, dissociation constant is given as
K = [A + ][B − ]/[AB] = (cα × cα)/c(1−α)
= cα 2 /(1−α) ...(1)
For very weak electrolytes,
α <<< 1, (1 −α) = 1
.·. K = cα 2
α= K / c ...(2)
Concentration of any ion = Cα = (K)(c).
1
From equation (2) it is a clear that degree of dissociation increases on dilution, (Q d α ). Thus, degree of
c
dissociation of a weak electrolyte is proportional to the square root of dilution.

4.5.2 Limitations of Ostwald's Dilution Law


The law holds good only for weak electrolytes and fails completely in the case of strong electrolytes. The value
of ‘α’ is determined by conductivity measurements by applying the formula Λ/Λ ∞ . The value of ‘α’
Λ / Λ °m
determined at various dilutions of an electrolyte when substituted in Eq. below, K = m gives a
Λ
1– m
Λ °m
P-125

constant value of K only in the case of weak electrolytes like CH 3 COOH, NH 4 OH etc. The cause of failure
of Ostwald’s dilution law in the case of strong electrolytes is due to the following factors:
1. The law is based on the fact that only a portion of the electrolyte is dissociated into ions at ordinary
dilution and completely at infinite dilution. Strong electrolytes are almost completely ionized at all
Λ
dilutions and m does not give accurate value of ‘α’.
Λ °m
2. When concentration of the ions is very high, the presence of charges on the ions appreciably affects the
equilibrium. Hence, law of mass action in its simple form cannot be applied in the case of high
concentration as well.
Example 4: A 0.01 M solution of acetic is 5% ionized at 25°C. Calculate its dissociation
constant.
Solution: According to Ostwald’s dilution law,
K a = α 2 /(1−α)V
α = 0.05, V = 1/0.01 = 100 litres
Hence, K a = 0.05 × 0.05/(1–0.05)100 = 2.63 × 10−5

Example 5: Calculate the H + ion concentration of a 0.02 N weak monobasic acid. The value
of dissociation constant is 4.0 × 10 −10 .
Solution: HA l H+ + A−
Applying Ostwald’s dilution law of a weak acid,
α = √K aV
K a= 4.0 ×10 −10 , V = 1/0.01 = 100 litres

α = √(4 × 10 −10 × 10 2 ) = 2 × 10 −4
Concentration of hydrogen ions
α/√V = (2×10 −4 )/100 = 2×10 −6 mol L −1
Concentration of hydrogen ions
= √(CK) = √(0.01 × 4 ×10 −10 ) = 2 × 10 −6 mol L −1
Example 6: The concentration of H + ions in 0.10 M solution of a weak acid is 1.0 × 10 −5 mol L
−1
. Calculate the dissociatison constant of the acid.
Solution: HA l H+ + A−
Initial concentration 0.1 0 0
Equil. conc. 0.1−1.0×10 −5 1.0×10 −5 1.0×10 −5
(mol L −1)
[HA] can be taken as 0.1 M as 1.0 × 10 −5 is very small.
Applying law of mass action,
K a = [H + ][A − ]/[HA] = (1.0×10 −5 × 1.0 ×10 −5 )/0.10

= 1 × 10 −9
P-126

Example 7: What will be the dissociation constant of 0.1 N aqueous ammonia solution in terms
of degree of dissociation ‘α’? What will be the value if the concentration is 0.01 N?
Solution: NH 4 OH l NH + 4 + OH −
At equilibrium (1−α) α α
Since the solution is 0.1 N,
V = 1/0.1 = 10 litre
[NH 4 OH]=(1−α)/10, [NH + 4 ] = α/10 and [OH − ] = α/10
For 0.01 N solution, K b remains the same at the same temperature but degree of dissociation value becomes
different.
Example 8: A 0.0128 N solution of acetic acid has Λ = 14 mho equiv −1 and Λ ∞ = 391 mho equiv
−1
at 25°C. Calculate the dissociation constant of the acid.
Solution: Degree of dissociation, α = Λ/Λ ∞ = 14/391 = 3.58×10 −2
Now applying Ostwald’s dilution law,
K a = α 2 /(1−α)V
α = 3.58 × 10 −2 and V = 1/0.0128 litre
So, K a = 3.58 × 10 −2 × 3.58 × 10 −2 × 0.0128 = 1.64 × 10 −5

4.6 Conductometry
Conductometry is a measurement of electrolytic conductivity to monitor a progress of chemical reaction.
Conductometry has notable application in analytical chemistry, where conductometric titration is a standard
technique. In usual analytical chemistry practice, the term conductometry is used as a synonym
of conductometric titration.

4.6.1 Conductometric Titrations


The principle involved in the conductometric titration is that electrical conductance depends upon the
number and mobility of ions. Acid-base titrations and redox titrations are often performed in which common
indicators like methyl orange, phenolphthalein are used to locate the end point. Similarly, starch solutions are
used for iodometric titration which is a type of redox titration. Conductometric titration does not require an
indicator to determine the end point. In this type of titration the electrolytic conductivity of the reation
mixture is continuously monitored as one reactant is added. The equivalence point is the point at which the
conductivity undergoes a sudden change. Marked increases or decrease in conductance are associated with
the changing concentrations of the two most highly conducting ions—the hydrogen and hydroxyl ions. The
method is used for titrating coloured solutions or homogeneous suspension which cannot be used with
normal indicators. Some of the cases of conductometric titrations are discussed below :
Case 1: Titration of strong acid versus strong base: In the first instance, the titration of a strong acid,
like hydrochloric acid, with a strong base, like sodium hydroxide is considered. The acid is taken in
conductivity vessel and the alkali in the burette. The conductance of hydrochloric acid is due to the presence
of hydrogen and chloride ions. As alkali is added gradually, the hydrogen ions are replaced by slow moving
sodium ions, as represented below:
+
[H + + Cl − ] + [ N a + OH − ] → Na + + Cl − + H 2 O
( op ) ( aq) ( aq) ( aq) ( aq) ( aq) ( l)
P-127

Hence, on continued addition of sodium hydroxide, the


conductance will go on decreasing until the acid has been
A D
completely neutralized. Any subsequent addition of alkali will

Conductance
result in introducing fast moving hydroxyl ions. The
conductance, therefore after reaching a certain minimum
value will begin to increase. On plotting the conductance
against the volume of alkali added, the points will lie on two B C
straight lines AB and CD (as shown here in Fig 10). The point
of intersection X of these two lines gives the volume of alkali X
required for the neutralization.
Case 2: Titration of weak acid versus strong base: It is End Point
required to titrate a weak acid like acetic acid against a strong
base. Conductance will be low initially on account of poor Volume of NaOH Added
dissociation of acetic acid On adding the alkali, highly ionized Fig. 10: Titration of a strong acid against
sodium acetate is formed and hence the conductance begins a strong base.
to increase.
The reaction taking place is,
CH 3 COOH + [ Na + + OH − ] →
( aq) ( aq) ( aq) Conductance
− +
CH 3 COO + Na + H 2 O
( aq) ( aq) ( l) End Point
When the acid is completely neutralized, further addition of
alkali introduces excess of fast moving hydroxyl ions. The x
conductance of the solution, therefore, begins to increase
even more sharply than before. On plotting the conductance
against the volume of the alkali added, the two lines obtained
will be as shown in Fig.11. The point of intersection X gives Volume of Alkali Added
the end point.
Fig.11: Titration of a weak acid against
Case 3: Titration of mixture of strong and weak acid a strong base.
versus strong base: When a mixture of a strong and a weak
acid is to be titrated against a strong base a combination of curves shown in Fig 10 and 11 is obtained. The
new graphical description is shown in Fig.12.
A mixture of HCl and CH 3 COOH is considered to be titrated
against NaOH. Hydrochloric acid, being a much stronSger
OH

acid, will get neutralized first. The titration of acetic acid will
HC

Na

commence only after hydrochloric acid Shas been


Conductance
lV

OH
S

completely neutralized. Hence a combination of curves as Na


NA

VS
mentioned above is obtained. This is represented in Fig.12. O H
CO
OH

While the point B corresponds to the neutralization of HCl, CH 3


the point C corresponds to the neutralizatrion of CH 3 COOH.
Case 4: Titration of strong acid versus weak base:
Finally, suppose it is required to titrate a strong acid like B C
hydrochloric acid, against a weak base, the ammonium Volume of NaOH Added
hydroxide. The conductance will fall at first due to the Fig.12: Titration of a mixture of HCl and
+
replacement of fast moving H + ions by slow moving NH 4 CH3COOH against NaOH.
ions. The reaction is shown below,
P-128

+ −
H + + Cl − + [ NH 4 OH ] → NH 4 + Cl + H 2O
( aq) ( aq) ( aq) ( aq) ( l)

Conductance
X

End Point

Volume of NH4OH Added


Fig. 13: Titration of a strong acid against
a weak base.

After neutralization of the acid, further addition of weakly ionized ammonium hydroxide will not cause any
appreciable change in the conductance. The curves obtained will be shown in Fig.13.

4.6.2 Application of Conductometric Titration


The solubility of poorly or sparingly soluble salts is expressed as the solubility product. It is the product of the
concentration of ions in the solution which are in equilibrium with the solid ion. These concentrations can be
determined via conductivity measurements. The dissociation of PbSO 4 is considered.
PbSO 4 l
2−
Pb 2+ + SO 4
(5) (aq) (aq)
As concentration of solid is a constant.
2−
Concentration of Pb 2+ = concentration of SO 4 =c
2+ 2−
K sp = [Pb aq ] [SO 4 aq ]

K sp = c 2

The measurement of the specific conductivity, Κ of the saturated solution leads to a value of the
concentration.
c = κ /Λ0
As, K sp = c 2

K sp = (κ / Λ0 )2

Here, Λ0 = Molar conductivity at infinite dilution.


P-129

Long Answer Type Questions 11. Specific conductance of a 0.12 N solution of an


Based on Specific Conductance, Equivalent electrolyte is 0.024 S cm −1. Find out its equivalent
Conductance and Molar Conductance conductance. [Lko. 2009]
1. Write the equation derived by Debye−Huckel. 12. The equivalent conductance of 0.1N solution of
How the equation can be verified experimentally? H3PO4 at 18°C is 96.5 ohm −1cm 2 eq −1. If at this
[Garhwal 2009]
temperature Λ° HCl = 378.3, Λ° NaCl = 109.0 and
2. Explain what is meant by specific, equivalent and
Λ° NaH PO = 70.0 S cm2 eq −1, calculate the
molecular conductivity of an electrolyte? How are 2 4

they related to each other? Why does the former degree of dissociation and dissociation constant for
decrease and later increase with dilution? the reaction:
H3PO4 ®

O
[Garhwal 2010] H+ + H2PO4
3. Write Debye-Huckel equation of strong electrolytes [Lko. 2010]
and explain how asymmetry effect and 13. The specific conductivity of a solution containing 1
electrophoretic effect can account for difference gm of BaCl 2 in 200 c.c. of water has been found to
between λ c and λ 0 ? [Garhwal 2008] be 0.0058 ohm −1 cm −1. What are molar and
4. Define specific conductance and equivalent equivalent conductivities of solution? (At. wt. of
conductance of an electrolyte. Explain why specific Ba=137 and Cl=35.5) [Kanpur 2011]
conductance of an electrolytic solution decreases 14. Discuss the Debye-Huckiel theory of strong
whereas its equivalent conductance increases with electrolytes. [D.D.U. 2009]
dilution. [Avadh 2010]
Based on Transport Number
5. Give an account of the Debye-Huckel theory pf
15. What is meant by transport number of an ion?
strong electrolytes. Explain symmetry effect and
Describe the method employed to determine the
electro phoretic effect. [Purv. 2009]
transport number of Ag + and NO3− ions in AgNO 3
6. Explain variation of specific conductance and
solution. [Garhwal 2010]
equivalent conductance with concentration.
[Purv. 2011] 16. Explain the moving boundary method for
7. State and explain the terms conductance, specific determining transport number of H + ion in HCl.
[Garhwal 2009]
conductance and equivalent conductance. Derive
a relationship between specific and equivalent 17. What is transport number? How transport number
conductance. [Lko. 2008] of Ag + ion can be determined by Hittorf’s method?
8. 0.5N solution of a salt placed between two [Garhwal 2008]
platinum electrodes 2.0 cm apart and of area of 18. How will you determine the transort number of an
cross-section 4.0cm 2 has a resistance of 25 ohms. ion by Hittorf's method? [Purv. 2008]
Calculate the equivalent conductivity of the 19. What is transport number? Discuss any one
solution. [Lko. 2008] method for the determinations of transport number
9. The equivalent conductivity at infinite dilution of of Ag + . [D.D.U. 2009]
NaNO 3 at 18°C is 100 mhos. If the ionic 20. Define transport number. Give a method for the
O−
conductance of NO3 ion in the solution is 58.7 determination of transport number.
+ [D.D.U. 2011]
mhos. Calculate the transport number of Na ions
in the solution. [Lko. 2008] 21. What do you understand by transport number?
10. Define specific conductance, equivalent Describe Hittroff's method for the determination of
conductance, ionic conductance and ionic mobility. transport number of Ag + & NO3− in a solution of
[Lko. 2009] AgNO 3 . [Meerut 2008]
P-130

22. Explain Hittorf's method to determine transport 33. (i) What is Ostwald's dilution law?
number of any ion (When electrodes are attacked). (ii) The specific conductance of 0.01 M solution
[Agra 2009] of acetic acid was found to be 0.016 Sm−1
23. (i) What is transport number? Describe Hittorf’s at 25°C . Calculate the degree of
method for the determination of transport dissociation of the acetic acid. Molar
number of silver ion. conductance of acetic acid at infinite dilution
(ii) Show that: λ°+ t °+ is 400.0 × 10−4 Sm2 mol −1 at 25°C.
=
λ°− t °− [Avadh 2010] (iii) For a salt of weak acid and strong base show
24. What is transport number? Describe Hittorf’s that:
method for the determination of transport number K
Kh = w
of silver ion. [Avadh 2008] Ka [D.D.U. 2001]
Based on Kohlrausch's Law Based on Conductonetry
25. Discuss the application of conductivity 34. (i) What is pH and pOH? Give two examples and
measurements in destermination of solubility uses.
product of sparingly solubie salts. [Purv. 2008] (ii) What are conductometric titrations? How is
it advantageous over volumetric
26. The specific conductance of a 0.01 N solution of
titrations? [Purv. 2011]
acetic acid at 25°C is 1.6×10 −4 mhos cm −1. The
ionic conductances of H + and CH3COO − ions are
Short Answer Type Questions
349.8 and 40.9 mhos cm 2 eq−1 respectively. Based on Specific Conductance, Equivalent
Calculate the degree of dissociation of acetic acid. Conductance and Molar Conductance
[Lko. 2011] 1. Give the salient features of Debye-Huckel theory of
27. Explain Kohlrausch's law of independent strong electrolyte. [Garhwal 2011]
migration. [D.D.U. 2008] 2. What do you know about cell constant?
[Garhwal 2010]
28. What is Kohlrausch's law? How does this law help
3. The specific conductivity of a decinormal solution
in determination of equivalent conductivity at
of KCl and 18°C is 0.0112 ohm −1 cm −1. The
infinite dilution? [D.D.U. 2011]
resistance of a conductivity cell containing the
29. State and explain Kohlraush's law of ionic solution at 18°C was found to be 55 ohm. Calculate
mobilities. Discuss its applications. [Meerut 2010] the cell constant. [Garhwal 2008]
30. What is Kohlrausch law? Discuss its various 4. A conductivity cell has electrodes 2 cm apart and
applications. [Meerut 2011, 12] area of each electrode is 4 cm 2 . It shows a
resistance of 35 ohm when filled with a electrolyte
Based on Arrhenius Theory of Electolytic
solution of 0.4 N strength. Find out equivalent
Dissociation
conductivity of the solution. [Garhwal 2008]
31. State Ostwald's dilution law and describe its
5. The resistance of a decinormal solution of a salt
limitations. [D.D.U. 2010] was found to be 32 ohm. Calculate its equivalent
32. Answer the following: conductivity. Cell constant = 1.8 cm −1.
(i) What are the main postulates of Arrhenius [Garhwal 2007]
theory of ionization? 6. Explain Debye-Huckel theory. [Garhwal 2006]
(ii) Why is it not possible to measure the limiting 7. What do you understand by equivalent
molar conductivity of a weak conductance? Explain the experimental method
electrolyte? for its measurement.
[Garhwal 2005]
(iii) Why is Ostwald’s dilution law not applicable
8. The resistance of a 0.01 N solution of an electrolyte
to strong electrolytes?
was found to be 210 ohms at 25°C. Calculate the
(iv) What is Debye-Huckel-Onsagar equation equivalent conductance of this solution. (Cell
for strong electrolyte? [Avadh 2010] constant = 0.88). [Garhwal 2005]
P-131

9. Define specific conductance. The specific 23. N


A conductivity cell was calibrated by using kCl
N 100
conductance of a acetic acid was found to be
100 (specific conductance k=0.1409 Ω −1m−1) in the
−1
0.000158 ohm cm −1 at 18°C. The molar cell and the measured resistance was 690 Ω.
conductance of acetic acid at infinite dilution at N
Determine the cell constant, A AgNO3 solution
18°C is 387 ohm −1 cm 2 mol −1. Calculate the 100
N in the same cell had a resistance of 700Ω. What is
degree of dissociation of acetic acid at 18°C.
100 the equivalent conductance of Ag NO3 .
[Avadh 2011] [D.D.U. 2008]
10. Why specific conductance decreases with dilution 24. What is equivalent conductance? Explain the terms
which equivalent conductance increases with involved in the relation. [D.D.U. 2009]
dilution of a solution? [Purv. 2008] Based on Transport Number
11. Explain Debye-Huckel-Onsager's equation for the 25. What is meant by Hittorf’s number of an ion?
strong electrolyte in short. [Purv. 2008] [Garhwal 2011]
12. Explain equivalent conductance and its variation 26. Describe the Hittorf’s method for the determination
with dilution. [Purv. 2009] of transport number. [Garhwal 2011]
13. What is cell constant? [Purv.2011; D.D.U. 2011] 27. Write equation for the relation between ionic
14. Conductance increases while specific conductance conductance and transport number.
decrease with dilution. Why? [Garhwal 2007]
[Lko. 2009]
15. State Debye-Huckel- Onsagar's equation for 28. What do you mean by transport number of an ion?
uni-univalent strong electrolyte such as KCl and Give one method of its determination?[Purv.2009]
discuss the theory given by them. [Lko. 2010] 29. What is transport number?
[Purv. 2011; Meerut 2010, 11, 12]
16. What is the relationship between specific
conductance and equivalent conductance? 30. Show that the sum of transport number of cations
Explain. [Kanpur 2008, 11] and anions is unity. [Lko. 2008]
17. Define relationship between ionic conductance 31. Write a short note on transport number.
[Kanpur 2009]
and ionic mobility. [Kanpur 2011]
32. Briefly explain 'Moving Boundary Method' for the
18. Define the specific conductivity and equivalent
conductivity and their unit. determination of transport numbers of K + and Cl −
[Kanpur 2011]
in KCL solution. [Lko. 2011]
19. A potential difference of 10 volts applied to the
end of a column of M/10AgNO3 solution 4cm. in Based on Kohlrausch's Law
diameter and 12 cm. in length gave current of 33. Equivalent conductance of HCl, CH 3 COONa and
0.01 amps. Calculate the specific and equivalent NaCl are 426.1, 91.0 and 126.4 Scm 2 eq −1
conductivities of the solution. [Kanpur 2012] respectively at infinite dilution. Calculate the
N equivalent conductance of acetic acid at infinite
20. A conductivity cell was calibrated by using KCl
100 dilution. [Garhwal 2012]
(Specific conductance k = 01409
. Ω −1m −1) in the 34. Explain Kohlrausch Law. [Garhwal 2010]
cell and the measured resistance was 690Ω.
N 35. Calculate the degree of dissociation of 0.1 N NaCl
Determine the cell constant. A AgNO3 solution. The equivalent conductance of this
100
solution in the same cell had a resistance of 700Ω. solution is 98.4 Scm 2 eq −1 and the ionic
What if the equivalent conductance of AgNO3 ? conductance of Na + and Cl − ions are 43.4 and
[D.D.U. 2008] 65.5 Scm 2 eq −1 respectively at infinite dilution.
21. Write a short notes on: [Garhwal 2010]
(i) Specific conductivity 36. Equivalent conductance of HCl, CH 3 COONa and
(ii) Equivalent conductivity [Kanpur 2010] NaCl are 426.1, 91.0 and 126.4 Scm 2 eq −1
22. Define specific conductance and molar respectively at infinite dilution. Calculate
conductance. [Purv. 2009] equivalent conductance at infinite dilution of acetic
P-132

acid. If equivalent conductance of acetic acid is 54. Write Short notes of the following
48.15 5 cm 2 eq−1 then calculate the degree of (i) Le Chatlier's Principle
dissociation of acetic acid. [Garhwal 2007] (ii) Ostwald's Dilution law [Agra 2008]
37. What is Kohlrausch’s law? With the help of this law 55. Give Ostwald's dilution law.
how the solubility of sparingly soluble salt is [Agra 2009; Meerut 2008]
determined? [Garhwal 2006] Based on conductometry
38. What is Kohlrausch's law? Explain its use for the 56. Write notes on
determination of equivalent conductance of acetic (i) Conductometric titrations (ii) Kohlrausch law
acid at infinite dilution. [Purv. 2010] [Garhwal 2008]
39. Why is it not possible to measure the limiting 57. Discuss any two applications of conductance
equivalent conductivity of a weak electrolte measurements. [Garhwal 2005]
experimentally? [Lko. 2011] 58. D.C. current can not be used for conductance
40. Write a short note on Kohlrausch law and discuss measurement of a solution. Why? [Lko. 2010]
its applications. [Kanpur 2008] 59. Explain the principle of conductometric titration
41. Explain Kohlrausch's law. taking example of HCl and NaOH.[Kanpur 2011]
[Kanpur 2010, 11]
42. The equivalent conductance of an electrolytic 60. Write short note on conductometric titration.
[D.D.U. 2009, 11]
solution is zero/maximum/minimum at infinite
61. How the ionic product of water (K w ) is determined
dilution. Justify correct answer. [D.D.U. 2010]
by conductance measurement ? [Lko. 2009]
43. The specific conductivity of a saturated solution of
62. Write short notes on:
AgCl is 1.33× 10 −6 Ohm −1cm −1. If the ionic
Moving boundary method for determination of
conductance of Ag + & Cl − ions are 59 and 74
transport number. [Lko. 2009]
S cm2 mol −1 Calculate the solubility of AgCl.
63. Explain the curve obtained in conductometric
[Meerut 2008]
titration between AgNO 3 and KCl. [D.D.U. 2010]
44. Calculate the ionic strength of a solution of
64. What is the principle of conductometric titrations?
0.0005M BaCl 2 solution. [D.D.U. 2008] Give their applications in acid/base titrations.
45. Calcualte the equivalent conductance at 20°C of [Garhwal 2011]
NH 4 OH at infinite dilution 20°C. [Agra 2009] 65. Explain conductometric titration of HCl and
46. How can the solubility of sparingly soluble salt can NH4OH graphically. [Purv. 2010]
be determined with the help of Kohlrausch's law?
Based on Arrhenius Theory of Electolytic
Dissociation
Very Short Answer Type Questions
Based on Specific Conductance, Equivalent
47. Explain Arrhenius theory of elctrolytic dissociation.
Conductance and Molar Conductance
[Garhwal 2005]
1. Define specific conductance.
48. Describe Debye-Huckel theory of strong
2. What is SI unit of specific conductance?
electrolytes. [Kanpur 2009, 11; D.D.U. 2011]
3. Define cell constant.
49. Explain Arrehenius theory and Kohlrausch's law.
4. What is the relation between ΛM and Λe?
[Agra 2008]
5. Explain the effect of dilution on specific
50. Write Ostwald's dilution law and its uses.
conductance (k).
[Kanpur 2008;10]
6. Write Debye Huckel Onsager equation.
51. Discuss the factors which influence the ionisation.
7. Explain electrophoretic effect.
[Kanpur 2008]
Based on Transport Number
52. Write short notes on Arrhenius theory of
electrolytic dissociation 8. Define transport number. [Meerut 2009, 11]
[Kanpur 2012]
53. Explain the following: Based on Kohlrausch's Law
Ostwald dilution law and its limitations. 9. Why the ionic mobility of H + is very high?
[Purv. 2009] 10. Write Kohlrausch's law. [Meerut 2009]
P-133

Based on Arrhenius Theory of Electolytic


Dissciation Fill in the Blank
11. What is the limitation of Arrhenius theory? 1. Out of specific, equivalent and molar
12. Explain Ostwald dilution law? conductivities, the quantity which decreases with
dilution is………. .
Objective Type Questions 2. Conductivity is the conductance of………of the
solution and its units are……… .
Multiple Choice Questions 3. Increase in molar conductivity with dilution is
1. On increasing dilution equivalent conductivity : because of increase in………. .
(a) Increases (b) Decreases 4. Specific conductance = cell constant × ……… .
(c) Remains unaltered (d) None of these. [Garhwal 2010]
[Garhwal 2009,11,12] 5. Cell constant of a conductivity cell is defined
2. Conductivity of 1 cm 3 of solution is called: as……… . [Garhwal 2008]
(a) Specific conductivity 6. With the increase of dilution, the specific
(b) Equivalent conductivity conductivity of electrolyte……… .
(c) Molar conductivity [Garhwal 2006]
(d) Observed conductivity. [Garhwal 2012] 7. Unit of equivalent conductance is ……… .
3. Conductivity of 1 cm 3 of solution is called : [Agra 2009]
(a) Specific conductivity 8. Fraction of the total current carried by an ion. is
(b) Equivalent conductivity called its ................... . [Garhwal 2008]
(c) Molecular conductivity
9. The degree of dissociation of sodium hydroxide is
(d) Resistance [Garhwal 2007]
………….. than that of ammonium hydroxide.
4. The value of degree of dissociation of any
[Garhwal 2006]
electrolyte will be :
(a) More than one (b) Less than one
(c) Equal to zero (d) None of the above. True and False
[Garhwal 2006] 1. The conductivity of an electrolyte is determined by
5. The unit of specific conductance is : a conductivity cell. [Garhwal 2010]
(a) Ohm −1 cm (b) Ohm cm −1 2. According to Debye-Huckel theory, the degree of
−1 −1 −1 dissociation of an electrolyte increases on dilution.
(c) Ohm cm (d) Gram cm
[Garhwal 2005] [Garhwal 2009]
6. Equivalent conductivity with dilution: 3. Transport numbers of cation and anion of an
(a) decreases (b) increases electrolyte are equal. [Garhwal 2010]
(c) becomes zero (d) None of these.
7. For any electrolyte, the sum of cation and anion 4. The ionic mobility of an ion is directly proportional
transport numbers is : to its conductivity. [Garhwal 2009]
(a) –2 (b) 0 (c) +1 (d) –1 5. The transport number of anion plus cation is unity.
[Garhwal 2010] [Garhwal 2008]
8. The method used for measuring the transport 6. Transport number cannot exceed.
number of silver ion is : 7. In conductometric titrations, it is essential to use
(a) Arrhenius method (b) Hittorf method suitable indicator for determining the end point.
(c) Perrin method (d) Berkeley method [Garhwal 2006]
[Garhwal 2006]
9. To which of the following species, Oswalds’s law is
applicable?
(a) CH 3 COOH (b) HNO 3
(c) HCl (d) H 2 SO 4
[Garhwal 2010]
P-134

Objective Type Questions

Multiple Choice Questions


1. (a) 2. (a) 3. (a) 4. (b)

5. (c) 6. (b) 7. (c) 8. (b)

9. (a)

Fill in the Blank


1. specific conductivity 2. 1 cm 3 & S m −1 3. dissociation of electrolyte

4. 1 5. Specific conductance x R 6. decreases


R

7. Sm 2 eq –1 or S cm 2 8. Transport Number 9. more


–1
eq

True and False


1. True 2. False 3. False 4. True

5. True 6. False 7. False

Hints & Solutions

Long Answer Type Questions


8. l 1 l 1 2
k = C× ⇒k = × ⇒k = ×
a R a 25 4
∴ k = 0.02 S cm −1
0.02
⇒ ΛE = × 103 = 40 S cm 2 eq−1
0.5
0.024
11. ΛE = × 103 = 200 S cm 2 eq−1
012
.
12. Λ0H PO = Λ0HCl + Λ0NaH PO − Λ0NaCl
3 4 2 4

= 378.3 + 70 − 109 = 339.3S cm 2 eq−1


Λ 96.5
So ,α = ⇒ α= = 0.284.
Λ0 339.3
α 2 C (0.284)2 01
.
And, Ka = = = 1. 12 × 10−2
1−α 1 − 0.284
P-135

1 1000
13. M= × = 0.024 M.
208 200
Similarly N = n − f × M ⇒ N = 2 × 0.024
= 0 . 048 N.
Now,
k 0.0058
ΛM = × 100 ⇒ ΛE = × 100
N 0.048

Λ E = 120.83 S cm 2 eq−1.
26. Λ0 = (λ0 CH 3 COO− ) + λ0 H + ) = 40.9 + 349 . 8
= 390 . 7 S cm 2 eq−1
Λ
α=
Λ0
k 1 . 6 × 10−4 × 103
Now, Λ= × 103 ⇒ Λ=
N 0 . 01
= 16 S cm 2 eq−1.
16
α= = 0.041
390 . 7

Short Answer Type Questions


1 l l
3. k= × ⇒k × R =
R a a
⇒ 0.0112 × 55 = l/a So, cell constant (l/a)
= 0.616 cm −1
1 l 1 2
4. k= × ⇒k = × = 0.014 S cm −1
R a 35 4
0 .014
Λ= × 103 ⇒ Λ = 35. 71 S cm 2 eq−1
0.4
k
5. Λ= × 103
N
1
and k= × 1 . 8 = 0.05625
32
0.05625
So, Λ= × 1000 = 562 .5 cm 2 eq−1.
0.1
1
8. k= × 0. 88 = 0 . 0042 S cm −1
210
k 0.00419
ΛE = × 1000 = × 1000 ~− 419.04 S cm 2 eq−1
N 0.01
0.00158
9. Λ= × 1000 = 15 .8 S cm 2 eq−1.
0.01
15 . 8
α = = 0 . 0408.
387
10
19. V =i R⇒ = R So, R = 1000 Ω.
0.01
1 l 1 12
k= × = × = 9. 5 × 10−4 S cm −1.
1000 a 100 Π (2)2
P-136

k 9.5 × 10−4 × 1000


Λ= × 1000 = = 9.5 S cm 2 mol −1
M 01
.
20. 1 l l
k= × ⇒ = k × R = 01409
. × 690
R a a
= 97.22 m −1.
1 l
Now, k= × (for AgNO3 )
R a
1
= × 97.22 = 01388
. S m −1 = 01388
. × 10−2 S cm −1
700
0.1388 × 10−2
So, ΛE = × 103 = 138 . 88 S cm 2 eq−1.
0.01
32. ΛoCH COOH = ΛoHCL + ΛoCH COONa − ΛoNaCl
3 3
= 4261
. + 91 − 126.4
. Scm 2 eq−1
= 3907
34. Λ0 = 43.4 + 65.5 = 108.9 S cm 2 eq−1.
Λ 98 . 4
α= = = 0.903
Λ0 108.9
35. Λ0 HOAC = Λ0 HCl + Λ0 ACONa − Λ0 NaCl
426.1+91–126.4=390.7 S cm 2 eq−1
48.15
α= = 0.1232.
390.7

K 1.33 × 10−6
42. Λ= × 103 ⇒ × 103 = Solubility
Solubility 59 + 74
n
1
Ci (Zi )2
44. I =
2 ∑
i=1
1
= [C 2+ × (2)2 + 2 × CCl − × (1)2 ]
2 Ba
1
= [5 × 10−4 × 4 + 2 × 15 × 10−4 × 1]
2
1
= [2 × 10−3 + 10−3 ] = 1.5 × 10−3 M .
2
S = 10−5 M.

Fill in the Blank


7. S cm2 eq −1 or S cm2 eq −1

mmm
UnitP-137
-III

C HAPTER 5
Solution

5.1 Introduction
A solution is a homogeneous mixture of two or more pure components whose composition can be varied
within certain limits. If the solution contains two components only, it is called a binary solution. The
component present in larger quantity is called the solvent and the other component is called the solute. This
chapter deals with binary solutions only. Depending upon the physical state of solute and solvent, nine
different types of solutions are possible. But our discussion will be limited to those solutions which are in
liquid phase or liquid solutions, containing a solid, liquid or gaseous solute in liquid solvent.

5.2 Vapour Pressure


When a pure liquid is kept in closed vessel the liquid evapourates to give the vapours of the corresponding
liquid. After sometime the liquid reaches a dynamic equilibrium with the vapours. The dynamic equilibrium
means rate of evapouration and rate of condensation are exactly the same. The pressure that the vapours
exert at equilibrium on the walls of the container or on the surface of the liquid is called the vapour pressure
of the liquid at that temperature.When the liquid is in equilibrium with vapour, Liquid l Vapour, the KP of
the system is
K P = P v , where P v is the vapour pressure.
Our understanding of chemical equilibrium that K P for a given equilibrium is a constant and only depends on
temperature. Therefore the vapour pressure of the liquid is a constant and does not depend on the nature of
the vessel used and that it only depends on temperature. Moreover, the vapour pressure of a liquid is
independent of the surface area exposed on top of the liquid surface.

If a non–volatile solute that dissolves in the liquid is added to the liquid, some of the particles would be
present on the surface of the liquid, thereby decreasing the number of solvent particles present per unit area
of the surface. This would decrease the vapour pressure of the liquid. This in no way violates our observation
made earlier that vapour pressure is unaffected by the surface area of the solvent exposed. It is because the
vapour pressure of a liquid does not depend on the surface area of the solvent exposed but only depends on
the number of solvent molecules present per unit area on the surface. It decreases only on dissolving a solute.

5.2.1 Raoult's Law


Raoult studied vapour pressures of a number of binary solutions of volatile liquids, such as benzene and
toluene, at constant temperature and gave the following generalization which is known as the Raoult’s law.
The partial pressure of any volatile component in a solution is directly proportional to its mole fraction in the
P-138

solution. Mathematically, the partial pressure equals to the vapour pressure of the pure component multiplied
by the mole fraction of that component in the solution at any temperature. So, for a component A,
PA = χ A PA° or in general, P = (χ)P °

Derivation of Raoult's law: The rate at which the component molecules leave the surface is proportional
to the number of such molecules per unit area on the surface, which in turn is proportional to the mole
fraction of that component.
Rate of vapourization = k χ component .
here k is a constant of proportionality and χ component is the mole fraction of the component in the solution.
The rate with which vapour molecules condense is proportional to their concentration in the gaseous phase,
which in turn is proportional to their partial pressure.
rate of condensation = k' Pcomponent
here k' is the constant of proportionality and Pcomponent is vapour pressure of the component. At equilibrium
the two rates would be equal. Therefore,
k χ component = k' Pcomponent
k
⇒ Pcomponent = χ component
k'
k
when the solvent is pure, χ component = 1 and Pcomponent = . If the pure vapour pressure of the solvent is
k'
k
denoted as P ° , then = P°.
k'
∴ P component = χ component P ° .

5.2.2 Vapour Pressure of a Solution Made up of Volatile Components


A binary solution is considered which is made of nA moles of a volatile liquid A and nB moles of a volatile
liquid B. If PA and PB are partial pressures of the two liquid components then, according to the Raoult’s law
PA = χ A PA° ...(1)

PB = χ BPB° ...(2)

here χ A is the mole fraction of the component A given by nA / (nA + nB), χ B is the mole fraction of the
component B given by nB / (nA + nB) and PA° and PB° are the vapour pressures of pure components A and B,
respectively. If the vapour behaves like an ideal gas, then, according to Dalton’s law of partial pressures, the
total vapour pressure P is given by
P = PA + PB ...(3)

or P = χ A PA° + χ B PB° ...(4)


In general, Raoult’s law may be expressed as
Pi = χ iPi° ...(5)
and the total vapour pressure may be expressed as
P = ∑ Pi ...(6)
P-139

5.2.3 Vapour Pressure of a Solution Containing Non−volatile Solute in Volatile Solvent


Raoult’s law states that on adding a non-volatile solute to a volatile solvent, the vapour pressure of the solvent
is lowered and it is directly proportional to the mole fraction of the solvent in the solution. Since the
solvent-solute interaction may also affect the vapour pressure of the solution, the above law is followed by
solutions in which the solute is very little, so the law is meant for highly dilute solutions. These solutions are
referred to as ideal solutions.
As, P = χ A PA° + χ B PB° so,
°
( )
Psolution = χ solvent Psolvent  (Q solute is non volatile)
°
⇒ Psolution = (1– χ solute ) Psolvent 
°
Psolution Psolvent – Psolution
⇒ = 1 – χsolute ⇒ χsolute =
° °
Psolvent Psolvent

5.3 Ideal Solution


It is known from different experiments that Raoult’s law is obeyed only approximately for most of the binary
solutions. The law is obeyed perfectly only in the case of ideal solutions. This gives a definition of an ideal
solution. A solution made up of two or more constituents is said to be ideal if it obeys Raoult’s law exactly at
all concentrations and at all temperatures.

5.3.1 Definition of Ideal Solution


It has been found that liquid pairs which are similar, generally form ideal solutions. This implies that
intermolecular forces in these solutions are completely uniform. So, the magnitude of solute - solvent
interactions is of the same value as solute - solute or solvent - solvent interactions. Some of the the binary
mixtures which obey Raoult’s law over the entire range of concentration and thus form ideal solutions are
ethylene bromide and ethylene chloride; n-hexane and n-heptane; n-butyl chloride and n–butyl bromide;
benzene and toluene and carbon tetrachloride and silicon tetrachloride etc.

5.3.2 Vapour Pressures of Ideal Solutions


The vapour pressure of an ideal binary solution of two components A and B having different mole fractions
are shown in figure.

P° Total Vapo
A ur Pressure
of P=
Par 0.2 PA° + 0
tial Solution P=χ ° .8 PB°
Vapour Pressure

PA Pre P
A A+χ P °
=χ s su B B PB°
AP ° re o
A fA

of B
res sure
ial P
Part P°
= χΒ Β

χA =1 χA =0
χB =0 Mole Fraction
χB =1
Fig. 1: Vapour pressures of ideal solutions
P-140

It is quite obvious from the graph that the partial pressure of each component against its mole fraction in the
solution is a straight line. Moreover, the total vapour pressure of the solution for any given composition is
equal to the sum of the partial vapour pressure of the two constituents. In fig-1 it is being presumed that B is
having lesser vapour presence than A.

The partial vapour pressure of component A is given by χ A PA° and that of component B is given by χ BPB° .
The total vapour pressure of the solution is given by χ A PA° +χ BPB° .

Let us again emphasize that ideality of a solution implies that a molecule of a component A will have the same
tendency to escape into the vapour phase whether it is surrounded,
1. Entirely by other molecules of its own type,
2. Entirely by the molecules of the component B or partly by its own type,
3. Entirely by the molecules of the component B. Thus, the intermolecular forces between A and A, B and
B and between A and B are necessarily the same. The escaping tendency of component A from an
ideal solution is the same as that from a pure liquid except that it is proportionally reduced as a result of
the lower mole fraction of A molecules in the solution.

5.3.3 Total Vapour Pressure in Terms of Mole Fraction of the Components in Vapour Phase
The total vapour pressure, P, is also related to the mole fractions y A and y B of the two components in the
vapour phase. The mole fractions in vapour phase y A and y B are defined as,
χ A,vap = y A and χ B,vap = y B
From the definition of partial pressure,
y A = pA / P and yB = pB / P
where P is the total vapour pressure of the solution. Hence,
χ A PA° = P( y A )

and χ BPB° = P(y B )

So using Raoult's law, P = χ Α PΑo + χ Β PBo

⇒ P = (1 − χ B)PA° + χ BP B° = PA° + χ B(P B° − P A° ) (Q1 − χ Β = χ Α )

P − PA°
⇒ χB =
PB° − PA°
Using this value of χ B
P − PA°
P(y B) = PB°
PB° − PA°

or P(y B)(PB° − PA° ) = (PB° P) − (PB° × PA° )

or [ ( ) ]
P y B PB° − PA° − PB° = − PA° PB°

PA° PB°
Hence, P=
(
PB° + PA° − PB° y B )
P-141

This equation can be simplified further and put in a better form such that,
° °
( °
1 PB + PA − PB y B
=
)
p P A° P B°

PB° (1 − yB ) + PA° y B PB° (y A ) + PA° (y B )


= =
PA° PB° PA° PB°
yA yB
= +
PA° PB°

The vapour is assumed to be in equilibrium with the solution. The mole fraction, y A , of a component A in the
vapour phase is given by the ratio of its partial pressure to the total pressure, P, of the solution, assuming that
the vapour behaves ideally and Dalton’s law of partial pressures is applicable to it.

yA =
PA
=
χ A PA°
=
(1 − χ B )PA°
PA + PB χ A PA° + χ BPB° (
PB° − PA° χ B + PA° )
PB χ BPB°
yB = =
PA + PB ( )
PB° − PA° χ B + PA°

Example 1: Calculate the mole fraction of benzene in a benzene – toluene ideal solution that
contains 60 mole percent benzene in the vapour phase.
° °
Pbenzene = 90mm Hg; Ptoluene = 45mmHg
Solution: Here we would use two laws in combination. One is the Raoult’s law and the other is the
Dalton’s law. According to the Raoult’s law,
°
Pbenzene = χ benzene Pbenzene
and according to the Dalton’s law,
Pbenzene = Ybenzene Ptotal

where y benzene is the mole fraction of benzene in the vapour phase and Ptotal is the total pressure of the
system. Therefore,
°
Ybenzene Ptotal = χ benzene Pbenzene
°
Ytoluene Ptotal = χ toluene Ptoluene

Since χ benzene + χ toluene = 1


and Ybenzene + Ytoluene = 1
°
Ybenzene Ptotal = χ benzene Pbenzene …(1)
°
(1 – Ybenzene )Ptotal = (1 − )
χ benzene Ptoluene …(2)
P-142

Dividing (1) and (2)


χ °
Ybenzene benzene Pbenzene
=
(1 − Ybenzene ) (1 − χ benzene ) Ptol° uene
0.6 χ benzene 90
∴ = ×
0.4 (1 − χ benzene ) 45
3
∴ χ benzene =
7
Example 2: The vapour pressure of water at room temperature is lowered by 5% by dissolving a
solute in it, then the approximate molality of solution is
(a) 2 (b) 1 (c) 4 (d) 3
P °−P
Solution: = 0.05

P °−P 1 n
∴ = = 0.0526 =
P° 1 N
−1
0.05
0.0526 × 1000
molality = = 2.92
18
Ans: (d)
Example 3: The mathematical expression of Raoult’s law is:
P °−P n P °−P N P °−P n P °−P
(a) = (b) = (c) = (d) = n×N
P° N P° n P N P°
Solution: According to Raoult’s law
P °−P n
=
P° N+n
P °−P n
This is simplified to give =
P N
Ans: (c)
Example 4: Two liquids having vapour pressures P1° and P2° in pure state in the ratio of 2:1 are
mixed in the molar ratio of 1:2. The ratio of their moles in the vapour state would be:
(a) 1:1 (b) 1:2 (c) 2:1 (d) 3:2
Y1 χ1 P1°
Solution: = ×
1 − Y1 1 − χ1 P2°
Y1 1 2
= ×
1 − Y1 2 1
1
Y1 =
2
Ans: (b)
Example 5: Two liquids A and B have vapour pressure in the ratio PA0 : PB0 = 2:3 at a certain
temperature. Assume A and B form an ideal solution and the ratio of mole fractions of A to B in
the vapour phase is 1 : 3, then the mole fraction of A in the solution at the same temperature is:
1 2 1 3
(a) (b) (c) (d)
3 3 4 4
P-143

Solution: If Y A and YB are respectively the mole fraction of A and B in the vapour phase, then
YA 1
=
YB 3
PA 1 PA P
= [∴ YA = and YB = B ]
PB 3 P P

PA° χ A 1
=
PB° χ B 3

χA 1 3 1
= × =
χB 3 2 2
χA 1
or, =
1 − χA 2
1
On solving χ A =
3
Ans: (a)

5.4 Solubility of Partially Miscible Liquids


Four types of partially miscible liquid-liquid systems have been observed. These are :
1. Those in which the partial miscibility increases on increasing the temperature. Examples include
phenol–water, aniline–water, aniline hexane, CH 3 OH–CS 2 and cyclohexane–methanol systems. At
and above a certain temperature, the liquids become completely miscible.
2. Those in which partial miscibility increases on lowering the temperature. Examples include
(C 2 H 5 ) 2 NH–H 2 O, and (C 2 H 5 ) 3 N–H 2 O systems. At and below a certain temperature, the two liquids
become completely miscible.
3. Those in which partial miscibility increases on both raising as well as lowering the temperature in
certain ranges. Examples are H 2 O – nicotine and H 2 O–β–picoline systems. These liquid pairs show
complete miscibility both above and below certain temperatures.
4. Those in which complete miscibility temperature cannot be obtained. A common example is ether
water system.

The temperature above or below which a pair of partially miscible liquids becomes miscible in all proportions,
is called consolute temperature or critical solution temperature (CST) for the pair. Some liquid pairs attain
complete miscibility above a certain temperature in which case they are said to have the upper critical
solution temperature UCST while some liquid pairs show complete miscibility below a certain temperature
when they are said to have lower critical solution temperature (LCST). On the other hand, some liquid pairs
show both UCST and LCST. All partially miscible liquid pairs have, in general, an upper or a lower CST.
However, the UCST cannot be realized if one or both the components boil out before it is reached and the
lower CST may not be observed if one or both the components freeze out before it is reached. Thus, under
atmospheric conditions none of the CSTs is observed for ether and water system. The critical solution
temperature for a few partially miscible systems is provided in a tabular manner below,
P-144

Table1: CST of different mixtures


Components %B UCST LCST
A B
Water Phenol 34 65.80 –
Cyclohexane Methanol 29 49.0 –
Mathanol CS 2 80 49.5 –
Aniline Hexane 52 59.6 –
Water (C2 H 5 )2 NH 13 – 43.0
Water (C2 H 5 )3 N 50 – 18.5
Water Nicotine 208.0 60.8
Water β −picoline 153.0 49.1
Glycerine m −toluidine 120.0 7.1

The two phases having dissimilar composition in P=1atm


equilibrium with each other at a given temperature
constitute a pair of conjugate phases. The composition TB
Vapour
points of the conjugate phases are joined by tie lines.
Temperature

TA
Any composition-temperature coordinates placing a
point in the area of parallel miscibility represent a

Phase
One liquid
Va
One Liquid

system of two liquid phases and a vapour phase. Li pou


qu r+ Vapour
The mass ratio of the liquid phases can be id
Phase

+Liquid
determined by locating the overall composition a c b
point on the tie line for the specified temperature.
Two Liquid Phases
Thus from figure which shows the boiling point-mole
fraction diagram for a mixture of partially miscible A Mole Fraction B
liquids water-aniline with an upper CST, it is
observed that Fig. 2: The boiling point-composition diagram
for a mixture of partially miscrible
Mass of water (a) bc liquids with an upper CST.
=
Mass of aniline (b) ac
The above proportionality is called the Lever rule. It is mentioned here that the presence of another substance
dissolved in one or both phases has a marked effect on the CST as well as the liquid phase composition at the
CST. A substance soluble in only one of the original liquids raises the upper CST and lowers the lower CST.
However, a substance soluble in both the original liquids tends to lower the upper CST and raise the lower
CST. As a matter of fact, substances, taken in sufficiently large amounts, may convert partial miscibility of a
liquid pair into complete miscibility.
Small amounts of impurities may produce large changes in the CST. This observation forms the basis of the
Crismer test which can detect and measure minute quantities of impurities. It is found that the change in the
CST value is a linear function of the concentrations of the impurities. Thus, 1% of NaCl present in water raises
its upper CST with phenol by 12°C. Amount of water in alcohol is estimated by measuring their CSTs with
cyclohexane. Sodium oleate is soluble in both phenol and water. Addition of 0.98% sodium oleate to the
phenol-water system, decreases the CST by 43.7°C. Industrially soaps are employed for making Lysol-like
disinfectants by making phenols completely miscible with water. A few representative binary systems of
partially miscible liquids like phenol-water, triethylamine-water and nicotine-water are discussed below.
P-145

5.4.1 Phenol Water System


Phenol and water are only partially miscible at ordinary
temperatures. Therefore, on shaking these two liquids
with each other, two saturated solutions of different c z
compositions, one of phenol in water and the other of 68.1°C
water in phenol, are obtained. As already mentioned,
such solutions of different compositions co-existing with

Temperature °C
one another are termed conjugate solutions. The mutual tie line y'
y
solubility of phenol and water increases with rise in 30°C a x b
temperature and, therefore, the concentration of phenol
in water as well as that of water in phenol goes on
increasing with rise of temperature and ultimately at a A B
certain temperature of two conjugate solutions change 0 36.1 100
into one homogeneous solution. Scuh temperature for Weight % Phenol
phenol-water system is found to be 68.1°C. Composition
Fig.3: Phenol-water system.
of the solution then is 36.10 per cent phenol and 63.90
per cent water. Above the consolute temperature, the two
liquids become miscible with each other in all proportions. The variation of mutual solubility of water and
phenol with temperature is represented graphically in figure 3.
At a fixed temperature, such as 30°C, the composition of each layer is fixed as indicated by the points a and b.
The point a gives the composition of aqueous layer and b that of the phenol layer. The line joining them is
known as tie line, as already mentioned.
When the temperature and composition of a mixture of water and phenol is represented by a point, say, x,
within the curve ACB, the system consists of two layers which are termed as conjugate solutions of
composition represented by the points a and b.
If the temperature and composition are represented by a point on the left of curve AC or on the right of the
curve CB, say as at y or y’, the moisture will consist only of one solution. At y it will be an unsaturated solution
of phenol in water and y’ it will be an unsaturated solution of water
in phenol. Further addition of phenol at y or of water ay y’ will
ultimately result in the separation of the two phases. Lastly, if the
temperature and composition are represented by a point, say, z,
Temperature°C

above the curves ABC, the system will consist again of one
homogeneous solution. But, since this point lies above the
consolute temperature at which the two liquids are miscible in all Two Phase
proportions, further addition of phenol to water or adding water to
phenol will not yield two layers. Thus the composition of such a
solution can have any value. 18.5°
5.4.2 Triethylamine−Water System
One Phase
Triethylamine and water constitute a system in which the mutual
solubility increases with decrease in temperature. It has a lower
O 100
consolute temperature. The mutual solubilities of the two liquids Weight Persent Et3N
are plotted in figure. The convolute temperature is 18.5°C. Above Fig.4: Triethylamine–water system
this temperature, the liquids give rise to two distinct layers but below
the temperature, they are completely miscible in all proportions.
P-146

5.4.3 Nicotine-Water System One Phase


Nicotine and water exhibit an upper as well as a lower consolute 208°C
temperature. In other words, these liquids are completely miscible

Temperature°C
above a certain critical temperature – it is also termed as upper
consolute temperature. Similarly, these two liquids components are
completely miscible below a certain critical temperature-lower Two Phases
consolute temperature. Between these two limiting temperatures,
they are partially miscible. The solubilities of these liquids in one
another at different temperatures are shown graphically in figure. 60°C
(fig. 5). One Phase
0 100
Evidently, the upper consolute temperature is 208°C and the lower Weight Persent Nicotine
consolute temperature is 60°C. Between these two temperatures, Fig.5: Nicotine–water system
mixtures of water and nicotine separate into two liquid layers.

5.4.4 Distillation of Immiscible Liquids


In the case of immiscible liquids, the addition of one liquid to the other does not alter the properties of either
liquid. So, each liquid exerts its own vapour pressure independent of the presence of the other liquid. Hence,
the total vapour pressure – P, above the mixture containing two immiscible liquids in any proportion, will be
the sum of the vapour pressures of the pure liquids at that temperature. Thus, P = PA° + PB° where PA° and PB°
are the vapour pressures of the pure liquids A and B, respectively, at the given temperature. As the boiling
point of any system is the temperature at which its total vapour pressure becomes equal to the prevalent
atmospheric pressure, the mixture will boil at the temperature at which the total vapour pressure of the liquid
mixture becomes equal to the atmospheric pressure. So, the condition of boiling requires,

PA° + PB° = prevalent atmospheric pressure

This temperature will be lower than the normal boiling point of either of the liquids alone. In other words, any
mixture of two immiscible liquids will boil at a temperature lower than that at which any pure constant boils.
Moreover, it is independent of the amounts of the different components. Another important concept is that
the boiling point of the mixture will remain constant as long as the two liquids are present together.

The relative proportion of the two liquids in the distillate can be calculated remembering that the number of
moles of each component present in the vapour phase is proportional to its vapour pressure. If n A and n B are
the number of moles of the components A and B in the vapour phase at the boiling point, then

n A / n B = PA° / PB°

If W A and WB represent the actual masses of the two components in the distillate and M A and M B are their
respective molar masses, then n A = W A / M A ,and n B = WB / M B . The above equation is re - written as

n A WA / M A W A  PA°   M A 
So, = ⇒ =    ...(1)
nB WB / M B WB  PB°   M B 
P-147

As a result, the masses of the liquids in the distillate will be in the ratio of their vapour pressures and molar
masses. The above principle is made use of in the process of steam distillation.

Steam distillation: It is used in the purification of organic compounds which have high boiling points. The
compounds must be immiscible or nearly so in water. A common example is that of aniline. Its normal boiling
point is 180°C. But it can be made to boil and distil over at a much lower temperature by passing steam
through it. The apparatus is shown in figure.

Safety Tube
To Sink

Steam
Substance

water
in
distillation Water
flask Distillate
Receiver
organic compound
Fig.6: Steam distillation

The steam is produced in a metal container. The solution to be distilled is placed in the round-bottomed flask,
clamped at an angle so as to prevent the solution from being splashed into the condenser. The tube carrying
the steam from the container dips below the liquid in the flask. The distillation flask is kept heated gently on a
sand bath in order to avoid too much condensation of water into it. The vapours of the organic compound
mixed with steam pass over and are condensed in the receiver as shown. The boiling in this manner takes
place at about 98.5°C which is lower than the boiling point of either water or aniline. The vapour pressures of
water and aniline at 98.5°C are 717 mm and 43 mm respectively. As the molar masses of the liquids are 18
and 93, respectively, the relative masses of the two liquids in the distillate will be given by
Mass of water 18 × 717
= = 3. 23
Mass of aniline 93 × 43

The process of distillation of immiscible liquids is generally used for the purification of the liquids which either
have very high boiling point or decomposes when heated to their normal boiling points. In general, water is
used as one of the immiscible liquids and the process is known as steam distillation. It has been seen from
equation. that distillation of an immiscible mixture of an organic liquid with water will give a distillate
containing a high proportion of the organic compound only when the molar mass of the compound is larger
than that of water and has appreciable vapour pressure at temperature near the boiling point of water.

5.4.5 Fractional Distillation


Distillation, generally speaking is a separation process. Fractional distillation is the separation, of a mixture
into its component parts, by heating them to a temperature at which one or more fractions of the compound
will vapourize. It is a special type of distillation process. Those mixtures, for which the boiling points of
P-148

individual components differ by less than 25 °C from each other under a pressure of one atmosphere, are
resolved into the individual components. If the difference in boiling points is greater than 25 °C, a simple
distillation process is used.As an example the distillation of a mixture of water and ethanol is considered.
Ethanol boils at 78.4 °C while water boils at 100 °C. So, by heating the mixture, ethanol which is the more
volatile component will concentrate to a greater degree in the vapour leaving the liquid. Some mixtures form
an azeotropes, where the mixture boils at a lower temperature than either component. In this example, a
mixture of 96% ethanol and 4% water boils at 78.2 °C; the mixture is more volatile than pure ethanol. For this
reason, ethanol cannot be completely purified by direct fractional distillation of ethanol-water mixtures.

Apparatus: Apparatus normally includes a heat source, Thermometer


such as a hot plate with a bath and ideally with a
magnetic stirrer, a distilling flask typically a round-bottom
flask, a receiving flask often also a round-bottom flask, a
fractionating column, a condenser, like a Liebig condenser,
some boiling chips also known as anti-bumping granules,
a thermometer and adapter if needed. The arrangement
Fractionating Water Out
of fractional distillation is shown below.
Column
Process: The mixture is put into the round bottomed Condenser
flask along with a few anti-bumping granules, and the
fractionating column is fitted into the top. The fractional
distillation column is set up with the heat source at the Round bottom
flask Water in
bottom on the still pot. As the distance from the stillpot
increases, a temperature gradient is formed in the
column; it is coolest at top and hottest at the bottom. As
the mixed vapour ascends the temperature gradient,
some of the vapour condenses and revapourizes along Bunsen burner

the temperature gradient. Each time the vapour


condenses and vapourizes, the composition of the more Fig.7: Fractional distillation
volatile component in the vapour increases. This distills
the vapour along the length of the column, and eventually the vapour is composed solely of the more volatile
component or an azeotrope. The vapour condenses on the glass platforms, known as trays, inside the
column, and runs back down into the liquid below, refluxing distillate. The efficiency in terms of the amount
of heating and time required to get fractionation can be improved by insulating the outside of the column.
The hottest tray is at the bottom and the coolest is at the top. At steady state conditions, the vapour and liquid
on each tray are at equilibrium. The most volatile component of the mixture exits as a gas at the top of the
column. The vapour at the top of the column then passes into the condenser, which cools it down until it
liquefies. Initially, the condensate will be close to the azeotropic composition, but when much of the ethanol
has been drawn off, the condensate becomes gradually richer in water. The process continues until all the
ethanol boils out of the mixture. This point can be recognized by the sharp rise in temperature shown on the
thermometer.

Vacuum distillation: Systems operate at reduced pressure, thereby lowering the boiling points of the
materials. Note that the use of anti-bumping granules will not work at reduced pressures.
P-149

Pressuremeter
By pump
Impure
liquid

Heat

Water
Pure
liquid
Fig. 8: Vacuum distillation on low pressure

Steam distillation: Steam distillation is a type of distillation for temperature sensitive materials like natural
aromatic compounds.Many organic compounds tend to decompose at high temperatures. Separation by
normal distillation would then not be an option, so steam is introduced into the distillation apparatus. The
steam carries small amounts of the vapourized compounds to the condensation flask along with itself, where
the condensed liquids phase separate, allowing for easy collection. This process effectively allows for
distillation at lower temperatures, reducing the degradation of the desired products. If the substances to be
distilled are very sensitive to heat, steam distillation may be applied under reduced pressure, thereby
reducing the operating temperature further. After distillation the vapours are condensed. Usually the
immediate product is a two-phase system of water and the organic distillate, allowing for separation of the
components by decantation or other suitable methods.

Principle: When a mixture of two practically immiscible liquids are cooled while being agitated to expose
the surface of one liquid to the vapour phase, each constituent independently exerts its own vapour pressure
as a function of temperature as if the other constituent were not present. Consequently, the vapour pressure
of the whole system increases. Boiling begins when the sum of the partial pressures of the two immiscible
liquids just exceeds the atmospheric pressure. In this way, many organic compounds insoluble in water can
be purified at a temperature well below the point at which decomposition occurs. For example, the boiling
point of bromobenzene is 156 °C and the boiling point of water is 100 °C, but a mixture of the two boils at
95°C. Thus, bromobenzene can be easily distilled at a temperature 61 °C below its normal boiling point.

5.5 Solutions of Gases in Liquids


Generally gases are soluble in water as well as in some other liquids. The amount of the gas dissolved in a liquid
is broadly termed as its solubility. The solubility is generally expressed in terms of absorption coefficient (a),
introduced by Bunsen. It is defined as the volume of gas converted to N.T.P. dissolved by unit volume of
solvent at the temperature of the experiment and under 1 atmosphere pressure of the gas. The absorption
coefficients of a few gases at 20°C in three common solvents are given in table 2.
P-150

Table 2: Absorption Co-efficients ‘a’ of some gasses at 20° temperature

Solvents Hydrozen Nitrogen Oxygen Carbon dioxide

Water 0.017 0.015 0.028 0.88

Ethanol 0.080 0.130 0.130 3.00

Benzene 0.060 0.104 0.165 .......

5.5.1 Factors Affecting Solubility of a Gas in a Liquid


The solubility of a gas in a liquid depends upon a number of factors. Some of these determinants are
discussed below,
1. The nature of the gas and the nature of the solvent,
2. The temperature of the gas-liquid system and
3. The pressure of the gas.
1. Nature of Gas and Nature of Solvent: The solubility of a gas in a given solvent varies considerably
with the nature of the gas. Generally speaking, the gases which are easily liquefiable, are more soluble
in common solvents. Thus, carbon dioxide is more soluble than hydrogen or oxygen in water or in any
other solvent. It is because the easily liquefiable gases have enhanced inter molecular forces. Apart
from such gases, the ones which are capable of forming ions in aqueous solution are much more
soluble in water than in other solvents. This is illustrated by the fact that gases like hydrochloric acid and
ammonia are highly soluble in water but not in benzene in which they are not capable of forming ions.
2. Effect of Temperature: It has been found that under a constant pressure, the solubility of a gas
diminishes with rise in temperature. The absorption coefficients of a few gases at different temperatures
are given in Table 3 in support of this statement. it is due to the that dissoltuion of a gas in a liquid
solvent ab exothernic press.

Table 3: Absorption Co-efficient of gases


Temperature°C Carbon dioxide Oxygen Nitrogen

0 1.713 0.0484 0.0239


10 1.194 0.0380 0.0196
20 0.878 0.0310 0.0164
30 0.665 0.0260 0.0138
40 0.530 0.0231 0.0118

gas + solvent (liquid) ® solution ∆H = – ve


So, increase of temperature shifts the equilibrium in the backward direction which means solubility is
decreasing.
3. Effect of Pressure: Although solubilities of solids in liquids are almost independent of pressure, those
of gases in liquids vary considerably with pressure. The effect of pressure on the solubility of gases is
discussed with the help of a mathematical law known as Henry’s law. William Henry found that the
solubility of a gas at a given temperature increases directly with the pressure of the gas. This conclusion
P-151

formed the basis of the law, known as, Henry’s law. The law states that, the mass of a gas dissolved
per unit volume of a solvent is proportional to the pressure of the gas in equilibrium with
the solution at constant temperature. If m is the mass of a gas dissolved per unit volume of a
solvent and P is the pressure of the gas in equilibrium with the solution, then at constant temperature,
m ∝ P or m = kP
where k is the proportionality constant known as
Henry's constant. If Henry’s law is valid, then the
graph obtained by plotting solubility of a gas against m
the equilibrium pressure of the gas at a constant
temperature should be a straight line. P
It has been found that most gases obeys Henry’s law, Fig. 9: Mass vs P graph for gases
provided:
(i) The pressure is not too high

(ii) The temperature is not too low and


(iii) The gas is not highly soluble and does not dissociate or enter into chemical combination with the
solvent. Thus, the law is valid for the solubility of hydrochloric acid and ammonia only in benzene in
which both the gases are sparingly soluble but not in water in which these are highly soluble and also in
which hydrochloric acid dissociates (HCl + H 2 O → H 3 O + + Cl − ) and ammonia enters into chemical
combination (NH 3 + H 2 O → NH 4 OH .
Henry’s law may also be stated in terms of the volume of the gas dissolved instead of the mass. Suppose m
gram of a gas occupying a volume V is dissolved in a given volume of the solvent when the equilibrium
pressure of the gas is P. Now keeping the temperature constant, if the pressure of the gas is increased to 2P,
the mass of the gas dissolved in the same volume of the solvent will be 2m which corresponds to a volume 2V.
1
At pressure 2P the volume will be × 2V = V. It is due to Boyle’s law, which states that volume varies
2
inversely as the pressure.
Thus, the volume of the gas dissolved will remain the same as before in spite of increase in pressure. Henry’s
law may, therefore, be also stated as the volume of a gas dissolved in a solvent at a given temperature is
independent of the pressure. The validity of this statement is illustrated by the data given in table 3 for the
solubility of carbon dioxide in water at 25°C under different pressure.
Table 4: Solubility of Carbon dioxide at Different Pressures
Pressure (mm of H g ) Mass of gas dissolved Volume of gas dissolved per ml of water as
per ml of water (g) measure under experimental conditions (c.c.)

271 0.270 0.825


495 0.492 0.825
755 0.751 0.826
927 0.922 0.826
1211 1.205 0.825
1350 9.343 0.824
P-152

When a mixture of a number of gases is brought in contact with a solvent, each constituent dissolves in
proportion to its own partial pressure. Henry’s law applying to each gas is independent of the presence of the
other gases.

Henry’s law is also alternatively stated as the pressure of a gas over a solution in which the gas is dissolved is
propotional to the mole fraction of the gas dissolved in solution. Thus,

P2 = K n(χ 2 )

here the subscript 2 refers to the gaseous solute and χ 2 is its mole fraction. The constant K n is known as the
Henry’s law constant. The value of K n can be obtained by plotting P2 / χ2 versus χ 2 and extrapolating to χ 2
= 0. The Henry’s law constants for some common gases are given in table 5.

Table 5: The Henry Law Constants (in torr) Kn for Gases at 25°C

Gas Solvents

Water Benzene

H2 5.34 × 107 . × 106


275

N2 6.51 × 107 . × 106


17

O2 3.30 × 107 –

CO2 1.25 × 106 8.57 × 104

CH 4 31.4 × 106 4.27 × 105

In general, according to Henry's law,


P = K H (χ)

The fact that the solubility of a gas is proportional to its partial pressure, is made use of in removing a
dissolved gas merely by bubbling an indifferent gas through the solution.

Example 6: The Henry law constant for H 2(g) in waer is 5.34 × 107 torr. Calculate the solubility
of this gas in water at 25°C if its partial pressure over the solution is 760 torr. Assume that the
density of the solution is the same as the density of the solvent.
Solution: According to Henry's law, 760 = 5.34 × 107 (m)
m = 1.42 × 10−5 man per unit volume

5.5.2 Henry's Law and Raoult's Law


The Henry’s law, therefore, is expressed as
χ 2 = K hP2
or P2 = χ2 / K h

where χ 2 is the mole fraction of the solute gas in the solution which is in contact with the gas above having
pressure equal to P 2 and K h is a proportionality constant.
P-153

Raoult’s law holds good in ideal solutions for solvent as well as solute if both are volatile. In the above
example, we can regard P 2 as the vapour pressure of the volatile solute gas when it is present to the extent of
mole fraction χ 2 in the solution. Applying Raoult’s law, it can be said that,

P2 = χ2P2°

where P2° is the vapour pressure of the pure solute gas at the temperature of the solution.

Henry’s Law and Raoult’s Law become identical if K h is equal to 1 / P2° . In an ideal solution, therefore,
the value of Henry’s law constant K h becomes equal to 1 / P2° . In that case, Raoult’s law may be taken as a
special case of Henry’s law. Under these conditions, all the solution which obey Raoult’s law must also
obey Henry’s law. This condition however, is approached only in a few cases representing ideal solutions of
gases in liquids. In several cases of solutions of gases in liquids, it can be shown that if Henry’s law applies to
the solute, Raoult’s law applies to the solvent. To show this Gibbs-Duhem-Margules equation is used.
According to Gibbs-Duhem-Margules equation,
d(log P1) d(log P2 )
− =0
d(log χ 1) d(log χ 2 )

where χ 1 and χ 2 are the mole fractions and P1 and P2 are the vapour pressures of the solvent and solute,
respectively. Let us assume that the solute obeys Henry’s law. Then
χ 2 = K h P2

Taking logs,
We have log χ2 = logk h + logP2
Differentiating eq., it can be said that
d(logχ2 ) = 0 + d(logP2 )
d(log P2 )
or =1
d(log χ 2 )

Similarly, from these equations, it follows that


d(logP1)
=1
d(log χ1)

Integrating eq. and taking antilogs,


P1 = Kχ 1

If χ 1 = 1, then it becomes pure solvent, P1 will become P1° or the vapour pressure of the pure solvent. Hence,
the value of the constant K in eq. is equal to P1° , equation is therefore written as

P1 = χ 1P1°

This equation is the expression of Raoult’s law for the solvent. Thus, for any solution, the solute obeys
Henry’s law within a certain range of concentration, the solvent obeys Raoult’s law over the same range
of concentration.
P-154

5.5.3 Non Ideal Solution−Azeotropes


An azeotrope is a mixture of two or more liquids in such a way that its composition cannot be changed by
distillation. This happens because, when an azeotrope is boiled, the vapour has the same proportions of
constituents as the unboiled mixture. As their composition is unchanged by distillation, azeotropes are also
referred as constant boiling mixtures in old text books. The word azeotrope is derived from the Greek words
meaning no change on boiling.

Each azeotrope has a characteristic boiling point. If the boiling point of an azeotrope is less than the boiling
points of any of its constituents, then it is a positive azeotrope. Sometimes the boiling point of an azeotrope
can be greater than the boiling point of any of its constituents, then it is a negative azeotrope. So, there are
two types of azeotropes on the basis of its boiling point.

The two types are,


1. Minimum boiling azeotrope or positive azeotrope and
2. Maximum boiling azeotrope or negative azeotrope.

Minimum-boiling or Positive Azeotrope: A solution that shows positive deviation from Raoult's
law forms a minimum boiling azeotrope or positive
azeotrope at a specific composition. An ethanol-water
mixture is obtained by fermentation of sugars. It is an Va Vapour only
po
azeotropic mixture because on fractional distillation, u
&L r r
it gives a solution containing a fixed amount of iq ou
ui
d Vap
B &
ethanol. Once this has been achieved, the liquid and T
A D i quid
vapour have the same composition. A well-known C L
example of a positive azeotrope is 95.63% ethanol E

and 4.37% water by weight. Ethanol boils at 78.4 °C, Liquid only
water boils at 100 °C, but the azeotrope boils at
X 0% Composition 100%
78.2 °C, which is lower than either of its constituents.
Y 100% 0%
In general, a positive azeotrope boils at a lower Fig. 10: Positive Azeotrope
temperature than any other ratio of its constituents.

The diagram (fig. 10) shows a positive azeotrope of hypothetical constituents, X and Y. The bottom trace
illustrates the boiling temperature of various compositions. Below the bottom trace, only the liquid phase is in
equilibrium. The top trace illustrates the vapour composition above the liquid at a given temperature. Above
the top trace, only the vapour is in equilibrium. Between the two traces, liquid and vapour phases exist
simultaneously in equilibrium. For example, heating a 25% X: 75% Y mixture, to a temperature AB would
generate vapour of composition B over liquid of composition A. The azeotrope is the point on the diagram
where the two curves touch. The horizontal and vertical steps show the mechanism of repeated distillations.
Point A is the boiling point of a non–azeotropic mixture. The vapour that separates at that temperature has
composition B. The shape of the curves demands that the vapour at B be richer in constituent X than the
liquid at point A. The vapour is physically separated from the vapour–liquid equilibrium and is cooled to
P-155

point C, where it condenses. The resulting liquid represented by point C is now richer in X than it was at point
A. If the collected liquid is boiled again, it progresses to point D, and so on. The stepwise progression shows
how repeated distillation can never produce a distillate that is richer in constituent X than the azeotrope. It is
important to note that starting to the right of the azeotrope point results in the same stepwise process closing in
on the azeotrope point from the other direction.

Maximum−boiling or Negative Azeotrope: A solution that shows negative deviation from Raoult’s law
forms a maximum boiling azeotrope or negative azeotrope at a specific composition. Nitric acid and water
maxima is an example of this class of azeotrope. This azeotrope has a fixed composition of nitric acid and
water by mass, with a boiling point of 393.5 K. Another example of a negative azeotrope is hydrochloric
acid at a concentration of 20.2% and 79.8% water by weight. Hydrogen chloride boils at -84 °C and water at
100 °C, but the azeotrope boils at 110 °C, which is higher than either of its constituents. In general, a negative
azeotrope boils at a higher temperature than any other ratio of its constituents.

Vopour only
B
T D A Vapour
& Liquid
& Vapour C
id
Liqu E Liquid only

X 0% Composition 100%
Y 100% 0%
Fig. 11: Negative Azeotrope

The diagram (fig. 11) above shows a negative azeotrope of hypothetical constituents, X and Y. Again the
bottom trace illustrates the boiling temperature at various compositions and again, below the bottom trace
the mixture must be entirely liquid phase. The top trace illustrates the condensation temperature of various
compositions, and again, above the top trace the mixture must be entirely vapour phase. The point, A, shown
here is a boiling point with a composition chosen very near to the azeotrope. The vapour is collected at the
same temperature at point B. That vapour is cooled, condensed, and it is collected at point C. Because this
example is a negative azeotrope rather than a positive one, the distillate is farther from the azeotrope than the
original liquid mixture at point A was. So the distillate is becoming poorer in constituent X and richer in
constituent Y than the original mixture. Because this process has removed a greater fraction of Y from the
liquid than it had originally, the residue must be poorer in Y and richer in X after distillation than before.

If the point, A had been chosen to the right of the azeotrope rather than to the left, the distillate at point C
would be farther to the right than A, which is to say that the distillate would be richer in X and poorer in Y than
the original mixture. So in this case too, the distillate moves away from the azeotrope and the residue moves
toward it. This is a definite characteristic of negative azeotropes. No amount of distillation, however, can
make either the distillate or the residue arrive on the opposite side of the azeotrope from the original mixture.
This is characteristic of all azeotropes.
P-156

1. Assuming ideal behaviour, what mass of a non-volatile solid of molar mass 128 g mol −1 would have to
be dissolved in 228 g octane (C 8 H 18 ) to lower the vapour pressure of octane by 20%?

2. What weight of solute (molecular mass 60) is required to dissolve in 90 g of water to reduce the vapour
pressure to 2/3 of pure water?

3. 5 g of a substance with molar mass 200 have been dissolved in 50 g of a solvent with molecular mass
60 and vapour pressure 400 torr. Find the vapour pressure of the solution.

4. The vapour pressure of pure benzene at 25°C is 639.7 mm of Hg and vapour pressure of solution of a
solute in benzene at the same temperature is 631.9 mm of Hg. Calculate the molality of the solution.

5. The vapour pressures at a certain temperature of two liquids (A) and (B) are 39 mm Hg and 27.5 mm
Hg respectively. The molecular weights are M A = 72 and M B = 87. The liquids are completely
miscible. What is the vapour pressure at the same temperature for a mixture of (A) and (B) in the weight
ratio of 1 : 2 ?

6. The vapour pressure of 2 % aqueous solution of an electrolyte XY at 373 K is 755 mm of Hg. Calculate
the molecular mass of the solute. Vapour pressure of pure water at 373 K is 760 mm of Hg.

7. A solution containing 30 gm of a non-volatile solute in exactly 90 gm water has a vapour of 21.85 mm


of Hg at 25°C. Further 18 gm of water is added to solution, the new vapour pressure becomes 22.15
mm of Hg at 25°C. Calculate
(i) Molecular weight of solute
(ii) Vapour pressure of water at 25°C.

8. Calculate the composition of the vapour in equilibrium with an ideal solution of ethylbenzene (Pe° = 10
torr) and methylbenzene (Pm° = 37 torr) in which the mole fraction of ethylbenzene in the liquid is 0.35.
Calculate the total vapour pressure of the solution.

9. The vapour pressure of a solution containing 2 mol of liquid (A) and 3 mol of liquid (B) at 60°C is 500
mm. When 2 mol of (A) is further added to the above solution, the vapour pressure decreases to 400
mm Hg at the same temperature.
(i) Calculate the vapour pressures of the pure liquids (A) and (B).
(ii) The vapours of the second solution are condensed. Calculate the composition of the vapours in
equilibrium with the condensate at 60°C.

10. A certain liquid mixture of the liquids (A) and (B) behaving ideally shows a vapour pressure of 70 mm
Hg at 25°C for a certain mole fraction (χ) of (A). For the same mole fraction (χ) for (B) in the mixture
the vapour pressure of the mixture is 90 torr at 25°C, if the difference between the vapour pressure of
the pure liquid = 40 torr. Calculate (χ).
P-157

Long Answer Type Questions 7. Using boiling point-composition diagram, explain


Based on Vapour Pressure the principle of fractional distillation. [Lko. 2011]
1. What is Raoult's law? Derive Raoult's law. What is 8. Write note on phenol-water system.
the total vapour pressure of a solution made up of [Garhwal 2006]
volatile components? Based on Solutions of Gasses in Liquids
2. Derive the mole fraction of a component in first 9. State and explain Raoult’s and Henry’s law. What
condensate. happens to consolute temperature when soap is
Based on Solubility of Partially Miscible Liquids added to phenol-water system?
3. (i) Explain the principle of fractional distillation 10. Discuss Henry’s law. [Garhwal 2006]
using composition-boiling point diagram. 11. What do you understand by isohydric solution?
(ii) A mixture of water and aniline boils at 95°C
[Kanpur 2012]
when the total vapour pressure is 720mm.
Find the composition of distillate. The 12. State Henry's law. What are the conditions under
vapour pressue of water at 95°C is 634 mm. which it holds good? [Agra 2008]

[Lko. 2008]
4. Explain solubility of partially miscible liquids. Discuss
Very Short Answer Type Questions
in detail the phenol water system. Based on Vapour Pressure
5. (i) What are azeotropes? 1. Define vapour pressure.
(ii) Distinguish between positive and negative 2. What is Raoult's law?
azeotropes. 3. Why the vapour pressure of solution decreases
Based on solutions of Gasses in Liquids when a non volatile solute is added to a volatile
solvent?
6. (i) Discuss distillation of immiscible liquids.
4. Define ideal solution.
(ii) What is Henry's law?
Based on Solubility of Partially miscible Liquids
7. (i) What are the factors on which the solubility
of a gas depends? Explain. 5. What are partially miscible liquids?
(ii) What is fractional distillation? 6. What are azeotropes?
Short Answer Type Questions 7. Is phenol- water a partially miscible system?

Based on Vapour Pressure Based on Solutions of Gasses in Liquids


1. Why vapour pressure of the solvent decreses when 8. Explain Henry's law.
a non-volute solute is added to it? [Lko. 2011] 9. What is the impact of temperature on solubility of a
gas?
2. Write short note on Raoult's law. [Purv. 2010]
10. Compare Henry's law and Raoult's law.
Based on Solubility of Partially miscible Liquids
3. Water and phenol are partially miscible to each
other. Explain.
Objective Type Questions
[Garhwal 2012]
4. Explain steam distillation. [Kanpur 2012] Multiple Choice Questions
5. Distinguish between fractional distillation and 1. For dilute solution, Raoult’s law states that:
steam distillation. [Lko. 2010] (a) The lowering of vapour pressure is equal to
6. What do you mean by azeotropic mixtures? mole fraction of solute
Discuss the temperature composition curve of (b) The relative lowering of vapour pressure is
azeotropes. [Lko. 2010] equal to the mole fraction of solute
P-158

(c) The relative lowering of vapour pressure is (a) Ptotal = P*A + (P*A − PB* )χ B
proportional to the amount of solute in
(b) Ptotal = PB* + (P*A − PB* )χ A
solution
(d) The vapour pressure of the solution is equal (c) Ptotal = PB* + (PB* − P*A)χ A
to the mole fraction of solvent
(d) Ptotal = PB* + (PB* − P*A)χ B
2. A liquid is in equilibrium with its vapour at its
7. Azeotropic mixture are :
boiling point. On the average, the molecules in the
two phases have equal: (a) Constant boiling mixtures

(a) Inter–molecular forces (b) Those which boil at different temperatures


(b) Potential energy (c) Mixture of two solids
(c) Total energy (d) None of the above. [Garhwal 2010]
(d) Kinetic energy
3. Two liquids A and B have vapour pressure in the Fill in the Blank
ratio Pa° : PB° = 1: 2 at a certain temperature. Suppose 1. The boiling point of any liquid ………….. with the
increase of atmospheric pressure.
that we have an ideal solution of A and B in the
[Garhwal 2005]
mole fraction ratio A : B = 1 : 2, the mole percent of
2. Benzene-toluene mixture form an ……… solution.
A in the vapour in equilibrium with the solution at
the given temperature is: 3. The liquid mixture that boils at a definite
temperature and distills completely at the same
(a) 20 (b) 25
temperature like a pure chemical compound is
(c) 35 (d) 75
called ………………. . [Garhwal 2007]
4. The vapour pressure of a given liquid will decrease
4. For phenol-water system miscibility rise with
if:
……… of temperature.
(a) Surface area of liquid is decreased
5. For .................... system, neither of the two CSTs is
(b) The volume of liquid in the container is observed under atmospheric conditions.
decreased
6. Water-aniline mixture is comprised of ...................
(c) The volume of the vapour phase is increased liquids.
(d) The temperature is decreased. 7. For any gas the increase in pressure will …………..
5. Raoult’s law is obeyed by each constituent of a is solubility. [Garhwal 2005]
binary liquid solution when:
(a) The forces of attractions between like True and False
molecules are greater than those between 1. The boiling point of water decreases at the hill top.
unlike molecules [Garhwal 2006]
(b) The forces of attractions between like o
P −P
2. According to Raoult's law, = χ solvent.
molecules are smaller than those between Po
unlike molecules
3. Water and phenol are partially miscible to each
(c) The forces of attractions between like other. [Garhwal 2005]
molecules are identical with those between
4. Benzene is highly soluble in water.
unlike molecules
5. All partially miscible liquids pairs have an upper
(d) The volume occupied by unlike molecules
CST.
are different.
6. Et 3 N – H 2 O constitute a system in which solubility
6. For a binary ideal liquid solution, the total pressure is inversely proportional to temperature.
of the solution is given as:
7. According to Henry's law, P = K H (χ).
P-159

Numerical Questions
1. 64 g 2. 150 g 3. 388 torr

4. 0.156 mol kg-1 5. 31.83 mm Hg 6. i = 1.9, a = 0.9

7. 67.83,23.78 8. Xethyl benzene = 0.127, 9. (i) = 149.12 mm Hg, =733.92 mm Hg


Xmethyl benzene = 0.873, (ii) 0.05
P = 27.55 torr

10. X = 0.25,

Objective Type Questions


Multiple Choice Questions
1. (b) 2. (c) 3. (a) 4. (d)

5. (c) 6. (b) 7. (a)

Fill in the Blank


1. increases 2. ideal 3. azeotrope

4. increase 5. ether-water 6. partially miscible

7. increase

True and False


1. True 2. False 3. True 4. False

5. False 6. True 7. True

Hints and Solutions


Numerical Questions
P ° –P n 100 – 80 W / 128
1. Using = , it can be said =
P° n +N 100 W /128 +228 /114

2 W
A– A
P ° –P n 3 60
2. As, = , so, =
P° n +N A W 90
+
60 18
Here A is vapour pressure of pure water and w is weight of solute.

P ° –P n 400 – P 5 / 200
3. Using = , =
P° n +N 400 5 / 200 +50 / 60
P-160

P ° –P 6397
. – 631.9
4. Using = χsolute, = χsolute
P° 6397.
χ 1000
and it is known that, m = solute ×
1 – χsolute 78

a / 72 29 / 87
5. Ptotal = PA +PB = χ A P ° +χ BP ° =   × 39 +
 × 27.5
A B  a / 72 +29 / 87  a / 72 + 29 / 87
760 – 755 2/ m
6. = Solving, we get M (molecular wt. of solute)
760 2 / m+98 /18

7. Two equations are formed,


P ° –21.85 30 / M P ° –21.15 30 / M
= ....(1) and =
P° 30 / M +90 /18 P° 30 / M +108 /18
We get P° and M,

8. Pt = χ e P °e +χ m Pm
° ⇒ Pt = 0.35 × 10 + 0.65 × 37
χ e Pe° 0.35 × 10
and Ye = =
Pt 0.35 × 10 + 0.65 × 37
2 ° 3 4 3
9. (i) × PA + × PB° = 500 and × PA
°
+ × PB° = 400 solving we get PA
°
and PB°
5 5 7 7
°
4 / 7 × PA °
(ii) YA = So, Pt = YA × PA + YB × PB°
°
(4 / 7)PA + (3 / 7)PB°
°
' YA × PA
Now, YA = YB' is mole fraction required.
° °
YA × PA + YB PB
°
10. 70 = (χ)PA + (1 – χ)PB° ...(1)
°
90 × (1 – χ)PA + (χ)PB° ...(2)
°
PA – PB° = 40 ...(3)

solving we get the value of χ

Long Answer Type Questions


p water mass of water /18
3. (ii) =
p aniline mass of aniline / 93
634 mass of water 93
= ×
720 – 634 mass of aniline 18
634 mass of water 93
= ×
86 mass of aniline 18
mass of water
So, = 1.426
mass of aniline

Multiple Choice Questions


°
χ A PA a×b
3. YA = ⇒ YA =
°
χ A PA + χ B PB° a × b + 2a × 2b

1
=
5
mmm
-IV
UnitP-161

C HAPTER 6
Electrochemistry-II

6.1 Introduction
An electrochemical cell is a device capable of either deriving energy from chemical reactions or facilitating
chemical reactions through the introduction of electrical energy. A common example of an electrochemical
cell is a standard 1.5-volt battery used in torch. Oxidation-reduction or redox reactions take place in
electrochemical cells. There are two types of electrochemical cells.
1. Galvanic or Voltaic Cells: Spontaneous reactions occur in Galvanic or voltaic cells.
2. Electrolytic Cells: Non spontaneous reactions occur in electrolytic cells.

Both types of cells contain electrodes where the oxidation and reduction reactions occur. In both galvanic
and electrolytic cells, oxidation occurs at the electrode termed the anode and reduction occurs at the
electrode called the cathode.

6.2 Galvanic Cell


A Galvanic cell or voltaic cell, named after Galvani, is an electrochemical cell that derives electrical energy from
spontaneous redox reactions taking place within the cell. It generally consists of two different metals connected
by a salt bridge, or individual half-cells separated by a porous membrane. As redox reactions are exothermic
process, so that the heat released in the process is converted into electrical energy in a Galvanic cell.

6.2.1 Set Up of a Galvanic Cell


An electrochemical cell consists of two electrodes, in contact with an electrolyte, which is an ionic conductor.
The electrolyte may be a solution, a liquid, or a fused solid. An electrode and its electrolyte comprise an
electrode compartment. The two electrodes may sometimes share the same electrolyte. In a given galvanic
cell, combination of any of the two electrodes can be used. The cell may even contain same types of
electrodes with different concentration of electrolytes.

Example of one such Galvanic cell is shown here,


Pt(s) | X 2 (1 atm) | X + (c 1M) || X + (c 2 M) | X 2 (1 atm) | Pt (s).
P-162

When an inert metal like platinum is part of the cell, it is present to act as a source of electrons, but takes no
other part in the reaction. If the electrolytes are different, the two compartments may be joined by a salt
bridge. Salt bridge contains concentrated electrolytic solution in agar-agar jelly that completes the electrical
circuit and enables the cell to function.

One of the simplest Galvanic cell, called Daniel cell is shown in the given figure. It consists of two redox
couples such as Cu 2+ | Cu at one end and Zn2+ | Zn at the other end. Since Zn has a higher oxidation
potential than Cu, it gets oxidized to Zn2 + at anode. At the same time Cu2 + gets reduced to Cu at the cathode
due to its higher reduction potential (or lower oxidation potential).

Salt Bridge: Salt bridge is U – shaped glass tube filled with a jelly like substance, agar – agar (a plant jel)
mixed with an electrolyte like KCl, KNO 3 , NH 4 NO 3 etc. The one important pre- requisite for an electrolyte to
be used in salt bridge is that the cation as well as anion part of the electrolyte must have same ionic mobility
and almost same transport number. So, if KCl is selected as a salt bridge electrolyte, it means that the
transport number of K + and Cl − are almost the same.

Some of the the important functions of the salt bridge are,


1. It connects the solutions of two half - cells and completes the inner circuit.
2. It prevents transference or diffusion of the solutions from one half cell to the other.
3. It keeps the solution of two half - cells electrically neutral.
4. It prevents liquid – liquid junction potential. The liquid junction potential is the potential difference
which arises between two solutions when they are in contact with each other.

Functioning of Galvanic Cell: The zinc rod which has been oxidized becomes negatively charged due to
the electron left by the zinc as shown here,
Zn(s)→Zn 2+ (aq) +2e −

At the Cu rod, Cu 2+ from the electrolytic solution gains electron and gets converted to Cu as shown below,
Cu 2+ (aq)+2e −→Cu(s)

The electrons are withdrawn from the Cu electrode, so the Cu electrode is assumed to be positively charged.
P-163
Galvanic cell

v
V
Oxidation Reducion
Zn Zn2++ 2e- Cu2++ 2e- Cu
High-resistance voltmeter
Anode Salt bridge Cathode
Zn
Zn Cu
Cu

-
e

-
e -
Zn2+ e
- Zn2+ 2+
e Cu 2+
Cu

2+ SO 2-
2+
Zn2+
Zn SO 2-
SO442- Cu2+ SO442-
Cu

Fig. 2: Function of Galvanic Cell

As these two half cells are connected through wires and an ammeter or voltmeter, electric current flows which
is indicated by a deflection in ammeter showing that a chemical reaction is taking place in the cell. During the
course of reaction, zinc rod gets dissolved and copper is deposited on the copper rod. Thus the concentration
of the anode solution increases while that of cathode solution decreases. The flow of electrons takes place
from the zinc rod to copper rod in the external circuit. The current flow is continuous as long as the electrical
connections and the salt bridge are maintained and sufficient amount of reactant solution remains.
In addition, the purpose of the salt bridge in a Galvanic cell is to complete the inner circuit as well as maintain
the electro neutrality of both the solutions as stated earlier. Since zinc ions are produced as electrons leave the
zinc electrode, this tends to produce a net positive charge in the left compartment. After sometime the
solution would become highly positively charged due to passage of Zn 2+ in the solution. Now, the solution
would repel any further Zn2 + continuous coming in the solution, so that cell would stop working. Similarly,
no Cu 2+ would get reduced at the cathode due to the attractive forces from the sulphate ions. Thus the net
charge accumulating in the two compartments would immediately stop the electron flow through the external
circuit, and the oxidation- reduction reaction would stop. The salt bridge keeps the solution electrically
neutral by passing appropriate amounts of cations or anions to the two half cell compartments. So if KCl is
considered to be part of salt bridge- then Cl − would go to Zn half cell compartment and eliminate excess Zn2 +
2−
Similarly, K + will eliminate excess SO 4 ions.

6.2.2 Cell Notation


Some important features of cell notation are,
1. Anode is written on the left of the notation and cathode is written on the right.
2. In the notation for cells, phase boundaries are denoted by vertical bar or slash.
3. Concentration of the electrolytes in the anode and cathode must be written in parenthesis.
4. In case of a gas, the partial pressures are to be indicated in atm or mm Hg.
5. A comma is used to separate two chemical species in the same solution.
6. A double vertical line '||' denotes an interface for which it is assumed that the liquid junction potential
(potential developed between the electrolyte and the electrode) has been eliminated.
P-164

Some of the representative examples in their standard cell notation are:


(i) Zn(s) | ZnSO 4 (aq)||CuSO 4 (aq)|Cu(s);
(ii) Pt | H 2 (p 1) | HCl (c M) || AgCl(s) | Ag;
(iii) Pt | Fe 2+ (c 1), Fe3 + (c2 ) || Ag + (c) | Ag;

A noteworthy point is that in some cell representations, the salt bridge is not indicated which means that the
electrolyte is common to both anode and cathode compartment, for example,
Pt ; H 2 (p 1) | HCl | H 2 (p 2 ) ; Pt

6.2.3 Cell Potential and Nernst Equation


Energy takes many forms, mechanical work like potential and kinetic energy, heat, radiation in the form of
photons, chemical energy, nuclear energy in the form of mass, and electric energy or E.M.F. It is important to
note that the E.M.F. is determined by the nature of the reactants and electrolytes, not by the size of the cell.
Electrochemistry deals with cell potential as well as energy of chemical reactions. The energy of a chemical
system drives the charges to move, which eventually gives rise to the cell potential of a Galvanic cell. The
energy aspect is also related to the chemical equilibrium. All these relationships are tied together in the
concept of Nernst equation. Nernst equation is used to relate either half-cell potential or emf of a cell with the
concentration of the related species. In a redox reaction, the energy released in a reaction due to movement
of charged particles gives rise to a potential difference. The maximum potential difference is called
the electromotive force, (E.M.F.), E.
Let us first consider a redox change occuring in an electrochemical cell,
m1A + m2B l n1C + n2D

where A, B, C and D are the reagents whose concentrations vary. It is presumed that these are either gases or
in solution phase.
RT [C ]n1 [D]n2
E cell = E °cell − ln
nF [A]m1 [B]m2

Putting T = 298 K, R = 8.314 J/mol K, F = 96500 C, we get


0.059 [C ]n1 [D]n2
E cell = E °cell − log
n [A]m1 [B]m2

Any spontaneous reaction, taking place in a Galvanic cell, occurs with a decrease in free energy. This
decrease in free energy brings in an equivalent amount of electrical work obtainable from a given system over
and above any PdV energy that can be delivered to the surrounding. This can be calculated by the total
charge driven through cell and the potential difference. Thus
− ∆G = Total charge × E.M.F. of cell [Q energy = charge × Potential]
− ∆G = nF × E cell
Daniel cell is represented as Zn(s)| Zn 2+ (c 1)|| Cu 2+ (c 2 )|Cu(s) assumes that Zn is the anode and Cu is the
cathode. Such an assumption would be true only if the cell potential (E cell ) is positive. The cell potential is
given in the following three ways,
E cell =E RP (Cathode)− E RP(Anode)
P-165

E cell = E OP(Anode) − E OP(Cathode)


E cell = E RP(Cathode) +E OP (Anode)

Here, E RP(Cathode) is the reduction potential of the cathode while E RP(Anode) is reduction potential of the
anode, E OP(Cathode) is the oxidation potential of the cathode while E OP (Anode) is the oxidation potential of
the anode.

6.2.4 How to Find the E.M.F. of Galvanic Cell


It is important to learn how to find the E.M.F. of Daniel cell using Nernst equation. Some of the cases are
discussed below as examples.

Example: Zn(s)| Zn 2+ (c 1)||Cu 2+ (c 2 )|Cu(s)


Daniel cell, which is a type of Galvanic cell, is represented as Zn(s)| Zn 2+ (c 1)||Cu 2+ (c 2 )|Cu(s).Since one
needs to represent the reduction potential of cathode and anode, the first requirement is to write the relevant
electrode reactions.
For cathode: Cu 2+ +2e −→Cu

The Nernst equation can be written as,


RT 1
E = E° − log
Cu2 + / Cu Cu2 + / Cu nF Cu2 +

E° is the standard reduction potential of the given half reaction, R is the universal gas constant T is the
Cu2+ / Cu
absolute temperature at which the cell works, F is the Faraday constant and n is the number of moles of
electrons as seen in the reaction. In these expressions, the concentration must be always in moles per liter
while the partial pressures in atmosphere units.
0.059 1
∴ E = E° − log
Cu2+ / Cu Cu2 + / Cu n [Cu 2 + ]

For anode: Zn 2+ +2e −→Zn


0.059 1
E =E ° − log
Zn2+ / Zn Zn2+ / Zn n [Zn2 + ]

As, E cell =E −E
Cu2 + / Cu Zn2 + / Zn

0.059 1 0.059 1
∴ E cell = E ° − log − E° 2+ + log
Cu2+ / Cu n [Cu2 + ] Zn / Zn n [Zn2+ ]
0.059 [Zn2 + ]
E cell =E ° 2+ − E° 2+ − log
Cu / Cu Zn / Zn n [Cu2 + ]
0.059 [Zn2 + ]
∴ E cell =E °cell − log (Here E °cell = E ° 2+ – E ° 2+ )
n 2+ Cu / Cu Zn / Ze
[Cu ]
° 0.059 c
E cell − log 1
E cell = 2 c2
P-166

Example: Pt|H 2 |HCl|| Cu 2+ |Cu


E cell has been defined as E RP(Cathode)− E RP(Anode). The Nernst expression holds good even if the number
of moles of electrons of the two half reactions are different. For example, consider the cell,
Pt|H 2 |HCl|| Cu 2+ |Cu
For cathode : Cu 2+ + 2e −→Cu
2+ 0.059 1
E Cu / Cu =E ° − log
Cu2+ / Cu 2 [Cu2 + ]
For cathode : H + + e −→ 1 H2
2
0.059 [PH2 ]1 /2
E = E° − log
H+ / H2 H+ / H2 1 [H + ]
1
0.059 1 0.059 [PH2 ] 2
∴ E cell = E° 2+ − E° + − log + log
Cu / Cu H / H2 2 [Cu2 + ] 1 [H + ]

It is also possible to balance the electrons in both the half cell reactions and then subtract E RP(Anode)from
E RP(Cathode). That is,
For anode : 2H + +2e −→H 2
0.059 PH2
E =E ° − log
H+ / H2 H+ / H2 2 [H + ]2
0.059 [H + ]2
E cell =E ° 2+ − E° + − log
Cu / Cu H / H2 2 [Cu2 + ]PH
2

However, it is important to note that in both the cases the E cell remains the same.

6.2.5 E.M.F. of the Cell and ∆G


The electrical work or the electrical energy is equal to the product of the E.M.F. of the cell and electrical
charge that flows through the external circuit as shown here,
W max = nFE °cell ...(1)

According to thermodynamics the free energy change is equal to the maximum work. In the cell, work is
done on the surroundings by which electrical energy flows through the external circuit, So
− W max = ∆G° ...(2)

∆G ° = − nFE °cell
from eq. (1) and (2)
In non standard conditions, ∆G = − nFE cell , Here ∆G = free energy change.

6.2.6 Relation Between Standard Free Energy Change and Equilibrium Constant
Let us assume that the redox change taking place in Daniel cell attains equilibrium. At equilibrium, the
reduction potential of the two electrodes becomes equal and E.M.F. of the cell comes to be zero. [E cell = 0].
Once again the cell reaction in a Daniel cell is considered,
P-167

Zn(s) + Cu 2+ (aq) l Zn 2+ (aq) + Cu(s)


The equilibrium constant for this reaction is given as
[Zn2 + ] e
K eq = ( Here 'e' refers to equilibrium concentrations).
[Cu2 + ] e

Applying Nernst equation to the complete cell reaction,


RT [Zn2 + ]
E cell = E °cell − ln
nF [Cu2 + ]
RT
∴ E °cell = ln K eq [as E cell = 0]
nF
∴ nFE °cell = RT ln K eq

− ∆G ° = RT ln K eq

∴ ∆G° = –2.303 RT log K eq


This relation is valid for any equilibrium constant like K W , K P , K c , Ksp etc. The following schematic relation is
°
a useful tool. The three parametres which are inter-related here are K, ∆G ° and E cell .
K

RT
E°cell = lnK
nF

∆G°=–nF E° cell

Fig 3: Interrelation between ∆G°, E° and equilibrium constant

6.2.7 Heat of Reaction in an Electrochemical Cell


Let n Faraday charge flows out of a cell having E.M.F. E, then
− ∆G = nFE …(1)
Gibbs – Helmholtz equation from thermodynamics is given as
 ∂∆ G 
∆G = ∆H + T   …(2)
 ∂T  P

From equation (1) and (2) it can be said,


 ∂(− nFE)  ∂E 
− nFE = ∆H + T   = ∆H − nFT  
 ∂T  P  ∂T  P

 ∂E 
∆H = − nFE + nFT  
 ∂T  P
P-168

 ∂E 
Here,   = Temperature coefficient of cell
 ∂T  P

 ∂E 
When   = 0 then ∆H = − nFE
 ∂T  P

Example 1: Zn | Zn 2+ (c1) || Zn 2+ (c 2) | Zn. For this cell ∆G is negative if:

(a) c1 = c 2 (b) c1 > c 2 (c) c 2 > c1 (d) None

0.0591 c 0.0591 c
Solution: E = E° − log 1 = log 2
2 c2 2 c1

to make ∆G = −ve
∆E = +ve
hence, c2 > c1
Ans: (c)

Example 2: The E °cell in which the reaction:

MnO −4 + Fe2+ + H + → Mn 2+ + Fe3 + + H 2O occurs is 0.59 V at 25 ° C.

The equilibrium constant for the reaction is:

(a) 50 (b) 10 (c) 10 50 (d) 10 5

Solution: For the given half-cell reaction

− nFE °cell = − RT In K eq

−5 × 96500 × 0.59 = −2.303 × 8.314 × 298 log K eq

K eq = 7.8 × 10 49 ≈10 50

Ans: (c)
Example 3: The E.M.F. of the cell Ag | AgCl (K SP ) (saturated solution ) ||
Cl − (c1)|AgCl (K SP )| Ag is given by:

0.059 K SP 0.059 c1
(a) E =− log (b) E = − log
1 c1 1 K SP

0.059 K sp 0.059 c1
(c) E = log (d) E = log
1 c1 1 K sp
P-169

Solution: The cell can be reduced to

 K 
Ag | AgCl (satd. solution) || Ag +  SP  | Ag

 [C l ]

The reactions occuring at 2 electrodes are

At anode: Ag → Ag +A + e −

At cathode: Ag +c + e − → Ag
_________________________________
Net reaction: Ag +c → Ag +A

[Ag +A ] [Ag +c ]
∴ E cell = E °cell − 0.059 log =0.059 log
+
[Ag c ] [Ag +A ]

[Since, it is a concentration cell for which E °cell = 0]

Ksp Ksp
∴ E cell =0.059 log =0.059 log

[Cl ] Ksp [Cl − ]

Ans: (b) & (c)

Example 4: What happens when electric current is passed through an electrochemical cell
Zn | Zn 2+ || Cu 2+ | Cu ?
(a) No current will pass through an electrochemical cell

(b) Zn will be oxidised to Zn 2+ and Cu 2+ is reduced to Cu.

(c) Zn 2+ will be reduced to Zn and Cu is oxidised to Cu 2+ ions.

(d) Zn 2+ will be reduced to Zn and Cu 2+ is reduced to Cu.

Solution: When electric current is passed through an electrochemical cell, the cell reaction is completely
reversed i.e., reduction takes place at anode and oxidation takes place at cathode. Therefore, in the
electrochemical cell Zn | Zn 2+ || Cu 2+ | Cu, Zn 2+ will be reduced to Zn and Cu will be oxidised to Cu 2+
ions.
Ans: (c)

6.3 Electrochemical Series and its Significance


The series is consisted of various species in the order of their standard reduction potential. Some of the
species with their standard reduction potential are represented below in a tabular manner. Importantly, in this
series the standard reduction potential of hydrogen is considered to be zero.
P-170

Table1: Electrochemistry
S.No. Half reaction E° in volts

1. Li + + e−→Li +2.87

2. Co 3+ +e −→Co 2+ +1.82

3. Pb 4+ +2e −→Pb 2+ +1.70

4. 3+ − +1.50
Au +3e →Au
5. − − +1.36
Cl 2 +2e →2Cl

6. O 2 +4H + +4e −→2H 2 O +1.229

7. Br2 + 2e− → 2Br − +1.07

8. 2Hg 2+ +2e −→Hg 2+


2
+0.92

9. Ag + +e −→Ag +0.799

10. Hg 2
2
+
+ 2e− → 2Hg +0.79

11. Fe 3+ +e−→Fe 2+ +0.77

12. O 2 +2H + +2e −→H 2 O 2 +0.69

13. I 2 +2e −→2I − +0.535

14. Cu 2+ +2e −→Cu +0.34

15. Cu 2+ +e −→Cu + +0.15

16. Sn 4+ +2e−→Sn2 + +0.13

17. + − 0.00
2H +2e →H 2

18. Pb 2+ +2e −→Pb −0.126

19. 2+ − −0.14
Sn +2e →Sn
20. 2+ −0.25
Ni +2e −→Ni
21. Co 2+ +2e−→Co −0.28

22. Fe 2+ +2e−→Fe −0.44

23. 2+ − −0.762
Zn +2e →Zn
24. − − −0.828
2H 2 O+2e →H 2 +2OH

25. Al 3+ +3e−→Al −1.66

26. 2+ − −2.37
Mg +2e →Mg
27. + − −2.71
Na +e →Na
28. Ca 2+ +2e−→Ca −2.87

29. K + +e−→K −2.93

30. − − −3.05
F 2 +2e →2F
P-171

6.3.1 Application
Electrochemical series is one of the most important documents. It deals with a number of information. Some
of these information are discussed below,
1. Electropositive Character of Metals: The electropositive character also depends on the tendency to
lose electron or electrons. Like reactivity, the electropositive character of metals decreases from top to
bottom in the electrochemical series. On the basis of standard reduction potential values, metals are
divided into three groups,
(i) Strongly Electropositive Metals: Metals having standard reduction potential near about –
2.0 volt or more negative like alkali metals, alkaline earth metals are strongly electropositive in
nature.
(ii) Moderately Electropositive Metals: Metals having values of reduction potentials between
0.0 and about – 2.0 volt are moderately electropositive. Al, Fe, Zn, Ni, Co etc. belong to this group.
(iii) Weakly Electropositive Metals: The metals which are below hydrogen and possess
positive values of reduction potentials are weakly electropositive metals. Some of the examples
are Cu, Hg, Ag, etc.
2. Reactivity of Metals: The activity of the metal depends on its tendency to lose electron or electrons,
subsequently form cation (M a+ ) . This tendency depends on the magnitude of standard reduction
potential. The metal which has high negative value of standard reduction potential readily loses the
electron or electrons and is converted into cation. Such a metal is said to be chemically active.
(i) Alkali metals and alkaline earth metals having high negative values of standard reduction potentials
are chemically active. These react with cold water and evolve hydrogen. These readily dissolve
in acids forming corresponding salts and combine with those substances which accept electrons.
(ii) Metals like Fe, Pb, Sn, Ni, Co, etc., having not so high values of negative standard reduction
potential do not react with cold water but react with steam to evolve hydrogen.
(iii) Metals like Cu, Ag and Au which lie below hydrogen are less reactive and do not evolve
hydrogen from water.
3. Displacement Reactions:The electrochemical series helps in predicting whether a given metal will
displace another, from its salt's solution. The metal having low standard reduction potential will
displace the metal from its salt’s solution which has higher value of standard reduction potential. A
metal having lower standard reduction potential has greater tendency to provide electrons to the
cations of the metal to be precipitated.
Displacement of one non metal from its salt solution by another non-metal can also be predicted with
the help of electrochemical series. A non-metal having high value of reduction potential will displace
another non-metal with lower reduction potential. The non-metal’s which possess high positive
reduction potentials have the tendency to accept electrons readily. These electrons are provided by the
ions of the non–metal having low value of reduction potential. Thus, Cl2 will displace bromine and
iodine from bromides and iodides. One such example is shown below.
Cl2 + 2KI → 2KCl + I2
2I − →I 2 +2e − .....(Oxidation)
Cl 2 +2e − → 2Cl − .....(Reduction)
The summary is that activity or oxidising nature of the non metal increases as the value of reduction
potential increases. So, higher the standard reduction potential, stronger is the oxidising agent. It is due
to this fact, F2 is strongest oxidising agent.
P-172

Another important aspect of displacement reaction is the displacement of hydrogen from dilute acids
by metals. The metal which can provide electrons to H + ions present in dilute acids for reduction,
evolve hydrogen from dilute acids.
Mn→ Mn n+ +ne − .....(Oxidation)
2H + + 2e − → H2 .....(Reduction)
The metal having negative values of reduction potential possess the property of losing electron or
electrons. Thus, the metals occupying top positions in the electrochemical series readily liberate
hydrogen from dilute acids and on descending in the series tendency to liberate hydrogen gas from
dilute acids decreases. The metals which are below hydrogen in electrochemical series like Cu, Hg, Au,
Pt, etc., do not evolve hydrogen from dilute acids.
Displacement of hydrogen from water can also be discussed with the help of electrochemical series.
Iron and the metals above iron are capable of liberating hydrogen from water. The tendency decreases
from top to bottom in electrochemical series. Alkali and alkaline earth metals liberate hydrogen from
cold water but Mg, Zn and Fe liberate hydrogen from hot water or steam.
4. Reducing Power of Metals: Reducing nature depends on the tendency of losing electron or
electrons. More is the negative reduction potential, more is the tendency to lose electrons. Thus
reducing nature decreases from top to bottom in the electrochemical series. The power of the reducing
agent increases, as the standard reduction potential becomes more and more negative. Sodium is a
stronger reducing agent than zinc and zinc is a stronger reducing agent than iron.
Alkali and alkaline earth metals are strong reducing agents. As such, Li is the strongest reducing agent.
5. Oxidising Power of Non-metals: Oxidising nature depends on the tendency to accept electron or
electrons. Higher the value of reduction potential, higher is the tendency to accept electron or electrons.
Thus, oxidising nature increases from top to bottom in the electrochemical series. The strength of an
oxidising agent increases as the value of reduction potential becomes more and more positive.
F2 is a stronger oxidant than Cl 2 , Br 2 and I 2 , similarly Cl 2 is a stronger oxidant than Br 2 and I 2 .
→ → → →
I2 Br2 Cl2 F2
+0.53 +1.06 +1.36 +2.85
----------------------------− − − − − − − →
Oxidising Nature Increases
6. Extraction of Metals: A more electropositive metal can displace a less electropositive metal from its
salt’s solution. This principle is applied for the extraction of Ag and Au by cyanide process. Silver from
the solution containing sodium argento cyanide, NaAg(CN) 2 , can be obtained by the addition of zinc
as it is more electro-positive than Ag.
2 NaAg(CN) 2 +Zn→Na 2 Zn(CN) 4 +2Ag

Example 5: The standard reduction potentials at 298 K for the following half reactions are given
against each:
Zn 2+ (aq) + 2e − l Zn(s) −0.762
Cr 3+
(aq) + 3e −
l Cr(s) −0.740
2H + (aq) + 2e − l H 2(g) 0.00
Fe 3+
(aq) + e −
l Fe 2+
(aq) 0.770
P-173

The strongest reducing agent is:


(a) Zn(s) (b) Cr(s) (c) H 2(g) (d) Fe 2+ (aq)

Solution: Zn 2+ (aq) is having minimum reduction potential. So oxidation potential of Zn(s) will be
maximum and it will be strongest reducing agent.
Ans: (a)

Example 6: A solution containing one mole per litre each of Cu(NO 3 ) 2, AgNO 3 , Hg 2(NO 3 ) 2,
Mg(NO 3 ) 2 is being electrolysed by using inert electrodes. The values of standard potentials are:

E° = 0.80 V, E° = 0.79 V, E° = 0.34 V, E° = −2.3V


Ag + / Ag Hg 2+|Hg Cu 2+|Cu Mg 2+|Mg
2

With increasing voltage, the sequence of deposition of metals on the cathode will be:
(a) Ag, Hg, Cu, Mg (b) Mg, Cu, Hg, Ag
(c) Ag, Hg, Cu (d) Cu, Hg, Ag

Solution: The metal ions will be get preferentially deposited on the cathode in order of their decreasing
reduction potentials. The order of deposition of metals will be Ag, Hg and Cu. Mg will not be deposited
because H + will be preferentially discharged.
Ans: (c)

Example 7: If E ° is x1, E ° is x 2 then E ° will be:


Fe 2+ / Fe Fe 3+ / Fe Fe 3 + / Fe 2 +

(a) 3x 2 − 2x1 (b) x 2 − x1 (c) x 2 + x1 (d) 2x1 + 3x 2

Solution: ∆G 3 = ∆G 2 − ∆G 1

−n3FE3° =− n2FE2° + n1FE1°

n2E2° − n1E1°
E3° = = 3x 2 − 2x 1
n3

Ans: (a)

6.3.2 Overpotential or Overvoltage


In electrochemistry, overpotential is the potential difference (voltage) between thermodynamically determined
reduction potential of a half cell reaction and the potential at which the redox event is experimentally
observed. For example, the overvoltage or overpotential value of chlorine gas is + 0.08 V. it means that the
gas is released at the respective electrode at a potential which is higher than its potential by 0.08 V. The term
is directly related to the voltage efficiency of a cell. In an electrolytic cell, the overpotential requires more
energy than thermodynamically expected to drive a reaction. In a galvanic cell, overpotential means less
energy is recovered than thermodynamics predicts. In each case the extra or missing energy is lost as heat.
Overpotential is customised to each cell design and varies across cells and operational conditions, even for
the same cell reaction. Generally speaking, overpotential is a characteristic of gaseous substances only.
P-174

Overpotential for certain gases on different electrodes at 25 °C

Material of the electrode Hydrogen Oxygen Chlorine

Platinum (platinized) – 0.07 V +0.77 V +0.08 V

Palladium –0.07 V +0.93 V

Gold –0.09 V +1.02 V

Iron –0.15 V +0.75 V

Platinum (shiny) –0.16 V +0.95 V +0.10 V

Mercury –0.85 V

Nickel –0.28 V +0.56 V

Graphite –0.62 V +0.95 V +0.12 V

Lead –0.71 V +0.81 V

Zinc –0.77 V

Bubble overpotential is a specific case of concentration overpotential. It is due to the evolution of gas at either
the anode or cathode. An example is the electrolysis of an aqueous sodium chloride solution—although
oxygen should be produced at the anode based on its potential, bubble overpotential causes chlorine to be
produced instead.Itforms the basis of the industrial production of chlorine and sodium hydroxide by
electrolysis from brine (aqueous NaCl).

6.3.3 Types of Electrodes


Before starting the discussion about different types of electrodes and other details about them, one must
understand the concept of reference electrodes. A reference electrode is an electrode which has a
stable and well-known electrode potential. In electrochemical methods, where it is necessary to
observe, measure, or control the potential of another electrode, it is necessary to use a reference electrode,
which maintains a potential that remains practically unchanged during the course of an electrochemical
measurement.

One such electrode, the normal hydrogen electrode, has been chosen as a reference electrode, relative to which
potentials of other electrodes and those of oxidation-reduction couples are often expressed. It is termed as a
primary reference electrode. By maintaining a constant pressure of hydrogen gas, the potential of a hydrogen
electrode can be used for determination of the activity of hydrogen ions in the tested solution. However in
practice, the determination of the hydrogen-ion activity (pH) is performed by using a glass electrode. The
hydrogen electrode itself is used only in fundamental studies and some non aqueous solutions. The hydrogen
electrode, however, remains important for providing a reference standard.

In practice, potentials are measured against reference electrodes that are easier to work with than the normal
hydrogen electrode. Such electrodes are known as secondary reference electrodes. The most common
electrodes are the calomel and silver–silver chloride electrodes. There are many applications of reference
electrodes. The simplest application is when the reference electrode is used as a half cell to build an
electrochemical cell. This allows the potential of the other half cell to be determined. The various kinds of
electrodes are summarized in the following table and illustrated one by one.
P-175

Table 2: Different types of electrodes


S.No Electrode type Designation Half-reaction
+
(a) Metal-metal ion M (aq) | M(s) M + (aq)+e− →M(s)
(b) Gas-ion electrode X + (aq) | X 2 (g)| Pt(s) X + (aq)+e−→ ½X 2 (g)

Pt(s)|X 2 (g) | X − (aq) ½X 2 (g)+e−→ X − (aq)

(c) Metal-insoluble X − (aq) | MX(s)|M(s) MX(s)+e−→M(s)+X − (aq)


salt-anion
(d) Redox electrode M 2+ (aq) , M + (aq)|Pt(s) M 2+ (aq)+e−→ M + (aq)

Gas Electrodes: Gas electrode is of so many types. Some of the important


types are discussed here. A simple schematic diagram of a gas electrode is
shown here, The reaction taking place is,
X 2 + 2e – → 2 X –
or X 2 → 2 X + + 2e –
1. Hydrogen Electrode: Hydrogen gas bubbled in a solution of an
acid like HCl forms an electrode of this type. Since hydrogen gas is
non-conducting, platinum or some other metal, which is inert and
hence cannot be attacked by the acid, is used for making electrical
contact in the circuit. Fig.4: Gas electrode
Thermochemical study shows that hydrogen is adsorbed at metal surphace
in atomic state. The reaction taking place are
½H 2 (g) → H(Pt)
H(Pt) → H + (aq) +e − (Pt)

The net reaction is


½H 2 (g) →H + (aq) + e − (Pt)
The electrode assembly is represented as (if the half cell is act as cathode)
H + (aq)|H 2 (g)|Pt
It is discussed in detail in later part of chapter.
2. Oxygen Electrode: In this electrode, oxygen gas at a given pressure
is introduced into a solution containing hydroxyl OH − ions. The Fig.5: Hydrogen electrode
electrode reaction is written as
O 2 (g) + 2H 2 O + 4e −→ 4OH − (aq)
The electrode acting as cathode is represented as
(aq)OH − |O 2 (g)|Pt
3. Chlorine Electrode: In this electrode, chlorine gas at a given pressure is bubbled into a solution of
HCl. The electrode acting as cathode is represented as
Cl − (aq)|Cl 2 (g)|Pt
and the electrode reaction is written as
½ Cl 2 (g) + e −→ Cl − (aq)
P-176

Example 8: The reduction potential of Cl 2 | Cl − electrode when the partial pressure of Cl 2 is 10


atm and [Cl − ] = 1 × 10 −3 M is (Given: Standard reduction potential of Cl 2 | Cl − = 1.36 V):
(a) 1.15 V (b) 1.10 V (c) 1.20 V (d) 1.567 V
− −
Solution: Cl 2 + 2e → 2Cl
0.059 [Cl -]2 0.059
E −= 1.36 – log = 1.36 − log (1 × 10 −7 ) = 1.567 V
Cl2 / Cl 2 PCl2 2
Ans: (d)

Metal-Metal Ion Electrodes: The electrodes used in the Daniel


cell are the examples of metal-metal ion electrodes. like Zn|Zn 2+ or
Cu|Cu 2+ etc.
Another example of this type is a silver-silver ion electrode which is
represented as Ag |Ag + . The anodic half-cell reaction is
Ag(s)→ Ag + (aq) + e −
Highly active metal reacts directly with water and thus cannot be
used for such electrodes. So, for preparation of such electrodes, very
active metals are avoided.
A metal electrode may be made of an amalgam of the metal in . -
mercury instead of the pure metal. Some of the irreproducible
effects associated with solid surfaces are eliminated when amalgam
electrodes are used. Amalgams offer the only route for the study of the
electrochemistry of highly reactive metals in this pure state.
Redox Electrodes: A redox electrode consists of ions of two
different oxidation states and a non-reactive metal, usually platinum.
The schematic version is shown here, The general reaction taking
place in electrode vessel is
M n+ + (n – m) e – → M m+

An example is Pt | Cr2 + (c 1), Cr 3+ (c 2 ). The half cell reaction with this


electrode as the anode is .
2+ 3+ −
Cr (aq)→Cr (aq)+e (Pt).

Redox electrode can also be made with organic molecules that can exist in two different oxidation states.A
biochemical system of hydroquinone (QH 2 ) and quinone(Q) forms a redox electrode which is abbreviated as
Pt | QH 2 , Q, H +
OH O
The half cell reaction is represented as
or QH 2 l Q +2H + + 2e − or
q + 2H++2e-
or C 6H6O 2 ® C 6H4O 2 + 2H+ + 2e –
This electrode is called quinhydrone electrode because of the charged OH O
complex formed between quinone and hydroquinone.
P-177

Glass Electrode: A glass electrode is a type of ion-selective electrode made of a doped glass membrane
that is sensitive to a specific ion. It is an important part of the instrumentation for chemical analysis and
physico-chemical studies. In modern practice, widely used membranous ion-selective electrodes are part of a
Galvanic cell. The electric potential of the electrode system in solution is sensitive to changes in the content of
a certain type of ions, which is reflected in the dependence of the electromotive force (EMF) of galvanic
element concentrations of these ions. Glass electrodes are commonly used for pH measurements. There are
also specialized ion sensitive glass electrodes used for determination of concentration of lithium, sodium,
ammonium and other ions.

coaxial cable

end cap end cap

back seal glass


back seal
foil shield
electrode
KCl
silver reference element
element electrode KCl
fill solution electrolyte
KCI 7.0 pH
pH mercuric
Outer chloride
electrodes
body tip: Hg Cl
sensitive
glass silver chloride liquid junction:
membrane tip: Agcl porous ceramic
bulb

Fig. 8: Glass electrodes paired with a reference electrode

Metal-Insoluble Metal Salt-Anion Electrodes: A simple schematic diagram of such kind of electrode is
shown here,
An example of this type of electrode is Cl − |AgCl|Ag. In this electrode a
solution of Cl − is taken and a rod of Ag coated with a paste of AgCl is dipped M(s),
into this solution. The reaction that take place are:
MX(s)
AgCl l Ag + + Cl − …(5)

Ag + + e − →Ag …(6)
_____________________
Net half cell reaction: AgCl + e −→Ag+Cl − …(7)
_____________________ X-(aq)
These electrodes are constructed for any metal with its insoluble metal salt
Fig .9: Metal-insoluble salt
dipped in a solution containing the common anion of the salt and not the
-anion electrode
common cation. Obviously the anion in the solution should come from
highly soluble electrolyte. Moreover, the cation in the solution should have a standard reduction potential less
than that of the electrode under consideration.
P-178

Calomel Electrode: Another metal- insoluble salt-anion electrode


is the calomel electrode which is used as a secondary reference
electrode. The calomel electrode is represented as Cl − (c) | Hg 2 Cl 2 |
Hg. The cathodic half cell reaction is ½Hg 2 Cl 2 (s) + e −→ Hg(l) + C
l − (aq). The Schematic diagram of calomel electrode is shown in fig
10 b. It can be easily set up in the laboratory. Highly pure Hg is Ag
placed in a glass tube as shown in figure (10 b).
Hg is covered by a paste of calomel Hg 2Cl2 . A solution of KCl is
introduced through a ride tube shown in the right. The concentration
of KCl is either 1N or 0.1N or a saturated solution. The Pt-wire is Cl-
used to set electrical contact. The net electrode reaction is,
Hg Cl + 2e– 2Hg + 2Cl – AgCl (Paste)
2 2
(Liquid) Fig.10 (a): Cl-|AgCl|Ag electrode
(Solid)

The electrode is reversible wrt Cl ions.

6.3.4 Standard Hydrogen Electrode (SHE) Saturated calomel


electrode
It is a primary reference electrode: The stanl dard hydrogen electrode
electrode abbreviated as SHE, is a redox electrode which forms connection
the basis of the thermodynamic scale of oxidation- reduction
potentials. To form a basis for comparison with all other electrode
reactions, standard electrode potential (E0 ) is considered to
be zero at all temperatures. Potentials of any other electrodes
are compared with that of the standard hydrogen electrode at saturated
the same temperature. Hydrogen electrode is based on the KCl solution
redox half cell reaction as shown below:
calomel mixture
2H + (aq) + 2e − → H 2 (g) Hg2Cl2
This redox reaction occurs at a platinum electrode. The glass frit
electrode is dipped in an acidic solution and pure hydrogen gas KCl crystals
is bubbled through it. The concentration of both the reduced Fiber wick for contact
form and oxidised form is maintained at unity. That implies that with external solution
the pressure of hydrogen gas is 1 bar and the activity of
Fig. 10 (b): Saturated Calomel electrode
hydrogen ions in the solution is unity. The activity of hydrogen
ions is the effective concentration, which is equal to the formal
concentration times the activity coefficient. These unit-less activity coefficients are close to 1.00 for very dilute
aquouos solutions, but are usually lower for more concentrated solutions. The Nernst equation should be
written as:
RT a +
E= ln H
F (PH / P0 )1 /2
2
2.303RT RT
or E=− pH − ln p H
F 2F 2 /p0

Where, a H+ is the activity of the hydrogen ions, PH is the partial pressure of the hydrogen gas, in pascals,
2
R is the universal gas constant, T is the temperature, in kelvins F is the Faraday constant, P0 is the standard
pressure =10 5 Pa.
P-179

6.3.5 Determination of pH
By Using Hydrogen Electrode: The potential of a hydrogen electrode in contact with a solution of H + ions
involving the reaction
1
H + +e − → H 2 (1 atm)
2
is given by Nernst equation,
 RT 
E el =E °el +2.303  +
 log [H ]
 F 
°
By convention, E(el) , the standard electrode potential of hydrogen electrode, is zero
2.303RT
∴ E el = log[H + ] = −0.0591pH (at 25° C)
F
Thus, the potential of a hydrogen electrode depends upon the pH of the solution with which it is in contact.
This can be determined by combining the hydrogen electrode with a reference electrode, say, calomel
electrode.
The complete cell is represented as
Pt, H 2 (1 atm), H + (c = unknown) || KCl (sat. soln.), Hg 2 Cl 2 (s), Hg.
The E.M.F. of the cell is determined potentiometrically. This is given by
E=E R −E L =0.2422−(−0.0591pH)
or 0.0591pH=E−0.2422
E − 0.2422
or pH=
0.0591
Here, 0.2422 V is the standard reduction potential of calomel electrode.

By Using Quinhydrone Electrode: The quinone-hydroquinone system involves the following equilibrium:
C6H4O2 + 2H++2e- C6H6O2
Quinone Hydroquinone
(Q) (QH2)
The potential developed on a platinum electrode immersed in this system is given by the Nernst equation as
2.303RT [QH2 ]
E = E °el − log
2F [Q][H + ]2
2.303RT [Q][H + ]2
= E °el + log
2F [QH2 ]
° 2.303RT [Q] 2.303RT
=E el + log + logH +
2F [QH2 ] F
Instead of taking quinone and hydroquinone, a small amount of qunihydrone, which is an equimolar mixture
of the two compounds - quinone (Q) and hydroquninone [QH] 2 , is taken. Since hydroquinone (QH 2 ) is a
weak acid, its ionization is very small particularly if the pH of the solution is less than 7. Therefore, the
[Q]
concentration of hydroquinone [QH 2 ] is the same as that of quinone [Q], the quantity is unity. The
[QH2 ]
middle term in the equation, above is therefore, reduces to zero. Hence
P-180

2.303RT
E el =E °el + [H + ]=E °el + 0.0591log[H + ]at 25 ° C
2F
=E °el − 0.0591pH

As the standard reduction potential of the quinhydrone electrode is, E °el = + 0.06996 V.
∴ E °el = + 0.06996 − 0.0591pH

Thus, the potential of the quinhydrone electrode, just as that of the hydrogen electrode, depends upon the
pH of the solution with which it is in contact. So, the quinhydrone electrode behaves as a reversible hydrogen
electrode. Consequently, this electrode can be used for measuring pH values of solutions. The quinhydrone
electrode is preferred to the hydrogen electrode as it can be set up easily by merely adding a pinch of
quinhydrone to the solution under examination and inserting a clean platinum electrode for making electrical
connection. The qunihydrone electrode is combined with a saturated calomel electrode to form a cell. The
combination may be represented as
Hg, Hg2 Cl2 (s),KCl(sat. soln.)|| H + (unknown conc.)Q,QH2 ;Pt

The E.M.F. of the above cell is given by


E=E R −E L =(0.6996−0.591pH)−0.2422 at 25° C
Or, 0.0591pH=0.6996−0.2422−E
0.6996 − 0. 2422 − E
∴ pH =
0.0591

Limitations of Qunihydrone Electrode: The quinhydrone electrode cannot be used for solutions of pH
more than 8. It is because:
1. Q
In more alkaline solutions, hydroquinone ionizes appreciably as an acid so, ratio is no longer
QH2
unity and,
2. Also gets oxidized partly by atmospheric oxygen.
Due to these facts, the equilibrium present between quinone and hydroquinone is distorted.

6.3.6 Activity and Mean Ionic Activity of an Electrolyte


In a number of cases discussed above, references have been made to activities of individual ions. But it
should be clearly understood that there is no method by which activities of individual ionic species can be
determined experimentally. The reason is that it is not possible to have a solution containing only one kind of
ions. By experiment, therefore, the quantity that is determined is activity a or mean ionic activity a ± of the
electrolyte concerned.
Consider a uni-univalent electrolyte, like HCl, in aqueous solution. Let a + and a − be the hypothetical
activities of H + and Cl − ions, respectively, in a solution of a given concentration, the activity a of the
electrolyte is defined by the expression,
a=a +a −
Similarly, mean ionic activity a ± of the electrolyte is defined by the expression
a ± = a +a −
P-181

Thus, while the activity of an electrolyte is given by the product of activities of component ions, the mean
ionic activity of the electrolyte is given by the geometric mean of activities of component ions. Squaring,
(a ± )2 = a +a −

Combining equations,
a=a +a − = (a ± )2

Mean Ionic Activity Coefficients of Electrolytes: Consider an electrolyte M X A Y → xM + + yA −


ionizing as
M X A y → xM + + yA −

For the sake of simplicity, the charge on each cation is represented by a single plus and that on each anion by
a single minus sign. Accordingly, the activity of the electrolyte in this case would be given by
a=(a + )x (a − )y

=(a ± )x + y (by definition)

If m is the molality of the electrolyte, then, molality of the cations, m + = xm and molality of the anions, m − =
ym. The activity and molality are known to be related by the expression
a=γm

where γ is the activity coefficient. Acitivity of the electrolyte M x A y , is, therefore, given by
y
a=(γ + m + ) x (γ − m − )

= (γ + x m) x (γ − ym) y

= x x y y (γ ± m) x + y

Where γ ± is the mean ionic activity coefficient related with γ + and γ − by the expression
(γ ± ) x + y = (γ + )x (γ − )y

For a uni-univalent electrolyte, HCl or NaCl, x = 1 and y = 1, Hence,


a=(a ± )2

a=(γ ± m)2

The quantity measured by experiment is either activity a or mean ionic activity of the electrolyte a ± at a given
molality m. Hence, mean activity coefficient γ ± of the electrolyte at a given concentration can be easily
calculated. For a uni-bivalent electrolyte, Na 2 SO 4 , x = 2 and y = 1. Hence,
a=(a ± )3

and, a=22 ×1(γ ± m)3 = 4(γ + )3 m 3


and so on. The concept of activity and activity coefficients has immense relevance in the discussion of
concentration cells.
P-182

6.4 Concentration Cells


The electrical energy arises from the chemical reactions which take place in the cells in the case of Galvanic
cells as discussed earlier. There is, however, another category of cells in which the E.M.F. arises not due to
any chemical reaction but due to difference in the concentration of the same species in the two half cells. This
difference in the concentration causes transfer of the matter from one half-cell to the other. These are called
concentration cells.
Types of concentration cell: Concentration cells are of two types
1. Electrode Concentration Cells
2. Electrolyte-Concentration Cells

6.4.1 Electrode Concentration Cells


In these cells, two electrodes at different concentrations are dipping in the same solution. However both the
electrodes are the same. One of the examples of a concentration cell of this category is a lead amalgam with
two different concentrations,
Hg-Pb(c 1),PbSO 4 (solution), Hg-Pb(c 2 )

The electrode reactions are:


RHE:Pb 2+ +2e − → Pb(c 2 ) (Reduction)
LHE:Pb(c 1) → Pb 2+ +2e − (oxidation)
Overall reaction: Pb(c 1) → Pb (c 2 )

The E.M.F. of cell is given by


 0.0591   ° 0.0591 
E=E R − E L = E °Pb − log c2  − E Pb − log c1
 2   2 
0.0591 c c
= log 1 = 0.02955log 1
2 c2 c2

Here, if c1 > c2 the E.M.F. is positive so that the whole process is spontaneous, so that lead will go
spontaneously from the high concentration to the low concentration amalgam.
Another example of the electrode-concentration cell is that of two hydrogen electrodes at unequal gas
pressures. Here, two hydrogen electrodes at unequal gas pressures are immersed in the same solution of
hydrogen ions thereby constituting an electrode concentration cell. This is represented as follows:
Pt | H 2 (p 1) | solution of H + ions, say, HCl | H 2 (p 2 ) | Pt

The reactions occurring are :


RHE: 2H + + 2e − → H 2 (p 2 ) (Reduction)
LHE: H2 (p 1) → 2H + + 2e − (Oxidation)

Overall reaction: H 2 (p 1) → H 2 (p 2 )
This reaction is independent of the concentration of the electrolyte, as observed. At moderate pressures, H 2
can be considered to be an ideal gas so that the ratio of the activities can be considered to be equal to the ratio
of the gas pressures. Hence, the Nernst equation may be written as
0.0591 p 
E=E ° − log  2  at 25° C
2  p1 
P-183

Since, both the left hand and right hand electrodes are the same, by definition, E° = 0, so,
p  p 
E=− 0.02955 log  2  = 0.02955 log  1 
 p1   p2 

When p 1 < p 2 , the E.M.F. is positive so that the whole process is spontaneous, being equivalent to the
expansion of H 2 gas.

Example 9: Calculate the E.M.F. of the electrode-concentration cell


Pt|H 2(p1),HCl,H 2(p 2)|Pt
at 25°C if p1 = 600 torr and p 2 = 400 torr.
Solution: The overall cell reaction is, H 2 (P 1)l H 2 (P2 )
0.0591 P 0.0591 400
∴ E= log 2 = log
2 P1 2 600

=5.19 × 10−3 Volts.

Example 10: Calculate the E.M.F. of the electrode-concentration cell


Hg−Zn(c1),Zn 2+ (aq),Hg−Zn(c 2)
at 25°C, if the concentrations of the zinc amalgam are : c1 = 2 g per 100 g of mercury and c 2 = 1 g
per 100 g of mercury.
Solution: The overall cell reaction is, Zn(c 1) l Zn (c 2 )
0.0591 c 0.0591 1
∴ E= log 2 = log
2 c1 2 2

= 5.19 × 10−3 Volts.

6.4.2 Electrolyte-Concentration Cells


In these cells, the two electrodes of the same metal are dipped in solutions of metal ions at different
concentrations or different activities. One such cell is represented below:
Zn,Zn2 + (a + )1||Zn2 + (a + )2,Zn

In this case, both the electrodes are of the same metal (Zn) and these are in contact with solutions of the same
ions Zn 2+ . The concentrations and hence activities of the ions, are, however, different. Let (a + ) 1 and (a + ) 2 be
the activities of zinc ions in the two electrolytes surrounding the electrodes. The two electrolytes, which are
generally ZnSO 4 solutions, are separated from each other by a salt bridge. This is represented by the double
bar put in between the two half-cells.

The reactions occurring are


At cathode: Zn 2+(a + )2 +2e − → Zn(s) (Reduction)

At anode: Zn(s)→ Zn (a + ) + 2e − (Oxidation)

Overall cell reaction : Zn 2+(a + )2 → Zn 2+(a + )1


P-184

The net process thus involves the transfer of Zn 2+ ions from the solution in which the activity is (a + )2 to the
solution in which the activity is (a + )1. According to Nernst equation, the reduction potential of RHE and LHE
are given by
° RT 1 ° RT
E R = E el − ln = E el + ln(a + )2
nF (a + )2 nF
RT 1 RT
and E L =E °el − ln = E °el + ln(a + )1
nF (a + )1 nF
RT (a + )2
∴ E cell = E = E R − E L = ln
nF (a + )1

For the process to be feasible, E.M.F. should be positive. Hence, (a + )2 >(a + )1. Substituting activities (a + )1
and (a + )2 by molalities m 1 and m 2 , it can be said that,
RT γ 2 m2
E cell ≡ E = ln (Qa = γ × m)
2F γ1m 1

Another similar concentration cell is considered, which is represented by


Pt| H 2 (g),H + (a + )1||H+ (a + )2 ,H2 (g)|Pt

In this case, both the electrodes are hydrogen electrodes which are in contact with hydrogen ions of different
activities. The two solutions, which are generally solutions of hydrochloric acid, are separated by a salt
bridge, as before.

The reactions taking place at the two electrodes are:


1
RHE: H + (a + )2 + e − → H2 (g) (Reduction)
2
1
LHE: H2 (g) → H + (a + )1 + e − (Oxidation)
2
Overall cell reaction : H + (a + )2 → H +(a + )1

The E.M.F. of the cell will be given, as before, by


E.M.F. = [reduction potential of right hand electrode] – [reduction potential of left hand electrode]. As the
standard reduction potentials on both the sides are the same, so,
 RT 1   ° RT 1   RT  (a + )2
E=E R − E L = E °el − ln  − E el − ln =  ln …(8)
 F (a + )2   F (a + )1   F  (a + )1

For E.M.F. to be positive, (a + )2 should be greater than (a + )1. Another example is considered now. A cell is
considered consisting of two cells connected back to back through silver electrodes:
Pt| H2 ,HCl(a 1),AgCl(s),Ag||Ag, AgCl(s),HCl(a 2 ),H2|Pt

The reactions taking place at the silver electrodes are,


1
RHE:Ag+H + (a 2 ) + Cl − (a 2 ) + e − → H2 (1 atm) + AgCl(s) (Reduction)
2
1
LHE: H2 (1 atm) + AgCl(s)→ Ag+H +(a1)+Cl − (a 1)+e − (Oxidation)
2
P-185

Overall reaction: H + (a 2 ) + Cl − (a 2 ) → H + (a1) + Cl − (a1)


HCl(a 2 )→HCl(a1)
Thus, the overall reaction involves no chemical change. It consists only of the transfer of HCl from activity a 2
to activity a 1.The E.M.F. of the cell is given by
RT 1  RT 1 
E cell ≡ E = E R − E L = E °el −
ln − E °el − ln 
F + −
(a H)2 (a Cl )2  F (a H)(a Cl )1 
+ −

2RT (a ± )2 (a )
= ln = 0.1182 log ± 2 at 25 ° C …(a)
F (a ± )1 (a ± )1

here (a ± )1 and (a ± )2 are the mean ionic activities of the HCl solutions in the two cells. Here, too, it can be
observed that when a 2 > a1, E is positive so that the process is spontaneous from right to left.The physical
significance is that the dilution of HCl from activity a 2 to activity a 1 is a spontaneous event, even though the
two solutions are not in contact with each other.

Example 11: Calculate the E.M.F. of the concentration cell consisting of zinc electrodes, one
immersed in a solution of 0.01 molality (number of moles dissolved per kg of the solvent) and
the other in a solution of 0.1 molality at 25°C. The two solutions are separated by a salt bridge.
The mean activity coefficient of the electrolyte may be assumed to be unity.
Solution: The cell is represented as,
Zn|Zn2 +||Zn2 +|Zn
(0.01 m) (01. m)
RT (a + )2 2.303 RT γ m
E= ln = log 2 2
nF (a + )1 2F γ1 m1
0.0591 0.1
= log = 0.0295 volts
2 0.01

Example 12: Find the potential difference between the hydrogen electrodes in the cell.
Pt,H 2(g),HCl,AgCl(s),Ag||Ag.AgCl(s),HCl,H 2(g),Pt
at 25°C. The activity coefficients of 0.01 m and 0.10 m solutions are 0.95 and 0.85, respectively.
Solution: The Potential difference between two H electrodes is
RT (a + )2 2.303 RT γ m
E= ln = log 2 2
nF (a + )1 nF γ1 m1
0.0591 0.1 × 0.85
= log
1 0.01 × 0.95
= 0.05624 volts

6.4.3 Types of Electrolyte Concentration Cells


Electrolyte concentration cells in which solutions of the same electrolyte of different concentrations are used
can be of two types,
1. The two solutions are separated from each other by means of a salt bridge or by some other means. In
this type, the two solutions are in direct contact with each other. The transference of ions from one
solution to the other takes place directly. Such cells are called concentration cells with transference.
P-186

2. In the second type, the two electrolytic solutions are not in direct contact with each other and the
transference of ions from one solution to the other does not take place directly. These are called
concentration cells without transference.
Here, both the types of concentration cells will be discussed in detail.
I. Concentration Cells with Transference: A concentration cell is considered which is formed by
combining two hydrogen gas electrodes in contact with HCl solutions of different concentrations. The
two solutions are in direct contact with each other, as shown:
Pt|H 2 (g), HCl(a 1)||HCl(a 2 ), H 2 (g)| Pt
1 atm 1 atm
The reaction on the left hand electrode involves oxidation and that on the right hand electrode involves
reduction, as usual.
The following changes are involved for the flow of one faraday of electricity:
1
Left hand electrode: H2 (g) → H + (a + )1 + e − ...(1)
2
1
Right hand electrode: H + (a + )2 + e − → H2 (g) ...(2)
2
Thus, H + ions are generated at the left hand electrode and consumed at the right hand electrode as the
current follows. Since the solutions are in direct contact with each other, the ions are free to move from
one solution to the other when current flows through the cell. In the present case, evidently, H + ions
move from the solution on the left hand side to that on the right hand side. Since anions move in
direction opposite to that in which cations move, Cl − ions migrate from right to left.
Let t − be the transport number of Cl − ion and t + (= 1 – t − ) that of H + ion in HCl. Then, for one faraday
of electricity passing through, t − faraday will be carried by Cl − ions and t+ faraday by H + ions.
According to Faraday’s second law, t − equivalent of Cl − ions will be transferred from the solution of
activity a 2 to the solution of activity a 1. This may be represented as
t_ Cl − (a_ )2 → t_ Cl − (a_ )1 …(3)
At the same time, t + equivalent of H + ions will be transferred from the solution of activity a 1 to that of
activity a 2 which may be represented as
t +H+ (a + )1 → t +H + (a + )2 …(4)
The net result for the flow of one faraday of electricity is summed up below:
Left Hand Electrode:
(a) Gain of 1 gram equivalent of H + ions by process ...(5)
(b) +
Loss of t + gram equivalent of H ions by process ...(6)
∴ Net gain of H + ions =(1-t + ) gram equivalent
= t_ gram equivalent
At the same time, net loss of Cl − ions = t_ gram equivalent by process (3)
Right Hand Electrode:
(a) Loss of 1 gram equivalent of H + ions by process (2)
(b) Gain of t + gram equivalent of H + ions by process (4)
P-187

∴ Net loss of H + ions=(1 − t + ) gram equivalent


=t − gram equivalent
At the same time, net loss of Cl − ions = t_ gram equivalent by process (3)
Thus, for every one faraday of electricity, there is net transfer of t_ gram equivalent of H + ions and t_
gram equivalent of Cl − ions from right to left or from the solution in which activity of HCl is a 2 to that in
which activity of HCl is a 1.
These changes are represented as
t_ H+ (a + )2 → t_ H + (a + )1

t_ Cl − (a − )2 → t −Cl − (a − )1

⇒ t – [H+ (a + ) 2 + Cl – (a – ) 2 ] → t – [ H+ (a + )1 + Cl – (a – )1 ]

The E.M.F. of concentration cell with transference, therefore, is given by


RT (a + )2 RT (a − )2
E w.t. = t_ ln + t− ln
F (a + )1 F (a − )1
Here, E w.t. is E.M.F. of cell with transference. If (a ± ) 1 and (a ± )2 are the mean ionic activities of the two
hydrochloric acid solutions, it follows by the definitions given earlier
(a ± )12 = (a + )1(a − )1and(a ± )22 = (a + )2 (a − )2
Therefore, it can be said that,
RT (a ± )22  RT  (a ± )2
E w.t.=t_ ln =(2t – )  ln
F (a ± )12  F  (a ± )1

Knowing that the activity of a uni-univalent electrolyte is given by a=(a ± )2 , it can be said that,
RT a 2
E w.t. = t_ ln
F a1
where a 2 and a 1, as already stated, are activities of HCl solutions of the right and left hand electrode,
respectively. The equation is used for calculating activity of an electrolyte at a given concentration from
the experimental value of E w.t. It is important to note that one of the solutions used should be of known
activity.
II. Concentration Cells Without Transference: In order to understand the setting up of such a cell,
consider a simple electrochemical cell such as
Pt| H2 (g),HCl(a1)|AgCl(s), Ag
1atm
Let the activity of H + ions in the solution be (a + )1 and that of Cl − ions be (a − )1. Since reduction takes
place on the right hand electrode and oxidation on the left hand electrode, the two half-cell reactions
will be as follows:
AgCl(s)+e − → Cl − (a − ) 1+Ag(s) ...(1)
Oxidation half-cell reaction:
1
H2 (g) → H +(a + )1+e − …(2)
2
P-188

The net reaction taking place in the cell for one faraday of electricity is obtained by adding equation (1)
and (2) given above. Thus,
1
H2 (g) + AgCl(s) → H +(a + )1 + Cl − (a_ )1 + Ag(s) …(3)
2
Now consider the same cell with the difference that the activity of HCl solution is now a 2 .
Pt,H2 (g),HCl(a 2 )|AgCl(s), Ag
The net cell reaction for one faraday of electricity will now be as follows:
1
H2 (g) + AgCl(s) → H + (a + )2 + Cl − (a − )2 + Ag(s) …(4)
2
Finally, consider the situation when the two cells are connected to each other in such a way that they
send current in opposite direction. Thus,
Pt, H2 (g),HCl(a1), AgCl(s), Ag|Ag, AgCl(s),HCl(a 2 ), H2 (g),Pt
1 atm 1 atm
The overall reaction of the combined cell for the passage of one faraday of electricity will, evidently, be
obtained by subtracting equation (4) from equation (3),
H + (a + ) 2 +Cl − (a − ) 2 → H + (a + )1 + Cl − (a − )1 ...(5)
Thus, for the flow of one faraday of electricity, the overall reaction is the transfer of 1 mole of each of
H + and Cl − ions or one mole of HCl, from a solution of activity a 2 to that of activity a1.
Hence, E.M.F. of such a cell would be given by
RT (a + )2 RT (a − )2
E w.o.t = ln + ln …(6)
F (a + )1 F (a − )1

RT (a ± )22
= ln …(7)
F (a ± )12
where (a ± )1 and (a ± )2 are the mean ionic activities of the electrolyte in the two solutions and the
subscript w.o.t. stands for ‘without transference’. It can be modified if AgCl / Ag / Cl – – the widdle
electrode is kept common to both and then the net reaction is, H+ (a 2 ) → H+ (a1) So,
RT (a ± ) 2
E w.o.t = ln …(8)
F (a ± )
1

where (a ± )1 and (a ± )2 are the mean activities of hydrochloric acid in the two solutions. It will be
observed that cell reaction does not involve transfer of electrolyte from one solution to the other
directly. It takes place indirectly. The cell is, therefore, a cell without transference. If the middle
electrode, viz., Ag, AgCl(s), is withdrawn, the two solutions of HCl will be in direct contact with each
other. The cell will then become a concentration cell with transference. It shows that cells which are
reversible with respect to cations (H + ions in the present case) can be converted into a concentration
cell without transference by interposing another electrode in between which is reversible with respect to
anions (Cl − ions in the present case).
Another example of a concentration cell without transference, in which the electrodes are reversible
with respect to a cation, is given below:
Zn, ZnSO 4 (a 1), PbSO 4 (s), Pb, PbSO 4 (s), ZnSO 4 (a 2 ),Zn
Here PbSO 4 is interposed in between two Zn electrodes thereby removing the direct contact and
making it a concentration cell without transference.
P-189

Concentration cells without transference in which end electrodes are reversible with respect to an
anion and in which an electrode reversible with respect to a cation is inserted in-between, are also
known. One such cell is a represented below:
Ag, AgCl(s), HCl(a1), H2 (g),HCl(a2 ), AgCl(s),Ag
The end electrodes are reversible with respect to Cl − ions while the intermediate electrode is reversible
with respect to H + ions.

6.4.4 Liquid Junction Potential (L.J.P.)


Equation derived for the E.M.F. of a concentration cell with transference includes the potential at the junction
of the two solutions of HCl as well. Equation for E.M.F. derived earlier when the two solutions are not in direct
contact with each other does not include the liquid junction potential. These equations may be reproduced as
RT (a ± )2
E w.t. = 2t − ln
F (a ± )1
RT (a ± )2
E w.o.t = ln
F (a ± )1
Hence, liquid junction potential E l is given by
RT (a ± )2
E LJP = E w.t.− E w.o.t =(2t − − 1) ln
F (a ± )1
RT (a ± )2
= |t_+(1 − t + ) − 1| ln
F (a ± )1
RT (a ± )2
= (t_ − t + ) ln
F (a ± )1
It is evident from the above equation is that the sign as well as the magnitude of L.J.P. depends on the
transference numbers of the anion and cation. If the transference numbers of the anion and cation of an
electrolyte are the same or nearly the same, so that, t − = t + then L.J.P. = 0 or negligibly small. If the
transference number of cation is greater than that of anion, so that, t + > t − then L.J.P. will be negative and if
reverse is the case, then L.J.P. will be positive and will add to the E.M.F. of the cell. Potassium chloride and
ammonium nitrate are amongst the electrolytes in which transference numbers of cations and anions are
nearly the same. The solutions of these electrolytes are, therefore, frequently used as salt bridge because the
liquid junction potential is then reduced to a minimum.

Example 13: Calculate the LJP at 298 K between two solutions of HCl having mean ionic
activities of 0.01 and 10 −3 respectively. The transport number of H + ion in given conditions is
t + = 0.83.
Solution: t + = 0.83; t − = 1 − 0.83 = 0.17
10−3
So, E LJP = (017
. − 0.83)(0.0591) × log 10 = 0.0039 Volts
10−2
Example 14: The E.M.F. oF the concentration cell with transference, viz., Pt; H 2 (1atm), HCl
(a ± = 0.009048); HCl (a ± = 0.01751); H 2 (1 atm); Pt is 0.02802 V at 25 ° C. The E.M.F. of the
corresponding cell without transference is 0.01696 V. Calculate the liquid junction potential,
E1 and the transport number of H + ion.
Solution: E LJP = 0.02802 − 0.01696 = 0.01106 V
P-190

0.009048
and E LJP = (t − − t + ) × 0.0591 × log
0.01751
0.01106 = (t − − t + ) (0.016946)
Solving t+ = 0.826

6.5 Potentiometric Titration


It is known that the potential of an electrode depends upon the concentration of the ion to which it is
reversible in accordance with Nernst equation. In a titration, there is change in ionic concentration which can
be followed by measuring the potential of a suitable electrode. The potentiometric titration is a type of
titration which involve the measurement of electrode potentials with the addition of the titrant. There are a
number of advantages of potentiometric titrations over the ordinary titrations involving the use of indicators.
Potentiometric titrations like conductometric titrations can be carried out in coloured solutions as indications
cannot be used in such cases. Apart from this, in ordinary titrations, one needs a prior information about the
relative strengths of acids and bases before a proper indicator is selected. However, no such information is
required in the case of potentiometric titrations.
Different types: The potentiometric titrations fall into three categories:
1. Acid-base titrations
2. Oxidation-reduction (redox) titrations
3. Precipitation titrations

6.5.1 Acid-Base Titration


A solution of HCl is titrated against NaOH. Any electrode whose potential
depends upon H + ion concentration like hydrogen electrode or
quinhydrone electrode is placed in the HCl solution. It is connected to a
E,Volts

reference electrode to form a Galvanic cell. If hydrogen electrode is used


as the H + indicating electrode and a saturated calomel electrode is used
as the reference electrode, then the Galvanic cell may be represented as

Pt| H 2 (1 atm)| H + (c = unknown) || KCl sat. soln | Hg 2 Cl 2 (s)| Hg


Volume of NaOH
(A)
The E.M.F. of the cell is measured potentiometrically. It is given by
Fig .11: Plot of E Versus Volume
E ≡ E=E R -E L = E calomel −E hydrogen of NaOH.

=0.2422−0.0591 log H + =0.2422 + 0.0591pH

It is presumed that 100 ml of 0.1 M HCl is titrated against the titrant which is 1 M NaOH.
The concentration of the titrant is usually 5 to 10 times higher than that of the solution to be titrated so that the
volume change is as small as possible. As the titration proceeds, the H+ ion concentration goes on
decreasing, so that pH of the solution goes on increasing, hence the E.M.F. of the cell goes on increasing. It is
evident that the E.M.F. of the cell will increase by 0.0591 volt for every ten-fold decrease in the concentration
P-191

of H + ions. Assuming, for the sake of simplicity of calculations, that there is no change in volume during the
titration, it is evident that the addition of first 9 ml of NaOH solution will give a change of 0.0591 volt.
However the addition of next 0.90 ml will produce the same change and the addition of next 0.09 ml will also
produce the same change and so on. Thus, the E.M.F. of the cell-changes slowly at first but more and more
rapidly as the end point approaches. After the end point, further addition of NaOH produces very little
change in the H + ion concentration and hence there is very little change in the E.M.F. of the cell. A plot of E
against the volume of NaOH added is shown in figure (fig. 11)

As can be seen, the E.M.F. of the cell initially rises gradually and thereafter
more rapidly near the equivalence point. Beyond the equivalence point,
the E.M.F. of the cell again increases slightly on adding more of NaOH.
Once the titration curve is obtained, the analyst has to determine, by
inspection, where the curve is steepest. A vertical line is drawn through ∆E
∆V
the sharp portion of the curve and the intersection of this line with the
volume axis is determined. There occurs some uncertainty in this
procedure and this will be reflected in the ultimate volume reading. For a
Volume of NaOH
reaction that goes to completion, the titration curve is so sharp near the (B)
equivalent point that the uncertainty is small. However, for a reaction Fig. 12: Plot of ∆E/∆V Versus
with small equilibrium constant, the precision with which the equivalence Volume of NaOH
point may be determined becomes poorer. (Fig.12) shows a plot of the
slope of the titration curve, that is, the change in E.M.F. with change in volume (∆E/∆V) against the volume of
the titrant.

The resulting curve rises to a maximum at the equivalent point. The


volume at the equivalence point is determined by drawing a vertical line +
from the peak to the volume axis. Of course, there is some uncertainty in
locating exactly the peak. The more complete the reaction, the sharper ∆2E0
the peak and hence the more accurate is the location of the equivalence ∆V2
point. (Fig.13) shows a plot of the change in the slope of a titration curve -
(∆ 2 E/∆V 2 ) against the volume of the titrant.
Volume of NaOH
At the point where the slope ∆ E/∆V is a maximum, the derivative of the
Fig. 13: Plot of ∆2E ⁄ ∆2V2 Versus
slope is zero. The equivalent point where is located by drawing a vertical Volume of NaOH

line from the point at which ∆2 E/∆V2 is zero on the volume axis. The steeper the portion of the curve joining
the maximum and minimum value of ∆2E/∆V2 , the more is the completion of the titration reaction.

6.5.2 Redox Titrations


Like acid-base titrations, the redox titrations are also carried out potentiometrically. In this case, the electrode
reversible with respect to H + ions is replaced by an inert metal) such as platinum wire, dipped in a solution
containing both the oxidized and the reduced forms of the same species. The electrode acts as an
oxidation-reduction electrode. Let us consider the redox reaction.
P-192

Fe2 + (aq) + Ce4 + (aq) → Fe3 + (aq)+ Ce3 + (aq)

Here, Fe 2+ ions is oxidized by Ce 4+ ions and the titration is carried out potentiometrically. Before adding Ce
4+
ions, the solution contains only Fe 2+ ions. On adding a small amount of Ce 4+ ions to the solution, a small
amount of Fe 2+ ions is oxidized to Fe 3+ ions. With the presence of both the Fe 2+ and Fe 3+ ions, the electrode
behaves as an oxidation reduction electrode whose potential is given by
RT [Fe3 + ]
E el = E °el + ln
F Fe2 +
[Fe3 + ]
Here, the electrode potential is controlled by the concentration ratio . For instance, if the ratio is equal
[Fe2 + ]
to 0.01, the electrode potential would be given by
E el = E °el + 0.059log(0.01) = E ° − 0.1182

[Fe3 + ]
With further addition of Ce 4+ ions, the ratio changes, thereby changing the value of E el . At the
[Fe2 + ]
equivalent point, [Fe2 + ]=[Ce4 + ] and [Fe3 + ]=[Ce3 + ] , so that the electrode potential at the equivalent point,
E eq. is given by
[Fe3 + ] [Ce4 + ]
E eq = E1° + 0.059log = E2° + 0.0591 log
2+
[Fe ] [Ce3 + ]
The above equations is rewritten as
[Fe3 + ]
E eq = E1° + 0.0591log
[Fe2 + ]

[Ce4 + ]
or E eq = E2° + 0.0591log
[Ce3 + ]
Adding and simplifying, keeping in mind that at the equivalence point,
[Fe2 + ] = [Ce4 + ]and [Fe3 + ] = [Ce3 + ]
(E1° + E2° )
E eq =
2
The numerical values of E1° and E2° are 0.77 V and 1.61 V, respectively. Beyond the equivalent point,
[Ce4 + ]
[Fe3 + ] = 0 . So, the electrode potential thereafter is controlled only by the ratio. For potentiometric
[Ce3 + ]
measurements, the oxidation-reduction electrode for example. Fe 3+ , Fe 2+ is combined with a reference
electrode like a saturated calomel electrode, to form a Galvanic cell which is represented as
Hg, Hg 2Cl2 (s), KCl(sat. soln.) || Fe2 + , Fe3 +

in accordance with the positions of these electrodes in the electrochemical series. Before the equivalent
point, the E.M.F. of the cell would be given by
[Fe3+ ]
E = E R − E L = E °el + 0.0591log − E calomel
[Fe2 + ]
P-193

[Fe3 + ]
=0.77+0.0591 log − 0.2422
[Fe2 + ]
And after the equivalence point, the E.M.F. of the cell is given by
[Ce4 + ]
E=1.61 +0.0591 log − 0. 2422
[Ce3 + ]
At the equivalent point, the E.M.F. of the cell is given by
077
. + 1.61
E= − 0.2422
2
=0.9478 V
The E.M.F. of the cell is measured potentiometrically at each stage of titration and the E.M.F. data thus
obtained are processed for the equivalence point. The redox titration curve is exactly similar to the acid-base
titration curve shown earlier in figure 11.

6.5.2.1 Precipitation Titrations


A solution of silver nitrate is standarised by titrating against a standard solution of potassium chloride. The
silver electrode is used as the indicator electrode in this case. The potential of the half-cell, Ag + , Ag, is
measured by connecting it with the calomel electrode, as usual. The solution is titrated against a standard
solution of potassium chloride. The strength of standard solution of potassium chloride is about 10 times
higher. As the reaction proceeds, the Ag + ions is gradually precipitated as silver chloride.
Ag + +NO 3− + K+ + Cl – → AgCl ↓ +K + + NO 3−
The concentration of Ag + ions goes on decreasing and hence the potential of the Ag + , Ag electrode goes on
decreasing continuously on the progressive addition of KCl solution.
E el = E °el + 0.0591log[Ag + ]

The electrode potential will change slowly at first but more and more rapidly as the end point approaches. At
the end point, the concentration of Ag + ions is very small. The concentration of Ag + ions is low only due to
Sparingly Soluble nature of AgCl. Hence, the change in electrode potential is maximum at the end point. If
the addition of KCl is continued further, the concentration of Ag + ions remains almost unaffected except for
very small decrease on account of the common ion effect. The addition of KCl beyond the end point,
therefore, causes only a miniscule change in the electrode potential.

6.5.3 Application of E.M.F. Measurements


The E.M.F. measurements find a number of useful application. Some of these are given below:

6.5.3.1 Determination of Activity Coefficients of Electrolytes


The E.M.F. measurement helps to determine the activity coefficient. It is considered to find out the activity
coefficient of hydrochloric acid. A cell is considered without liquid junction containing HCl. The two
electrodes are so chosen that one is reversible with respect to the cation of the electrolyte (in this case, the H +
ion) and the other is reversible with respect to the anion (the Cl − ion). Evidently, the first electrode is the
hydrogen electrode and the other can be the silver-silver chloride electrode. Accordingly, the cell
arrangement is as follows:
Pt| H 2 (g)(1 atm) | HCl(m) AgCl(s), Ag(s)
P-194

Here m is the molality of HCl solution.


The cell reaction is
1
H2 (g) + AgCl(s) → Ag(s) + H + (m) + Cl − (m)
2
The E.M.F. of the cell at 25°C is given by
a Ag a + a −
E=E ° −0.0591log H Cl
(a H2 )1 /2 a AgCl
= E ° − 0.0591 loga a
H+ Cl −
As already discussed,
a a = (a ± )2 = (γ ± , m)2 = γ 2± m2
H+ Cl −
Here γ ± and m are the mean ionic activity coefficient and molality of HCl, respectively. Substituting,
E = E ° − 0.0591 logγ2± m 2
=E ° − 0.1182 log γ ± − 0.1182 log m
Rearranging
E+0.1182 log m=E ° − 0.1182 logγ ±
The two unknowns E° and γ ± can be determined by measuring the E.M.F. of the cell over various
concentrations of HCl, including dilute concentrations. At infinite dilution, m = 0 and γ ± = 1 so that
log γ ± =0. Thus, a plot of E + 0.1182 log m versus m, extrapolated to m = 0 gives E° as the y-intercept.
Knowing the value of E°, the mean ionic activity coefficient γ ± of HCl at any other concentration can be
determined from the E.M.F. data of the cell at that concentration.
Alternatively, the Debye-Huckel limiting law (DHLL) can be used, so that
logγ ± =-0.509 |z +z −|µ 1/ 2 (µ is ionic strength)
Substituting for the log γ ± term,
E+0.1182 log m = E ° + 0.0602µ1 /2 c (Q|Z+Z –|= 1for HCl)
1/ 2
Thus, a plot of E + 0.1182 log m versus µ will give a straight line at low concentrations where the limiting
law is valid. The extrapolation of this plot to µ 1/ 2 = 0 gives E° as the y-intercept of the line.
In practice, however, an extension of the DHLL is needed to make a satisfactory linear extrapolation. For a
uni-univalent electrolyte in dilute aqueous solution at 25°C,
log γ ± =−0.509m 1/ 2 + bm

here b is an empirical constant, 'm' is molality of


solution in mol / Kg.
Substituting this equation and rearranging the (E°in Volts)
terms, we obtain
E+0.1182 log m−0.602m 1/ 2
=E '=E ° −(0.1182b)m
This equation shows that the left hand side
which has been designated as E’, when plotted
against m, will give a straight line whose
Fig.14: Determination of Ag|AgCl potencial
intercept at m = 0, is E°. This graphical relation by extrapolation
is shown below (Fig. 14)
P-195

Figure shows the plot of E’ versus m. The extrapolation to m = 0 gives E° = 0.2224 V. This is the E.M.F. of
the cell with HCl at unit activity. It follows that this is the standard electrode potential of the Ag, AgCl electrode
since the other electrode is the standard hydrogen electrode for which the standard electrode potential is zero,
by definition. One can use similar cells for determining the standard reduction potential of different
electrodes.
Example 15: Calculate the mean ionic activity coefficient of 0.1 molal HCl at 25°C from the
fact that the E.M.F. of the cell.
Pt;H 2(g)|HCl(m)|AgCl(s),Ag(s)
is 0.3524 V and the extrapolated value of E° is 0.2224 V.
Solution: E + using 0.1182 logm = E °−0.1182 log γ ±

so, 0.3524 + 0.1182 log 0.1=0.2224 − 0.1182 log γ ±


Solving γ ± = 0794
.
Example 16. Consider the following cell :
Ag(s), Ag + (a=0.001m)|| Ag + (c=0.1m, a=x), Ag(s)
Its E.M.F. at 25 ° C is +0.1110 V, (a) Write cell reaction and (b) calculate activity coefficient of
the Ag + ion in 0.1 m solution.
Solution: Cell reaction involved is
Ag + (c=0.1 m, a=x) → Ag + (a=0.001m)
And, E= −0.0591 log 10−3 / x
Solving x=0.076
0.076
so, γ= = 076
.
01
.
Example17.The E.M.F. of the cell
Cd(s), CdCl2 (m=0.02)| AgCl(s), Ag(s)
is found to be 0.780 V at 25°C. Using the standard potentials, viz.,E ° = −0.403V and
Cd 2 + ,Cd
E° =+0.222V , calculate the mean ionic activity coefficient of CdCl 2 at this temperature.
AgCl, Cl −

Solution: For the given cell,


E ° cell = 0.222 − (−0.403) = 0.625V
For dissociation of CdCl2 ,
a cdcl2 = (a ± )3 = (γ + m)(2 γ − m)2

= 4(γ ± )3 m 3
For the given cell,
0.0591
E=E °− log(a + )(a − )2
2
=0.78=0.625 − 0.0295 log 4(γ ± )3 m3
Solving γ ± = 0. 558
P-196

6.6 Ionic Equilibrium


The concepts of chemical equilibrium provide an excellent theoretical framework which applies best to
equilibria among ionic species present in aqueous solution or in short ionic equilibria. The objective of this
chapter is to impart thorough understanding of such equilibria involving ions. As a matter of fact, these
equilibria deserve a special attention because of their great industrial, analytical and biochemical importance.
The characteristics of ionic equilibrium are same as that of chemical equilibrium.

6.6.1 Basic Terms in Ionic Equilibrium


Concept of Conjugate Acid-base Pair: Any acidic substance after loss of H + changes to another
substance called conjugate base of that acid. Similarly a basic substance after gain of H + changes to another
substance called conjugate acid of that base.
Table 3: Some of the conjugate acid base pairs
Acid Conjugate base Base Conjugate acid

HCl Cl H 2O H3O +
HNO 3 NO3− OH − H 2O

CH 3 COOH CH 3 COO ROH ROH2+

H 2 SO 4 HSO4− NH2− NH3

HSO4− SO2
4
− NH 3 NH4+

It must be noted that the strengths of conjugate acid and base pairs are inversely related to each other. So that
the stronger the acid weaker is the conjugate base and vice versa. In fact, product of dissociation constants of
acid and its conjugate base is equal to K w at that temperature.
∴ Ka × K b = K w
For example, in case of HF,
(Ka ) HF × (K b) = 10 –14 at 25°C
F–

Strength of Acids and Bases and pH Scale: Acidic strength means the tendency of an acid to give H +
ions in water and basic strength means the tendency of a base to give OH − ions in water. So greater is the
tendency to give H + or OH − ions, more will be the acidic or basic strength of acid or base.
Many properties of aqueous solutions depend on the concentration of H + ions of the solutions and therefore
there is a need to express these concentrations in simpler terms. For this purpose, Sorenson introduced the
concept of pH.
pH = – log a (where a is the activity of H + ions).
H+ H+

Activity of H + ions is the concentration of H + ions in a solution. For concentrated solutions, the activity would
be much less than the concentration itself. As activity = activity co-efficient × concentration. For dilute
solution, activity = concentration, because activity co-efficient is equal to 1.
Therefore, the earlier given expression of pH can be modified for dilute solutions as,
pH = – log [H + ]. This assumption can only be made when the solution is very much dilute, [H + ] ≤ 1M. For
higher concentration of H + ions, one needs to calculate the activity experimentally and then calculate the pH.
P-197

Thus, for dilute solutions,


pH = − log [H + ]
pOH = − log [OH − ]
Auto Ionization of Water: Pure water acts as a very weak electrolyte so, it gets ionised very weakly to give
H + and
OH − ions. So, dissociation of water can be shown as,
H 2O l H + + OH − .
Now, applying law of chemical equilibrium,
[H + ] × [OH − ]
K=
[H2O]
here K is equilibrium constant called dissociation constant of water. As water is weakly dissociated, its
concentration remains practically constant through out dissociation so,
K × [H 2 O] = [H + ] × [OH − ]
K w = [H + ] × [OH − ] where K w is ionic product of water.
Taking log on both sides of the above equation,
Since the concentration of pure water is constant, so when it is multiplied by K, product will also be a constant
called ionic product of water (K w ).The value of K w is approximately 10−14 at 298 K.
Therefore, [H + ] × [OH − ] = 10−14 at 298 K ⇒ pH + pOH = 14
Example18: At -50°C, the self-ionization constant (ion product) of NH 3 is K NH 3 =
[NH 4+ ][NH 2− ] = 10−30 . How many amide ions are present per mm 3 of pure liquid ammonia?
(a) 600 ions/mm 3 (b) 6 × 10 6 ions/mm 3
(c) 6 × 10 4 ions/mm 3 (d) 60 ions/mm 3
Solution: 2NH 3 (l) + l NH4+ + NH−2
1-x x x
(Neglecting x in comparison to 1 since K NH3 is very small).
K NH3 = x 2 = 10 −30

thus, x = 10 −15 M = [NH −2 ]


10−15
[NH2− ]= 10 −15 moles/lit = moles/mm 3
6
10
23
[NH −2 ]= 10 −21
× 6 × 10 ions/mm 3 = 600 ions/mm 3
Ans: (a)
pH Scale: Based on K w value of water at 298 K, Sorenson introduced a scale called pH scale or pOH scale
to measure the acidic or basic strength of acids or bases. The range of the pH scale is from 0 to 14. As
ionization of water gives equal amount of H + and OH − , one can say
[H + ] × [H + ] = 10−14 or [OH − ] × [OH − ] = 10−14
[H + ] = 10 −7 or [OH − ] = 10−7
P-198

pH = − log 10 −7 or pOH = − log


pH = 7 or pOH = 7
For pure water, at all temperatures
[H + ]2 = [OH − ]2 = K w
[H + ] = [OH − ] = K w
pH = pOH = − log K w

For pure water, [H ] is always equal to [OH − ] so it is called neutral. So, neutral pH of any solution at 298 K
+

taking water as solvent is 7. For acidic or basic solutions the [H + ] and [OH − ] are not equal but their product is
always equal to K w at that temperature. The variation of K w with temperature is expressed as
K w2 ∆H  1 1
log = −
K w 2.303 R  T1 T2 
1

here K w 2 and K w 1 are the ionic product of water at temperature T 2 and T 1 respectively, ∆H is the enthalpy
change for ionization of H 2 O into H + and OH − ions and R is the gas constant.
Common ion effect: If to an ionic equilibrium, we add any ion which appears in the equilibrium reaction,
the equilibrium will shift in a direction opposite to that in which that ion appears. So, For example, the
suppression of dissociation of weak acids and bases in the presence of common ions is known as common ion
effect. The common ion can be furnished by either a weak or strong acid or base respectively. As an example,
the dissociation of acetic acid is considered,
CH3COOH ® CH3COO – + H+
If H+ or CH3COO – is added from outside, the equilibrium will tend to shift in the backward direction by
following Le Chatelier's principle. So, the dissociation of acetic acid is suppressed - it is known as common ion
effect.

6.6.2 Determination of pH of Acids


Dissociation of Strong Acid: A strong acid is that which ionises completely. One example is considered. If
the concentration of HCl is c, so the [H + ] coming from acid is also c. Therefore,
pH = − log c
Now let us calculate the pH of 10 M HCl, 10−4 M HCl, 10−5 M HCl, 10−6 M HCl and 10−7 M HCl. For 10−3
−3

M HCl, the dissociation takes place like below,


HCl → H+ + Cl −
10 −3 M 10 −3 M 10−3 M
As H + is 10−3 M,so, pH = −log 10−3 = 3. Similarly,
for ; 10−4 M HCl, pH = −log 10−4 = 4
for 10−5 M HCl, pH = −log 10−5 = 5
for 10−6 M HCl ; pH = −log 10 −6 = 6
and for 10−7 M HCl ; pH = −log10−7 = 7
But the pH of 10−7 M HCl cannot be 7, because it directly implies that the solution would be neutral. Thus, for
10−7 M HCl, [H + ] from water is also added and then the pH is calculated.
P-199

Method – I: We already know the concentration of H + ions from pure water. It is 10−7 M. So this
contribution is added to the H + ion concentration from HCl to get the total H + ion concentration.
∴ (H + ) T = (H + ) HCl + (H + ) water = 10−7 M + 10−7 M = 2 × 10−7 M
∴ ` pH = – log (H + ) T = – log 2 × 10 −7 = 6.6989
Though everything looks fine, but there is a mistake. It can be seen that 10−7 M H + from HCl causes
common ion effect to the dissociation of water. So, H + obtained from water will be less than 10−7 M.
Therefore, the method given above is an approximation.
Method – II: This method yields the value of H + from water in the presence of a common ion. It is very clear
that water would dissociate less in the presence of HCl. Let the amount of water dissociating be x' in the
presence of 10−7 M HCl.
∴ H2O l H+ + OH −
At equilibrium: x' +10 −7 x'
+ − −14
[H ] [OH ] = 10
⇒ (x' +10 −7) (x')= 10−14
Calculating for x' , we get x' = 0.618 × 10 −7 M. One can clearly see the common ion effect in action.
Water,which normally dissociates to give 10 −7 M H + ion, has now yielded lesser H + ions equal to 0.618 ×
10−7 M.
Therefore, [H + ] T = 10−7 + 0.618 × 10−7 = 1.618 × 10 −7 M

∴ pH= – log ( 1.618 × 10−7 ) = 6.7910


Now, let us answer the question as to why one needs to take the H + ions of water into account for calculating
the pH of 10−7 M HCl while it was never considered for calculating the pH of 10 −6 M, 10−5 M, 10−4 M, 10−3 M,
10−2 M and 10−1M HCl.
It can be seen that the H + ions from water has decreased due to the common ion effect. Greater the
concentration of the common ion added, greater will be the effect. Therefore, for concentrations greater than
or equal to 10−6 M, the H + from water will be even less than 0.618 × 10−7 M. As such, the contribution is so
small in comparison to the [H + ] from HCl, that the contribution is ignored. So, finally it is concluded that
H + ions of water needs to be considered only if concentration of acid is <10 −6 M.
The same logic applies for bases also.
Example19: While calculating the pH of 10 −7 M HCl, we took into account the common ion
effect of HCl on water. Why did we not take the common ion effect of water on HCl?
Solution: Let the K w of water be
Therefore, in pure water,
x 2 = 10−10 ; x = 10−5
Let us calculate the [H + ] from water in the presence of 10−7 M HCl.
(x ' + 10 −7 ) x ' = 10 −10
∴ x ' = 0.995 × 10−5 M
The percentage decrease in the concentration of H + of water is
10−5 − 0.995 × 10−5
× 100 = 0.5%
10−5
P-200

When the K w of water was 10−14 the concentration of pure water was 10−7 M while that in the presence of
10−7 M HCl was 0.618 × 10 −7 M.
10−7 − 0.618 × 10−7
The percentage decrease was = × 100 = 38.2%
10−7
This implies that greater the value of K, lesser will be ability to experience the common ion effect. This is why
the common ion effect of H 2 O was not taken on HCl.
Dissociation of Weak Acid: A weak acid is that which dissociate partially in water and soon comes in
equilibrium with its ions. One of the examples is dissociation of acetic acid as shown below,
CH 3 COOH l CH 3 COO − + H +
[CH3COO − ] × [H + ]
Ka =
[CH3COOH]
The equilibrium constant is called Ka or acid dissociation constant. Let the [H + ] coming from water is x and
the degree of dissociation of weak acid is α.
CH 3 COOH l CH 3 COO − + H +
Initial concentration c 0 0
Conc. at equilibrium (c - cα) (cα) (cα + x)
α × (cα + x)
Such that, Ka =
1− α
Moreover, there is one additional equilibrium of dissociation of water,
H 2O l H + + OH −
(cα + x) x
For this dissociation,
K w = (c α + x ) × (x)
Now if we know c, Ka and K w we can calculate α and x.
pH = − log [H + ] total = −log (c α + x)
If [H + ] water is very small, x is neglected. Quantitatively, C α must be more than or equal to 10−6 , so that x can
be neglected. When x is negligible, Then
cα 2
Ka =
1−α
If α << 1 then 1- 0 ≈ 1. In general, if α is less than or equal to 0.1, it is neglected
K
Ka = Cα2 or α= a
c
 Ka 
pH = −log c α = −log c ×  = − log Ka c
 c 
Example20: Calculate the pH of 10 −6 M CH 3 COOH. Ka (CH 3 COOH) = 1.8 × 10 −5.
Solution: We solve this problem by two methods.
Method 1: Since [H + ] due to a weak acid is given by
[H + ] = Ka c

= 1.8 × 10−5 × 10−6 = 4.24 × 10−6


pH = 5.37
P-201

Though we seemed to have solved it correctly, there is an error that we have made. This error can be
highlighted by considering the pH of 10 −6 M HCl. We can see that the pH would be 6. Now, we know that
lesser pH implies higher concentration of H + ions. So how can a weak acid having the same concentration as
a strong acid disociate to give the same concentration of H + ions.
The mistake has occurred in the assumption that α is very much small compared to 1. In fact by making this
assumption the α that we calculate is,
Ka 1.8 × 10−5
α= = = 4.24
c 10−6
This value is not possible as α cannot be more than 1, ever. Now, we will calculate the pH without this
assumption, by following a separate method
c α2 10−6 α 2
Method 2: Ka = , 1.8 × 10 −5 =
1− α 1−α
Solving for α, α = 0.95
∴ [H + ] = c α = 0.95 × 10 −6 = 9.5 × 10 −7

Of course, we need to take the [H + ] from water also as [H + ]acid < 10 −6 M.


∴ (x' + 9.5 × 10 −7 ) x' = 10 −14 (Where x' is the amount to which water dissociates in the presence of 10 −6 M
CH 3 COOH).
x' = 1.04 × 10 −8
∴ [H ] T = 9.5 × 10 −7 + 1.04 × 10 −8
+

pH = 6.01
Note: This illustration shows a need to know as to when is the assumption that ∝ is very small compared to 1,
valid. This is done in the following manner.
Ka
First of all, calculate the value of α from the expression α = .
c
If this value of α comes out to be less than or equal to 0.1, then the assumption is valid. If the value of α >0.1,
then the assumption is not valid and one has to Solve α using the quadratic expression.
Example 21: The ionization constant of HCO 2H is 1.8 × 10 −4 . What is the percent ionization of
a 0.001 M solution ?
(a) 66% (b) 42% (c) 34% (d) 58%
Solution: HCO 2 H l H +
+ HCO −2
0.001-x x x
α is > 0.1 which means x is not ignorable. check for α
x2 Ka
∴ Ka = 1.8 × 10 −4 = α=
0.001 − x c

1.8 × 10−4
Solving by quadratic, α= = 0.42
0.001
x = 3.4 × 10 −4
ionized HCO 2H 3.4 × 10−4
∴ % ionization = × 100 = × 100 = 34%
total HCO 2H 0.001
Ans: (c)
P-202

Dibasic or Diprotic and Polyprotic Weak Acids: Dibasic acid is that acid which has the tendency to
give two H + ions per molecule of that acid like H 2 SO 4 , H 2 S, H 3 PO 3 etc. Let us calculate pH of a diprotic
acid H 2 A whose dissociation constants for first and second proton are Ka1 and Ka 2 respectively. Let us also
assume that concentration of the disprotic acid is c.
H 2A l H+ + HA −
c (1 - α 1) c α1 c α1
HA −
l H +
+ A 2−
c α 1(1 - α 2 ) cα 1α 2 cα 1α 2
[H + ]total × [HA − ]total (c α1 + c α1α2 ) × [c α1(1 − α2 )] (α1 + α1α2 ) × [cα1(1 − α2 )]
Ka 1 = = =
[H2 A] c × (1 − α1) (1 − α1)
..
[H + ]total × [A 2 − ]total (c α1 + c α1α2 ) × (c α1α2 ) (c α1 + c α1α2 ) × α2
Ka 2 = = =
− c α1 × (1 − α2 ) (1 − α2 )
[HA ]total
From above equations we can calculate α 1 and α 2
pH = – log[H + ] total = − log (c α 1 +c α 1 α 2 )
In this case, we have neglected [H + ] coming from water.
Determination of pH of Bases: The pH of bases can also be calculated as we have done for acids. In case
of a base, instead of H + we take OH − ions, instead of Ka we take K b and instead of pH we calculate pOH.
Then pH can be calculated as pH = 14 – pOH or pH = pK w − pOH.
As an example, let us consider dissociation of BOH ( a general base).
BOH ® B+ + OH –
Initial concent C – –
Conc. dissociated αC – –
Conc, at eqlm. C(1–α ) αC αC
α
α C
So, Kb =
1– α
And OH – = α C
Example 22: The pH of a solution obtained by dissolving 5 × 10 – 4 moles of Ca(OH)2 (strong
electrolyte) to 100 ml solution at 298 K will be:
(a) 11 (b) 12 (c) 9.8 (d) 2
−4
5 × 10 × 1000
Solution: [Ca(OH) 2 ] = = 5 × 10 −3 mole L−1
100
[OH − ] = 2 × 5 × 10 −3 = 10 −2 mole L −1
10 −14
[H + ] = = 10 −12
10 −2
pH = −log 10 −12 = 12
Ans: (b)
P-203

6.6.3 Hydrolysis of Ions in Solution


The reaction of an ion with water is called hydrolysis of ion. Either H 3 O + or OH − is produced as a result of
hydrolysis.
Hydrolysis of Anions: Anions are functioning as a base when react with water and hydrolyzed as follows:
H 2O + B − l HB + OH −
The extent of hydrolysis of a given anion depends on its basic strength. The anions which are stronger base
than OH − will show complete hydrolysis in aqueous medium. For example,
H 2 O + H − →H 2 + OH −
The anions which are weaker base than OH − hydrolyse to a limited extent in aqueous medium.
For example,
CN − + H 2 O l HCN + OH −
Kw
K = Kh =
Ka
Here, K w = Ionic product of water; Ka = Dissociation constant of the conjugate acid.
Hydrolysis of Cations: The cations which are stronger acid than H 3 O + will show complete hydrolysis.
Example is PH4+ ion.
PH4+ + H 2 O → H 3 O + + PH 3
The cations which are weaker acid than H 3 O + ion , show hydrolysis to a limited extent. For example, NH4+ .
NH4+ + 2H2O l NH4OH +H3O +
Kw
K = Kh =
Kb
Here, K w = Ionic product of water; K b = Dissociation constant of the conjugate base

6.6.4 Hydrolysis of Salts and their pH


When a salt is dissolved in water, it first undergoes dissociation to give constituent ions. Now if these ions
correspond to the ions of weak acid or base, they react with H + or OH − of water to give that acid or base at
equilibrium. This process is called hydrolysis. The salts which undergo hydrolysis are,
1. Salts of weak acid with strong base
2. Salts of strong acid with weak base
3. Salts of weak acid with weak base
Salts of Weak Acid with Strong Base: Let us consider the salt of acetic acid with sodium hydroxide - CH
3 COONa. The aq. solution of salt of weak acid with strong base is alkaline or basic. Let h be the degree of
hydrolysis of the anion of weak acid and K h is its hydrolysis constant.
hydrolysis
CH 3 COO − + H 2 O l CH 3 COOH + OH −
Concentration at equilibrium c(1- h) ch ch

[CH3COOH] × [OH ] [CH3COOH][OH − ] × [H + ]
Kh = =

[CH3COO ] [CH3COO − ] × [H + ]
P-204

Kw ch × ch ch2
Kh = = = = ch2 (if h ≤ 0.1)
Ka c × (1 − h) 1 − h
Kw
h= (Here, h is degree of hydroysis)
Ka c

From above expression, we can calculate h. Now, the expression of pH will be desired as follows [OH − ] = ch
pOH = − log (ch) or pH = pK w – pOH
Kw K
[H + ] = = w

[OH ] ch
K  K w × Ka
pH = −log w  = − log if h ≤ 0.1
 ch  c
1 1 1
pH = pK w + pKa + log(c)
2 2 2
where c is the concentration of the anion undergoing hydrolysis.
Example 23: Calculate the pH at the equivalence point of the titration between 0.1M
CH 3 COOH ( 25 ml) with 0.05 M NaOH. Ka (CH 3 COOH) = 1.8 × 10 −5.
Solution: At equivalence point, the equivalents of NaOH added would be equal to the equivalents of
CH 3 CO 2 H and the same equivalents of CH 3 CO 2 Na is formed. This salt would undergo hydrolysis. As the
salt is made up of weak acid and strong base,
K w Ka
∴ [H + ] =
c
First of all we would calculate the concentration of the salt, CH 3 COONa. For reaching equivalence point,
N 1V 1 = N 2 V 2
∴ 0.1 × 25 = 0.05 × V 2
V 2 = 50 ml
01
. × 25 01
.
Therefore, [CH 3 COONa] = =
75 3
10 −14 × 1.8 × 10 −5
∴ [H + ] = = 2.32 × 10 −9
0.1 / 3
pH = – log 2.32 × 10 −9 = 8.63
Salts of Strong Acid and Weak Base: Let us consider the salt ammonium chloride. Here NH4+ ion will be
hydrolysed as it is ion of weak base. The aq. solution of salt of strong acid with weak base is acidic. Let the
degree of hydrolysis be h.
NH 4 Cl → NH4+ + Cl −
NH4+ + H 2 O l NH 4 OH + H +
Intial concentration : C
Conc. at equilibrium: c (1 − h) ch ch
+
[NH4OH] × [H ] [NH4OH] × [H + ] × [OH − ]
Kh = =
[NH4+ ] [NH4+ ][OH − ]
P-205

K w ch × ch h2C
Kh = = =
K b c(1 − h) 1 – h
if h ≤ 0.1, then,
K w ch × ch ch2
Kh = = =
K b c(1 − h) 1 − h
 Kw 
pH = −log[H + ] = − log (c × h) = −log c ×
 K 
 b × c

Kw × c
pH = −log
Kb
1 1 1
pH = pK w − pK b − log(c)
2 2 2
where c is the concentration of conjugate acid of weak base or the ion undergoing hydrolysis.
Salt of a Weak Acid with Weak Base: Let the salt is CH 3 COONH 4 . It is consisted of the weak acid
CH 3 COOH and the weak base NH 4 OH. The ions get hydrolysed according to the reaction,
CH 3 COO − + NH4+ + H 2 O l NH 4 OH + CH 3 COOH
Initial Concentration c c 0 0
Concentration At equi. c(1–h) c(1–h) ch ch
[NH4OH][CH3COOH]
Kh=
[NH4+ ][CH3COO − ]
Multiplying and dividing by H + and OH − and rearranging,
[NH4OH][CH3COOH][H + ][OH − ] [H+ ][OH -]
Kh = =
[NH4+ ][CH3COO − ][H + ][OH − ] [CH3COO − ][H+ ] [NH+ -
4 ][OH ]
×
[CH3COOH] [NH4OH]
Kw
∴ Kh =
Ka K b
Substituting the concentration terms,
Kw ch × ch (ch)2 h2
Kh = = = =
2
Ka K b c(1 − h)c(1 − h) [c(1 − h)] (1 − h)2
h Kw
∴ =
1− h Ka K b
There is an important issue that needs clarification before we move on further. In this case, we can see that
both the ions - cation and anion get hydrolysed to produce a weak acid and a weak base. Hence, one cannot
predict whether the solution is acidic, basic or neutral. It has been approximately considered that the degree
of hydrolysis of both the ions is the same. Now, to calculate the pH of this solution, the hydrolysis reaction is
re-considered,
4 + H 2O l
CH 3 COO − + NH+ NH 4 OH + CH 3 COOH,
CH 3 COOH is present in the solution. This implies that the equilibrium between CH 3 COOH, CH 3 COO − and
H + would exist.
∴ CH 3 COOH l CH 3 COO − + H +
P-206

In fact, the equilibrium between NH 4 OH, NH4+ and OH − also exists.


Now, the pH of the solution is calculated as,
CH 3 COOH l CH 3 COO − + H +
ch c(1–h)
[CH3COO -][H+ ] c(1- h)(H+ )
Ka = =
[CH3COOH] ch
 h 
∴ [H + ] = Ka × 
1- h 
Kw  h 
Substituting , for  as derived earlier,
Ka K b 1 − h 
 h  Kw K w Ka
[H + ] = Ka ×  = Ka × =
1- h  Ka K b Kb
K w × Ka 1 1 1
pH = - log[H + ] = - log = pK w + pKa − pK b
Kb 2 2 2
If Ka > K b the solution is acidic, if Ka < K b the solution will be basic and if Ka = K b the solution will become
neutral.
Example 24: The hydrolysis constant for ZnCl 2 will be:
Kw K 2w K 2w Kb
(a) K h = (b) K h = (c) Kh = (d) K h =
Kb Kb K 2b K 2w
where K b is effective dissociation constant of base Zn ++
Solution: Zn ++ + 2H 2 O l Zn(OH) 2 + 2H +
[Zn(OH)2 ][H+ ]2
∴ Kh =
[Zn++ ]
Zn(OH) 2 l Zn ++ + 2OH −
[Zn++ ][OH -]2
∴ Kb =
[Zn(OH)2 ]
K w = [H + ] [OH − ]
K2w
∴ = Kh
Kb
Ans: (b)
Example 25: Consider the reaction A − + H 3 O + l HA + H 2O. The Ka value for acid HA is 1.0
−6
× 10 . What is the value of K for this reaction.
(a) 1.0 × 10 6 (b) 1.0 × 10 −8 (c) 1.0 × 10 8 (d) 1.0 × 10 −6
Solution: The reaction is the reverse of the ionization reaction of HA, hence the equilibrium constant is the
reciprocal of Ka .
[HA] 1 1
K= = = =1.0 × 10 6
− + K −6
[A ][H ] a 1.0 × 10
Ans:(a)
P-207

Example 26: The degree of dissociation of anilinium acetate is:


(a) independent of initial concentration
(b) directly proportional to initial concentration
(c) inversely proportional to initial concentration
(d) inversely proportional to square root of initial concentration
3 CH3CO 2 l
C 6H5NH+ --
Solution: C 6H5NH+ --
3 + CH3CO 2

c (1–a) cα cα
2

K eq = assuming ∝ to be small, (1 − α) ≈ 1
(1 − α)
K eq
∴ α=
c
Ans: (d)

6.6.5 Buffer Solution


A buffer solution is a solution which resists any change in its pH value when either a small addition of acid or
base is added or it is diluted. Although the pH of buffer solutions changes on these additions but the change is
marginal or very less. Buffer solution is primarily of two types:
1. Acidic buffer solution
2. Basic buffer solution
Apart from these two types, there is one more type of buffer solution known as neutral buffer solution. As
such, salts of weak acid and weak base is considered as a neutral buffer solution. Presently, the discussion will
be limited to acidic and basic buffer solutions only.
Acidic Buffer solution: The acidic buffer solution is defined as a mixture of a weak acid and its conjugate
base or its salt with a strong base. Let us consider an acid buffer containing acetic acid and sodium acetate. By
the way, acetate ion CH 3 COO − is a conjugate base of acetic acid. The two dissociations are shown below,
CH3COONa is undergoing complete dissociation andCH3COON undergoing a partial dissociation.
CH 3 COONa → CH 3 COO − + Na +
CH 3 COOH l CH 3 COO − + H +
Expression of pH of acidic buffer solution: If the above two dissociations are considered, it can be
observed that due to the presence of CH 3 COO − ions of salt, the dissociation of CH 3 COOH is suppressed, as
suggested by the concept of common ion effect. So one can understand that [CH 3 COO − ] at equilibrium is
entirely due to the [CH 3 COO − ] coming from the salt-sodium acetate. Applying the Ostwald law to the
dissociation of acetic acid,
CH 3 COOH l CH 3 COO − + H +
[CH3COO -]total × [H+ ]total
Ka =
[CH3COOH]
[CH3COOH]
[H + ] = Ka ×
[CH3COO − ]total
P-208

[CH3COOH]
pH = − log H + = − logKa − log
[CH3COO --- ]
[CH3COO --]
pH = pKa + log
[CH3COOH]
[Salt]
pH = pKa + log
[Acid]
It is known as Herderson's equation.
Buffer Action: Buffer action refers to removal of externally added H + or OH − by internal mechanism, so
that the constancy of pH is maintained. Buffer action also means the nullifying of the impact of adding water
to the pH of a buffer solution. For illustration, an acidic buffer solution is considered. The acidic buffer
solution is consisted of acetic acid and sodium acetate. The following ions are obtained at the equilibrium,
CH 3 COOH l CH 3 COO − + H +
CH 3 COONa → CH 3 COO − + Na +
Now, it is to be observed that what is the impact of adding H + or OH − ions from outside.
1. Add H + : If one adds any acid (H + ) to this buffer, it will be consumed by CH 3 COO − as shown below,
CH 3 COO − + H + l CH 3 COOH
And, the addition of H + does not change the pH of solution much.
2. Add OH − : If OH − is added, this time, it will be consumed by CH 3 COOH, as shown below,

CH 3 COOH + OH → CH 3 COO − + H 2 O
So, there is again the ph of the solution remains unchanged.
3. Add water: If one dilutes the solution by adding water, the concentration of H+ will decrease. This
will shift the equilibrium in forward direction to give more H + and so compensate the decrease in H +
concentration, so that pH remains constant.
Basic Buffer Solution: It is a solution of a weak base and its salt with a strong acid. Let us consider a
buffer solution containing NH 4 OH and NH 4 Cl. Following dissociation will take places,
NH 4 Cl → NH4+ + Cl −
NH 4 OH l NH +4 + OH −
[Salt]
The pH is given by pOH = pK b + log , then the pH of the solution is calculated using the formula, pH
[Base]
= pK w − pOH. The buffer action may be explained as in the case of acid buffer.
To consider the buffer action in case of basic buffer solution, a simple example is taken. In the buffer solution
of NH4OH + NH4Cl, first H+ ions are added.
1. Add H+ ion: The two components are dissociating as suggested above. So,
NH 4 OH l NH +4 + OH −
NH 4 Cl → NH4+ + Cl −
When H+ ion is added it is consumed by undissocided NH 4 OH as,
NH 4 OH + H + l NH +4 + H 2 O
Such that the net impact on ph of solution is almost nothing.
P-209

2. Add OH – ion: The added OH – ion is consumed by NH +4 , so that,


NH +4 + OH − l NH 4 OH
Again, the impact on pH is negligible.
3. Add water: As water is added, the overall volume increases so that the concentration of OH −
decreases. To maintain a constant concentration the equilibrium (NH 4 OH l NH +4 + OH − ) moves
rightwards and the ph is maintained.
Neutral Buffer Solution: A solution of a salt of weak acid with weak base (CH 3 COONH 4 ) also acts as a
buffer. The dissociation is shown below,
CH 3 COONH 4→ CH 3 COO − + NH+
4

The buffer action is very simple.When an acid is added to it, H + combines with CH 3 COO − to give
CH 3 COOH.
CH 3 COO − + H + l CH 3 COOH
Similarly, when a base is added, the OH combine with NH4+ to give NH 4 OH.

NH4+ + OH − l NH 4 OH
So, the net impact on pH of solution is nothing. The pH of this type of solution has been discussed already in
the case of hydrolysis of such salts, comprised of weak acids and weak bases.
Buffer Capacity: Although it is difficult to give an exact limit up to which a buffer can be used, it is generally
accepted that a buffer solution can be used for practical purposes when its [Salt]/[Acid]. Concentration ratio
lies within the range of 10 to 0.1.Therefore, it can be concluded that a particular weak acid or base can be
employed for making useful buffer solution of pH or pOH lying with the range of p Ka ± 1 or p k b ± 1. For
example, acetic acid (p K = 4.75 at 25°C) and sodium acetate mixture can be used for preparing buffer
a
solutions whose pH values are roughly in the range of 3.75 to 5.75. Outside this range, the buffer capacity of
acetic acid and sodium acetate solution is too small to be used for any practical purpose.
Buffer capacity is defined as the number of equivalents of acid or base required by one litre of a buffer
solution for changing its pH by one unit.
Number of moles of H+or OH - added per litre
Buffer capacity =
Change in pH
Thus higher the amount of acid or base used to produce a definite change in pH of a buffer solution, higher
will be its buffer capacity.
Example 27: CH 3 COOH (50 ml, 0.1M) is titrated against 0.1M NaOH solution. Calculate the
pH at the addition of 0 ml, 10 ml, 20 ml, 25 ml, 40 ml, 50ml and 60 ml of NaOH. Ka of
CH 3 COOH is 2 × 10 −5.
Solution: (i) When 0 ml of NaOH is added, the pH is due to acetic acid,
∴ [H + ] = Ka × C = 2 × 10−5 × 01
. = 2 × 10−6
1
= – log 2 × 10−6 = −
[log 2 − 6] = 3 − 015
. = 2.85
2
(ii) When 10 ml of NaOH is added, it reacts with CH 3 COOH to produce salt and water. The solution is
then a buffer.
[Salt]
∴ pH = pKa + log
[Acid]
P-210

10 × 01
. 1
= 4.699 + log = 4.699 + log = 4.0969
50 × 01
. − 20 × 01
. 4
(iii) When 20 ml of NaOH is added.
20 × 01
. 2
pH = pKa + log = 4.699 + log = 4.5229
50 × 01
. − 20 × 01
. 3
(iv) When 25 ml of NaOH is added,
25 × 01
.
pH = 4.699 + log = 4.699
50 × 01
. − 25 × 01
.
(v) When 40 ml of NaOH is added,
40 × 01
.
pH = 4.699 + log = 4.699 + log 4 = 5.3011
50 × 01
. − 40 × 01
.
(vi) When 50 ml of NaOH is added,
Here, if we use the buffer equation, pH would be = ∞
But we can’t use the buffer equation as there is no acid. Therefore, we will use the hydrolysis equation.
K w Ka
∴ [H + ] =
c
01
.
c= [ Q Total Volume is 100 ml and millimoles of salt is 50 × 0.1]
2
10−14 × 2 × 10−5 × 2
[H + ] =
01
.
pH = 8.699
(vii) When 60 ml of NaOH is added, the excess OH − ion from NaOH would suppress the hydrolysis of
CH 3 COO − ion. So we can ignore the contribution of OH − ion from the hydrolysis of CH 3 COO − ion.
01
. × 10
∴ [OH − ] = [ 10 ml of OH − ion is in excess]
110
1
=
110
pOH = 2.0414
∴ pH = 14 – 2.0414= 11.9586
Example 28: A weak base (BOH) with K b = 10 −5 is titrated with a strong acid HCl. at 3 / 4th
of the equivalence point, pH of the solution is
(a) 5 + log 3 (b) 5 - log 3 (c) 14 - 5 - log 3 (d) 8.523
Solution: Let the initial equivalent of BOH are x.
BOH + HCl → BCl + H 2 O
x 3/4x 0 0
3 x 3 3
x- x= 0 x x
4 4 4 4
[Salt] 3x × 4
pOH = pK b + log = 5 + log
[Base] 4×x
pH = 14 - 5 - log3 = 8.523
Ans: (c) & (d)
P-211

6.6.6 Solubility and Solubility Product


Solubility is defined as the maximum amount of a salt which can be dissolved in a given amount of solvent at
a particular temperature. It is generally expressed in the amount of solute in grams that can be dissolved in
100 g of solvent. Other units of solubility are moles of solute that can be dissolved in one litre solution and
grams of solute that can be dissolved in one litre of solution. The solutions which contain maximum possible
amount of solute that can be dissolved in it are called saturated solutions. Let us consider solubility of any
general salt A xBy . Let the solubility is s mol per litre.
A x B y (s)l xA y + (aq) + yB x − (aq)

Concentration at equilibrium: xs ys
For saturated solution at equilibrium,
y
[A y + ]x × [Bx − ]y [xs]x × [ys]
K= =
[A xBy ] [A xBy ]

K [A x B y ] =(xs)x × (ys)s

As concentration of solid A x B y remains constant, it can be written that,


Ksp = (x x ) × (y y ) × (s)x + y

where Ksp is called solubility product.


Solubility Product and Precipitation: As discussed above, for a salt like solid A x B y , the solubility
product Ksp is defined as
Ksp = (x x ) × (y y ) × (s)x + y

It is important to note that in the above expression the term on right hand side is ionic product if the solution is
not saturated and it becomes solubility product if the solution is saturated. It must be noted that,
1. If the ionic product is more than Ksp, the solution is called super saturated. The equilibrium will shift in
backward direction and some amount of salt is thrown out of solution and finally an equilibrium is
developed in such a way that the ionic product becomes equal to solubility product. This situation is
known as precipitation.
2. If ionic product is less than Ksp the solution is unsaturated.
3. If ionic product is equal to Ksp the solution is saturated.
Different Cases of Calculating Solubilities
1. Solubility of a salt of strong acid with strong base in pure water
Let us consider a salt CaF 2 . Let the solubility of the salt is s mol per litre.
CaF 2 (s) l Ca 2+ (aq) + 2F − (aq)
s mol/L 2s mol/L
2+ − 2
Ksp = [Ca ] × [F ]

Ksp = s × (2s) 2 =4s 3

Ksp
s =3
4
2. Solubility of a salt of strong acid with strong base in presence of common ion
Let us calculate the solubility of AgCl in 0.1 M AgNO 3 solution
P-212

AgNO 3 →Ag + + NO −3
AgCl(s) l Ag + (aq) + Cl − (aq)
Let the solubility of AgCl under these conditions be s′ mol per litre. So
[Ag + ] AgCI = s′ mol L −1

[Ag + ]AgNO3 = 0.1 mol L−1

[Ag + ] Total = (0.1 + s′) mol L−1


Ksp = [Ag + ] Total × [Cl − ]
Ksp = (0.1 + s′) × (s′)
s' can be calculated if we know Ksp.
Example 29: One litre of a saturated solution of CaCO 3 is evaporated to dryness due to which
7.0 mg of residue is left. The solubility product for CaCO 3 is:
(a) 4.9 × 10 −8 (b) 4.9 × 10 −5 (c) 4.9 × 10 −9 (d) 4.9 × 10 −7
7 × 10−3
Solution: Moles of CaCO 3 in residue = = 7 × 10−5
100
Moles of CaCO 3 in 1 litre solution = 7 × 10 −5
CaCO 3 (s) l Ca 2+ + CO 23 −
7 × 10 −5 7 × 10 −5
Ksp = [Ca 2+ ] × [CO 23 − ]

=7 × 10 −5 × 7 × 10 −5 = 4.9 × 10 −9
Ans: (c)
Example 30: M(OH) x has Ksp 4 × 10 −12 and solubility 10 −4 M. Then the value of x is
(a) 1 (b) 2 (c) 3 (d) –4
Solution: M(OH) x will ionize in the way
M(OH) x l M +x + x OH −
10 −4 x × 10 −4
∴ Ksp = [M + x] [OH − ] x

∴ (10 −4 ) (x × 10 −4 ) x = 4 × 10 −12
By inspection we get this relation will hold good when x = 2
Ans: (b)
Example 31: The precipitate of Ag 2CrO 4 (Ksp = 1.9 × 10 −12) is obtained when equal volumes of
the following are mixed:
+
(a) 10 −4 M Ag + 10 −4 M CrO2−
4 (b) 10 −2 M Ag + + 10 −3 M CrO 2−
4
(c) 10 −5M Ag + + 10 −3 M CrO2−
4 (d) 10 −4 M Ag + + 10 −5 M CrO 24 −
Solution: Precipitation occurs when the ionic product exceeds the Ksp value.
When equal volumes of two solutions are mixed the concentration of each is reduced to half. Therefore,
In first case,
P-213

2
1  1  1
Ionic product, I. P. =  × 10−4   × 10−4  = × 10−12 = 1 . 25 × 10−13
2  2  8
As, I.P. < Ksp
∴ no precipitation occurs.
In second case,
2
1  1  1
I.P. =  × 10−2   × 10−3  = × 10−7 = 1.25 × 10−8
2  2  8
As, I.P. > Ksp
∴ Precipitation occurs.
In third case,
2
1  1  1
I.P. = × 10−5   × 10−3  = × 10−13 = 1 . 25 × 10−14
2  2  8
As, I. P. < Ksp
∴ no precipitation occurs.
In fourth case,
2
1  1  1
I. P. =  × 10−4   × 10−5  = × 10−13
2  2  8

= 1.25 ×10 −14


As, I. P. < Ksp
∴ no precipitation occurs.
Ans: (b)
Selective Precipitation: Selective precipitation of ions from a mixture in the form of salts which are
partially soluble can be done by adding common ions drop by drop. Let us consider selective precipitation of
Cl − and CrO 24 − ions in the form of AgCl and Ag 2 CrO 4 from a mixture having 0.01 M Cl − and 0.02 M CrO 24 − .
We know that precipitation takes place only if the ionic product exceeds solubility product. Let us first
calculate the concentration of Ag + required to make their ionic products equal to solubility products.
Ag + required by Cl − so that AgCl can be precipitated
AgCl(s) l Ag + (aq) + Cl − (aq)
Ksp(AgCl) = [Ag + ] × [Cl − ]
Ksp (AgCI) 10−10
[Ag + ] = = = 10−8
[CI− ] 0.01

Ag + required by CrO 24 − for its precipitation to take place:


Ag 2 CrO 4 (s) l 2 Ag + (aq) + CrO 24 − (aq)
Ksp(Ag 2 CrO 4 )= [Ag + ] 2 × [CrO 2−
4 ]

Ksp(Ag 2CrO 4 ) 10−13


[Ag + ] = = = 2.23 × 10−6
[CrO 24 − ] 0.02
P-214

If one adds a salt having Ag + (like AgNO 3 ), the concentration of Ag + increases and it becomes equal to 10−8 .
Any further addition of Ag + results into increase in ionic product of AgCl than its Ksp value. However, the
ionic product of Ag 2 CrO 4 will still be less than its Ksp. So precipitation of only AgCl takes place as long as [Ag
+
] reaches 2.23 × 10 −6 . Now further addition of Ag + will result into an ionic product greater than its solubility
product. So, any further addition of Ag + causes AgCl and Ag 2 CrO 4 to precipitate simultaneously.

6.7 Different Types of Commercial Cells


6.7.1 Fuel Cells
Fuel cells are Galvanic cells in which chemical energy of fuels is directly converted into electrical energy.
Conventional conversion of chemical energy of fuels into electrical energy is carried out by burning the fuel,
Subsequently using the heat energy to raise steam which is then used for spinning the turbines connected
with electric generators. The efficiency of this process, which cannot exceed that of a reversible carbon
engine, varies from 20 to 40 percent. Fuel cells on the other hand, convert about 75 percent of the available
chemical energy into electrical energy. It can be of different types. Some of those are,
1. Hydrogen Oxygen Fuel Cell: A common type of fuel cell is based on the combustion of hydrogen to
form water:
2H2 (g) + O 2 (g) → 2H2O(1)
This is known as hydrogen oxygen fuel cell. A Water
schematic diagram of the cell is show in figure Anode(-) (+) cathode
below. aqueous electrolyte,
It consists of two electrodes made of porous (NaOH (aq))
graphite impregnated with a catalyst (platinum,
silver or a metal oxide). The electrodes are
placed in aqueous solution of KOH or NaOH. H2 O2
Oxygen and hydrogen are continuously fed into
the cell, as shown, under a pressure of about 50 porous partition
atm. The gases diffuse into the electrode pores
and so does the electrolyte solution. The half-cell
reactions which occur at the electrodes are as
follows: Fig.15: Schematic diagram of H2 – O2 fuel cell

Oxidation Half-cell Reaction: Hydrogen is oxidized to H + ions which are neutralized by the OH − ions of
the electrolyte :
H 2 → 2H + + 2e −
2H + +2OH − → 2H 2 O
The net oxidation half-cell reaction is
H 2 +2OH − → 2H 2 O + 2e −
Reduction Half-cell Reaction: Reduction half-cell reaction involves the reduction of oxygen to OH − ions :
O 2 (g)+2H 2 O+4e − → 4OH −
The net cell reaction is thus given by
P-215

2H 2 (g)+O 2 (g) → 4H 2 O
Summarily, if these things are represented, then it can be said that,
At Anode : H 2 + 2OH − (aq) → 2H 2 O(l) + 2e −
At Cathode : O 2 + 2H 2 O + 4e − → 4OH − (aq)
______________________________
Net reaction : 2H 2 (g)+O 2 (g) → 2H 2 O(I)
______________________________
The E.M.F. of the cell is found to be 1 volt. The water produced vaporizes off since the cell is operated at
temperature above 100°C. This can be condensed and used.
Hydrogen-Oxygen Fuel Cells in Manned Space Flights: The hydrogen-oxygen fuel cells are playing
important role in some of the manned space flights. The electrolyte used in these cells is an ion-exchange
material and not a solution of KOH or NaOH. The ion-exchange material which is used in the form of a
membrane allows easy passage of protons which react with oxygen and electrons to form water. The actual
mechanism is, however, quite complex. Fuel cells are associated with a number of advantages. Their
efficiency is very high. About 75 per cent of chemical energy can be converted into electrical energy. The
individual cells can be stacked and connected in series to generate higher voltages. They are also very light.
However, there are a number of engineering problems which shall have to be solved before fuel cells become
practical sources of electrical energy. Once it is done, fuel cell technology would bring revolution in the area
of energy production.
2. Hydrocarbon-Oxygen Fuel Cells:Fuel cells based on the combustion of hydrocarbons such as CH
4 , C 2 H 6 , C 3 H 8 , etc., in the presence of catalysts, have also been operated. The half-cell reactions with
propane as the fuel are as follows:
Oxidation half-cell Reaction :
C 3 H 8 + 6H 2 O l 3CO 2 + 20 H + + 20e −
Reduction Half-cell Reaction :
O 2 + 2H 2 O + 4e − l 4OH −
Overall cell Reaction :
C3H8 + 5O2 l 3CO2 + 4H2 O
The catalyst in this case is essentially platinum. This makes the fuel cell operation highly expensive.

6.7.2 Primary Cells


A cell which acts as a source of electricity without being previously charged by an electrical current from an
external source is called primary cell. In such a cell electrical energy is obtained at the expense of chemical
energy only as long as the active materials are present. Dry cell is an example of primary cell. Some of the
examples of primary cells are discussed below,
1. Dry Cell: The most commonly used flashlight battery is the dry cell, also called the Leclanche cell,
after its inventor, Georges Leclanche. The anode of a dry cell is a zinc can, which is usually covered
with a steel jacket to shield it from the atmosphere. The cathode is a graphite rod which serves as an
inert electrode. The graphite rod is in the center of the cell and is surrounded by a thick paste containing
MnO 2 and powdered graphite. It is the MnO 2 that is reduced at the cathode. The electrolyte is a moist
paste of a saturated solution of NH 4 Cl, ZnCl 2 and some inert filler. The cell is not really dry, water is an
essential component of the electrolytic paste. At the anode, Zn is oxidized to the +2 state, and at the
cathode MnO 2 is reduced to the +3 state. There are a number of different chemical substances formed
involving the +3 oxidation state of manganese, including Mn 2 O 3 (s), Mn2O 24 − ,and MnO(OH). We will
write the cell reactions showing the formation of Mn 2 O 3 only at the cathode.
P-216

At anode: Zn(s) → Zn 2+ + 2e −
At cathode: 2MnO 2 (s) + 2NH +4 + 2e − → Mn 2 O 3 (s) + 2NH 3 + H 2 O
_________________________________________________________
Net reaction: Zn(s) + 2MnO 2 (s) + 2NH +4 → Mn 2 O 3 (s) + Zn2 + + 2NH 3 + H 2 O
_______________________________________________________________
The voltage of a dry cell battery is 1.5 V. The dry cell is not rechargeable, as the Zn 2+ ions formed at the
anode migrate through the electrolytic paste and combine with NH 3 produced at the cathode to form
the complex ion, Zn(NH 3 ) 2+4 .
One of the problems with the dry cell is that the electrolytic paste is acidic, since NH 4 Cl is an acidic salt.
Thus there is a direct reaction between Zn and NH +4 that slowly eats away the zinc can.
Zn(s) +2NH4+ → Zn 2+ + H 2 (g) + 2NH 3
So, flashlight battery that has never been used but has been sitting on the shelf for more than a year
may split and leak as the Zn metal is converted to Zn2 + ions by above reaction.
An improved form of the dry cell is the alkaline dry cell, in which the NH 4 Cl is replaced by KOH. It is
more expensive than the acid form, but it lasts longer because there is no corrosion of the Zn by NH +4
ions.
2. Corrosion: Corrosion is basically an electrochemical phenomenon. The rusting of iron, the tarnishing
of silver, development of a green coating on copper and bronze are some of the examples of corrosion.
Corrosion causes enormous damage to building, bridges, ships and to many other articles made of
iron.
In corrosion, a metal is oxidized by loss of electrons to oxygen and forms metal oxide. Corrosion of
iron, which is commonly known as rusting , occurs in presence of water and oxygen (air). Although the
chemistry of corrosion is complex but it is understood that at one spot of an iron object oxidation
occurs and that spot behave as an anode.
Anode: Fe(s) → Fe 2+ (aq) + 2e − ; (E ° 2+ = −0.44 V)
Fe / Fe
Electrons released at anode move through the metal and go to another spot on the metal and reduce the
oxygen in presence of H + ions. H + is believed to be available from H 2 CO 3 formed due to dissolution of CO 2
in water. This spot works as a cathode:
Cathode: O 2 (g) + 4H + (aq) + 4e − → 2H 2 O(l); (E° = 1.23 V)
Here Fe 2+ ions move through water on the surface of the iron object. If water present is saline, it will help
more in carrying the current in the miniature cell thus formed and will enhance corrosion. The overall
reaction of the miniature cell is the sum of the cathode and anode reactions as follows:
2Fe(s) + O 2 (g) + 4H + (aq) → 2Fe 2+ (aq) + 2H 2 O(l); (E °cell = 1.67 V)
The Fe 2+ ions are further oxidized by atmospheric
Air
oxygen to Fe 3+ (as Fe 2 O 3 ) and comes out as rust in the Rust O2(g)
form of a hydrated iron (III) oxide expressed as Fe 2 O 3 , (Fe2O3)
xH 2 O. Fe2+ water
4Fe 2+ (aq)+O 2 (g)+4H 2 O(l) → 2Fe 2 O 3 (s)+8H + cathode anode
e–
H + ions produced in the above reaction help further in
rusting. Impurities present in iron also enhance rusting by O2+4H++4e– 2H2O Fe Fe2++2e–
setting a number of miniature cells. Very pure iron does iron
not rust quickly.
Factors which promote corrosion: There are a
number of factors which promote corrosion. Some of the Fig. 16: Corrosion
prominent factors are,
P-217

1. Presence of impurities: Presence of impurities in metals generally enhances the chances of


corrosion. Pure metals do not corrode, so, very interestingly, a pure sample of iron does not corrode.
2. Reactivity of the metal: it is being observed that more active metals are readily corroded.
3. Presence of electrolytes: Electrolytes also increase the rate of corrosion. For example, iron rusts
faster in saline water which contains a number of electrolytes than in pure water.
4. Strains in metals: Corrosion takes place rapidly at bends, scratches, nicks and cuts in the metal.
5. Presence of air and moisture: Most commonly, air and moisture accelerate the rate of corrosion.
Presence of gases like SO 2 and CO 2 in air catalyse the process of corrosion because these gases
facilitate the formation of acids like sulphuric acid or carbonic acid. Iron when placed in vacuum does
not rust.
Prevention of corrosion: Prevention of corrosion is very important because enormous damage occurs to
buildings, bridges, ships and all other objects made of metals especially iron. Crores of rupees are spent every
year on replacing the rusted structures. Prevention of corrosion not only saves money but also helps in
preventing accidents such as bridge collapse etc.Some of the methods and processes generally employed for
prevention of corrosion are briefly explained below :
1. Barrier Protection: The metal surface is not allowed to come in contact with moisture, oxygen and
carbon dioxide. This can be achieved by the following methods:
(i) The iron surface is coated with non-corroding metals such as nickel, chromium, aluminium, etc.
by electroplating or tin, zinc, etc. by dipping the iron article in the molten metal. This again shuts
out the supply of oxygen and water to iron surface. The iron surface is coated with phosphate or
other chemicals which give a tough adherent insoluble film which does not allow air and
moisture to come in contact with iron surface.
(ii) The metal surface is coated with paint which keeps it out of contact with air, moisture etc. till the
paint layer develops cracks.
(iii) Normally, by applying film of oil and grease on the surface of the iron tools and machinery, the
rusting of iron is prevented. It is because it keeps the iron surface away from the contact with
moisture, oxygen and carbon dioxide.
2. Sacrificial Protection: Sacrifical protection means covering the iron surface with a layer of metal
which is more active or electropositive than iron and thus prevents the iron from losing electrons. The
more active metal loses electrons in preference to iron and convertes itself into ionic state. With the
passage of time, the more active metal gets consumed but so long as it is present there, it will protect the
iron from rusting and does not allow even the nearly exposed surface of iron to lose electrons or
undergo oxidation. The metal which is most often used for covering iron with more active metal is zinc
and the process is called galvanization. The layer of zinc on the surface of iron, when comes in contact
with moisture, oxygen and carbon-dioxide in air, a protective invisible thin layer of basic zinc
carbonate. ZnCO 3 .Zn(OH) 2 is formed due to which the galvanized iron sheets lose their luster and also
tends to protect it from further corrosion.
Iron can be coated with copper by electro-deposition from a solution of copper sulphate or with tin by
dipping into molten metal. Now, if the coating is broken, iron is exposed and iron being more active
than both copper and tin, is corrode*. Here, iron corrodes more rapidly than it does in the absence of
tin.At times, zinc, magnesium and aluminium powders mixed with paints provide decorative protective
coatings also.
P-218

3. Using anti-rust solutions:These are alkaline phosphate and alkaline chromate solutions. The
alkalinity prevents availability of hydrogen ions. In addition, phosphate is depositedas an insoluble
protective film of iron phosphate on the iron. These solutions are used in car radiators to prevent
rusting of iron parts of the engine.
4. Electrical Protection: Cathodic Protection: The iron object to be protected from corrosion is
connected to a more active metal either directly or through a wire. The iron object acts as cathode and
the protecting metal acts as anode. The anode is gradually used up due to the oxidation of the metal to
its ions due to loss of electrons. These electrons are transferred through the wire to H + ions present
around the iron object and thus protect it from rusting. As iron object acts as cathode, it is called
cathodic protection. The iron object gets protection from rusting as long as some of the active metal is
present. Metals widely used for protecting iron objects from rusting are magnesium, zinc and
aluminium which are called sacrificial anodes.
Magnesium is oftenly employed in the cathodic protection of iron pipes buried in the moist soil, canals,
storage tanks, etc. Pieces of magnesium are buried along the pipeline and connected to it by the
wire.Magnesium acts as anode whereas iron pipe acts as cathode.

1. Find out the e.m.f. of the following cell at 25°C:


Ag | AgNO 3 (0.01 N) || AgNO 3 (0.1 N)Ag
Assume that 0.1 N and 0.01 N AgNO3 are 85% and 95% dissociated, respectively. [Garhwal 2009]

2. Derive Nernst equation for electrode potential of metal. Calculate the E.M.F. of an amalgam cell at
60°C as under:
Zn(Hg) c 1/ ZnSO 4 solution / Zn(Hg) c 2
where c 1 = 6.08 × 10 −5 gm/atom/litre, c 2 = 21.28 × 10 −5 gm/atom/litre,

F = 96500 Coulomb, R = 8.314 Joules [Avadh 2011]

3. What is e.m.f. of a concentration cell consisting zinc electrodes in which one is immersed in a solution
of 0.05 molar concentration and other in a solution of 0.5 molar concentration of its ion at 25°C. The
two solutions are connected by a salt bridge. [Avadh 2011]

4. Calculate standard E.M.F. of the cell : Zn(s) | Zn(aq)++ || Cu(aq)++ | Cu(s)


Given : E°(Cu 2+ +/Cu) = 0.34 Volt, E°Zn/Zn 2+ + = –0.76 Volt [Garhwal 2010]

5. (i) Describe the construction of standard hydrogen electrode. With an example give its use.
(ii) The standard electrode potentials of zinc and silve electrodes are –0.76 and 0.80 volts
respectively. Formulate the Galvanic cell which will generate e.m.f. Give reasons for this
formulations. [Avadh 2010]

6. Establish the relation between solubility and solubility product. Find out the solubility product at 25°C,
if solubility of PbSO 4 in water at 25°C is 0.048gm/litre. (Molecular weight of PbSO 4 = 303).
7. What is buffer solution? Derive Henderson’s equation for pH value of acidic buffer solution.
P-219

8. Compute the pH of a solution at 25°C which is twice as alkaline as pure water.


9. In what volume ratio 0.02 M HCl and 0.02 M BOH (a weak base) should be mixed to make 200 mL of
buffer of pH = 7? K b for BOH = 8.1 × 10 −8 .

10. Calculate the pH at the equivalence point of the titration between 0.1M CH 3 COOH (25 ml) with 0.05
M NaOH. Ka (CH 3 COOH) = 1.8 × 10 −5 .

11. Calculate the pH of a solution which is obtained by adding 100 ml of 0.1 M HCl and 9.9 ml of 1 M
NaOH.
12. Calculate the percentage of hydrolysis of 0.003 M aqueous solution of NaOCN. (Ka for HOCN = 3.33
× 10 −4 M).
13. The solubility product of PbBr 2 is 8 × 10 −5 . If the salt is 80% dissociated in saturated solution, find the
solubility of the salt. Atomic weights of Pb and Br are 208 and 80 amu respectively.
14. Calculate (i) the degree of hydrolysis of ammonium acetate. The dissociation constant for NH 4 OH is 4
× 10 −5 and that of CH 3 COOH is 1.8 × 10 −5 . (ii) also calculate the pH of the resulting solution.

15. The concentration of HCN and NaCN in a solution is 0.01 M each. Calculate the concentration of
hydrogen and hydroxyl ions if the dissociation constant of HCN is 7.2 × 10 −10 .
16. What volume of 0.10 M sodium formate solution should be added to 50 ml of 0.05 M formic acid to
produce a buffer solution of pH 4.0 ? pK a for formic acid is 3.80.
P-220

Long Answer Type Questions


7. (i) With a suitable example explain working of a
Based on Galvanic Cell
reversible electrode.
1. Calculate the E.M.F. of a zinc-silver cell at 25° C
(ii) Define activity coefficient and give an
when activity of Zn2 + ion is 0.5 and the activity of
appropriate method for its determination.
Ag + ion is 10. Standard reduction potential at 25°
[D.D.U. 2010]
C are:
8. Define single electrode potential. Derive Nernst
Ag + , Ag electrode=+ 0.799 volt, Zn2 + , Zn equation for it and discuss the effect of electrolytic
electrode=− 0.760 volt. [Agra 2008] concentration on electrode rotential. How will you
2. The standard electrode potential of the cell: establish the sign of electrode potential?
[Meerut 2012]
Zn(s)|ZnNO3 (aq)||AgNO3 (aq)|Ag(s)
9. (i) Briefly describe the different types of
is + 1.561 V at 298 K. State the overall cell reaction
reversible electrodes.
and calculate ∆ rG°. [D.D.U. 2008]
(ii) Derive Nernst equation for single electrode
3. (i) What are reversible and irreversible cells?
potential. [Lko. 2010]
Explain each of them with suitable
examples. 10. (i) Describe a calomel reference electrode.
(ii) Derive Nernst equation for measuring Explain its use in the determination of single
E.M.F. of a cell. electrode potential.
[Lko. 2011]
(ii) Write the cell reaction and calculate the
4. Explain reversible and irreversible cells. Discuss the
standard E° of the following cell at 25°C :
application of E.M.F measurements in
determination of pH of a solution. Zn|Zn++ (a = 1)||Cd++ (a = 1)|Cd
The standard E.M.F. of the Daniel cell involving the Given : E ° Zn| Zn++ = 0.763 volt and
cell reaction
E ° Cd|Cd++ = 0.403 volt.
l
[Alld. 2008]
Zn(s) + Cu2 +
(aq)
Zn2 +
(aq)
+ Cu(s)
Based on Concentration Cells
Calculate the equilibrium constant of the cell 11. (i) What are concentration cells? Derive an
reaction at 25°C. [Alld. 2011] expression for e.m.f of a concentration
5. (i) Derive Nernst equation for the cell E.M.F.. cell without transport.
What is the cell E.M.F. at equilibrium? (ii) Corrosion is an electrochemical
(ii) The E.M.F. of the cell corresponding to the phenomenon. Explain. [Avadh 2010]
reaction:
12. Explain the liquid junction potential and
Zn(s) + 2H + → Zn+2 (0.1M) + H2 (1 atm) concentration cell. Derive expressions for ∆G, ∆H
and ∆S in terms of E.M.F. of a cell and temperature
is 0.28 volt at 25° C. Write the half cell
reaction and calculate the pH of the solution coefficient of E.M.F. . [Avadh 2009]
at the hydrogen electrode. 13. Discuss concentration cells by giving suitable
E ° ZnlZn +2
= 0.761 Volt. examples. Calculate e.m.f. of concentration cell
[Alld. 2012]
without transference. [Kanpur 2010]
Based on Electrochenical Series and its Significance
14. (i) Define concentration cell. Derive an
6. Describe the construction and working of the
expression for E.M.F. of a concentration
following electrodes:
cell without transportation.
(i) Hydrogen electrode (ii) Calomel electrode
(ii) What is liquid junction potential? Can it be
(iii) Quinhydrone electrode (iv) Glass electrode.
[Avadh 2009] completely removed? [Purv. 2008]
P-221

15. (i) What is a concentration cell? Obtain the (ii) Calculate the pH value for the solution of
E.M.F. of a concentration cell without 0.001 M hydrochloric acid. [Purv. 2009]
transference. 25. Write short notes on any three of the following:
(ii) What is meant by a reversible cell? Explain (i) Concentration cell
the workability of a reversible cell with a (ii) Buffer solution and mechanism of action
suitable example. [D.D.U. 2011] (iii) Potentiometric titration
16. What is concentration cell? Derive an expression of (iv) Hydrogen electrode
E.M.F. of a concentration cell without transport. (v) Henderson− Hazel equation. [Purv. 2011]
With the help of E.M.F. of conc. cell how to 26. (i) What are concentration cell? Drive an
determine pH value. [Meerut 2008] expression of e.m.f. for concentration cell
without transference.
17. (i) Derive the expresion for E.M.F. of two types
(ii) Calculate the pH of 0.01 M solution of a
of concentration cells and obtain the
expression for the liquid junction potential. monobasic acid (K a =7.2 × 10−10 ). [Agra 2009]
(ii) How will you determine pH of a solution 27. (i) Derive an expression for e.m.f. of a
using hydrogen electrode? [Lko. 2009] concentration cell.
Based on Patentiometric Titration (ii) What is pH? Discuss any one method for its
18. State and explain determination. [D.D.U. 2009]
(i) Over potential and hydrogen overvoltage. 28. (i) Discuss the mechanism of hydrolysis of salts.
(ii) Potentiometric titration. (ii) For the hydrolysis of salts of a weak acid and a
(iii) Electrochemical series. [Avadh 2008] strong base prove that,
K
Kh = w
19. Write notes on any two of the following Ka [D.D.U. 2009]
(i) Potentiometric titration
(ii) Electrochemical series 29. (i) Discuss the factors which influence the
ionization.
(iii) Liquid junction potential [Purv. 2009]
(ii) What do you understand by the E.M.F. of a
20. (i) Write short notes on the following. cell? How can be it measured
(a) Polarisation experimentally?
(b) Over-voltage (iii) Give the relation between electrical energy
(ii) Derive Nernst equation for measuring E.M.F and chemical energy. [Meerut 2012]
of a cell. [Agra 2008] 30. At 25°C, ionisation constant of acetic acid is
Based on Ionic Equilibrium 1. 8 × 10−5 and ionic product of water is 1 × 10−14 ,
21. What is meant by buffer solution’? Explain the Calculate the hydrolysis constant and degree of
buffer action of a mixture of acetic acid and sodium hydrolysis of 0.1 molar solution of sodium acetate
acetate. [Avadh 2008] at 25° C. [Lko. 2008]
22. What do you mean by degree of hydrolysis and salt 31. (i) Derive Henderson equation for pH of (i)
hydrolysis? Derive a relation between hydrolysis acidic and (ii) basic buffer solution.
constant an dissociation constant of base for the (ii) Derive the expression for K h , h and pH for
salt of strong acid and weak base.[Garhwal 2005] hydrolysis of weak acid and strong base.
23. At 25°C dissociation constant for NH 4 OH is 1.8 × [Lko. 2009]
10 −5 . Calculate the degree of dissociation of 0.01 Based on Different Types of Commercial Cells
M solution of ammonium chloride. (At 25°C KW = 32. Write note on any two of the following:
1 × 10 −14 ). [Garhwal 2005] (i) Overpotential and hydrogen overvoltage
24. (i) What is meant by pH of a solution? What (ii) Theories of corrosion
are buffer solutions? Derive (iii) Application of glass electrode in pH
Henderson's equation to calculate the pH of determination. [Purv. 2008]
buffer solutions. 33. Write notes on any two of the following:
P-222

(i) Quinhydrone Electrode 15. Explain reversible electrodes. [Garhwal 2010]


(ii) Corrosion 16. What is standard electrode potential and give its
(iii) Hydrogen over voltage [Meerut 2009] application? [Garhwal 2010]
17. How is pH of a solution determined with the help
Short Answer Type Questions of:
Based on Galvanic Cell (a) hydrogen electrode (b) quinhydrone electrode.
1. What do you understand by E.M.F. of any cell and [Garhwal 2009]
how is it measured? Why a galvanic cell stops 18. Write electrode reactions responsible for evolution
working after some time? [Garhwal 2012] of H 2 and O 2 at the respective electrodes during
electrolysis of aqueous NaCl solution.
2. Write note on electromotive force and its
[Garhwal 2008]
calculations. [Garhwal 2009]
19. Describe the hydrogen and calomel electrode.
3. In the cell Zn(s) | Zn 2+ (aq) || Cu 2+ (aq) | Cu(s) [Garhwal 2007]
with the help of oxidation and reduction show 20. Write a short note on the concept of activity.
which of the electrodes would act as anode and [Garhwal 2006]
which as cathode? [Garhwal 2008] 21. Describe the main uses of electrochemical series of
4. Calculate standard e.m.f. of a cell which involves the elements. [Garhwal 2006]
the following cell reaction: 22. Describe quinhydrone electrode. Derive an
Zn + 2Ag + → Zn2 + + 2Ag [Kanpur 2009] equation for representing its electrode potential.
How pH of any solution is measured with its help?
5. What is e.m.f. of a concentration cell consisting of Also explain the limitation of this electrode.
zinc electrodes in which one is immersed in a [Garhwal 2006]
solution of 0.05 molar concentration and other in a
23. Write a note on the significance of electrochemical
solution of 0.5 molar concentration of its ion at
series. [Garhwal 2011]
25° C ? [Kanpur 2010]
24. Write a short note on glass electrode.
6. What is Nernst equation for e.m.f. of a cell? [Kanpur 2008]
[Kanpur 2011] 25. Predict whether zinc and silver react with 1 N
7. How ∆G is related with e.m.f. of a cell and sulphuric acid to give out hydrogen gas or not.
equilibrium constant (k) of the reaction Given that standard reduction potential of zinc and
respectively? [Kanpur 2008] silver are −0.76 and 0.80 volts respectively.
[Kanpur 2009]
8. Write Nernst equation for electrode potential.
26. What do you understand by reference electrode?
[Purv. 2011]
How will you determine pH of a solution with the
9. What is the function of salt bridge in the galvanic help of standard hydrogen electrode?
cell? How is it constructed? [D.D.U. 2008] [Kanpur 2011]
Based on Electrochemical Series and its Significant 27. Explain different types of reversible electrodes and
10. Write electrode reactions responsible for evolution derive Nernst equation for a reversible cell.
H 2 and O 2 at cathode and anode respectively [Purv. 2010]
during electrolytsis of aqueous H 2 SO 4 . 28. Write down a schematic representation of standard
[Garhwal 2012] hydrogen gas electrode along with the reactions
11. Explain the working of hydrogen electrode. involved there in. [D.D.U. 2011]
[Garhwal 2012]
29. Write short notes on single electrode potential.
12. Draw the sketch of glass electrode and how e.m.f of
[Meerut 2008]
a cell is measured experimentally.[Garhwal 2011]
30. Predict whether Fe and Au react with IM sulfuric
13. Write short notes on the example of a reference
acid to give out a hydrogen gas. [Kanpur 2012]
electrode.
14. Write a note on standard hydrogen electrode. 31. Define standard electrode potential. What is its
[Garhwal 2010] value for hydrogen electrode? [Lko. 2009]
P-223

32. Generally the aqueous solution of salts except few 48. Dissociation amount of a 0.01 N solution of
are not neutral. [Lko. 2010] monobasic acid is 10%. Calculate dissociation
constant of the acid.
33. Copper is not able to displace H + ions from acid [Kanpur 2009]
solutions, why? [Lko. 2010] 49. Explain buffer action of acetic buffer and basic
buffer solutions with examples. [Kanpur 2009]
Based on Concentration Cells
34. What are concentration cells? 50. Determine hydrolysis constant, % hydrolysis,
[Purv. 2010]
hydrogen ion concentration, and pH for 0.1 M
35. What are concentration cells ? Construct a cell
NH4Cl solution. [Kanpur 2009]
without transference to determine the mean
activity and mean activity coefficients of HCl 51. Calculate pH of 10−8 M KOH solution.
having molality of m. [D.D.U. 2008] [Kanpur 2010]

36. What are concentration cells? Classify them 52. How will you calculate hydrolysis constant of a salt
and write expressions for their E.M.F. formed by weak acid and strong base?
[Meerut 2010] [Kanpur 2010]

Based on Patentiometric Titration 53. Write a short notes on:


37. Explain with reasoning the nature of curve that (i) Solubility Product
would be obtained when a mixture of HCl and CH (ii) Overvoltage [Kanpur 2010]
3 COOH is titrated against NaOH. 54. Explain the buffer action of acidic buffer and basic
[Garhwal 2009] buffer solutions with examples. [Kanpur 2011]
Based on Ionic Equilibrium
55. Calculate pH of a solution which has 3 g of acetic
38. Explain the buffer action of a solution of acetic acid acid and 8.2 g of sodium acetate per litre. K a for
and sodium acetate. [Garhwal 2010] acetic acid at 25° is 1. 8 × 10−5 .
[Kanpur 2011]
39. Derive Henderson equation for pH of a buffer
56. Explain mechanism of buffer action. [Purv. 2008]
solution. [Garhwal 2012]
57. What are buffer solutions? [Purv. 2010]
40. Calculate the pH of a solution containing 2 g of
NaOH per litre of water. 58. If pH of a solution is 5.3 what will be its pOH value?
41. Calculate the pH of 0.010 N HCl and 0.010 N [Purv. 2010]
NaOH. [Garhwal 2010] 59. Define salt hydrolysis and derive expression
42. Calculate the pH of the solution containing 4 g of for-hydrolysis constant of a salt of weak acid and
NaOH per litre of water (Na=23). [Kanpur 2008] strong base in terms of K w and K a . [Purv. 2010]

43. Calculate the percentage hydrolysis of a 0.010 N 60. What do you understand by hydrolysis of a salt?
solution of sodium salt of organic acid. The ionic [Purv. 2011]
product of water is 10−14 and dissociation constant 61. What is the nature of CH3COONa solution?
of organic acid is 10−8 . [Kanpur 2008]
[Agra 2000]
62. What is solubility and solujbility product?
44. Calculate the hydrolysis constant of a salt formed
[Agra 2009]
by strong acid and weak base. [Kanpur 2008]
63. What is buffer solution? Explain with a suitable
45. Explain the buffer action of acidic buffer and basic
example. [D.D.U. 2009]
buffer solution with examples. [Kanpur 2008]
64. Write short notes of the following:
46. Write short notes on common ion effect.
[Kanpur 2008] (i) Nernst equation

47. Calculate the pH value of a solution at 25°C in (ii) Buffer action [D.D.U. 2010]
which 0.02 g mole of acetic acid per litre and 0.02 g 65. Why does an aqueaus solution of ammonium
mole of sodium acetate per litre are present. At acetate give a neutral response towards litmus
25°C, the dissociation constant of acetic acid is 1.8 while that of ammounium cynide gives slightly
× 10−5 . [Kanpur 2008] alkaline response? Explain briefly. [D.D.U. 2010]
P-224

66. Write down Henderson's equation and explain all Very Short Answer Type Questions
terms in the equation. [D.D.U. 2011] Based on Galvanic Cell
67. The dissociation constant of CH3COOH at 18°C is 1. In Daniel cell, electricity flows in which
−5 direction?
1.8 × 10 . Calculate pH of N /100 solution of
CH3COOH. 2. What is salt bridge?
[Meerut 2008]
3. Why solid NaCl can not be taken as an electrolyte
68. (i) What is solubility product? Discuss its in salt bridge?
applications. Write Nernst equation for reduction reaction at the
4.
(ii) The solubility of Na3PO4 is found to be following electrode Zn2 + / Zn.
2.0 × 10−4 mol /l at 25° C. Calculate the 5. What is the inter-relation between cell e.m.f. and
solubility product of the salt.[Meerut 2009] ∆G ?
69. Define ionic product of water. How it is Based on Electrochenical Series and its Significance
determined? 6. What is electrochemical series? Give one
[Meerut 2009, 12]
application.
70. What is solubility product? Discuss its applications.
7. What is a calomel electrode?
[Meerut 2010, 11, 12]
8. Explain mean ionic activity.
71. Describe Henderson equation. [Kanpur 2012] Based on Concentration Cells
72. What do you understand by common ion effect? 9. What is electrode concentration cell?
Explain why we pass H2S gas in basic medium for Based on Ionic Equilibrium
analyzing IV group? [Kanpur 2012] 10. Explain hydrolysis of salt?
73. What will be the pH value of a solution obtained by 11. What is a buffer solution?
mixing 5g glacial acetic acid and 15g of sodium 12. The aqueous solution CH 3COONa is acidic or
acetate, making volume of one litre? basic. Explain. [Meerut 2008]
[Kanpur 2012] 13. Give an example of acidic buffer. [Meerut 2010]
74. Calculate the pH of a buffer solution containing 0.1 Based on Different Type of commercial Cells
M acetic acid and 0.15 M sodium acetate. 14. Write the reaction involved in H2 − O2 fuel cell.
Dissociation constant of acetic acid=1. 8 × 10−5 . 15. Explain reaction involved in corrosion.
[Lko. 2011]
Based on Different Types of Commercial Cells Objective Type Questions
75. What is corrosion? Explain theory of rusting of
iron. [Garhwal 2010] Multiple Choice Questions
76. Describe the factors which affect corrosion. 1. For a spontaneous redox reaction the E.M.F.
[Kanpur 2008] should:
77. Explain the factors preventing corrosion. (a) Be zero (b) Have positive sign
[Kanpur 2010] (c) Have negative sign (d) None of these
[Garhwal 2011]
78. Write short notes on any two of the following:
2. A solution contains 10 ml of 0.1 N NaOH and 10
(i) Buffer and its mechanism of function.
ml of 0.05 N H 2 SO 4 , pH of this solution is:
(ii) Glass Electrodes. (a) 1 (b) Less than 7
(iii) Electrochemical aspects of corrosion. (c) Greater than 7 (d) Zero
[D.D.U. 2008] 3. Which one of the following has zero electrode
79. Give some applications of e.m.f. measurements. potential?
[Alld. 2009] (a) Standard calomel electrode
80. Describe the application of E.M.F. measurement (b)Standard hydrogen electrode
for the determination of solubility product and (c) Silver-silver chloride electrode
dissociation constant of an acid. [Alld. 2010] (d) Glass electrode [Garhwal 2006]
P-225

4. Which of the following is a redox reaction? solution. Which other pair of substances from the
2+ 3+ − following may have a similar property?
(a) Fe →Fe +e
2+ (a) HCl and NaCl (b) NaOH and NaNO 3
(b) 2 Fe +Cl2 →2Fe3 +2Cl −
(c) KOH and KCl (d) NH 4 OH and NH 4 Cl
(c) 2Cl2 +2e−→2Cl −
13. One litre of a buffer solution containing 0.01 M NH
(d) Sn2 + → Sn4 + + 2e − [Garhwal 2010] 4 Cl and 0.1 M NH 4 OH having pK b of 5 has pH of :

5. The hydrogen ion concentration in weak acid of (a) 9 (b) 10 (c) 4 (d) 6
dissociation constant K a and concentration c is 14. A precipitate is formed when :
nearly equal to: (a) The solution becomes saturated
Ka c (b) The ionic product is less than the solubility
(a) (b)
c Ka product

(c) K ac (d) K ac (c) The ionic product is nearly equal to the


solubility product
6. At 90°C, pure water has [H 3 O + ] = 10 −6 mol litre
(d) The ionic product exceeds the solubility
−1
. The value of Kw at 90°C is: product.
(a) 10 −6 (b) 10 −8 15. Solubility product of A 2 B is 4 × 10 −9 (mol/lit) 3 . Its
(c) 10 −12 (d) 10 −14 solubility is:
7. The pH of a solution is 5.0. To this solution (a) 10 −3 M (b) 4 1/ 3 × 10 −3 M
−4
sufficient acid is added to decrease the pH to 2.0. (c) 10 M (d) 2 × 10 −5 M
The increase in hydrogen ion concentration is: 16. The precipitate of CaF 2 (K sp = 1.7 × 10 −10 ) is
(a) 100 times (b) 1000 times
obtained when equal volumes of the following are
(c) 2.5 times (d) 10 times mixed:
8. The compound whose 0.1 M solution is basic is: (a) 10 −4 M Ca 2+ + 10 −4 M F −
(a) Ammonium acetate (b) Ammonium chloride (b) 10 −2 M Ca 2+ + 10 −3 MF −
(c) Ammonium sulphate (d) Sodium acetate (c) 10 −5 M Ca 2+ + 10 −3 M F − \
9. About buffer solution which is correct? (d) 10 −3
M Ca 2+
+ 10 −5
MF −
(a) It contains a weak acid and its conjugate base 17. 100 ml of 0.02 M benzoic acid (pK a = 4.2) is
(b) It contains a weak base and its conjugate acid titrated using 0.02 M NaOH. pH after 50 ml and
(c) It shows little change in pH on adding small 100 ml of NaOH have been added are:
amount of acid or base. (a) 3.50, 7 (b) 4.2, 7 (c) 4.2, 8.1 (d) 4.2, 8.25
(d) None of the above. 18. Which of the following reactions is correct ?
1
10. An acidic buffer solution can be prepared by (a) pH + =0 (b) pH = log[H + ]
mixing solution of: [H + ]
1
(a) Sodium acetate and acetic acid (c) log pH = [H + ] (d) pH = log
(b) Ammonia and ammonium hydroxide [H + ]
[Garhwal 2012]
(c) Sulphuric acid and sodium sulphate
19. An acidic buffer solution can be prepared by
(d) Sodium chloride and sodium hydroxide.
mixing the solution of :
11. A certain buffer solution contains equal
(a) Sodium acetate and acetic acid
concentration of X − and HX. The K b for X − is 10
(b) Ammonium chloride and ammonium
−10
. The PH of the buffer is: hydroxide
(a) 4 (b) 7 (c) 10 (d) 14 (c) Sulphuric acid and sodium hydroxide
12. A mixture of weak acid (say acetic acid) and its salt (d) Sodium chloride and sodium hydroxide
with a strong base (say sodium acetate) is a buffer [Garhwal 2012]
P-226

20. Buffer solution can be obtained by mixing the


following aqueous solution: True and False
(a) CH 3 COONa and excess HCl 1. A cell works, if its e.m.f. is positive.
(b) CH 3 COONa and CH 3 COOH [Garhwal 2011]
(c) NaCl and HCl 2. In Daniell cell oxidation reaction takes place at zinc
electrodes. [Garhwal 2006]
(d) CH 3 COOH and excess NaOH
[Garhwal 2010] 3. The Galvanic cell converts electrical energy into
21. The extent of ionisation increases: chemical energy. [Garhwal 2007]
(a) With the increase in the concentration of the 4. Reference electrodes are of two types.
solute. [Garhwal 2010]
(b) On addition of excess of water to the solution 5. The chemical properties of a metallic conductor
change after electricity passes through it.
(c) On decreasing the temperature of the solution
[Garhwal 2008]
(d) On Stirring the solution vigorously 6. Calomel electrode is the main metal-metal salt ion
22. The nature if solution obtained by salt hydrolysis of electrode. [Garhwal 2010]
Sodium Sulphate is : 7. With the help of Nernst equation, the potential of
(a) Acidic (b) Basic an electrode is determined.
[Garhwal 2005, 12]
(c) Netural (d) Acidic or basic Solution
8. When sodium acetate is added into an aqueous
solution of acetic acid the pH of the solution does
Fill in the Blank not change. [Garhwal 2010]
1. Anode is an electrode at which ………………
9. Standard hydrogen electrode is a primary
occurs. [Garhwal 2012]
reference electrode.
2. Electrolysis is due to ……...... formation in salt
10. Potentiomeric titration does not not require an
solution. [Garhwal 2011]
indicator.
3. In electrolysis the oxidation takes place at ………
11. Dissociation of weak electrolytes is higher.
electrode. [Garhwal 2010] [Garhwal 2010]
4. Calomel electrode is a ....................... reference 12. The solubility product of Ag 2 CrO 4 is equal to
electrode.
[Ag + ] 2 [CrO42− ].
5. If the H + ion concentration in a solution is 10 −5 [Garhwal 2007]
mole/litre, then the pH of the solution is 13. pH value of 0.01M – HCl solution will be 4.0.
………………. . [Garhwal 2007] [Garhwal 2006]
6. LJP = ................. . 14. pH of an acidic buffer solution can not be more
7. Concentration cell is of ..................... types. than 7 at 25°C.
8. Plot of potentiometric titration for HCl–NaOH 15. pH of 10 –8 M KOH is more than 7.
titration is ...................... . 16. The pH of an acidic buffer solution is given by
9. For AgBr, K sp = ............. [Acid]
pH=pKa +log .
10. NH 4 NO 3 hydrolysis gives rise to .................. [Salt]
solution.
11. CH3COONH4 is a .................... buffer solution.
12. The p H value of any alkaline solution is ................
than. [Garhwal 2005]
P-227

Numerical Questions
8. 7.3 9. 2.2 3:1::BOH:HCl 10. PH=8.634
11. 3.04 12. 0.01% 13. 12.486 gl −1

14. (i) 3.73 × 10 −3 15. [H + ]=7.2 × 10−10 , 16. 39.625 ml


(ii) 7.173 [OH − ]=1.39 × 10 −5

Objective Type Questions

Multiple Choice Questions


1. (b) 2. (c) 3. (b) 4. (b) 5. (d) 6. (c)
7. (b) 8. (d) 9. (c) 10. (a) 11. (a) 12. (d)
13. (b) 14. (d) 15. (a) 16. (b) 17. (c) 18. (d)
19. (a) 20. (b) 21. (b) 22. (c)

Fill in the Blank


1. Oxidation 2. Ion 3. Anode

4. Secondary 5. 5 6.  RT (a ± )2 
(2t –) – 1 In
 F (a ± )1 

7. two 8. 9. S2

VNaOH

10. acidic 11. neutral 12. more

True and False


1. True 2. True 3. False 4. True
5. False 6. True 7. True 8. False
9. True 10. True 11. False 12. True
13. False 14. False 15. True 16. False
P-228

Hints and Solutions

Long Answer Type Questions


1. The Galanic cell is Zn|Zn 2+ ||Ag + |Ag

and the cell reaction is, LHE: Zn → Zn 2+ +2e –

RHE: 2Ag + +2e – → 2Ag



Overall cell reaction : Zn + 2Ag + → Zn 2+ +2Ag

° ° 0.0591 Zn2+
So, E cell =E cell – log
2 (Ag + )2

0.0591 0. 5
=(E °Ag + /Ag – E ° )– log
Zn2+ / Zn 2 (10)2
0.0591
=(0.799+0.76) – log 5 × 10 –3
2
0.0591
=1.559 – (–2.3) = 1.627 V
2
2. LHE : Zn → Zn 2+ + 2e –

RHE : 2Ag + + 2e – → Ag



overall reaction : Zn+2Ag + → Zn 2 + 2Ag

∆G° = – nFE° = –2 × 96500 × 1.561 = –301.27 KJ/mol.


0.0591
4. E° cell = log K eq.
2
2E ° cell
= log K eq.
0.0591

5. (b) LHE : Zn → Zn 2+ +2e –

RHE : 2H + +2e – → H 2



overall reaction : Zn+2H + → Zn 2+ + H 2


0.0591 01
.
0.28 = (0+0.761) – log
2 + 2
(H )
01
. 01
.
16.27 = log ⇒ 1.8 × 10 16 =
(H + )2 (H + )2
.
01
⇒ (H + ) 2 = = 0.055 × 10 –16
1.8 × 1016
P-229

= 5.5 × 10 –18

H + = 2.34 × 10 –9

As the solution is highly dilude pH is close to 7.



10. (b) LHE : Zn → Zn 2 + + 2e

RHE : Cd 2+ + 2e → Cd



Overall reaction : Zn + Cd 2+ → Zn 2+ +Cd.

E° cell = E ° –E ° = –0.403 – (–0763


. )
Cd2+ / Cd Zn2+ / Zn
= 0.36 V.

Kw 10–14
23. h= = = 2.35 × 10 – 4
Kb × C 1.8 × 10–5 × 10–2

24. (ii) ph=–log 10 10 –3 =3

26. H+ = α C = K a × C = 7.2 × 10–10 × 10–2

=2.68 × 10 – 4

pH = 4 – log 2.68 = 3.57

30. K 10–14
Kh= w = = 5.5 × 10–10
Ka 1.8 × 10–5

Kh 5. 5 × 10–10
h= = = 7.45 × 10–5
C 01
.

Short Answer Type Question


0.0591 0.5
5. E= log = 0.0295 V.
2 0.05
25. Zn will produce H 2 (g).

2 1
40. C = × = 0.05 ⇒ pOH = –log 5 × 10–2
OH– 40 1

=2– log 5 = 1.3

So, pH= 14 – 1.3 = 12.7

41. C =10 –2 pH=2,


H+

C =10 –2 pOH = 2 ∴pH=12


OH–

42. 4 1
C = . ⇒ pOH =1 ∴pH=13
× = 01
OH– 40 1
P-230

43. Kw 10–14
h= = = 10–2
Ka × C 10 8 × 10–2

47. Since, it is an acidic buffer solution, so,


[salt]
pH = pKa +log 10 ⇒ pH = pKa = 4.76
[Acid]

48. α 2 c (01
. )2 (0.01) 10–4
Ka = = = . × 10–5
= 11
1– α 1 – 01. 0.9

51. C =10 –8 , As this cone is very small so contribution from water has to be considered so, net
OH–
OH ≅ 10 –8 +10 –7 ≅ 11 × 10 –8 (appoximately)

POH= 8– log11 = 6.98

pH = 7.02.

55. [salt] 8.2 / 82


pH= pKa + log = 4.76 + log
[Acid] 3 / 60
01
.
= 4.76 + log = 5.06.
0.05

58. pH + pOH = 14 pOH = 14 – 5.3 = 8.7

67. H+ = K a × C = 1.8 × 10–5 × 10–2

= 4.24 × 10 – 4

pH = 4 – log 4.24 = 3.372

68. (b) K s p = (35)3 (S) = 27.54 = 27(2 × 10–4 )4

= 4.32 × 10 – 14

73. [salt] 5 / 60
pH = pKa + log = 4.76 + log
[Acid] 15 / 82
1 82
= 4.76 + log × = 4.41.
12 15

74. [salt] 015


.
pH= pKa + log = 4.76 + log
[Acid] 01 .
3
= 4.76 + log = 4.93.
2

Very Short Answer Type Questions


1. Cu to Zn
3. As the transport number of Na + and Cl − are not same.
❍❍❍
UnitP-231
-IV

C HAPTER 7
Phase Equilibrium

7.1 Different Basic Terms


7.1.1 Phase
A phase is defined as any homogeneous and physically distinct part of a system which is bounded by a surface
and is mechanically separable from other parts of the system. Examples of various types of phases are :
1. A gas mixture constitutes a single phase since gases are completely miscible.
2. Immiscible liquids constitute different phases. Thus, carbon tetrachloride (CCl 4 ) and water (H 2 O),
which do not mix with each other, from two phases.
3. A system consisting of a liquid in equilibrium with its vapour constitutes two phases,like, the liquid
phase and the vapour phase, each phase being separated from the other by a distinct boundary.
4. Ice-liquid, water-water vapour constitute a system containing three phases.
5. Completely miscible liquids such as water and alcohol and benzene and chloroform constitute
one-phase systems.
Consider the decomposition of calcium carbonate into CO 2 and CaO.
CaCO 3 (s) q CaO(s) + CO 2 (g)
Here there are two solids phases and one gaseous phase. So it is a three-phase system. When various phases
are in equilibrium with one another in a heterogenous system, there can be no transfer of energy or mass
from one phase to another. This means that for an equilibrium, the various phases must be at the same
temperature and pressure.

7.1.2 Component
The number of components of a system at equilibrium is defined as the smallest number of independently
variable constituents by means of which the composition of each phase can be expressed either directly or in
terms of chemical equations. A few examples are considered .Water exists in three phase, as shown here,
ice q liquid q vapour
However, the composition of each phase can be expressed in terms of H 2 O. Hence, it is a one-component
system.
Sulphur exists in four phases– rhombic sulphur, monoclinic sulphur, liquid and vapour. But since the
composition of each phase can be expressed in terms of sulphur only, it is a one-component system.
P-232

Now, an aqueous sucrose solution is considered. The composition of the solution phase can be expressed by
specifying the amounts of sugar and water. Hence, it is a two-component system.
In a chemically reactive system involving reactions between various species, the situation is slightly different.
Consider the thermal decomposition of CaCO 3 in a closed vessel.
CaCO 3 (s) q CaO(s) + CO 2 (g)
The system contains three phases – CaCO 3 (solid), CaO (solid) and CO 2 (gas). However, as a result of the
existence of equilibrium, the number of components is also two. If CaO and CO 2 are chosen, then the
composition of the solid CaCO 3 can be given by CaO + CO 2 such that both the species are having one mole
each.
If CaCO 3 and CO 2 are taken as the components, then the composition of solid CaO can be given by CaCO 3
– CO 2 and so on.
In a chemically reactive system, the number of components is given by
C=N–m–n–R
here N is the number of chemical species, m is the number of independent equilibrium conditions, n is the
number of relations between concentrations due to initial conditions and R is the number of independent
chemical reactions.

Example 1: Explain why KCl – NaCl – H 2O is regarded as a 3–component system whereas KCl –
NaBr – H 2O should be regarded as a 4–component system.

Solution: The number of component is given by C=N–E; have N is number of species and E is the number
of independent equations velating their concentration.
For KCl–NaCl –H 2 O system N=3 (three species).
E=0 So, C=3–0=3
For KCl–NaBr –H 2 O system N=5 (KCl, NaBr, H 2 O, NaCl, KBr)
E=1 as KCl +NaBr q KBr + NaCl
So, C=5–1 = 4.
Example 2: For each of the following systems, determine the number of components:
(i) NH4Cl(s),NH + (aq),Cl − (aq),H2O(liq),H2O(g),NH3 (g), H3O+ (aq),
4

OH (aq),NH4OH(aq),
(ii) NH4Cl(s),NH3 (g),HCl(g), where the partial pressure of NH 3 is equal to the partial
pressure of HCl as is the case when the gaseous mixture is formed by the sublimation of
NH4Cl(s).
(iii) NH4Cl(s),NH3 (g),HCl(g), where the partial pressure of NH 3 is not necessarily equal to
the partial pressure of HCl.
Solution: (i) N= 8 (as number of species is 8–H 2 O (liq) and H 2 O (g) is counted as one species).
E=5; as shown below,
P-233

(a) NH 4 Cl q NH+
4 +Cl

(b) 4 +H 2 O q
NH+ NH 3 +H 3 O +
(c) NH 3 +H 2 O q NH 4 OH
(d) 2H 2 O q H 3 O +OH –
+

(e) Electro neutrality of solution achieved by merging of ions.


So, C=N – E = 8 – 5 = 3
(ii) N = 3 (NH 4 Cl,NH 3 , HCl)
E = 2 as shown here
(a) NH 4 Cl q NH 3 +HCl
(b) pNH 3 = pHCl
So, C= 3–2=1
(iii) N=3 (NH 4 Cl, NH 3 , HCl)
E=1 as shown here NH 4 Cl q NH 3 +HCl
C=3–1=2.

7.1.3 Degree of Freedom (F)


The degree of freedom of a system is defined as the number of independent variables such as temperature,
pressure and concentration which must be specified in order to define the system completely. Thus, the state
of a pure gas can be specified by the two variables P and T or P and V, since the third variable can be
calculated from the equation of state. This means that a pure gas has two degrees of freedom.
Now, a one-component system having two phases is considered. The example is furnished by liquid water in
equilibrium with its vapour. The system will have only one degree of freedom because at a given
temperature, the equilibrium vapour pressure of water can have only one fixed value. Thus, if temperature is
specified, pressure becomes known automatically and vice–versa. One has to mention only one variable,
either temperature or pressure. The system is, therefore, monovariant.
Finally, a one–component system having three phases is considered. The example is furnished by ice, liquid
water and vapour coexisting at the freezing point of water. This system will have no degree of freedom. The
reason is that these three phases can coexist only at one particular temperature under one particular pressure.
The mere statement that these three phases coexist defines the system completely. Thus if F=0, the system is
called invariant; if F=1,univariant or monovariant,if F=2, bivariant and if F=3, trivariant, and so on.
It is concluded that :
1. The greater the number of components in a system, the greater is the number of degrees of freedom for
a given number of phases.
2. The greater the number of phases, the smaller is the number of degrees of freedom, such as T, P and
composition, required to specify the system completely.
3. For a given number of components, the number of phases is maximum when the number of degrees of
freedom is zero. Thus, for a one-component system, the maximum number of phases is three and for a
two-component system, the maximum number of phases is four.
P-234

7.1.4 Conditions for Equilibrium between Phases


The following conditions must be satisfied for the existence of equilibrium between various phases in a
multiphase system.

7.1.4.1 Thermal Equilibrium


All the phases must be at the same temperature otherwise there will be flow of heat from one phase to
another. Consider two phases α and β at temperature T α and Tβ , respectively. Let S α and Sβ be the entropies
of the two phases and let dq be the heat transferred from phase α to phase β at equilibrium. Then the entropy
change of the system is given by
dS = dSα + dSβ = 0 (at equilibrium)
dq dq
Since dSα = − and dSβ = +
T T
dq dq
∴ − + =0
Tα Tβ

∴ Τα = Τβ

7.1.4.2 Mechanical Equilibrium


All the phases must be under the same pressure otherwise the volume of one phase will increase at the
expense of another. It can be proved as below. It is supposed that α–phase is expanded into β–phase by the
volume change dV. Then, the change in the Helmholtz free energy at constant temperature is given by,
dA = −dA α + dA β = 0 (at equilibrium)
But dA α = −Pα dV and dA β = −PβdV (at constant T)
Since at equilibrium, dA α = −dA β hence Pα = Pβ

7.1.4.3 Chemical Equilibrium


For a system of many phases at equilibrium, the chemical potential of a component i is the same in all the
phases. We can prove it as follows.
Consider a closed system of P phases designated as α, β, γ , …. P containing C components designated as 1, 2,
3, …. C, in equilibrium. It is assumed that each phase is at the same constant temperature are pressure.
The Gibbs free energy of each phase is a function of T, P and the composition of the phase, i.e.,
G α = f(T,P, ni )α 
G β = f(T,P, ni )β 

... ... ...  where i = 1,2,3,...... i, ......C
... ... ... 

G P = f(T,P, ni )P 

(The subscript P designates P th phase)


The Gibbs free energy change for the system is the sum of the Gibbs free energy changes of the various
phases. Thus,
dG = dG α + dG β + dG γ +......
P-235

Again, for a multicomponent system, we have


dG = − SdT + VdP + ∑i µ idni
At constant T and P, dT = 0 and dP = 0 so that
(dG)T,P = ∑i µ idni
For an infinitesimal transfer of mass from one phase to another,
dG = ∑ µ i,α dni,α + ∑ µ i,βdni,β + ∑ µ i,γ dni,γ
i=1 i=1 i=1
Also, for a closed system at equilibrium, dG = 0.
∴ ∑i µ i,αdni,α + ∑i µ i,βdni,β + ∑i µ i,γ dni,γ +... = 0
Since the system is closed, the total mass of each component at equilibrium is constant. Hence,
dn 1,α + dn 1,β + dn 1,γ +...+ dn 1,P = 0 
dn 2,α + dn 2,β + dn 2,γ +...+ dn 2,P = 0 

−−− 
dni,α + dni,β + dni,γ +...+ dni,P = 0 

−−− 

dn C,α + dn C,β + dn C,γ +...+ dn C,P =0 
In order that above equatione 16 remains zero for all variations of n, subject to the restrictions
imposed above, we must have the following requirements met with :
µ 1,α + µ 1,β + µ 1,γ +...+µ 1,P 
µ 2,α + µ 2,β + µ 2,γ +...+µ 2,P 

−−− 
µ C,α + µ C,β + µ C,γ +...+µ C,P 
It follows from the above that for a multiphase system at equilibrium the chemical potential, µ i , of a
component i is the same in every phase.
The phase rule was enunciated by J.Willard Gibbs.

7.1.5 The Gibbs Phase Rule


The Phase rule was enunciated by J. willard Gibbs.

7.1.5.1 The Rule and its Derivation


It states that if the equilibrium in a heterogenous system is not affected by gravity or by electrical and
magnetic forces, the number of degrees of freedom, F, of the system is related to the number of components,
C and the number of phases, P, existing at equilibrium with one another by the equation,
F=C–P+2

7.1.5.2 Derivation of the Phase Rule


Let a system of C components (C 1, C 2 , C 3 , ... C C ) is distributed between P phase (α, β, γ , δ,...P), as shown in
fig. (1).
P-236

It is assumed that passage of a component from one phase to another does not constitute a chemical reaction.
The state of each phase of the system is completely specified by the two variables, temperature and pressure
and also by the composition of each phase. In other words, the state of each phase is specified by
T, P, (X1,α , X 2,α ,..., X C,α ),(X1,β , X 2,β ,..., X C,β ),(X1,P , X 2,P ,..., X C,P )
Where X i ' s are compositions of the components. The total number of variables is thus CP + 2. However, all
the variables are not independent since in each phase, the sum of the mole fraction must equal unity.
X1,α + X 2,α + X 3,α +...= X1,β + X 2,β + X 3,β+...,etc.= 1
In other words,
C1 C2 C3 ... ... CC
∑i Xi,P = 1 (i = 1, 2, 3, …. C)
P
for all the P phases separately. There are thus P relations
C1 C2 C3 ... ... CC
of this type. Again, for complete equilibrium to exist
between the phases, the chemical potential of each species δ
must be the same in each phaseis
C1 C2 C3 ... ... CC
µ 1,α = µ 1,β = µ 1,γ =... = µ 1,P 
γ
µ 2,α = µ 2,β = µ 2,γ =... = µ 2,P 
µ C,α = µ C,β = µ C,γ =... = µ C,P  C1 C2 C3 ... ... CC

It is observed that there are P – 1 separate equations for β


each component. Hence, for C components, the number C1 C2 ... ...
C3 CC
of such equations is C(P – 1). Also, the equilibrium
conditions for the chemical reactions required that the α
chemical affinity, A f , for each reaction at equilibrium must ... ...
C1 C2 C3 CC
be zero, so that,
A f,i = 0 (I = 1, 2, 3, …., r ′)
Fig. 1: A system of C components distributed
This means that there are r ′ equations of this type. Hence through P phases
total number of restricting conditions is
P + C(P − 1) + r'
The degree of freedom, F, is given by the difference between the number of variables required to specify the
state of the system = CP + 2, and the number of restriction imposed by their inter-dependence. Thus,
F = (CP + 2) − (P + CP − C + r' )
= 2 + (C − r' ) − P
This equation is known as Gibbs phase rule. If the system is non-reactive, so that no reaction takes place in it,
then equations of the type A f = 0 will be absent, hence, r ′ = 0. So, the phase rule becomes
F=C −P +2
It is being observed from this equation that for a system having a given number of components, the greater
the number of phases, the smaller will be the number of the degrees of freedom. Thus, for a one-component
system like water, the maximum number of phases that can coexist at equilibrium is three so that for this
equilibrium, the number of degrees of freedom is zero. In general, for a system having a given number of
components where number of phases is maximum, the degree of freedom is zero. On the other hand, for a
system with a given number of phases, the larger the number of components, the greater will be the number
of the degrees of freedom. In these cases, variables such as T, P and concentration must be specified in order
to describe the state of the system completely.
P-237

The phase rule does not tell us anything regarding the composition of matter. It merely states that systems
having the same number of the degrees of freedom behave alike thermodynamically. The phase rule is a very
important generalization that enables us to investigate highly complex heterogeneous equilibria.
Example 3: Determine the number of degrees of freedom in each of the following systems.
Suggest the variables that could correspond to these degrees of freedom.
(i) Liquid water and water vapour in equilibrium.
(ii) Liquid water and water vapour in equilibrium at pressure of 1 atm.
Solution: From Gibbs phase rule it can be said F=C – P+2
(i) As C=1 So, F= 1–2+2=1
So, only one variable either T or P need to be given.
(ii) As pressure is 1 atm (a constant), so F reduces by 1.
So, Gibbs phase rule can be stated as,
F=C–P+1
As, C=1 and P=2 F= 1–2+1=0
So, the system is invariant.
Example 4: Calculate the number of components and the number of the degrees of freedom in
(i) An aqueous solution of glucose.
(ii) An aqueous solution of acetic acid.
(iii) An aqueous solution of sodium chloride.
(iv) A mixture of H 2(g), O 2(g) and H 2O(g).
Solution: (i) C=2, and P = 1, so F= 2–1+2=3
The variables are temperature, pressure and concert ration.
(ii) C=2, P=1, so F=3
And the variable are once again temperature, pressure and concentration like the previous case.
(iii) C=2, P=1 so, F=3
Variables are temperature, pressure and concentration.
(iv) It we are dealing with a mixture of 3 gases,
so, C=3. Since, P=1, so, F=3–1+2=4
It means the variables are temp. Pressure and concentration of two of three components.
If we deal with a mixture of gases produced by decomposition of water vapour (H 2 O q H 2 + O 2 )
1
2
; then C=1, since the information about other two can be obtained by the equilibrium constant.
pH2O
K=
1
(pH2 )(pO 2 )2
and the known stoichiometry pH 2 =2pO 2 . So, C=1, p=1, hence F=1–1+2. Variables are
temperature and pressure.

7.2 One Component System


Since the minimum number of phases in any system is 1, it is evident from the phase rule equation F = C – P
+ 2 that, for a one–component system, the maximum number of degrees of freedom can be only 2, as shown
here,
P-238

F=C–P+2=1–1+2=2
The one component system is, thus, bivariant. It can be completely defined by specifying two variables. The
two variables are temperature and pressure. If a one-component system has two phases in contact with each
other, the degree of freedom will be 1 :
F=C–P+2=1–2+2=1
The system is then said to be monovariant. It can be completely defined by specifying only one variable -
either temperature or pressure. If temperature is fixed, pressure of the system is fixed automatically and
vice-versa. Thus, the system consisting of a liquid in contact with its vapour like is monovariant.
H2O(l) q H2O(g)
If temperature is fixed, the pressure of the vapour is fixed automatically. This is because at each temperature,
there can be one and only one vapour pressure. Other examples of monovariant systems are :
(i) A solid in contact with its vapour,
H2O(s) q H2O(g)
(ii) A solid in contact with its liquid,
H2 O (s) q H2O(g)
ice
Some important one-component systems will now be discussed.

7.2.1 Water System


Water, as already mentioned, can exist in three possible phases, namely, solid, liquid and vapour. Hence,
there can be three forms of equilibria, as shown below,
(i) Liquid q Vapour
(ii) Solid q Vapour
(iii) Solid q Liquid
Each equilibrium involves two phases. The phase diagram for the water system is shown in figure.
C

218 atm Critical pressure A


Fusio

e
rv

Liquid b
Pressure (not to scale)

Solid
Cu
n

Ice Water
cur v

c y
x
e

z
1 atm X Y
Z Z′
O
4.58 mm.
Triple Vapour
A′ Point
m
Sublimation
n c Critical
Curve s Temperature
B u M.P. B.R.
0° 0-0075° 100° 374°
Temperature (not to scale)
Fig. 2: The phase diagram for the water system
P-239

The curve OA represents the equilibrium between liquid water and vapour at different temperatures. It is
called the vapour pressure curve of water as it gives the vapour pressure of water at different temperatures. It
can be seen that, for any given temperature, there exists one and only one vapour pressure. Similarly, for
each vapour pressure, only one temperature can be maintained. Thus, the degree of freedom of the system is
one, as predicted by the phase rule:
F=C–P+2=1–2+2=1
At 100°C, the vapour pressure of water equals the pressure of the atmosphere which is 760 mm. This is,
therefore, the boiling point of water. The curve OA extends as far as the critical temperature of water (374°C)
since above this temperature liquid water cannot exist. The variation of vapour pressure with temperature is
quantitatively given by Clapeyron-Clausius equation,
P2 ∆H v  T2 − T1 
ln =  
P1 R  T1T2 
The curve OB represents the equilibrium between ice and vapour. It is called the vapour pressure curve of ice
or sublimation curve of ice. Its lower end B extends to absolute zero. Again, as can be seen, for each
temperature there can be one and only one pressure and similarly for each pressure, one and only one
temperature can be maintained. In other words, the degree of freedom is 1.
The variation of vapour pressure of ice with temperature, is again, quantitatively given by Clapeyron-
Clausius equation,
Example 5: At 100°C, the specific volumes of water and steam are respectively 1 c.c. and 1673
c.c. Calculate the change in vapour pressure of the system by 1°C change in temperature. The
molar heat of vaporization of water in this range may be taken as 9.70 kcal.
Solution: Vmolar of H 2 O(l) = 18 cm 3 /mol = 18 ×10 –6 m 3 /mol .
Vmolar of H 2 O(g) = 18×1673×10 –6 m 3 /mol=30114×10 –6 m 3 /mol
cal J J
∆H vap = 9700 × 4.184 = 40584.8
mol cal mol
dP 40584.8
And, =
dT m3
373(30114 – 18) × 10 –6
mol
As, dT= 1K,
So, dP=0.00361×10 6 N/m 2 = 0.0356 atm
=27.08 mm of Hg
Since, along OA, the two phases are liquid and vapour and along OB, the two phases are solid and vapour, at
the point O, where the two curves meet, three phases, namely, solid, liquid and vapour, will coexist. Such a
point is known as the triple point. The temperature and pressure at the triple point of water are 0.0075°C and
4.58 mm, respectively. According to the phase rule, O is an invariant point, the degree of freedom being zero,
as shown below,
F=C–P+2=1–3+2=0
The curve OC represents the equilibrium between ice and water. It is called the fusion curve of ice as it
indicates the temperature and pressure at which solid, ice and liquid water can coexist in equilibrium. In other
words, this curve shows the effect of pressure on the melting point of ice. As can be seen, the line OC is
inclined towards the pressure axis which indicates that the melting point of ice is lowered by increase of
pressure. As can be predicted from Clapeyron-Clausius equation,
P-240

dT T(Vl − Vs )
=
dP ∆H f
Since density of ice is less than that of water, Vs is greater than Vl . In other words, the expression on the right
hand side of the above equation is negative. Hence, dT/dP should also have a negative sign. This means that
the increase of pressure must lower the freezing point of water and vice versa. It can be easily shown with the
help of the above equation that freezing point of water is lowered by 0.0075°C by 1 atm increase in pressure.
Thus, while the freezing point of water at a pressure of 4.58 mm is +0.0075°C, at a higher pressure of 760
mm, it is reduced to 0°C.
Example 6: The specific volumes of ice and water at 0°C are 1.0907 cm 3 and 1.0001 cm 3 ,
respectively. What would be the change in melting point of ice per atm increase of pressure?
Heat of fusion of ice = 79.8 cal g −1.
Solution: Vmolar of H 2 O(s) = 18 ×1.0907×10 –6 m 3
Vmolar of H 2 O(l) = 18×1.0001×10 –6 m 3
g cal J
∆H f of ice = 18 × 79.8 × 4.184
mol g cal
= 6009.9 J/mol.
Increase of pressure, dP=1atm=101325 N/m 2
dT=?
dT T(Vl – Vs ) 273K[–0.0906] × 18 × 10 –6
And, = =
dP ∆H f 6009.9 J / mol
=–0.0075 K.
Along the curve OC, there are two phases, namely, ice and water. Therefore, according to the phase rule, the
system should be univariant. This is seen to be actually the case as, for any given pressure, melting point will
have one definite value. The curve OC must meet the other two curves at the triple point O.
(i) Significance of Regions Between the Lines: It is presumed that the state of equilibrium is
represented by a point X on the line OA. The two phases in contact are liquid and vapour. If the
temperature is kept constant and pressure increased, the vapour will be compressed wholly into the
liquid phase. This change is represented by the dotted line XY in figure 2. Similarly, if pressure is kept
constant and temperature is decreased, the vapour will change into liquid again. This change is
represented by the dotted line xc. Thus, the area above the curve OA represents exclusively the liquid
phase, as shown.
Coming back to the point X, if the pressure is maintained constant and the temperature is increased,
the liquid will change completely into vapour. The change is represented by the dotted line XY.
Similarly, if the temperature is kept constant and the pressure is diminished, liquid will again be
converted completely into the vapour phase as represented by the dotted line xz. Thus, the area or
region below the curve OA represents the vapour phase only.
Similarly, if a system represented by a point s on the solid-vapour equilibrium curve (OB) is subjected
to increase of temperature at a constant pressure (along ST) or decrease of pressure at a constant
temperature (along SU), it will change completely into vapour. Thus, any point lying below the curve
OB also represents the vapour phase only.
Lastly, if the system at S is subjected to increase of pressure at constant temperature along sm or
decrease of temperature at constant pressure along SN, the vapour will condense completely into solid
phase. Thus, the area lying above the curve OB will represent the solid phase exclusively.
P-241

It may be noted that if a point representing the state of equilibrium of the system lies within a particular
region, the number of phases is 1; if on a line, the number of phases is 2 and if on a point where the
lines meet, the number of phases is 3.
(ii) Metastable Equilibrium: Sometime it is possible to cool water or any liquid below its freezing
temperature without the separation of ice. The water is then said to be super-cooled and can be kept as
such almost indefinitely if the presence of ice or any other solid phase is carefully avoided. The vapour
pressure curve of liquid water AO can, therefore continue below the point O, as shown by the dotted
curve OA’. The liquid q vapour system along the curve OA’ is said to be in metastable equilibrium
because as soon as a small particle of ice is brought in contact with the supercooled liquid, the entire
liquid solidifies. It will be seen from the phase diagram that the curve OA’ lies above the curve OB.
Thus, the metastable system has a higher vapour pressure than the stable one at the same temperature.
(iii) Effect of Temperature and Pressure: The significance of the equilibrium diagram can be further
understood by following the changes that occur on altering the temperature or pressure of the system. It
is desired to know the effect of heating ice when it is under a pressure of 1 atm and at a certain
temperature represented by the point X in the fig. (2). As the system is bivariant, the temperature can
have any value at the same pressure. Therefore, heating the ice slowly, at constant pressure will shift
the system along the line XY. At Y fusion takes place. The liquid phase also appears. The system has
now two phases and, therefore, one degree of freedom. Now, the temperature cannot alter without the
alteration of pressure. The effect of continued heating will be simply to cause fusion of ice at constant
temperature. When fusion is complete, when ice has changed completely into water, the system again
has only one phase and becomes bivariant. The temperature begins to rise along YZ. At Z vaporization
begins, the vapour phase also makes its appearance. The number of phases rises to 2 and the system
becomes monovariant.
The continued heating will not alter the temperature as the pressure has been kept constant. The only
effect will be to convert more and more of the liquid into vapour. When the liquid phase disappears
completely, the system will have only one phase - the vapour phase and will become again bivariant.
The temperature of the vapour will rise along the line ZZ’.
(iv) Triple Point: It is interesting to note the effect of heating and applying pressure on the system when all
the three phases coexist as at the triple point O. The effect of heat will be simply to cause more and
more of the solid to melt but there will be no rise in temperature or pressure until the whole of the solid
has completely changed into liquid. When this happens, the system has only two phases - liquid and
vapour. The system changes from non-variant to univariant. Therefore, further addition of heat will
cause a rise of temperature. The equilibrium will shift along the curve OA.

7.2.2 Carbon Dioxide System


The phase diagram for carbon dioxide system has three distinct areas in which carbon dioxide can exist either
as solid, liquid or gas. It is being shown below in fig. (3).
AB is the sublimation curve along which solid carbon dioxide is in equilibrium with the gas, BD is the
vaporisation curve along which liquid carbon dioxide is in equilibrium with the gas while BC is the fusion
curve along which solid and liquid carbon dioxide are in equilibrium with each other. B is the triple point at
which all the three phases of carbon dioxide coexist in equilibrium with one another. The temperature of CO
2 at this point is –57°C while the pressure is 5.2 atm. A slight variation in temperature or pressure at this point
may result in the disappearance of one of the two phases. For example, a small increase in temperature will
result in the disappearance of the solid phase and the equilibrium will shift along the curve BD while a slight
decrease in temperature will result in the disappearance of the liquid phase and the equilibrium will shift
along the curve BA. If the temperature is kept constant and the pressure is increased, the gaseous phase will
disappear and the equilibrium will shift along the curve BC.
P-242

C D

Solid Liquid

Pressure (not to scale)


5.2 atm.
B
Gas

1 atm.
A
–78° –57°
Temperature (not to scale)
Fig. 3: The Phase Diagram for the Carbon Dioxide System
The phase diagram of carbon dioxide resembles that of water in showing three distinct areas for solid, liquid
and gaseous phases. But it differs essentially from latter in several respects. In the first place, the fusion curve
slopes away from the pressure axis. This indicates that increase of pressure raises the melting point of solid
carbon dioxide is greater than that of solid carbon dioxide.
The second point to be noticed is that solid carbon dioxide can exist in equilibrium with its liquid form only at
a very high pressure equal to 5.2 atom. This is unlike the water system in which ice and water can exist in
equilibrium even at a very low pressure equal to 4.58 mm of Hg. The third point of difference is that the
vapour pressure of solid carbon dioxide even at extremely low temperature is very high and many times
higher than that of ice.
A significant feature of carbon dioxide system is that even at 1 atm pressure, carbon dioxide gas can be
directly solidified without the appearance of liquid phase merely on cooling to –78°C. It is for this reason that
solid carbon dioxide is commonly known as dry ice.
Polymorphy: The existence of a given substance in more than one crystalline form possessing different
physical properties is known as polymorphism. Polymorphism occurs in a number of elements and
compounds. When it occurs in elements, it is frequently referred to as allotropy.
Each polymorphic form constitutes a separate phase. The temperature at which one form changes into
another, at a given pressure, is known as the transition temperature. For example, monoclinic sulphur, when
cooled under a pressure of one atm, changes into rhombic sulphur at 95.6°. At the same time, rhombic
sulphur when heated under a pressure of one atm, changes into monoclinic sulphur at 95.6°C. Thus, 95.6°C
is the transition temperature at which one form of sulphur changes reversibly into the other. Polymorphic
forms which can undergo reversible transformations into one another at the transition temperature are said to
be enantiotropic and the phenomenon is known as enantiotropy.
There are cases when polymorphic forms do not undergo reversible transformations into one another. For
example, diamond can be converted into graphite under suitable conditions of temperature and pressure but
the reverse is not possible. Similarly, white phosphorus can be readily transformed into red form under
suitable conditions but the reverse cannot take place in a direct manner. Such polyphormic forms which
cannot be reversibly transformed into one another are said to be monotropic and the phenomenon is known
as monotropy.
P-243

7.2.3 Sulphur System


Sulphur primarily exists in two crystalline D
forms, rhombic (S R ) and monoclinic
(S M ), with 95.6°C as the transition
temperature at one atm pressure at which Sulphur 150°, 1290 atm.
C
they can be transformed into one another. Rhombic
Below 95.6°C, rhombic is the stable form Liquid E

sm
while above it, monoclinic is the stable

M
variety. At 95.6°C, both forms are in

e of S
Sh
equilibrium with each other. Each form

ve
has its own characteristic melting point. Solid
Pressure (not to scale)

Fusion curv

ur
Thus, under a pressure of one atm,

eC
ur
melting point of rhombic sulphur is 114°C

s
res
ic
while that of monoclinic sulphur is

oc l i n

rP
120°C. The liquid form of sulphur (S L )

ou
e
Sublimation

Mo n
ur v

p
undergoes some sizeable changes in

Va
Curve of SR

nc
colour and viscosity when heated and B Vapour
ultimately boils at 444.7°C. Summarily, tio
nsi
A′
sulphur exists in four possible phases,
Tra

two solids (S R and S M ), one liquid (S L )


on
and one vapour (S V ) phase. However, m ati M
i S
O bl of
all the four phases cannot coexist at the Su r ve
same times since the number of phases A Cu
coexisting in a one-component system 95.6° 114° 120°
cannot exceed three, as already shown. Temperature °c (not to scale)
The phase diagram of sulphur system is
Fig. 4: The phase diagram for the sulphur system
represented in fig. 4.
Curve AO is the sublimation curve of rhombic sulphur and gives the vapour pressure of rhombic sulphur at
different temperatures. The two phases in equilibrium are rhombic sulphur and the vapour. The equilibrium
(S R q S V ) is monovariant. Therefore, at one temperature, there can be one vapour pressure only.
The point O is the transition temperature (95.6°C) at which rhombic sulphur changes into monoclinic
sulphur. O is thus a triple point at which three phases, two solids and the vapour (S R – S M – S V ) coexist in
equilibrium. This is a non-variant point.
The curve OB is the sublimation curve of monoclinic sulphur. It gives vapour pressure of monoclinic sulphur
at different temperatures. As the number of phases is 2, the system is monovariant. The Clapeyron-Clausius
equation can be used for the quantitative study of the variation of vapour pressure of solid sulphur (S R or
S M ) with temperature. The point B is the melting point (120°C) of monoclinic sulphur. This is another triple
point at which three phases – monoclinic sulphur, liquid and vapour sulphur (S M – S L – S V ) are in
equilibrium. This is a non-variant point. The curve BE is the vapour pressure curve for liquid sulphur. The
two-phase equilibrium (S L q S V )is monovariant.
The variation of vapour pressure of liquid sulphur with temperature can be studied, again, with the help of
Clapeyron-Clausius equation. The curve OC is the transition curve which gives the effect of pressure on the
transition temperature of rhombic sulphur into monoclinic sulphur. The equilibrium involved along the curve
is S R q S M . Both the phases are solid. The system is monovariant. Since transformation of rhombic into
monoclinic sulphur is accompanied by increase of volume, the increase of pressure causes a rise in the
P-244

transition temperature. This can be predicted with the help of Clapeyron-Clausius equation which, when
applied to S R q S M equilibrium may be put as
dP ∆H t
=
dT T(VB − VA )
where ∆H t is the molar heat of transition. Since density of monoclinic sulphur (1.95 g cm −3 ) is less than that
of rhombic sulphur (2.05 g cm −3 ), V B is larger than V A . The right hand side of the above equation is,
therefore, positive. Hence, dP/dT should also have a positive sign. This means that increase of pressure raises
the transition temperature. The curve OC, therefore, slopes away from the pressure axis.
Example 7: The following equations give the vapour pressures of ice and water:
ln Pvapour (ice) = – (6140.1/T) + 24.00 …(1)

ln Pvapour (water) = – (5432.8/T) + 21.41 …(2)

where P is in mm Hg.Calculate (i) the temperature and pressure at the triple point of water (ii)
the molar enthalpies of sublimation, vaporization and fusion at the triple point.
Solution: (i) At triple point, the vapor pressure of ice and water will be equal, so,
 1  1
–6140.1   + 24 = –5432.8   + 21.41
 T  T
solving T=707.3/2.59=273.08 K.
The vapor pressure at the triplet point can be obtained by putting T in either of two equations. P=4.55
mm
(ii) Clapeyron equation states that
– ∆H m
ln p = +1
RT
Comparing with (1) & (2)
∆Hsublimation KJ
= –61401
. ⇒ ∆Hsublim = 51.05
R mol
∆H vap KJ
And, = –5432.8 ⇒ ∆H vap = 45.17 =
R mol
And, at triple paint, ∆Hsublim = ∆H f + ∆H vap
KJ
∆H f = 51.05 – 45.17 = 5.88 .
mol
Example 8: As supercooled water freezes sponteaneously, its temperature rises to 0°C. Find the
source of heat for the process:
H 2O(l) (–10°C) →H 2O (s) (0°C)?
Solution: In the given case, ∆H=0. The energy Liberated in freezing process warms the system to O°C.
Example 9: (i) Distinguish between the triple point and the freezing point of a pure substance.
(ii) For most pure substances which is suitable to be higher – the triple point or the freezing
point?
Solution:(i) The triple point is the point where solid, liquid and vapour are in equilibrium with one another.
The freezing point is the temperature at which solid and liquid phases are in equilibrium under 1 atm
pressure.
(ii) The freezing point is higher.
P-245

7.3 Two-Component System


For the two-component solid-liquid equilibria, some of the important cases are shown here,
1. The two components are miscible in the liquid state.
2. The two components are only partially miscible in the liquid state. The present discussion is limited to
the former case.
Under this category, the following cases are discussed:
(i). The two components are not miscible in the solid state and form a eutectic mixture.
(ii). The two components form stable compound with congruent melting point.
(iii). The two components form a compound with incongruent melting point.
Some of these systems are discussed at length by dividing into certain types like,
Type-a Simple eutectic systems
Type- b Systems in which two components form a stable compound,
Before starting the discussion one must have an idea of reduced phase rule.
Reduced Phase Rule: For a 2 component system, the phase rule becomes,
F = C – P + 2,
F = 2 – P + 2, so,
F = 4 – P, since the least number of phases, P, in any system is 1, the maximum number of degrees of
freedom is 3. Summarily, 3 variables are required to describe a system. Now, three variables are difficult to be
plotted, one of the variables is held constant. The degrees of freedom is reduced by 1 unit, so phase rule
equation changes to
F’ = C – P + 1, it is known as reduced phase rule.

7.3.1 Simple Eutectic System


The general phase diagram of such a system is present in.
Here, in the diagram the points A and B
represent the melting points of the a a′ B
components A and B. As increasing c
quantities of B are added to A, the freezing
Liquid melt b′
point of A falls along the curve AC. Similarly,
A
as increasing quantities of A are added to B, b
the freezing point of B falls along the curve t1 × Eutectic
Temperature

BC. Thus, along the curve AC, the solid A is Point


in equilibrium with the liquid solution of the Solid A+ solution
component B in A. This is called the freezing t2 Solid B+Solution
Y
point curve of the component A. Similarly, D′
along the curve BC, solid B is in equilibrium D d C d′
with the liquid solution of the component A Solid A Solid B
in B. This represents the freezing point curve + Solid a + Solid B +
of the component B. The number of phases Eutectic (Eutectic) Eutectic
along AC, as well as along BC, is two. 100% A Composition 100% B
Since, all measurements are done at
Fig. 5: A simple eutectic system at constant pressure
constant atmospheric pressure, the reduced
phase rule equation can be applied. Thus, F' = C − P + 1 = 2 − 2 + 1 = 1 Hence, the system is monovariant.
P-246

The composition varies with temperature along AC or BC, as the case may be. The two curves intersect at
some point C where both the solids A and B must be in equilibrium with the liquid phase. The number of
phases is 3. Applying the reduced phase rule equation, F' = 0 Thus, the system at C has no degree of
freedom. It is invariant. This means that under a given pressure, the system consisting of two solids and their
liquid solution, A – B – L, can exist only at a definite temperature and that the composition of the liquid
solution phase is definite as well. The point C, as can be seen, is the lowest temperature at which liquid can
exist in equilibrium with the solids A and B. Since the mixtures of A and B of composition corresponding to
point C, has the lowest melting point, the point C is known as the eutectic point. The word eutectic means
easy melting. In the area above the lines AC and BC, the two components are present only as a
homogeneous liquid solution. As the system consists of only one phase in this area, it is bivariant. So, in order
to define any point in this area, it is essential to specify the temperature as well as the composition. It is
understood that the pressure has been kept constant.
A liquid mixture of composition represented by a point a is cooled at a constant pressure. The temperature
will fall without any change of composition until the point b on the freezing point curve AC is reached. Around
this temperature, which corresponds to t 1, the solid A separates out. Assuming pressure to be constant, the
system now consists of two phases and is, therefore, monovariant . The temperature will fall only with change
in composition of the liquid phase. Therefore, as cooling continues, the component A keeps on separating
out and the solution becomes relatively richer in B. The temperature and the composition of solution both
change along the curve bC. Thus, at temperature t 1, solid A is in equilibrium with solution of composition x
and at temperature t 2 it is in equilibrium with solution of composition y. It is evident, therefore, that in area
ACD, solid A is in equilibrium with solutions of varying composition given by the curve AC depending upon
the temperature.
When the eutectic temperature is reached at d, the second solid B also begins to crystallize out. The system
now has 3 phases and, therefore, at constant pressure, it becomes invariant. On further cooling the system,
solid A and solid B separate out together in a fixed ratio so that the composition of the solution remains
constant as indicated by the point C. The temperature also remains constant. When the solution phase has
been completely solidified and the system consists only of a mixture of solid A and solid B, it becomes
monovariant and continued cooling results in fall of temperature below the line DD’ into the area within
which only the two solids coexist. Similarly, if the composition of the original liquid is on the right side of the
eutectic point, as represented, say, by the point a’, similar series of changes will be observed on cooling. In
this case, however, on reaching the point b’ on the freezing point curve BC, the solid B will separate out. As
cooling continues, B keeps on separating and the solution now becomes richer in A. The composition and the
temperature both change along the curve b’C. Thus, in the area BCD’, solid B is in equilibrium with solutions
of varying composition. Again, when the eutectic temperature is reached at the point d’, A also begins to
separate out. The system now becomes invariant. On withdrawing heat further from the system, the
temperature remains constant and solid A and solid B continue to separate in the same ratio. As a result, the
solution composition also remains constant. Ultimately, the solution phase solidifies completely as before and
thereafter the temperature of the system can fall below the line DD’ into the area of coexistence of two solids
A and B.
A special case is considered when the liquid has the same composition as that of the eutectic. This is
represented by the point C. On cooling, no solid will separate out until the eutectic temperature is reached. At
this temperature, both solids A and B crystallize out simultaneously. The temperature and composition of the
solution remain constant until the system gets completely solidified.
P-247

Thermal Analysis: It is observed on reference to fig. 5, that when a liquid mixture that is its melt consisting
of A and B is allowed to cool, either of two solids A or B begins to separate out as soon as the temperature
falls to a point on the line AC or BC, as the case may be. The number of phases is now two - the liquid mixture
and the solid A or B. If cooling is continued thereafter, the given solid continues to deposit with the
corresponding fall in temperature till the eutectic temperature - point C is reached. At this point, both the
solids separate out and the number of phases rises from a
two to three. The system now becomes invariant and, Freezing point
therefore, the temperature remains constant. One
b

Temperature
important point to be remembered in this connection is Eutectic
that the separation of the solid A or B from the liquid d d´
solution is an exothermic process. The heat evolved in
the process is similar to the heat given out when the same
substance changes from liquid to solid state at its freezing
point. As a result of this, the rate of cooling is diminished e
as soon as the separation of solid A or B commences to
take place. Thus, the rate of cooling along bd is slower Time
than that along ab until the eutectic point d is reached. Fig. 6: A Cooling Curve in a Two Component
After this, temperature remains unchanged. The above System forming a Eutectic.
facts can be represented by plotting fall of temperature
with time as in fig. 6.
Such a graph is called a cooling curve. In the figure, the rate of cooling along ab is quite rapid. This is when
separation of a solid has not yet commenced. At b one of the solids begins to separate. The rate of cooling
along bd, therefore, slows down. The cooling curve thus shows a distinct break at the point b. At d, the
eutectic point is reached. The temperature, therefore, remains constant along dd until the solidification has
taken place completely. With the disappearance of the liquid phase, the system now consisting only of solid
phase, becomes univariant and further cooling results in fall of temperature along de. Thus, there are three
breaks in the cooling curve. The first one occurs at the freezing point of the mixture where the solid first
commences to form. The second break occurs at the eutectic point and the third one occurs when the mixture
gets completely solidified.
This discussion provides a basis for the principle on which the method of thermal analysis used in the study of
solid-liquid phase equilibria involving metals, is based. In actual practice, a number of mixtures of metals A
and B of different composition lying between 100% A and 100% B are prepared. These are heated above
their respective melting points so that there is only one phase - liquid phase in every case. Each liquid is then
allowed to cool slowly and the temperature is recorded after small intervals of time. In this way several cooling
curves are obtained. The first break in each curve occurs at the freezing point, that is, at the commencement
of the freezing of the mixture of that particular composition. As the mixtures differ in their compositions, their
freezing will commence at different temperatures. The second break occurs at the point at which the
temperature remains constant. This gives the eutectic temperature which will be the same in all the mixtures
irrespective of their initial compositions. The freezing points of pure A and pure B are also determined in a
similar manner. The equilibrium diagram for the system of two metals - A and B under examination is thus
determined. This is illustrated in the figure below. There are many two-component systems which follows the
general behavior as depicted in fig. 7. They include pairs of metals, metallic alloys, organic compounds and a
number of salts and water.
P-248

B B

A
Temperature A

1 2 3 4 5 6 C

Time 0.1% 1 2 34 5 6 100%


Composition
Fig. 7: Cooling Curves for Different Compositions in a Two-Component
System forming a Eutectic
Two cases, one involving metals – lead and silver system, the other involving a salt and water for example,
CuSO 4 –H 2 O, and some more systems are discussed here at length.
1. Lead–Silver System: These metals are completely miscible in liquid state and do not give rise to any
compound formation. The equilibrium diagram, therefore, is similar to that shown in figure. The
various features are illustrated in fig. 8.
The phases co-existing in the various areas B 961°
or regions are also shown in the fig.8. Pure Liquid melt
form of lead melts at 327°C and the
addition of silver lowers its melting point
Temperature

along AC. Thus, AC is the freezing point a


curve of lead. Pure silver metal melts at A
327°
961°C and the addition of lead lowers its liquid Liquid+Solid Silver
melting point along BC. Thus, BC is the + B
freezing point curve of silver. Along AC, Solid lead 303°
303° c
solid lead and solutions coexist while along Solid lead C Solid silver
BC, solid silver and solution coexist. The + +
system, at constant pressure, is Eutectic Eutectic
monovariant along AC as well as along 100% Pb 2.6% Ag Composition 100% Ag
BC. C is the eutectic point where the three 0% Ag 0% Pb
phases, solid lead, solid silver and their Fig. 8: The phase diagram for lead-silver caption system
liquid solution (or melt), coexist. It is an
invariant point. The temperature at the eutectic is 303°C and the composition of the solution phase is 2.6
percent silver, as shown in the figure.
(i) Desilverisation of Lead: Phase diagram of lead-silver has a special significance in connection with
the desilverisation of lead. The argentiferous lead consisting of a very small percentage of silver is first
heated to a temperature well above its melting point so that the system consists only of the liquid phase
represented, say, by the point a. Subsequently, it is allowed to cool. The temperature of the melt will fall
along the line ab. As soon as the point b is reached, lead will start to crystallize out and the solution will
contain relatively increasing amounts of silver. Further, cooling will shift the system along the line bc.
Lead continues to separate out and is constantly removed by ladles. The melt continues to be richer
and richer in silver until the point C is reached where percentage of silver swells to 2.6%. Thus, the
original argentiferous lead which might have contained 0.1 percent or even less of silver, is now
enriched up to 2.6 percent of this metal. The process of enrichment of the relative proportion of silver in
the alloy is known as Pattinson’s process.
P-249

2. Potassium Iodide – Water System: Potassium iodide water system is a typical example of binary
system involving a salt and water which form eutectic mixture. An essential feature of a salt-water
system is that the melting point of the salt is usually very high, even higher than the critical temperature
of water. So, interestingly, it is not possible to represent melting points of both the components in the
equilibrium diagram as has been done in the case of the binary system involving metals. The
equilibrium diagram of potassium iodide – water system is shown in fig. 9.

C B
a′

Solution
b′

A
Temperature

0° F.P
A . Cu
rv
b eo Solution+Solid KI
fW
at
Solution+ ice er

–23°
C
Eutectic+ice Eutectic+solid KI

Solid KI+ice
0 Composition (%KI) 100

Fig. 9: The Phase Diagram for the Potassium Iodide-Water System


Its resemblance with the general diagram for binary systems forming eutectics is quite evident. However, the
melting point of potassium iodide is not realised in practice. A is the melting point of ice. At a pressure of 1 atm
and in the presence of dissolved air, this temperature is 0°C. As KI is added, some of it dissolves in the water
which was in contact with ice. The number of components rises to two. As the experiments are generally
conducted at atmospheric pressure, the vapour pressure of water can be neglected and hence the vapour
phase can be ignored. The system, therefore, is considered to have two phases - ice and solution. Duly, the
degree of freedom of the system at constant pressure, will be 1. Thus, corresponding to each temperature,
there will be a definite composition of the solution. In other words, if the temperature changes, there will have
to be a change in concentration of the solution. Similarly, if the composition of the solution changes, there is a
corresponding change in temperature. If the addition of KI is continued, there will be a change in the
concentration of the solution and there will be a corresponding change in the temperature of the system.
This is represented by the curve AC which is known as the freezing point curve of water. Along this line, the
solution containing potassium iodide in water is in contact with ice. At the point C, the solution becomes
saturated and, therefore, potassium iodide also spearates out as another solid phase. The system now
consists of three phases - solid ice, solid KI and solution in equilibrium. The point C, therefore, is invariant at a
constant pressure. This is the eutectic point of the system. The temperature at this point is –23°C which is the
lowest temperature that can be attained in this system.
P-250

If the system at C is heated, ice starts melting and solid potassium iodide will pass into solution in the same
ratio in which it is already present in the solution so that composition of the solution remains unchanged. The
temperature of the system does not rise so long as the number of phases is 3. The heat supplied to the system
is utilised in bringing about the change of state of ice into water. If heat continues to be supplied to the system
and if KI is present in excess, ultimately ice will disappear. The system is left with two phases-solid KI and
solution and will become univariant. The temperature will now rise but there will be a corresponding change
in composition. The curve CB is traced out. Since solid potassium iodide is in contact with its solution in water
along the curve CB, this is known as the solubility curve of potasium iodide. The sharp rise of the curve shows
that the solubility of KI increases slowly with rise of temperature. It is noted that along both the curves AC and
CB, the solutions of potasium iodide of varying compositions are in equilibrium with a solid phase. The solid
being ice along AC and KI along CB. The two curves meet at the eutectic point C which has been discussed
above. In the area lying above the curves AC and CB, only solution phase can exist. The existence of various
phases in different regions of the equilibrium diagram is shown in the figure (fig. 9). If a solution represented
by the point a is cooled, the temperature will fall along ab without any change of composition as the system is
bivariant. On reaching b, ice separates out. The system will now become univariant.
Therefore, the composition of the solution will change with temperature. Accordingly, on continued cooling,
the composition of the solution will move along the freezing point curve AC until the eutectic point C is
reached when potasium iodide will also separate out. The number of phases now increases to 3 and the
degree of freedom at constant pressure falls to zero. Ultimately, the solution will freeze to give the eutectic
mixture at a constant temperature.
If a solution of composition - a’, lying on the right of the eutectic point, is cooled, KI will begin to crystallise out
as soon as the point b’ is reached. The composition will henceforth change with the temperature along bc and
more of potassium iodide will continue to separate out until at the eutectic point C, ice also begins to deposit.
Eventually, the whole of the solution will freeze to give the eutectic mixture consisting of 52% potassium
iodide and 48% ice at a constant temperature. Finally, a solution of composition represented by c is
considered which lies vertically above the eutectic point. When such a solution is cooled, the temperature will
continue to fall along cC without any change in composition until the eutectic point C is reached when ice and
KI both begin to separate out simultaneously.
It is observed from the above discussion that all solutions on cooling ultimately show a halt in temperature at
the eutectic point. Further, when a solid mixture of the same composition as the eutectic mixture, is heated, it
melts sharply at the eutectic temperature. Hence, the mixture of potassium iodide and ice deposited at the
euteic point was at one time considered to be a definite compound of the form of a salt hydrate. This was
given the name cryohydrate. However, a closer look showed that physical properties such as density and
heat of solution of the eutectic solid were almost exactly equal to the mean values of the two constituents,
namely, potassium iodide and ice, indicating that it is a mixture. Further, when the solid was examined under
a powerful microscope, both the constituents were seen to lie as separate crystals. Therefore, the eutectic
solid is a mixture and not a compound.
Freezing Mixtures: It is observed that the addition of salt to ice produces an appreciable fall in temperature.
It can be explained easily by means of a phase diagram of the kind shown in figure. If some salt, such as
potassium iodide or sodium chloride, is added to ice in contact with a small amount of water at 0°C then,
some of the salt will dissolve in the water. Subsequently, there will be three phases - salt, ice and solution in
contact with each other. Now three phases can coexist in stable equilibrium only at the eutectic temperature.
This temperature, in most cases, is well below the normal melting point of ice. Hence, ice will melt and the salt
P-251

will dissolve in the water thus produced so that the composition varies along the line AC. Now, melting of the
ice as well as dissolution of the salt are accompanied by absorption of heat. If the system is not in a position to
absorb heat from the surroundings, its temperature will fall. This fall, which takes place along the line AC, will
continue until the eutectic point is reached or till either salt or ice is used up completely. The lowest
temperature that can be attained in the presence of excess of a salt and ice depends upon the eutectic
temperature of the system. Table 1 below deals with the eutectic temperatures and eutectic compositions of
some of the common systems as shown below,
Table: 1. Eutectic composition and temperature of common systems
System Composition Eutectic Temperature
(% salt in the eutectic) (°C)

NH 4 Cl and Ice 20.1 –16.0

NH 4 NO 3 and Ice 43.0 –18.0

NaNO3 and Ice 33.3 –18.1

NaCl.2H 2 O and Ice 23.0 –22.0

KI and Ice 52.0 –23.0

CaCl 2 .6H 2 O and Ice 15.2 –55.9

7.3.2 Systems in which Two Components form a Stable Compound


There are again numerous systems including metals, organic compounds and inorganic salts dissolved in
water, which fall under this category. The two components of the system at a certain stage enter into chemical
combination with one another forming stable compounds. Thus, a number of intermetallic compounds when
the two components are metals are formed. Similarly, several double compounds are formed when the two
components are organic compounds. In the case of systems involving salt and water, a number of salt
hydrates are formed.
Such systems are of two kinds depending upon whether the compound formed has a congruent or an
incongruent melting point. So, there are two classes
1. Class I. Formation of Compounds with Congruent Melting Points
2. Case II. Formation of Compounds with Incongruent Melting Points
1. Class I. Formation of Compounds with Congruent Melting Points: A compound which melts
sharply at a constant temperature into a liquid of the same composition as the solid, is said to possess a
congruent melting point. Considering a general case, let A and B be the two components and AB a
stable solid compound formed by their chemical combination. The phase diagr am will be of the type
shown in figure (fig.10). In this system there are three different solid phases, namely A, B and the
compound AB. Accordingly there will be three fusion or freezing point curves AC, BE and CDE. While
along AC, the solid A is the equilibrium with the liquid phase and along BE, the solid B is in equilibrium
with the liquid phase. Similarly, along the central portion CDE, the solid compound AB is in
equilibrium with the liquid phase, at different temperatures. The maximum point D of the curve is the
congruent melting point of the compound because the solid and liquid phases now have the same
composition.
P-252

Naturally, at this temperature, the two component system


has become a one-component system because both solid Liquid D Liquid
A
and liquid phases contain only the compound AB.
t x x

So Liq
Therefore, accordingly to the phase rule, D is a 1

lid uid
non-variant point. This represents a definite temperature

A
Temperature
Compound AB

+
just like the melting points A and B of the pure + Liquid

id +
components. The congruent melting points D of C

qu E
Li lid
compound AB has been shown to lie above the melting

So
points of the pure components A and B. But, it is not
Solid A +
essential. There are different types of systems known in Solid B +
Compound AB
which the melting point of the compound formed lies Compound AB
above, below or in-between the melting points of the pure
D
components. 100 % A Molar composition 100 % B
In the above fig. the compound has been shown to
Fig.10: Formation of compounds with
contain equimolar amounts of A and B. This is not Congruent Melting Points
necessarily so in every system. There are two eutectic

points in such a system as represented by C and E in the figure. At C, the solids A and AB are in equilibrium
with the liquid phase while at E, the solids B and AB are in equilibrium with the liquid phase.
It will be seen that at a certain temperature, say, t, the liquid phase can have two compositions, X and X 1 in
equilibrium with the same solid AB. In other words, the compound AB can have two solubilities at the same
temperature. This paradox can readily be explained if the diagram is looked upon as made up of two parts
with DD’ as the dividing line. The left half of the diagram shows two-component system, A and AB while the
right half shows the two–component system, B and AB. On the left hand side, the curve DC represents the
usual depression in freezing point of the compound AB on the addition of A while on the right hand side, the
curve DE represents the usual depression in freezing point of the compound AB on the addition of B. The
names of the various phases in different areas of the diagram are represented in the figure 10.
The two-component systems which form one or more compounds with congruent melting points include
aluminium-magnesium, zinc-magnesium, gold-tin, mercury-thallium and phenol-aniline systems. Amongst
the salt hydrates, FeCl 3 – H 2 O system falls into this category.
(i) Ferric Chloride – Water System: Ferric chloride– water provides an interesting case in which a
number of hydrates with congruent melting points are formed. There are four stable hydrates of
composition, Fe 2 Cl 6 .12H 2 O–dodecahydrate, Fe 2 Cl 6 .7H 2 O–heptahydrate, Fe 2 Cl 6 .5H 2 O–pentahy;
–drate and Fe 2 Cl 6 .4H 2 O– tetrahydrate. Double formula Fe 2 Cl 6 is used for ferric chloride in order to
avoid the use of a fractional number for the molecules of water in the case of hepta and pentahydrates.
The complete equilibrium diagram of the system is shown in fig. 11.
The point A represents the freezing point of water (0°C) under atmospheric pressure. On adding a small
amount of ferric chloride, a solution of appropriate composition is obtained and the temperature falls
along the curve AB. Since, the system now has two components and two phases, ice and solution,
ignoring the vapour phase, the degree of freedom applying modified phase rule is 1. Therefore, with
change in temperature, there will have to be change in composition, at constant pressure. Hence, on
increasing amounts of ferric chloride, there will be fall in temperature alongwith change in composition
P-253

till the eutectic temperature B is reached where the dodecahydrate, Fe 2 Cl 6 .12H 2 O, separates out as a
new solid phase. The system consists of 3 phases and, therefore, B is an invariant point. Generally the
vapour phase is ignored. The temperature at this point is 55°C. Evidently, this is the lowest temperature
that can be attained in this system.

Solution L

l6
J

FeC
2
G
K
C H
a b c dE e f g
h
F
D
Temperature

A
O
5H2O

4H2O
7H2O
12H2O
IC
E

–55°
B

Composition
Fig. 11: The Phase Diagram for the Ferric
chloride–Water system

On continued addition of ferric chloride and heating, ice will disappear ultimately and the number of phases
becomes 2. So, the system becomes monovariant. Now the temperature will rise and with it the composition
of the solution will change along the curve BCD. Thus, BCD is the solubility curve of Fe 2Cl6 .12H 2 O. At C,
the liquid and the solid phases have the same composition, namely, Fe 2Cl6 .12H 2 O. Therefore, C is taken as
the congruent melting point of the dodecahydrate. The significance of the retroflex portion of the solubility
curve can be and CD, in fact, represent the effect of adding water and ferric chloride, respectively, in lowering
the congruent melting point of dodecahydrate.

At D, the heptahydrate, Fe 2Cl6 .7H 2 O, is separates out as a new solid phase. The system again becomes
invariant thereby giving rise to D, as the second eutectic point. Similarly, DEF is the solubility curve of
Fe 2Cl6 .7H 2 O with E as its congruent melting point. F is the third eutectic point where the three phases which
coexist are heptahydrate, pentahydrate and solution.

FGH is the solubility curve of Fe 2Cl6 .5H 2 O with G as its congruent melting point and H as the fourth eutectic
point where the three phases coexisting are pentahydrate, tetrahydrate and solution. Then follows the
solubility curve HJK of Fe 2Cl6 .4H 2 O, J being the congruent melting point of this hydrate. At K, the
anhydrous ferric chloride separates out. K, therefore, is the fifth eutectic point. Thereafter follows the
solubility curve KL of the anhydrous ferric chloride. No other solid phase separates out along this line. In
addition to the five eutectic points, B, D, F, H and K, at which the system is non–variant (i.e., F’ = 0), there
are four other points also at which the system is non–variant. These are the congruent melting points C, E, G
and J.
P-254

Table: 2. FeCl 3 – water system

Point Temperature Solid Phase Moles of Fe 2Cl 6 per


(°C) 100 moles H 2O

A 0 Ice 0
Eutectic p.B –55.0 Ice,Fe 2 Cl 6 12H 2 O 2.75
Cong.m.p.C 37.0 Fe 2 Cl 6 .12H 2 O 8.33 (1:12)
Eutectic p.D 26.0 Fe 2Cl6 .12H2O,Fe2Cl6 .7H2O 12.15
Cong.m.p.E 32.5 Fe2Cl6 .7H2O 14.28 (1:7)
Eutectic p.F 30.0 Fe 2Cl6 .7H2O,Fe2Cl6 .5H2O 15.12
Cong.m.p.G 56.0 Fe2Cl6 .5H2O 20.00 (1:5)
Eutectic p.H 55.0 Fe 2Cl6 .5H2O,Fe2Cl6 .4H2O 20.32
Cong.m.p.J 73.5 Fe2Cl6 .4H2O 25.00 (1:4)
Eutectic p.K 66.0 Fe 2Cl6 .4H2O,Fe2Cl6 29.9

The temperatures, compositions and the names of the solid phases coexisting at the five eutectic points and
the four congruent melting points are given in Table 2. The freezing point of ice is also included.
Isothermal Evaporation: The isothermal evaporation of an unsaturated solution of ferric chloride presents
some interesting features. A dilute solution is allowed to evaporate gradually at a constant temperature of say,
30°C. a represents the composition and temperature of the solution. If the temperature is maintained
constant, the concentration of the solution keeps on changing gradually along the line ah on evaporation. In
the start, the solution becomes more and more concentrated until at the point b, the dodecahydrate separates
out. The solution is now saturated with respect to this compound. Since the system has only 1 degree of
freedom, the composition of the solution remains unchanged because the temperature remains constant.
The process is known as isothermal evaporation. This continues until the solution has completely evaporated
leaving only the solid Fe 2Cl6 .12H 2 O behind. On continued evaporation, water is knocked out from the
hydrate and a solution of higher composition c begins to appear. The amount of this solution goes on
increasing until the whole mass has liquefied and the solution has the composition represented by c. Thus, by
continuous removal of water by isothermal evaporation, the solution at a has first changed completely to
solid and then completely to liquid. If the evaporation is continued further in a similar manner, the solution
will change completely into solid at d and into liquid and e. The third solidification, this time of the
pentahydrate, will begin at f followed by completely solidification at g which will be subsequently followed by
the conversion of solid pentahydrate into solid tetrahdyrate and ultimately into solid anhydrous ferric
chloride. These changes, through unusual, can be readily interpreted in the light of the phase rule.
2. Case II. Formation of Compounds with Incongruent Melting Points: There are systems in
which the compounds formed by the combination of two components, instead of melting congruently,
decompose when heated giving a new solid phase and a solution with a composition different from
that of the solid phase. Such a compound is said to have an incongruent melting point. Its
decomposition at this temperature is known as transition reaction or meritectic reaction. It is
represented by the equation,

q
S1 S2
+solution (or melt)
Original solid new solid
P-255

The incongruent melting point is, therefore, also known as transition temperature or meritectic or
peritectic temperature. Since during the peritectic reaction, as represented by the above equation, the
number of phases is 3 – two solid and one liquid, the system is invariant as the pressure is fixed. Hence,
the temperature as well as the composition of all the phases remains fixed.
The general phase diagram for this type of systems
may be represented graphically in fig.12. Liquid B
It is presumed that. A and B be the two components Liquid
which combine to form a compound AB 2 . The points +
A x

Temperature
A and B represent the respective melting point of the E Solid B
two pure components while D represents the x D
incongruent melting point or transition temperature Liquid
Solid A + Solid B
of the compound AB 2 . Here, this compound breaks
+ Compound +
down to give a new compound. This point lies below Compound
Liquid AB2
E which may be taken as the hypothetical congruent AB2
C
melting point of the compound AB 2 . AC is the fusion Solid A +
curve of A along which the solid is in equilibrium with Compound AB2
the liquid. BD is the fusion curve of B along which 100% A Molar Composition 100% B
the solid B is in equilibrium with the liquid. Similarly,
Fig. 12: Formation of Compound with
CD is the fusion curve of the compound AB 2 along Incongruent Melting Point
which the solid AB 2 is in equilibrium with the liquid.
On cooling a liquid of composition x, A is the first
solid to separate out at the point x ′.
Further cooling then proceeds with change of composition along x ′C. At C the compound AB 2 commences
to form. The system now consists of two solids and one liquid phase. It excludes the vapour phase by
applying the reduced phase rule the system at C is non-variant. If a liquid of composition y is cooled down,
the first solid to separate out will be solid B at the point y ′. Further cooling will proceed along y ′D. At D, the
peritectic reaction takes place and the compound AB 2 separates out as shown below,
Solid B + solution q Solid AB2
Since the system will have now three phases, it will again become non-variant, the transition of solid B into
compound AB 2 will take place at constant temperature till the whole of the solid B disappears. Point D is,
thus, the peritectic point. Picric acid–benzene and sodium–potassium are some examples of two component
systems which give rise to compounds with incongruent melting points.

7.3.3 Calculation of the Eutectic Point and Eutectic Composition


When a two–component solution is cooled, the component which is present in excess amount separates out
as solid at the freezing point of the solution. The variation of the freezing point with composition is given by
the following expressions,
d lnX A = (∆H fA / RT2 )dT

d lnX B = (∆H fB / RT2 )dT

where the ∆H f s are the enthalpies of fusion of the two components and X A and X B are their mole fractions. It
is assumed that the components A and B behave ideally and ∆H f is independent of temperature, so that after
integrating the above equations for the limits T = T m , the melting point and T = T e, the eutectic point, to
obtain
P-256

∆H fA  1 1 
− ln (X A )e =  − 
R  Te TmA 

∆H fB  1 1 
and – ln (X B)e =  − 
R  Te TmB 

Using these two equations, the value of T e, the eutectic point as well as the eutectic composition (X A ) e and (X
B) e can be determined.

7.3.4 CuSO 4 And Water System


Two component systems in which solid and gas phases exist in equilibrium are important only when the
components combine to form one or more compounds with each other. The particular cases of interest are
furnished by salt hydrates and metallic ammoniates. As an illustration, one such system in which the two
components are CuSO 4 and H 2 O is considered here. In this system, there can be, under different conditions,
four solid phases, namely,

1. CuSO4 .5H 2 O

2. CuSO4 .3H 2 O

3. CuSO4 .H 2 O and

4. CuSO4 anhydrous.

If the pentahydrate is gradually dehydrated at a given temperature, the following dissociation reactions are
possible one after the other:

CuSO4 .5H 2 O(s) q CuSO4 .3H 2 O (s)+2H 2 O(g)

CuSO4 .3H 2 O (s) q CuSO4 .H 2 O (s)+2H 2 O(g)

CuSO4 .H 2 O (s) q CuSO4 (s)+H 2 O(g)

Importantly, in each state of the process there are three phases – two solid phases and one vapour phase, the
degree of freedom is unity. Thus, if temperature is kept constant, the vapour pressure of water must also
remain constant. If CuSO4 .5H 2 O crystals are taken in a vessel which is connected to a vaccum pump and
also to a manometer and the evacuation starts, the pressure of water vapour can be read on the manometer
from time to time. If the vessel is placed in a thermostat maintained at, say, 50°C, it will be seen that the
vapour pressure remains constant at 45.4 mm. This continues as such till the entire amount of pentahydrate -
CuSO4 .5H 2 O has changed into trihydrate - CuSO4 .3H 2 O. This state of affairs is represented by the line AB
in figure. After this the pressure drops to 30.9 mm, from the point B to C as shown. At this stage, the trihydrate
starts dissociating to monohydrate - CuSO4 .H 2 O. The pressure remains constant at this level along the line
CD. It drops to point E, corresponding to 4.4 mm pressure, only when the entire trihydrate has change to
monohydrate. Along EF, the monohydrate is dissociating into anhydrous copper sulphate. The pressure
remains constant at 4.4 mm till the complete of this process.
P-257

Temperature=50°C
CuSO4.5H2O ! CuSO4.3H2O+2H2O
45.4 mm B
A
Vapour Pressure

CuSO4.3H2O ! CuSO4.H2O+2H2O
30.9 mm D
C

CuSO4.H2O ! CuSO4+H2O
4.4 mm
E F
5H2O 4H2O 3H2O 2H2O H2O H2O
Composition
Moles of water per mole of CuSO4
Fig. 13: Phase diagram representing dehydration sulphate crystals at 50°C

Example 10: For a two–component system, ∆H f,A = 500 cal mol −1 and Tm, A = 400°C. If the
eutectic temperature is 350°C, calculate the solubility in terms of the fractions of B in A at
350°C.
Solution: As it is known,
∆H f,A  1 1
– = ln(X A )e
R  Tm.A Te 

500 × 4.184  1 1 
So, ln (X A ) e= –
8.314  673 623

(X A )e = 0.93 and (X B)e = 0.07


Example 11: Calculate the eutectic temperature and eutectic composition for a binary
solid–liquid system if ∆H f,A = 500 cal mol −1, ∆H f,A = 1000 cal mol −1 and the melting points of
pure A and B are 400°C and 600°C, respectively.
Solution: As it is known
500 × 4.184  1 1
ln (X A )e = –
8.314 
673 Te 

1000 × 4.184  1 1
And, ln (X B)e = –
8.314 873 Te 

And, XB = 1– XA
Solving (X A )e = 0.647 and (X B)e = 0.352
Te = 38 ° C .
P-258

Long Answer Type Questions 15. State phase rule and explain the terms involved.
Based on Different Basic Terms Draw well labeled phase diagram of Mg-Zn system.
1. What is Gibbs phase rule? State the number of Discuss the main features of this diagram.
phases and components in the following systems : [Garhwal 2009]
(i) Water in a beaker at ordinary temperature 16. (i) Derive Gibbs Phase rule thermodynamically.
(ii) Solution of sugar in a beaker at ordinary (ii) Explain the phase diagram to silver-lead system.
temperature. [Garhwal 2007] [Garhwal 2008]
Based on One Component System 17. State the phase rule and discuss all the terms
2. Construct the phase diagram of water system and involved in it. Apply the phase rule to lead silver
explain it. [Garhwal 2008] system. [Alld. 2010]
3. (i) Define component. phase and degree of 18. (i) Derive Gibbs's phase rule.
freedom. (ii) Discuss phase diagram of water in brief.
(ii) Draw and discuss phase diagram, of water
(iii) Discuss phase diagram of Pb-Ag.
system. [Alld. 2008; Agra 2009] [D.D.U. 2009, 11; Purv. 2008]
4. State the phase rule and discuss its application to 19. (i) Briefly discuss the phenomenon of eutectic
sulphur system. [Alld. 2009] and peritectic meltings.
5. State phase rule and discuss its application to water (ii) Draw the phase diagram of a system when
system. Draw the phase diagram. [Alld. 2012] solids A and B form a congruantly melting
6. (i) Discuss the phase diagram of water system. compound AB 2 . Label all the phases and
(ii) Explain the degree of freedom with equilibria involved in the phase diagram.
example. [Purv. 2010] (iii) How many phases and components are
7. Draw a labelled phase diagram of sulphur system present in a system represented by the
and discuss its main features. [Lko. 2008] equilibrium:

CaCO 3 (s) Q
8. (i) Write phase rule equation and define all
terms involved in equation. CaO(s)+CO 2 (g)
(ii) Draw the phase diagrams of Pb-Ag system [D.D.U. 2008]
and discuss the salient features of the 20. (i) Distinguish between first order and second
diagram. [D.D.U. 2010] order phase transition.
9. What is phase rule for carbon–dioxide system? (ii) Show that in a two component system
Give the phase diagram for carbon–dioxide eutectic point is invariant.
system. Explain dry ice and its uses. Short Answer Type Questions
[Kanpur 2012] Based on Different Basic Terms
10. (i) Phase diagram of simple electric system. 1. Explain degree of freedom by an example.
(ii) Phase diagram of water system. [Lko. 2009] [Garhwal 2010]
Based on Two Component System 2. Explain the term components and degree of
11. What is phase rule equation for Pb-Ag system? freedom. [Garhwal 2011]
Give the phase diagram for Pb-Ag system and
3. Write the equation for phase rule.
explain the theory of Pattinson’s process for
desilverization of Ag. [Garhwal 2005]
[Garhwal 2010]
4. Explain the term phase, components and degree of
12. Discuss the phase diagram of Bi–Cd system.
freedom. [Garhwal 2011; Kanpur 2009, 11]
[Garhwal 2011]
13. Draw and explain the phase diagram of a two 5. Define 'Phase' and 'Component'. [Purv. 2010,11]
component system of solids A and B forming a 6. Define phase and components with examples.
compound AB with a congruent melting point. [D.D.U. 2009]
[Garhwal 2010] 7. Estimate the number of components present in
14. Explain the terms eutectic and triple point. KCl–NaCl–H 2 O system? [D.D.U. 2011]
Calculate the number of phases, components and Based on One Component System
degrees of freedom for the following: 8. Draw a phase diagram for sulphur system. Name
(i) S (s) S (m) S (l) S (g) all the curves, regions and points theorem. What
(ii) CaCO 3 (s) → CaO (s) + CO 2 (g) would be the degree of freedom at the triple point?
[Garhwal 2010] [Garhwal 2010]
P-259

9. Explain lower and upper consulate temperature 28. What do you understand by two components
diagrammatically by graph and how impurity system? Explain lead-silver system.
affects these, discuss. [Garhwal 2012] [Garhwal 2005]
10. Write the condensed phase rule equation. 29. Explain the difference between melting point, triple
point and congruent melting point.
[Garhwal 2009]
[Garhwal 2009]
11. Draw a self-explanatory phase diagram of sulphur.
30. What are congruent and incogruent melting
[Garhwal 2009]
points? [Purv. 2008,09]
12. Draw a self-expanatory phase diagram of water.
31. Discuss congruent m.p. with an example.
[Garhwal 2008]
[Purv. 2011]
13. In reference to phase equilibrium, explain:
32. An eutectic mixture has a definite composition and
(i) Eutectic point (ii) Triple point melting point yet it is not a compound. Why?
(iii) Degree of freedom (iv) Condensed [Lko. 2010]
phase rule 33. Explain first order phase transition.
[Garhwal 2008]
14. What will be the number of components in sulphur [D.D.U. 2009]
system? [Garhwal 2006] 34. Draw phase diagram for Mg–Zn system. Discuss
application of phase rule on it. Explain congruent
15. Explain the following :
melting point. [Kanpur 2010]
All four phases cannot exist in sulphur system.
35. Explain system with upper critical solution
[Garhwal 2010]
temperature by giving one example. What is the
16. Discuss phase diagram of sulphur system. effect of impurities on critical solution temperature?
17. (i) Brief description of phase diagram of [Kanpur 2011]
sulphur system.
(ii) Component and degree of freedom in
phase study. [Purv. 2011]
Very Short Answer Type Questions
Based on Different Basic Terms
18. Ammonium chloride is heated in a closed vessel. It
can be either on componcnt or a two componcnt 1. Define a phase.
system. Explain. [Lko. 2008, 09]
2. What is phase rule?
3. Define a component.
19. What is meant by triple point of water system? Why
it is different from normal melting point of ice? 4. Define degrees of freedom.
[Lko. 2011] Based on one component system
20. What is a triple point and how many phases are in 5. What is a triple point?
equilibrium at triple point in a one component 6. What is reduced phase rule?
system? [D.D.U. 2008] 7. Explain metastable equilibrium with respect to a
water system.
21. Draw phase diagram of water. [D.D.U. 2009]
Based on Two Component System
22. A triple point is trivariant or invariant or univariant.
8. Define eutectic point.
Give correct answer. How many phases are present
9. Give one example of a two component system.
at invariant point? [D.D.U. 2010]
10. How many solid phases are presant in CuSO 4 and
Based on Two Component System water system?
23. Draw a self-explanatory phase diagram of CuSO 4
– H 2 O system. [Garhwal 2012]
24. Draw the phase diagram of Pb-Ag system. Objective Type Questions
[Garhwal 2009]
25. Draw phase diagram of Ferric chloride water Multiple Choice Questions
system and explain the following : 1. NH 4 Cl(s) → NH 3 (g) + HCl(g) is a system
(i) Congruent melting point (ii) Eutectic point containing :
(iii) Isothermal evaporation of ferric chloride (a) Two components (b) One component
solution at 31°C [Garhwal 2009] (c) One phase (d) Three phases.
26. Discuss the phase diagram of ferric chloride-water [Garhwal 2011]
system. [Garhwal 2007] 2. Reduced phase rule equation is :
27. What do you understand by two component (a) F = C – P + 2 (b) F = C – P + 1
system? Explain ferric chloride-water system. (c) F = C + P + 2 (d) F = C + P + 1
[Garhwal 2006] [Garhwal 2010]]
P-260

3. Reduced phase rule equation is :


6. Below eutectic temperature ……………. phase
(a) F = C – P + 2 (b) F = C – P + 1
disappears. [Garhwal 2008]
(c) F = C + P + 2 (d) F = C + P + 1
[Garhwal. 2012]
4. In sulphur system, the number of component is : True and False
(a) Zero (b) One (c) Two (d) Three. 1. For a system F is maximum, when P = 1.
[Garhwal 2005] [Garhwal 2011]
2. The eutectic temperature for all binary system is the
same. [Garhwal 2008]
Fill in the Blank 3. Water is one component and two phase system.
1. The triple point has the degree of freedom
[Garhwal 2010]
……………… . [Garhwal 2012]
4. Reduced phase rule equation can be applied to CO
2. Water cooled below its freezing point is called ……
2 system.
[Garhwal 2008]
3. CO 2 is a ………… component stystem. 5. For an equilibrium H 2 O (l) a H 2 O(g), the
4. The temperature of triple point in water system is number of degrees of freedom is 1.
........... [Agra 2009]
5. Dissociation of ammonium chloride in a closed
vessel is a ……………. component system.
[Garhwal 2009]

Objective Type Questions

Multiple Choice Questions


1. (a) 2. (a) 3. (b) 4. (b)

Fill in the Blank


1. zero 2. super cooled 3. 1 4. 0.0075°C

5. 2 6. liquid

True and False


1. True 2. False 3. True 4. False

5. True

Hints and Solutions

True and False


4. It is applied to 2 component system.
mmm

You might also like